*NURSING > EXAM > Maternity and Pediatric Nursing 4th Edition Ricci Kyle Carman test bank (All)

Maternity and Pediatric Nursing 4th Edition Ricci Kyle Carman test bank

Document Content and Description Below

Maternity and Pediatric Nursing 4th Edition Ricci Kyle Carman Test Bank Chapter 1 Perspectives on Maternal and Child Health Care 1 . The United States ranks 50th in the world for maternal mortalit... y and 41st among industrialized nations for infant mortality rate. When developing programs to assist in decreasing these rates, which factor would most likely need to be addressed as having the greatest impact? A ) Resolving all language and cultural differences B ) Assuring early and adequate prenatal care C ) Providing more extensive womens shelters D ) Encouraging all women to eat a balanced diet 2 . When integrating the principles of family-centered care, the nurse would include which of the following? A ) Childbirth is viewed as a procedural event B ) Families are unable to make informed choices C ) Childbirth results in changes in relationships D ) Families require little information to make appropriate decisions 3 . When preparing a teaching plan for a group of first-time pregnant women, the nurse expects to review how maternity care has changed over the years. Which of the following would the nurse include when discussing events of the 20th century? A ) Epidemics of puerperal fever B ) Performance of the first cesarean birth C ) Development of the x-ray to assess pelvic size D ) Creation of free-standing birth centers 4 . After teaching a group of students about pregnancy-related mortality, the instructor determines that additional teaching is needed when the students identify which condition as a leading cause? A ) Hemorrhage B ) Embolism C ) Obstructed labor D ) Infection 5 . The nurse is working with a group of community health me plan to address the special health needs of women. Which conditions would the group address as the major problem? mbers to develop a of the following A ) Smoking B Heart disease ) C Diabetes ) D Cancer ) 6 . When assessing a family for possible barriers to health care, the nurse would consider which factor to be most important? A ) Language B ) Health care workers attitudes C ) Transportation D ) Finances 7 . After teaching a group of nursing students about the issue of informed consent. Which of the following, if identified by the student, would indicate an understanding of a violation of informed consent? A ) Performing a procedure on a 15-year-old without consent B ) Serving as a witness to the signature process C ) Asking whether the client understands what she is signing D ) Getting verbal consent over the phone for emergency procedures 8 . The nurse is trying to get consent to care for an 11-year-old boy with diabetic ketoacidosis. His parents are out of town on vacation, and the child is staying with a neighbor. Which action would be the priority? A ) Getting telephone consent with two people listening to the verbal consent B ) Providing emergency care without parental consent C ) Contacting the childs aunt or uncle to obtain their consent D ) Advocating for termination of parental rights for this situation 9 . After teaching nursing students about the basic concepts of family-centered care, the instructor determines that the teaching was successful when the students state which of the following? A ) Childbirth affects the entire family, and relationships will change. B ) Families are not capable of making health care decisions for themselves. C ) Mothers are the family members affected by childbirth. D ) Childbirth is a medical procedure. 1 A nursing instructor is preparing a class discussion on the trends in health care 0 and health care delivery over the past several centuries. When discussing the . changes during the past century, which of the following would the instructor be least likely to include? A Disease prevention ) B ) Health promotion C ) Wellness D ) Analysis of morbidity and mortality 1 A nurse is assigned to care for an Asian American client. The nurse develops a 1 plan of care with the understanding that based on this clients cultural . background, the client most likely views illness as which of the following? A Caused by supernatural forces. ) B A punishment for sins. ) C Due to spirits or demons. ) D From an imbalance of yin and yang ) 1 A nurse is developing a plan of care for a woman to ensure continuity of care 2 during pregnancy, labor, and childbirth. Which of the following would be most . important for the nurse to incorporate into that plan? A Adhering to strict, specific routines ) B Involving a pediatric physician ) C Educating the client about the importance of a support person ) D Assigning several nurses as a support team ) 1 A nursing instructor is preparing a class discussion on case management in 3 maternal and newborn health care. Which of the following would the instructor . include as a key component? Select all that apply. A Advocacy ) B Coordination ) C Communication ) D Resource management ) E Event managed care ) 1 After teaching a group of students about the concept of maternal mortality, the 4 instructor determines that additional teaching is needed when the students state . which of the following? A The rate includes accidental causes for deaths. ) B It addresses pregnancy-related causes. ) C The duration of the pregnancy is not a concern. ) D The time frame is typically for a specified year. ) 1 A group of students are reviewing the historical aspects about childbirth. The 5 students demonstrate understanding of the information when they identify the . use of twilight sleep as a key event during which time frame? A 1700s ) B 1800s ) C 1900s ) D 2000s ) 1 A nurse is providing care to a woman who has just delivered a healthy newborn. 6 Which action would least likely demonstrate application of the concept of . family-centered care? A Focusing on the birth as a normal healthy event for the family ) B Creating opportunities for the family to make informed decisions ) C Encouraging the woman to keep her other children at home ) D Fostering a sense of respect for the mother and the family ) 1 When discussing fetal mortality with a group of students, a nurse addresses 7 maternal factors. Which of the following would the nurse most likely include? . Select all that apply. A Chromosomal abnormalities ) B Malnutrition ) C Preterm cervical dilation ) D Underlying disease condition ) E Poor placental attachment ) 1 8 . A nurse is preparing a presentation for a local community group about health status and childrens health. Which of the following would the nurse include as one of the most significant measures? A ) Fetal mortality rate B ) Neonatal mortality rate C ) Infant mortality rate D ) Maternal mortality rate 1 9 . A group of students are reviewing an article describing information related to indicators for womens health and the results of a national study. Which of the following would the students identify as being satisfactory for women? Select all that apply. A ) Smoking cessation B ) Colorectal cancer screening C ) Violence against women D ) Health insurance coverage E ) Mammograms 2 0 . A nurse is preparing a presentation for a local womens group about heart disease and women. Which of the following would the nurse expect to address when discussing measures to promote health. A ) Women have similar symptoms as men for a heart attack. B ) Heart disease is no longer viewed as a mans disease. C ) Women experiencing a heart attack are at greater risk for dying. D ) Heart attacks in women are more easily diagnosed. 2 1 . A nurse is working to develop a health education program for a local community to address breast cancer awareness. Which of the following would the nurse expect to include when describing this problem to the group? Select all that apply. A ) White women have higher rates of breast cancer than African American women. B ) African American women are more likely to die from breast cancer at any age. C ) Survival at any stage is worse among white women. D Women living in South America have the highest rates of breast cancer. ) E Breast cancer is the leading cause of cancer mortality in women. ) 2 2 . A group of nursing students are reviewing information about factors affecting maternal, newborn, and womens health. The students demonstrate understanding of the information when they identify which of the following deficiencies as being associated with poverty? Select all that apply. A ) Literacy B ) Employment opportunities C ) Mobility D ) Political representation E ) Skills Answer Key 1. B 2. C 3. D 4. B 5. B 6. D 7. A 8. A 9. A 10. D 11. D 12. C 13. A, B, C, D 14. A 15. C 16. C 17. B, C, D 18. C 19. B, E 20. C 21. A, B 22. A, B, C, D, E Chapter 2 Family-Centered Community-Based Care 1 . The nurse is caring for a 2-week-old newborn girl with a metabolic disorder. Which of the following activities would deviate from the characteristics of family-centered care? A ) Softening unpleasant information or prognoses B ) Evaluating and changing the nursing plan of care C Collaborating with the child and family as equals ) D Showing respect for the familys beliefs and wishes ) 2 . The nurse is providing home care for a 6-year-old girl with multiple medical challenges. Which of the following activities would be considered the tertiary level of prevention? A ) Arranging for a physical therapy session B ) Teaching parents to administer albuterol C ) Reminding parent to give a full course of antibiotics D ) Giving a DTaP vaccination at the proper interval 3 . A nursing student is reviewing information about documenting client care and education in the medical record and the purposes that it serves. The student demonstrates a need for additional study when the nurse identifies which of the following as a reason? A ) Serves as a communication tool for the interdisciplinary team. B ) Demonstrates education the family has received if legal matters arise. C ) Permits others access to allow refusal of medical insurance coverage. D ) Verifies meeting client education standards set by the Joint Commission. 4 . A pregnant client tells her nurse that she is interested in arranging a home birth. After educating the client on the advantages and disadvantages, which statement would indicate that the client understood the information? A ) I like having the privacy, but it might be too expensive for me to set up in my home. B ) I want to have more control, but I am concerned if an emergency would arise. C ) It is safer because I will have a midwife. D ) The midwife is trained to resolve any emergency, and she can bring any pain meds. 5 . The nurse is making a home visit to a client who had a cesarean birth 3 days ago. Assessment reveals that the client is complaining of intermittent pain, rating it as 8 on a scale of 1 to 10. She states, Im pretty tired. And with this pain, I havent been drinking and eating like I should. The medication helps a bit but not much. My mom has been helping with the baby. Her incision is clean, dry, and intact. Which nursing diagnosis would the nurse identify as the priority for this client? A ) Impaired skin integrity related to cesarean birth incision B ) Fatigue related to effects of surgery and caretaking activities C ) Imbalanced nutrition, less than body requirements related to poor fluid and food intake D ) Acute pain related to incision and cesarean birth 6 When caring for childbearing families from cultures different from ones own, . which of the following must be accomplished first? A Adapt to the practices of the familys culture ) B Determine similarities between both cultures ) C Assess personal feelings about that culture ) D Learn as much as possible about that culture ) 7 After teaching a group of students about the changes in health care delivery and . funding, which of the following, if identified by the group as a current trend seen in the maternal and child health care settings, would indicate that the teaching was successful? A Increase in community settings for care ) B Decrease in family poverty level ) C Increase in hospitalization of children ) D Decrease in managed care ) 8 . The nurse would recommend the use of which supplement as a primary prevention strategy to prevent neural tube defects with pregnant women? A ) Calcium B ) Folic acid C Vitamin C ) D Iron ) 9 . Which action would the nurse include in a primary prevention program in the community to help reduce the incidence of HIV infection? A ) Provide treatment for clients who test positive for HIV B ) Monitor viral load counts periodically C ) Educate clients in how to practice safe sex D ) Offer testing for clients who practice unsafe sex 1 0 . When assuming the role of discharge planner for a woman requiring ventilator support at home, the nurse would do which of the following? A ) Confer with the clients mother B ) Teach new self-care skills to the client C ) Determine if there is a need for back-up power D ) Discuss coverage with the insurance company 1 1 . When comparing community-based nursing with nursing in the acute care setting to a group of nursing students, the nurse describes the challenges associated with community-based nursing. Which of the following would the nurse include? A ) Increased time available for education B ) Improved access to resources C ) Decision making in isolation D ) Greater environmental structure 1 2 . After teaching a group of students about the different levels of prevention, the instructor determines a need for additional teaching when the students identify which of the following as a secondary prevention level activity in communitybased health care? A ) Teaching women to take folic acid supplements to prevent neural tube defects B ) Working with women who are victims of domestic violence C ) Working with clients at an HIV clinic to provide nutritional and CAM therapies D ) Teaching hypertensive clients to monitor blood pressure 1 A nursing instructor is describing trends in maternal and newborn health care. 3 The instructor addresses the length of stay for vaginal births during the past . decade, citing that which of the following denotes the average stay? A 2448 hours or less ) B 7296 hours or less ) C 4872 hours or less ) D 96120 hours or less ) 1 4 . Which of the following statements is accurate regarding womens health care in todays system? A ) Women spend 95 cents of every dollar spent on health care. B ) Women make almost 90% of all health care decisions. C ) Women are still the minority in the United States. D ) Men use more health services than women. 1 5 . A nurse is educating a client about a care plan. Which of the following statements would be appropriate to assess the clients learning ability? A ) Did you graduate from high school; how many years of schooling did you have? B ) Do you have someone in your family who would understand this information? C Many people have trouble remembering information; is this a problem for you? ) D Would you prefer that the doctor give you more detailed medical information? ) 1 6 . A nurse is developing cultural competence. Which of the following indicates that the nurse is in the process of developing cultural knowledge? Select all that apply. A ) Examining personal sociocultural heritage B ) Reviewing personal biases and prejudices C ) Seeking resources to further understanding of other cultures D ) Becoming familiar with other culturally diverse lifestyles E ) Performing a competent cultural assessment F ) Advocating for social justice to eliminate disparities. 1 7 . A nurse is engaged in providing family-centered care for a woman and her family. The nurse is providing instrumental support with which activity? A ) Explaining to the woman and family what to expect during the birth process. B ) Assisting the woman in breathing techniques to cope with labor contractions. C Reinforcing the womans role as a mother after birth ) D Helping the family obtain extra financial help for prescribed phototherapy ) 1 8 . A nurse is considering a change in employment from the acute care setting to community-based nursing. The nurse is focusing her job search on ambulatory care settings. Which of the following would the nurse most likely find as a possible setting? Select all that apply. A ) Urgent care center B ) Hospice care C ) Immunization clinic D ) Physicians office E ) Day surgery center F ) Nursing home 1 9 . A nursing instructor is presenting a class for a group of students about community-based nursing interventions. The instructor determines that additional teaching is needed when the students identify which of the following? A ) Conducting childbirth education classes B ) Counseling a pregnant teen with anemia C ) Consulting with a parent of a child who is vomiting D ) Performing epidemiologic investigations 2 0 . During class, a nursing student asks, I read an article that was talking about integrative medicine. What is that? Which response by the instructor would be most appropriate? A ) It refers to the use of complementary and alternative medicine in place of traditional therapies for a condition. B ) It means that complementary and alternative medicine is used together with conventional therapies to reduce pain or discomfort. C ) It means that mainstream medical therapies and complementary and alternative therapies are combined based on scientific evidence for being effective. D ) It refers to situations when a client and his or her family prefer to use an unproven method of treatment over a proven one. 2 1 . While a nurse is obtaining a health history, the client tells the nurse that she practices aromatherapy. The nurse interprets this as which of the following? A ) Use of essential oils to stimulate the sense of smell to balance the mind and body B ) Application of pressure to specific points to allow self-healing C ) Use of deep massage of areas on the foot or hand to rebalance body parts D ) Participation in chanting and praying to promote healing. 2 2 . A pregnant woman asks the nurse about giving birth in a birthing center. She says, Im thinking about using one but Im not sure. Which of the following would the nurse need to integrate into the explanation about this birth setting? (Select all that apply.) A ) An alternative for women who are uncomfortable with a home birth. B ) The longer length of stay needed when compared to hospital births C ) Focus on supporting women through labor instead of managing labor D ) View of labor and birth as a normal process requiring no intervention E ) Care provided primarily by obstetricians with midwives as backup care 2 3 . A nurse practicing in the community is preparing a presentation for a group of nursing students about this practice setting. Which of the following would the nurse include as characteristic of this role? A ) Greater emphasis on direct physical care B ) Broader assessment to include the environment C ) Increased dependency on physician D ) Limited decision making and support 2 A nurse is preparing a teaching plan for a woman who is pregnant for the first 4 time. Which of the following would the nurse incorporate into the teaching plan . to foster the clients learning? (Select all that apply.) A Teach survival skills first ) B Use simple, nonmedical language ) C Refrain from using a hands-on approach ) D Avoid repeating information ) E Use visual materials such as photos and videos ) 2 A group of nurses are reviewing the steps for developing cultural competence. 5 The students demonstrate understanding when they identify which of the . following as the final step? A Cultural knowledge ) B Cultural skills ) C Cultural encounter ) D Cultural awareness ) Answer Key 1. A 2. A 3. C 4. B 5. D 6. C 7. A 8. B 9. C 10. C 11. C 12. A 13. A 14. B 15. C 16. C, D 17. D 18. A, D, E 19. D 20. C 21. A 22. A, C, D 23. B 24. A, B, E 25. C Chapter 3 Anatomy and Physiology of the Reproductive System 1 . When describing the menstrual cycle to a group of young women, the nurse explains that estrogen levels are highest during which phase of the endometrial cycle? A ) Menstrual B ) Proliferative C ) Secretory D ) Ischemic 2 . After teaching a group of adolescent girls about female reproductive development, the nurse determines that teaching was successful when the girls state that menarche is defined as a womans first: A ) Sexual experience B ) Full hormonal cycle C ) Menstrual period D ) Sign of breast development 3 . A client with a 28-day cycle reports that would expect the clients next menses to she ovulated begin on: on May 10. The nurse A ) May 24 B May 26 ) C May 30 ) D June 1 ) 4 . Which female reproductive tract structure would the nurse describe to a group of young women as containing rugae that enable it to dilate during labor and birth? A ) Cervix B ) Fallopian tube C ) Vagina D ) Vulva 5 . After teaching a group of pregnant women about breast-feeding, the nurse determines that the teaching was successful when the group identifies which hormone as important for the production of breast milk after childbirth? A ) Placental estrogen B ) Progesterone C ) Gonadotropin-releasing hormone D ) Prolactin 6 . The nurse is assessing a 13-year-old girl who has had her first menses. Which of the following events would the nurse expect to have occurred first? A ) Evidence of pubic hair B ) Development of breast buds C ) Onset of menses D ) Growth spurt 7 . When describing the ovarian cycle to a group of students, which phase would the instructor include? A ) Luteal phase B ) Proliferative phase C ) Menstrual phase D ) Secretory phase 8 . The nurse is explaining the events that lead up to ovulation. Which hormone would the nurse identify as being primarily responsible for ovulation? A ) Estrogen B ) Progesterone C Follicle-stimulating hormone ) D Luteinizing hormone ) 9 . The nurse is teaching a health education class on male reproductive anatomy and asks the students to identify the site of sperm production. Which structure, if identified by the group, would indicate to the nurse that the teaching was successful? A ) Testes B ) Seminal vesicles C ) Scrotum D ) Prostate gland 1 0 . The nurse is creating a diagram that illustrates the components of the male reproductive system. Which structure would be inappropriate for the nurse to include as an accessory gland? A ) Seminal vesicles B ) Prostate gland C ) Cowpers glands D ) Vas deferens 1 1 . The nurse is preparing an outline for a class on the physiology of the male sexual response. Which event would the nurse identify as occurring first? A ) Sperm emission B ) Penile vasodilation C ) Psychological release D ) Ejaculation 1 2 . A woman comes to the clinic complaining that she has little sexual desire. As part of the clients evaluation, the nurse would anticipate the need to evaluate which hormone level? A ) Progesterone B ) Estrogen C ) Gonadotropin-releasing hormone D ) Testosterone 1 3 . A nurse is con reproductive include as an ducting a class for a group of teenage girls about female anatomy and physiology. Which of the following would the nurse external female reproductive organ? Select all that apply. A ) Mons pubis B Labia ) C Vagina ) D Clitoris ) E Uterus ) 1 4 . When describing the hormones involved in the menstrual cycle, a nurse identifies which hormone as responsible for initiating the cycle? A ) Estrogen B ) Luteinizing hormone C ) Progesterone D ) Prolactin 1 5 . A nursing instructor is describing the hormones involved in the menstrual cycle to a group of nursing students. The instructor determines the teaching was successful when the students identify follicle-stimulating hormone as being secreted by which of the following? A ) Hypothalamus B ) Anterior pituitary gland C Ovaries ) D Corpus luteum ) 1 6 . A woman comes to the clinic for an evaluation. During the visit, the woman tells the nurse that her menstrual cycles have become irregular. Ive also been waking up at night feeling really hot and sweating. The nurse interprets these findings as which of the following? A ) Menopause B ) Perimenopause C ) Climacteric D ) Menarche 1 7 . After teaching a group of students about female reproductive anatomy, the instructor determines that the teaching was successful when the students identify which of the following as the site of fertilization? A ) Vagina B ) Uterus C ) Fallopian tubes D ) Vestibule 1 8 . A woman comes to the clinic complaining of a vaginal discharge. The nurse suspects that the client has an infection. When gathering additional information, which of the following would the nurse be least likely to identify as placing the client at risk for an infection? A ) Recent antibiotic therapy for an upper respiratory infection B ) Last menstrual period about 5 days ago. C ) Weekly douching D ) Frequent use of feminine hygiene sprays. 1 9 . A group of nursing students are reviewing information about the male reproductive structures. The students demonstrate understanding of the information when they identify which of the following as accessory organs? (Select all that apply.) A ) Testes B ) Vas deferens C ) Bulbourethral glands D ) Prostate gland E ) Penis 2 0 . A nurse is examining a female client and tests finding would the nurse interpret as normal? the clients vaginal pH. Which A ) 4.5 B ) 7 C ) 8.5 D ) 10 2 1 . When describing the male sexual response to a group of students, the instructor determines that the teaching was successful when they identify emission as which of the following? A ) Semen forced through the urethra to the outside B ) Movement of sperm from the testes and fluid into the urethras C ) Dilation of the penile arteries with increased blood flow to the tissues. D ) Bodys return to the physiologic nonstimulated state 2 2 . A nurse is de adolescent he as which of scribing the structure and function of the reproductive system to an alth class. The nurse describes the secretion of the seminal vesicles the following? A ) Mucus-like B ) Alkaline C ) Acidic D ) Semen Answer Key 1. B 2. C 3. A 4. C 5. D 6. B 7. A 8. D 9. A 10. D 11. B 12. D 13. A, B, D 14. B 15. B 16. B 17. C 18. B 19. B, C, D 20. A 21. B 22. B Chapter 4 Common Reproductive Issues 1 . After discussing various methods of contraception with a client and her partner, the nurse determines that the teaching was successful when they identify which contraceptive method as providing protection against sexually transmitted infections (STIs)? A ) Oral contraceptives B ) Tubal ligation C ) Condoms D ) Intrauterine system 2 . When discussing contraceptive options, which method would the nurse recommend as being the most reliable? A ) Coitus interruptus B ) Lactational amenorrheal method (LAM) C Natural family planning ) D Intrauterine system ) 3 . A client comes to the clinic with abdominal pain. Based on her history the nurse suspects endometriosis. The nurse expects to prepare the client for which of the following to confirm this suspicion? A ) Pelvic examination B ) Transvaginal ultrasound C ) Laparoscopy D ) Hysterosalpingogram 4 . A client is to receive an implantable contraceptive. The nurse describes this contraceptive as containing: A ) Synthetic progestin B ) Combined estrogen and progestin C ) Concentrated spermicide D ) Concentrated estrogen 5 . The nurse discusses various contraceptive methods with a client and her partner. Which method would the nurse explain as being available only with a prescription? A ) Condom B ) Spermicide C ) Diaphragm D ) Basal body temperature 6 . When developing a teaching plan for a couple considering contraception options, which of the following statements would the nurse include? A ) You should select one that is considered to be 100% effective. B ) The best one is the one that is the least expensive and most convenient. C ) A good contraceptive doesnt require a physicians prescription. D ) The best contraceptive is one that you will use correctly and consistently. 7 . Which of the following measures would the nurse include in the teaching plan for a woman to reduce the risk of osteoporosis after menopause? A ) Taking vitamin supplements B ) Eating high-fiber, high-calorie foods C Restricting fluid to 1,000 mL daily ) D Participating in regular daily exercise ) 8 . When teaching a group of postmenopausal women about hot flashes and night sweats, the nurse would address which of the following as the primary cause? A ) Poor dietary intake B ) Estrogen deficiency C ) Active lifestyle D ) Changes in vaginal pH 9 . A client states that she is to have a test to measure osteoporosis. The nurse would most likely plan to bone mass prepare the to help diagnose client for: A ) DEXA scan B ) Ultrasound C ) MRI D ) Pelvic x-ray 1 0 . The nurse is reviewing the medical records of several clients. Which client would the nurse expect to have an increased risk for developing osteoporosis? A ) A woman of African American descent B ) A woman who plays tennis twice a week C ) A thin woman with small bones D ) A woman who drinks one cup of coffee a day 1 1 . Which of the following would the nurse emphasize when teaching postmenopausal women about ways to reduce the risk of osteoporosis? A ) Swimming daily B ) Taking vitamin A C ) Following a low-fat diet D ) Taking calcium supplements 1 2 . Which finding would the nurse expect to find in a client with endometriosis? A ) Hot flashes B Dysuria ) C Fluid retention ) D Fever ) 1 3 . After the nurse teaches a client about ways to reduce the symptoms of premenstrual syndrome, which client statement indicates a need for additional teaching? A ) I will make sure to take my estrogen supplements a week before my period. B ) Ive signed up for an aerobic exercise class three times a week. C ) Ill cut down on the amount of coffee and colas I drink. D ) I quit smoking about a month ago, so that should help. 1 4 . A woman has opted to use the basal body temperature method for contraception. The nurse instructs the client that a rise in basal body temperature indicates which of the following? A ) Onset of menses B ) Ovulation C ) Pregnancy D Safe period for intercourse ) 1 5 . A woman using the cervical mucus ovulation method of fertility awareness reports that her cervical mucus looks like egg whites. The nurse interprets this as which of the following? A ) Spinnbarkeit mucus B ) Purulent mucus C ) Postovulatory mucus D ) Normal preovulation mucus 1 6 . The nurse is reviewing the laboratory test results of a client with dysfunctional uterine bleeding (DUB). Which finding would be of concern? A ) Negative pregnancy test B ) Hemoglobin level of 10.1 g/dL C ) Prothrombin time of 60 seconds D ) Serum cholesterol of 140 mg/dL 1 7 . A nurse is preparing a class for a group of women at a family planning clinic about contraceptives. When describing the health benefits of oral contraceptives, which of the following would the nurse most likely include? (Select all that apply.) A ) Protection against pelvic inflammatory disease B ) Reduced risk for endometrial cancer C ) Decreased risk for depression D ) Reduced risk for migraine headaches E ) Improvement in acne 1 8 . After teaching a group of students about the different methods for contraception, the instructor determines that the teaching was successful when the students identify which of the following as a mechanical barrier method? (Select all that apply.) A ) Condom B ) Cervical cap C ) Cervical sponge D ) Diaphragm E ) Vaginal ring 1 9 . After assessing a woman who has come to the clinic, the nurse suspects that the woman is experiencing dysfunctional uterine bleeding. Which statement by the client would support the nurses suspicions? A ) Ive been having bleeding off and on thats irregular and sometimes heavy. B ) I get sharp pain in my lower abdomen usually starting soon after my period comes. C ) I get really irritable and moody about a week before my period. D ) My periods have been unusually long and heavy lately. 2 0 . After teaching a group of students about premenstrual syndrome, the instructor determines that additional teaching is needed when the students identify which of the following as a prominent assessment finding? A ) Bloating B ) Tension C ) Dysphoria D ) Weight loss 2 1 . A nurse is describing the criteria dysphoric disorder (PMDD). Wh a mandatory requirement for the needed for the diagnosis of premenstrual ich of the following would the nurse include as diagnosis? A ) Appetite changes B ) Sleep difficulties C ) Persistent anger D ) Chronic fatigue 2 2 . When reviewing the medical record of a client diagnosed with endometriosis, which of the following would the nurse identify as a risk factor for this woman? A ) Low fat in the diet B ) Age of 14 years for menarche C ) Menstrual cycles of 24 days D ) Short menstrual flow 2 3 . A client who has come to the clinic is diagnosed with endometriosis. Which of the following would the nurse expect the physician to prescribe as a first-line treatment? A ) Progestins B ) Antiestrogens C ) Gonadotropin-releasing hormone analogues D NSAIDs ) 2 4 . A woman comes to the clinic because she has been unable to conceive. When reviewing the womans history, which of the following would the nurse least likely identify as a possible risk factor? A ) Age of 25 years B ) History of smoking C ) Diabetes since age 15 years D ) Weight below standard for height and age 2 5 . A couple comes to the clinic for a fertility evaluation. The male partner is to undergo a semen analysis. After teaching the partner about this test, which client statement indicates that the client has understood the instructions? A ) I need to bring the specimen to the lab the day after collecting it. B ) I will place the specimen in a special plastic bag to transport it. C ) I have to abstain from sexual activity for about 12 days before the sample. D ) I will withdraw before I ejaculate during sex to collect the specimen. 2 A nurse is preparing a class for a group of young adult women about emergency 6 contraceptives (ECs). Which of the following would the nurse need to stress to . the group. Select all that apply. A ) ECs induce an abortion like reaction. B ) ECs provide some protection against STIs C ) ECs are birth control pills in higher, more frequent doses D ) ECs are not to be used in place of regular birth control E ) ECs provide little protection for future pregnancies. Answer Key 1. C 2. D 3. C 4. A 5. C 6. D 7. D 8. B 9. A 10. C 11. D 12. B 13. A 14. B 15. A 16. B 17. A, B, E 18. A, B, C, D 19. A 20. D 21. C 22. C 23. D 24. A 25. C 26. C, D, E Chapter 5 Sexually Transmitted Infections 1 . The nurse is developing a plan of care for a client who is receiving highly active antiretroviral therapy (HAART) for treatment of HIV. The goal of this therapy is to: A ) Promote the progression of disease B ) Intervene in late-stage AIDS C ) Improve survival rates D ) Conduct additional drug research 2 . A woman who is HIV-positive is receiving HAART and is having difficulty with compliance. To promote adherence, which of the following areas would be most important to assess initially? A ) The womans beliefs and education B ) The womans financial situation and insurance C ) The womans activity level and nutrition D ) The womans family and living arrangements 3 . When developing a teaching plan for a community group about HIV infection, which group would the nurse identify as an emerging risk group for HIV infection? A ) Native Americans B ) Heterosexual women C ) New health care workers D ) Asian immigrants 4 After teaching a group of adolescents about HIV, the nurse asks them to identify . the major means by which adolescents are exposed to the virus. The nurse determines that the teaching was successful when the group identifies which of the following? A Sexual intercourse ) B ) Sharing needles for IV drug use C ) Perinatal transmission D ) Blood transfusion 5 The nurse reviews the CD4 cell count of a client who is HIV-positive. A result . less than which of the following would indicate to the nurse that the client has AIDS? A 1,000 cells/mm3 ) B 700 cells/mm3 ) C 450 cells/mm3 ) D 200 cells/mm3 ) 6 When obtaining the health history from a client, which factor would lead the . nurse to suspect that the client has an increased risk for sexually transmitted infections (STIs)? A Hive-like rash for the past 2 days ) B Five different sexual partners ) C Weight gain of 5 lbs in 1 year ) D Clear vaginal discharge ) 7 Assessment of a female client reveals a thick, white vaginal discharge. She also . reports intense itching and dyspareunia. Based on these findings, the nurse would suspect that the client has: A Trichomoniasis ) B Bacterial vaginosis ) C Candidiasis ) D Genital herpes simplex ) 8 A client with trichomoniasis is to receive metronidazole (Flagyl). The nurse . instructs the client to avoid which of the following while taking this drug? A Alcohol ) B Nicotine ) C Chocolate ) D Caffeine ) 9 A woman gives birth to a healthy newborn. As part of the newborns care, the . nurse instills erythromycin ophthalmic ointment as a preventive measure related to which STI? A Genital herpes ) B Hepatitis B ) C Syphilis ) D Gonorrhea ) 1 0 . Which findings would the nurse expect to find in a client with bacterial vaginosis? A ) Vaginal pH of 3 B ) Fish-like odor of discharge C ) Yellowish-green discharge D ) Cervical bleeding on contact 1 1 . A pregnant woman diagnosed with syphilis comes to the clinic for a visit. The nurse discusses the risk of transmitting the infection to her newborn, explaining that this infection is transmitted to the newborn through the: A ) Amniotic fluid B ) Placenta C Birth canal ) D Breast milk ) 1 2 . The nurse encourages a female client with human papillomavirus (HPV) to receive continued follow-up care because she is at risk for: A ) Infertility B ) Dyspareunia C ) Cervical cancer D ) Dysmenorrhea 1 3 . A client is diagnosed with pelvic inflammatory disease (PID). When reviewing the clients medical record, which of the following would the nurse expect to find? (Select all that apply.) A ) Oral temperature of 100.4 degrees F B ) Dysmenorrhea C ) Dysuria D ) Lower abdominal tenderness E ) Discomfort with cervical motion F ) Multiparity 1 4 . Which instructions would the nurse include when teaching a woman with pediculosis pubis? A ) Take the antibiotic until you feel better. B ) Wash your bed linens in bleach and cold water. C ) Your partner doesnt need treatment at this time. D ) Remove the nits with a fine-toothed comb. 1 5 . A client with genital herpes simplex infection asks the nurse, Will I ever be cured of this infection? Which response by the nurse would be most appropriate? A ) There is a new vaccine available that prevents the infection from returning. B ) All you need is a dose of penicillin and the infection will be gone. C ) There is no cure, but drug therapy helps to reduce symptoms and recurrences. D ) Once you have the infection, you develop an immunity to it. 1 6 . A nurse is preparing a presentation for a group of women at the clinic who have been diagnosed with genital herpes. Which of the following would the nurse expect to include as a possible precipitating factor for a recurrent outbreak? (Select all that apply.) A ) Exposure to ultraviolet light B ) Exercise C ) Use of corticosteroids D ) Emotional stress E ) Sexual intercourse. 1 7 . After teaching a class on sexually transmitted infections, the instructor determines that the teaching was successful when the class identifies which statement as true? A ) STIs can affect anyone if exposed to the infectious organism. B ) STIs have been addressed more on a global scale. C ) Clients readily view the diagnosis of STI openly. D ) Most individuals with STIs are over the age of 30. 1 A group of students are reviewing information about STIs. The students 8 demonstrate understanding of the information when they identify which of the . following as the most common bacterial STI in the United States? A Gonorrhea ) B Chlamydia ) C Syphilis ) D Candidiasis ) 1 A nurse is assessing a client for possible risk factors for chlamydia and 9 gonorrhea. Which of the following would the nurse identify? . A Asian American ethnicity ) B Age under 25 years ) C Married ) D Consistent use of barrier contraception ) 2 0 . A nurse at a local community clinic is developing a program to address STI prevention. Which of the following would the nurse least likely include in the program? A ) Outlining safer sexual behavior B ) Recommending screening for symptomatic individuals C Promoting the use of barrier contraceptives ) D Offering education about STI transmission ) 2 1 . After teaching a class on preventing pelvic inflammatory disease, the instructor determines that the teaching was successful when the class identifies which of the following as an effective method? A ) Advising sexually active females to use hormonal contraception B ) Encouraging vaginal douching on a weekly basis. C ) Emphasizing the need for infected sexual partners to receive treatment D ) Promoting routine treatment for asymptomatic females as risk 2 2 . A group of nursing students are reviewing information about vaccines used to prevent STIs. The students would expect to find information about which of the following? A ) HIV B ) HSV C ) HPV D ) HAV E HBV ) 2 3 . A mother brings her 12-year-old daughter in for well-visit checkup. During the visit, the nurse is discussing the use of prophylactic HPV vaccine for the daughter. The mother agrees and the daughter receives her first dose. The nurse schedules the daughter for the next dose, which would be given at which time? A ) In 2 month B ) In 2 months C ) In 3 months D ) In 4 months 2 4 . A woman comes to the clinic complaining of a vaginal discharge. The nurse suspects trichomoniasis based on which of the following? (Select all that apply.) A ) Urinary frequency B ) Yellow/green discharge C ) Joint pain D ) Blister-like lesions E ) Muscle aches 2 5 . A nurse is teaching a women with genital ulcers how to care for them. Which statement by the client indicates a need for additional teaching? A ) I need to wash my hands after touching any of the ulcers. B ) I need to abstain from intercourse primarily when the lesions are present. C ) I should avoid applying ice or heat to my genital area. D ) I can try lukewarm sitz baths to help ease the discomfort. Answer Key 1. C 2. A 3. B 4. A 5. D 6. B 7. C 8. A 9. D 10. B 11. B 12. C 13. B, C, D, E 14. D 15. C 16. A, C, D, E 17. A 18. B 19. B 20. B 21. C 22. C, D, E 23. B 24. A, B 25. B Chapter 6 Disorders of the Breasts 1 . The nurse is developing the discharge plan for a woman who has had a left-sided modified radical mastectomy. The nurse is including instructions for ways to minimize lymphedema. Which suggestion would most likely increase the womans symptoms? A ) Wear gloves when you are doing any gardening. B ) Have your blood pressure taken in your right arm. C ) Wear clothing with elasticized sleeves. D Avoid driving to and from work every day. ) 2 . A laboratory technician arrives to draw blood for a complete blood count (CBC. for a client who had a right-sided mastectomy 8 hours ago. The client has an intravenous line with fluid infusing in her left antecubital space. To obtain the blood specimen, the technician places a tourniquet on the clients right arm. Which action by the nurse would be most appropriate? A ) Assist in holding the clients arm still. B ) Suggest a finger stick be done on one of the clients left fingers. C ) Tell the technician to obtain the blood sample from the clients left arm. D ) Call the surgeon to perform a femoral puncture. 3 . The nurse determines that a woman has implemented prescribed therapy for her fibrocystic breast disease when the client reports that she has eliminated what from her diet? A ) Caffeine B ) Cigarettes C ) Dairy products D ) Sweets 4 . When assessing a client with suspected breast cancer, which of the following would the nurse expect to find? A ) Painful lump B ) Absence of dimpling C ) Regularly shaped mass D ) Nipple retraction 5 . A woman who has undergone a right modified-radical mastectomy returns from surgery. Which nursing intervention would be most appropriate at this time? A ) Ask the client how she feels about having her breast removed. B ) Attach a sign above her bed to have BP, IV lines, and lab work in her right arm. C ) Encourage her to turn, cough, and deep breathe at frequent intervals. D ) Position her right arm below heart level. 6 . A breast biopsy indicates the presence of malignant cells, and the client is scheduled for a mastectomy. Which nursing diagnosis would the nurse most likely include in the clients preoperative plan of care as the priority? A ) Risk for deficient fluid volume B ) Activity intolerance C Disturbed body image ) D Impaired urinary elimination ) 7 . A 42-year-old woman is scheduled for a mammogram. Which of the following would the nurse include when teaching the woman about the procedure? A ) The room will be darkened throughout the procedure. B ) Each breast will be firmly compressed between two plates. C ) Make sure to refrain from eating or drinking after midnight. D ) A small needle will be inserted to get a sample for evaluation. 8 . During a clinical breast examination, the nurse palpates a well-defined, firm, mobile lump in a 60-year-old womans left breast. The nurse notifies the physician. Which of the following would the nurse anticipate the physician to order next? A ) Mammogram B ) Hormone receptor status C ) Fine-needle aspiration D ) Genetic testing for BRCA 9 . A client with advanced breast cancer, who has had both chemotherapy and radiation therapy, is to start hormonal therapy. Which agent would the nurse expect the client to receive? A ) Progestins B ) Tamoxifen C ) Cortisone D ) Estrogen 1 0 . As part of discharge planning, the nurse refers a woman to Reach to Recovery. This groups primary purpose is to: A ) Help support women who have undergone mastectomies B ) Raise funds to support early breast cancer detection programs C ) Provide all supplies needed after breast surgery for no cost D ) Collect statistics for research for the American Cancer Society 1 A woman with breast cancer is undergoing chemotherapy. Which of the 1 following side effects would the nurse interpret as being most serious? . A Vomiting ) B Hair loss ) C Fatigue ) D Myelosuppression ) 1 A woman comes to the clinic reporting a nipple discharge. On examination, the 2 area below the areola is red and slightly swollen, with tortuous tubular swelling. . The nurse interprets these findings as suggestive of which of the following? A Fibrocystic breast disorder ) B Intraductal papilloma ) C Duct ectasia ) D Fibroadenoma ) 1 3 . When performing a clinical breast examination, which would the nurse do first? A ) Palpate the axillary area. B ) Compress the nipple for a discharge. C ) Palpate the breasts. D Inspect the breasts. ) 1 4 . Evaluation of a woman with breast cancer reveals that her mass is approximately 1.25 inches in diameter. Three adjacent lymph nodes are positive. The nurse interprets this as indicating that the woman has which stage of breast cancer? A ) 0 B ) I C ) II D ) III 1 5 . After teaching a woman how to perform breast self-examination, which statement would indicate that the nurses instructions were successful? A ) I should lie down with my arms at my side when looking at my breasts. B ) I should use the fingerpads of my three middle fingers to apply pressure to my breast. C ) I dont need to check under my arm on that side if my breast feels fine. D ) I need to work from the center of my breast outward toward my shoulder. 1 6 . A nurse is working with a woman who has been diagnosed with severe fibrocystic breast disease. When describing the medications that can be used as treatment, which of the following would the nurse be least likely to include? A ) Tamoxifen B ) Bromocriptine C ) Danazol D ) Penicillin 1 7 . A group of students are reviewing information about benign and malignant breast masses. The students demonstrate understanding when they identify which of the following as indicating a benign breast mass. (Select all the apply.) A ) Painless B ) Unilateral location C ) Firm consistency D ) Absence of dimpling E ) Fixed to chest wall 1 The nurse is developing a plan of care for a woman with breast cancer who is 8 scheduled to undergo breast-conserving surgery. The nurse interprets this as . which of the following? A Removal of nipple and areolar area ) B Lump removal followed by radiation ) C Entire breast removal without lymph nodes ) D Axillary lymph node removal ) 1 A woman comes to the clinic and asks the nurse about when she should have 9 her first mammogram. Using the recommendations of the American Cancer . Society, which would the nurse suggest? A 30 years ) B 35 years ) C 40 years ) D 45 years ) 2 0 . After teaching a group of nursing students about the different types of chemotherapeutic agents used to treat breast cancer, the instructor determines that the teaching was successful when the students identify which of the following as an example of a selective estrogen receptor modulator (SERM)? (Select all that apply.) A ) Tamoxifen B ) Letozole C Raloxifene ) D Exemestane ) E Anastrozole ) 2 1 . A woman diagnosed with breast cancer is to receive trastuzumab. Which of the following would the nurse incorporate into the explanation about how this drug works? A ) It blocks the effect of the HER-2/neu protein inhibiting the growth of cancer cells. B ) The drug blocks the conversion of androgens to estrogens C ) It interferes with hormone receptors that allow estrogen to enter a cell D ) The drug ultimately attacks areas where micrometastasis has occurred. 2 2 . A nurse is conducting a class on breast cancer prevention. Which statement would the nurse most likely include in the discussion? A ) Most often a lump is felt before it is seen. B ) Early breast cancer usually has some symptoms. C ) If the mass is not painful, it is usually benign. D If lump is palpable, it has been there for some time. ) 2 3 . When describing programs for breast cancer screening, the nurse include breast self-examination (BSE). Which of the following most accurately reflects the current thinking about breast self-examination? A ) BSE is essential for early breast cancer detection. B ) A woman performing BSE has breast awareness. C ) BSE plays a minimal role in detecting breast cancer D ) A clinical breast exam has replaced BSE. 2 4 . During a wellness visit to the clinic, a woman asks the nurse if there is anything she can do to reduce her risk for developing breast cancer. Which of the following would the nurse most likely include? (Select all that apply.) A ) Eating three servings of fruit daily B ) Keeping weight gain under 11 pounds after age 18 C ) Eating at least seven portions of complex carbohydrates daily D ) Limiting the intake of refined sugar products E ) Using salt liberally when cooking 2 5 . A woman comes to the clinic and tells the nurse that she has read an article about certain foods that have anticancer properties and help boost the immune system. The nurse identifies A ) Garlic B ) Soybeans C ) Milk D ) Leeks E ) Flax seeds Answer Key 1. C 2. B 3. A 4. D 5. C 6. C 7. B 8. A 9. B 10. A 11. D 12. C 13. D 14. C 15. B 16. D 17. C, D 18. B 19. C 20. A, C 21. A 22. D 23. B 24. B, C, D 25. A, B, D, E Chapter 7 Benign Disorders of the Female Reproductive Tract 1 . A woman is admitted for repair of cystocele and rectocele. She has nine living children. In taking her health history, which of the following would the nurse expect to find? A ) Sporadic vaginal bleeding accompanied by chronic pelvic pain B ) Heavy leukorrhea with vulvar pruritus C ) Menstrual irregularities and hirsutism on the chin D Stress incontinence with feeling of low abdominal pressure ) 2 . To assist the woman in regaining control of the urinary sphincter for urinary incontinence, the nurse should teach the client to do which of the following? A ) Perform Kegel exercises daily. B ) Void every hour while awake. C ) Limit her intake of fluid. D ) Take a laxative every night. 3 . When developing the plan of care for a woman who has had an abdominal hysterectomy, which of the following would be contraindicated? A ) Ambulating the client B ) Massaging the clients legs C ) Applying elasticized stockings D ) Encouraging range-of-motion exercises 4 Which of the following would the nurse include when teaching women about . preventing pelvic support disorders? A Performing Kegel isometric exercises ) B ) Consuming low-fiber diets C ) Using hormone replacement D ) Voiding every 2 hours 5 . A client is diagnosed with an enterocele. The nurse interprets this condition as: A ) Protrusion of the posterior bladder wall downward through the anterior vaginal wall B ) Sagging of the rectum with pressure exerted against the posterior vaginal wall C ) Bulging of the small intestine through the posterior vaginal wall D ) Descent of the uterus through the pelvic floor into the vagina 6 . A woman is scheduled for an anterior and posterior colporrhaphy as treatment for a cystocele. When the nurse is explaining this treatment to the client, which of the following descriptions would be most appropriate to include? A ) This procedure helps to tighten the vaginal wall in the front and back so that your bladder and urethra are in the proper position. B ) Your uterus will be removed through your vagina, helping to relieve the organ that is putting the pressure on your bladder. C ) This is a series of exercises that you will learn to do so that you can strengthen your bladder muscles. D ) These are plastic devices that your physician will insert into your vagina to provide support to the uterus and keep it in the proper position. 7 . The nurse would be least likely to find which of the following in a client with uterine fibroids? A ) Regularly shaped, shrunken uterus B ) Acute pelvic pain C ) Menorrhagia D ) Complaints of bloating 8 . A client with polycystic ovarian syndrome (PCOS. is receiving oral contraceptives as part of her treatment plan. The nurse understands that the rationale for this therapy is to: A ) Restore menstrual regularity B ) Induce ovulation C ) Improve insulin uptake D ) Alleviate hirsutism 9 . When teaching a woman how to perform Kegel exercises, the nurse explains that these exercises are designed to strengthen which muscles? A ) Gluteus B ) Lower abdominal C Pelvic floor ) D Diaphragmatic ) 1 0 . A postmenopausal woman with uterine prolapse is being fitted with a pessary. The nurse would be most alert for which side effect? A ) Increased vaginal discharge B ) Urinary tract infection C ) Vaginitis D ) Vaginal ulceration 1 1 . When preparing the discharge teaching plan for the woman who had surgery to correct pelvic organ prolapse, which of the following would the nurse include? A ) Care of the indwelling catheter at home B ) Emphasis on coughing to prevent complications C ) Return to usual activity level in a few days D ) Daily douching with dilute vinegar solution 1 2 . A woman with polycystic ovary syndrome tells the nurse, I hate this disease. Just look at me! I have no hair on the front of my head but Ive got hair on my chin and upper lip. I dont feel like a woman anymore. Further assessment reveals breast atrophy and increased muscle mass. Which nursing diagnosis would most likely be a priority? A ) Situational low self-esteem related to masculinization effects of the disease B ) Social isolation related to feelings about appearance C ) Risk for suicide related to effects of condition and fluctuating hormone levels D ) Ineffective peripheral tissue perfusion related to effects of disease on vasculature 1 3 . After teaching a local womans group about incontinence, the nurse determines that the teaching was successful when the group identifies which of the following as characteristic of stress incontinence? A ) Feeling a strong need to void B ) Passing a large amount of urine C ) Most common in women after childbirth D ) Sneezing may be an initiating stimulus 1 A woman is being evaluated for pelvic organ prolapse. A postvoid residual 4 urine specimen is obtained via a catheter. Which residual volume finding would . lead the nurse to suspect the need for further testing? A 50 mL ) B 75 mL ) C 100 mL ) D 120 mL ) 1 After teaching a woman with pelvic organ prolapse about dietary and lifestyle 5 measures, which of the following statements would indicate the need for . additional teaching? A If I wear a girdle, it will help support the muscles in the area. ) B I should take up jogging to make sure I exercise enough. ) C I will try to drink at least 64 oz of fluid each day. ) D I need to increase the amount of fiber I eat every day. ) 1 6 . After teaching a group of students about genital fistulas, the instructor determines that the teaching was successful when the students identify which of the following as a major cause? A ) Radiation therapy B ) Congenital anomaly C Female genital cutting ) D Bartholins gland abscess ) 1 7 . A nurse is providing care to a female client receiving treatment for a Bartholins cyst. The client has had a small loop of plastic tubing secured in place to allow for drainage. The nurse instructs the client that she will have a follow-up appointment for removal of the plastic tubing at which time? A ) 1 week B ) 2 weeks C ) 3 weeks D ) 4 weeks 1 8 . After undergoing diagnostic testing, a woman is diagnosed with a corpus luteum cyst. The nurse anticipates that the woman will require: A ) Biopsy B ) No treatment C ) Oral contraceptives D ) Glucophage 1 9 . A nurse is teaching a client how to perform Kegel exercises. Which of the following would the nurse include? (Select all that apply.) A ) Squeeze your rectal muscles as if you are trying to avoid passing flatus. B ) Tighten your pubococcygeal muscles for a count of 10. C ) Contract and relax your pubococcygeal muscles rapidly 10 times. D ) Try bearing down for about 10 seconds for no more than 5 times. E ) Do these exercises at least 5 times every hour. 2 0 . After teaching a group of determines that the teachi leiomyomas as which of students about pelvic organ prolapse, the instructor ng was successful when the group identifies the following? A ) Cysts B ) Pelvic organ prolapse C ) Fistula D ) Fibroid 2 A nurse is assessing a female client and suspects that the client may have 1 endometrial polyps based on which of the following? . A Bleeding after intercourse ) B Vaginal discharge ) C Bleeding between menses ) D Metrorrhagia ) 2 After teaching a group of students about ovarian cysts, the instructor determines 2 that the teaching was successful when the students identify which type of cyst as . being associated with hydatiform mole? A Theca-lutein cyst ) B Corpus luteum cyst ) C Follicular cyst ) D Polycystic ovarian syndrome ) 2 3 . A nurse is reading a journal article about care of the woman with pelvic organ prolapse. The nurse would expect to find information related to which of the following? (Select all that apply.) A ) Rectocele B ) Fecal incontinence C ) Cystocele D ) Urinary incontinence E ) Enterocele 2 4 . A nurse is reviewing the medical record of a client. Which of the following would lead the nurse to suspect that the client is experiencing polycystic ovarian syndrome? (Select all that apply.. A ) Decreased androgen levels B ) Elevated blood insulin levels C ) Anovulation D ) Waist circumference of 32 inches E ) Triglyceride level of 175 mg/dL F ) High-density lipoprotein level of 40 mg/dL 2 A group of students are preparing a class presentation about polyps. Which of 5 the following would the students most likely include in the presentation? . A Polyps are rarely the result of an infection. ) B ) Endocervical polyps commonly appear after menarche. C ) Cervical polyps are more common than endocervical polyps. D ) Endocervical polyps are most common in women in their 50s. Answer Key 1. D 2. A 3. B 4. A 5. C 6. A 7. B 8. A 9. C 10. D 11. A 12. A 13. D 14. D 15. B 16. C 17. C 18. B 19. A, C 20. D 21. D 22. A 23. A, C, E 24. B, C, E 25. D Chapter 8 Cancers of the Female Reproductive Tract 1 . The nurse would refer a client, age 54, for follow-up for suspected endometrial carcinoma if she reports which of the following? A ) Use of oral contraceptives between ages 18 and 25 B ) Onset of painless, red postmenopausal bleeding C ) Menopause occurring at age 46 D ) Use of intrauterine device for 3 years 2 Which of the following instructions would the nurse include when preparing a . woman for a Pap smear? A Refrain from sexual intercourse for 1 week before the test. ) B ) Wear cotton panties on the day of the test. C ) Avoid taking any medications for 24 hours. D ) Do not douche for 48 hours before the test. 3 . A woman comes to the clinic for a routine checkup. A history of exposure to which of the following would alert the nurse that she is at increased risk for cervical cancer? A ) Hepatitis B ) Human papillomavirus C ) Cytomegalovirus D ) Epstein-Barr virus 4 . A client is scheduled to have a Pap smear. After the nurse teaches the client about the Pap smear, which of the following client statements indicates successful teaching? A ) I need to douche the night before with a mild vinegar solution. B ) I will take a bath first thing that morning to make sure Im clean. C ) I will not engage in sexual intercourse for 48 hours before the test. D I will get a clean urine specimen when I first wake up the morning of the test. ) 5 . Which finding obtained during a client history would the nurse identify as increasing a clients risk for ovarian cancer? A ) Multiple sexual partners B ) Consumption of a high-fat diet C ) Underweight D ) Grand multiparity (more than five children) 6 . A client is scheduled for cryosurgery to remove some abnormal tissue on the cervix. The nurse teaches the client about this treatment, explaining that the tissue will be removed by which method? A ) Freezing B ) Cutting C ) Burning D ) Irradiating 7 Which of the following statements best indicates that a client has taken self-care . measures to reduce her risk for cervical cancer? A ) Ive really cut down on the amount of caffeine I drink every day. B ) Ive thrown out all my bubble baths and just use soap and water now. C ) Every time I have sexual intercourse, I douche. D ) My partner always uses a condom when we have sexual intercourse. 8 . A client is suspected of having endometrial cancer. The nurse would most likely prepare the client for which procedure to confirm the diagnosis? A ) Transvaginal ultrasound B ) Colposcopy C ) Pap smear D ) Endometrial biopsy 9 . Which of the following descriptions would the nurse include when teaching a client about her scheduled colposcopy? A ) A gel will be applied to your abdomen and a microphone-like device will be moved over the area to identify problem areas. B ) A solution will be wiped on your cervix to identify any abnormal cells, which will be visualized with a magnifying instrument. C ) Scrapings of tissue will be obtained and placed on slides to be examined under the microscope. D After you receive anesthesia, a small device will be inserted into your abdomen ) near your belly button to obtain tissue samples. 1 0 . The nurse is preparing a presentation for a local womens group about ways to reduce the risk of reproductive tract cancers. Which of the following would the nurse include? A ) Blood pressure evaluation every 6 months B ) Yearly Pap smears starting at age 40 C ) Yearly cholesterol screening starting at age 45 D ) Consumption of two to three glasses of red wine per day 1 1 . The daughter of a woman who has been diagnosed with ovarian cancer asks the nurse about screening for this cancer. Which response by the nurse would be most appropriate? A ) Currently there is no reliable screening test for ovarian cancer. B ) A Pap smear is almost always helpful in identifying this type of cancer. C ) Theres a blood test for a marker, CA-125, that if elevated indicates cancer. D ) A genetic test for two genes, if positive, will identify the ovarian cancer. 1 Which of the following would the nurse be least likely to suggest when teaching 2 a group of young women how to reduce their risk for ovarian cancer? . A ) Pregnancy B ) Oral contraceptives C ) Feminine hygiene sprays D ) Breast-feeding 1 3 . A woman is diagnosed with adenocarcinoma of the endometrium in situ. The nurse interprets this as indicating which of the following about the cancer? A ) Spread to the uterine muscle wall B ) Found on the endometrial surface C ) Spread to the cervix D ) Invaded the bladder 1 4 . When preparing a woman with suspected vulvar cancer for a biopsy, the nurse expects that the lesion would most likely be located at which area? A ) Labia majora B ) Labia minora C Clitoris ) D Prepuce ) 1 5 . When describing the various types of reproductive tract cancers to a local womens group, which of the following would the nurse identify as the least common type? A ) Vulvar B ) Vaginal C ) Endometrial D ) Ovarian 1 6 . When assessing a female client for the possibility of vulvar cancer, which of the following would the nurse most likely expect the client to report? (Select all that apply.) A ) Abnormal vaginal bleeding B ) Persistent vulvar itching C ) History of herpes simplex D ) Lesion on the cervix E Abnormal Pap smear ) 1 7 . A nurse is reviewing the medical record of a woman diagnosed with vulvar cancer. Which of the following would the nurse identify as a risk factor for this cancer? (Select all that apply.) A ) Age under 40 years B ) HPV 16 exposure C ) Monogamous sexual partner D ) Hypertension E ) Diabetes 1 8 . A nurse is assisting with the collection of a Pap smear. When collecting the specimen, which of the following is done first? A ) Insertion of the speculum B ) Swabbing of the endocervix C ) Spreading of the labia D ) Insertion of the cytobrush 1 9 . The plan of care for a woman diagnosed with a suspected reproductive cancer includes a nursing diagnosis of disturbed body image related to suspected reproductive tract cancer and impact on sexuality as evidenced by the clients statement that she is worried that she wont be the same. Which of the following would be an appropriate outcome for this client? A ) Client will verbalize positive statements about self and sexuality. B ) Client will demonstrate understanding of the condition and associated treatment. C ) Client will exhibit positive coping strategies related to diagnosis. D ) Client will identify misconceptions related to her diagnosis. 2 0 . During a routine health check-up, a young adult woman asks the nurse about ways to prevent endometrial cancer. Which of the following would the nurse most likely include? (Select all that apply.) A ) Eating a high-fat diet B ) Having regular pelvic exams C ) Engaging in daily exercise D ) Becoming pregnant E ) Using estrogen contraceptives 2 1 . After teaching a group of students about cervical cancer, the instructor determines that the teaching was successful when the students identify which of the following as the area included with a cone biopsy? A ) Clitoris B ) Uterine fundus C ) Ovarian follicle D ) Transformation zone 2 2 . A woman is scheduled for diagnostic testing to evaluate for endometrial cancer. The nurse would expect to prepare the woman for which of the following? A ) CA-125 testing B ) Transvaginal ultrasound C ) Pap smear D ) Mammography 2 3 . A nurse is conducting a class for a local wom for a Pap smear. One of the participants asks her first Pap smear? The nurse responds by her first Pap smear at which age? ans group about recommendations , At what age should a woman have stating that a woman should have A ) 18 B 21 ) C 25 ) D 28 ) 2 4 . An instructor is describing the development of cervical cancer to a group of students. The instructor determines that the teaching was successful when the students identify which area as most commonly involved? A ) Internal cervical os B ) Junction of the cervix and fundus C ) Squamous-columnar junction D ) External cervical os 2 5 . A client has an abnormal Pap smear that is classified as ASC-US. Based on the nurses understanding of this classification, the nurse would expect which of the following? A ) Immediate colposcopy B ) Testing for HPV C ) Repeat Pap smear in 4 to 6 months D Cone biopsy ) Answer Key 1. B 2. D 3. B 4. C 5. B 6. A 7. D 8. D 9. B 10. C 11. A 12. C 13. B 14. A 15. B 16. B, C 17. B, D, E 18. C 19. A 20. B, C, D 21. D 22. B 23. B 24. C 25. C Chapter 9 Violence and Abuse 1 . The nurse is presenting a class at a local community health center on violence during pregnancy. Which of the following would the nurse include as a possible complication? A ) Hypertension of pregnancy B ) Chorioamnionitis C ) Placenta previa D ) Postterm labor 2 . Which approach would be most appropriate when counseling a woman who is a suspected victim of violence? A ) Offer her a pamphlet about the local battered womens shelter. B ) Call her at home to ask her some questions about her marriage. C ) Wait until she comes in a few more times to make a better assessment. D Ask, Have you ever been physically hurt by your partner? ) 3 . When describing an episode, the victim reports that she attempted to calm her partner down to keep things from escalating. This behavior reflects which phase of the cycle of violence? A ) Battering B ) Honeymoon C ) Tension-building D ) Reconciliation 4 . A battered pregnant woman reports to the nurse that her husband has stopped hitting her and promises never to hurt her again. Which of the following is an appropriate response? A ) Thats great. I wish you both the best. B ) The cycle of violence often repeats itself. C ) He probably didnt mean to hurt you. D ) You need to consider leaving him. 5 Which of the following nursing actions would be least helpful for a client who is . a victim of violence? A ) Assist the client to project her anger. B ) Provide information about a safe home and crisis line. C ) Teach her about the cycle of violence. D ) Discuss her legal and personal rights. 6 . When describing the cycle of violence to a community group, the nurse explains that the first phase usually is: A ) Somehow triggered by the victims behavior B ) Characterized by tension-building and minor battery C ) Associated with loss of physical and emotional control D ) Like a honeymoon that lulls the victim 7 . Which of the following statements would be most appropriate to empower victims of violence to take action? A ) Give your partner more time to come around. B ) Rememberchildren do best in two-parent families. C ) Change your behavior so as not to trigger the violence. D You are a good person and you deserve better than this. ) 8 . When a nurse suspects that a client may have been abused, the first action should be to: A ) Ask the client about the injuries and if they are related to abuse. B ) Encourage the client to leave the batterer immediately. C ) Set up an appointment with a domestic violence counselor. D ) Ask the suspected abuser about the victims injuries. 9 . Which of the following would the nurse describe as a characteristic of the second phase of the cycle of violence? A ) The batterer is contrite and attempts to apologize for the behavior. B ) The physical battery is abrupt and unpredictable. C ) Verbal assaults begin to escalate toward the victim. D ) The victim accepts the anger as legitimately directed at her. 1 In addition to providing privacy, which of the following would be most 0 appropriate initially in situations involving suspected abuse? . A ) Allow the client to have a good cry over the situation. B ) Tell the client, Injuries like these dont usually happen by accident. C ) Call the police immediately so they can question the victim. D ) Ask the abuser to describe his side of the story first. 1 1 . When the nurse is alone with a client, the client says, It was all my fault. The house was so messy when he got home and I know he hates that. Which response would be most appropriate? A ) It is not your fault. No one deserves to be hurt. B ) What else did you do to make him so angry with you? C ) You need to start to clean the house early in the day. D ) Remember, he works hard and you need to meet his needs. 1 2 . When developing a presentation for a local violence, the nurse is planning to include st its effects on children. In what percentage abused are the children battered also? community organization on atistics on intimate partner abuse and of the cases in which a parent is A ) 50% to 75% B ) 25% to 50% C 10% to 25% ) D Less than 5% ) 1 3 . The primary goal when working with victims of intimate partner violence is to: A ) Convince them to leave the abuser soon B ) Help them cope with their life as it is C ) Empower them to regain control of their life D ) Arrest the abuser so he or she cant abuse again 1 4 . Teaching for victims who are recovering from abusive situations must focus on ways to: A ) Enhance their personal appearance and hairstyle B ) Develop their creativity and work ethic C ) Improve their communication skills and assertiveness D ) Plan more nutritious meals to improve their own health 1 5 . During a follow-up visit to the clinic, a victim of sexual assault reports that she has changed her job and moved to another town. She tells the nurse, I pretty much stay to myself at work and at home. The nurse interprets these findings to indicate that the client is in which phase of rape recovery? A ) Disorganization B ) Denial C ) Reorganization D ) Integration 1 6 . A nurse is assessing a rape survivor for post-traumatic stress disorder. The nurse asks the woman, Do you feel as though you are reliving the trauma? The nurse is assessing for which of the following? A ) Physical symptoms B ) Intrusive thoughts C ) Avoidance D ) Hyperarousal 1 A group of students are preparing a class discussion about rape and sexual 7 assault. Which of the following would the students include as being most . accurate? (Select all that apply.) A Most victims of rape tell someone about it. ) B Few women falsely cry rape. ) C Women have rape fantasies desiring to be raped. ) D A rape victim feels vulnerable and betrayed afterwards. ) E Medication and counseling can help a rape victim cope. ) 1 8 . After teaching a class on date rape, the instructor determines that the teaching was successful when the class identifies which of the following as the most common date rape drug? A ) Gamma hydroxybutyrate B ) Liquid ecstasy C ) Ketamine D ) Rohypnol 1 9 . A nurse is caring for a woman who was recently raped. The nurse would expect this woman to experience which of the following first? A ) Denial B ) Disorganization C Reorganization ) D Integration ) 2 0 . A group of nurses are researching information about risk factors for intimate partner violence in men. Which of the following would the nurses expect to find related to the individual person? (Select all that apply.) A ) Dysfunctional family system B ) Low academic achievement C ) Victim of childhood violence D ) Heavy alcohol consumption E ) Economic stress 2 A nurse is working with a victim of intimate partner violence and helping her 1 develop a safety plan. Which of the following would the nurse suggest that the . woman take with her? (Select all that apply.) A Drivers license ) B Social security number ) C Cash ) D Phone cards ) E Health insurance cards ) 2 A nurse is presenting a discussion on sexual violence at a local community 2 college. When describing the incidence of sexual violence, the nurse would . identify that a woman has which chance of experiencing a sexual assault in her lifetime? A One in three ) B One in six ) C Two in 15 ) D Three in 20 ) 2 After teaching a class on sexual violence, the instructor determines that the 3 teaching was successful when the class identifies which of the following as a . type of sexual violence. (Select all that apply.) A Female genital cutting ) B Bondage ) C Infanticide ) D Human trafficking ) E Rape ) 2 A nurse is reading a journal article about sexual abuse. Which age range would 4 the nurse expect to find as the peak age for such abuse? . A 710 years ) B 812 years ) C 1418 years ) D 1822 years ) 2 After teaching a group of students about sexual abuse and violence, the 5 instructor determines that the teaching was successful when the students . describe incest as involving which of the following? A Sexual exploitation by blood or surrogate relatives ) B Sexual abuse of individuals over age 18 ) C Violent aggressive assault on a person ) D Consent between perpetrator and victim. ) Answer Key 1. B 2. D 3. C 4. B 5. A 6. B 7. D 8. A 9. B 10. B 11. A 12. A 13. C 14. C 15. C 16. B 17. B, D, E 18. D 19. B 20. B, C, D 21. A, B, C, E 22. B 23. A, B, C, D, E 24. B 25. A Chapter 10 Fetal Development and Genetics 1 . While talking with a pregnant woman who has undergone genetic testing, the woman informs the nurse that her baby will be born with Down syndrome. The nurse understands that Down syndrome is an example of: A ) Multifactorial inheritance B ) X-linked recessive inheritance C ) Trisomy numeric abnormality D ) Chromosomal deletion 2 . A nurse is describing advances in genetics to a group of students. Which of the following would the nurse least likely include? A ) Genetic diagnosis is now available as early as the second trimester. B ) Genetic testing can identify presymptomatic conditions in children. C ) Gene therapy can be used to repair missing genes with normal ones. D ) Genetic agents may be used in the future to replace drugs. 3 After teaching a group of students about fetal development, the instructor . determines that the teaching was successful when the students identify which of the following as providing the barrier to other sperm after fertilization? A Zona pellucida ) B Zygote ) C Cleavage ) D Morula ) 4 A nurse is teaching a class on X-linked recessive disorders. Which of the . following statements would the nurse most likely include? A Males are typically carriers of the disorders. ) B No male-to-male transmission occurs. ) C Daughters are more commonly affected with the disorder. ) D Both sons and daughters have a 50% risk of the disorder. ) 5 A pregnant woman undergoes maternal serum alpha-fetoprotein (MSAFP) . testing at 16 to 18 weeks gestation. Which of the following would the nurse suspect if the womans level is decreased? A Down syndrome ) B Sickle-cell anemia ) C Cardiac defects ) D Open neural tube defect ) 6 The nurse is developing a presentation for a community group of young adults . discussing fetal development and pregnancy. The nurse would identify that the sex of offspring is determined at the time of: A Meiosis ) B Fertilization ) C Formation of morula ) D Oogenesis ) 7 . When describing amniotic fluid to a pregnant woman, the nurse would include which of the following? A ) This fluid acts as transport mechanism for oxygen and nutrients. B ) The fluid is mostly protein to provide nourishment to your baby. C ) This fluid acts as a cushion to help to protect your baby from injury. D ) The amount of fluid remains fairly constant throughout the pregnancy. 8 Assessment of a pregnant woman reveals oligohydramnios. The nurse would be . alert for the development of which of the following? A ) Maternal diabetes B ) Placental insufficiency C ) Neural tube defects D ) Fetal gastrointestinal malformations 9 . A cou the vi statin how ple comes to the clinic for preconception counseling and care. As part of sit, the nurse teaches the couple about fertilization and initial development, g that the zygote formed by the union of the ovum and sperm consists of many chromosomes? A ) 22 B ) 23 C ) 44 D ) 46 1 0 . A woman just delivered a healthy term newborn. Upon assessing the umbilical cord, the nurse would identify which of the following as normal? (Select all that apply.) A ) One vein B ) Two veins C One artery ) D Two arteries ) E One ligament ) F Two ligaments ) 1 1 . After teaching a pregnant woman about the hormones produced by the placenta, the nurse determines that the teaching was successful when the woman identifies which hormone produced as being the basis for pregnancy tests? A ) Human placental lactogen (hPL) B ) Estrogen (estriol) C ) Progesterone (progestin) D ) Human chorionic gonadotropin (hCG) 1 2 . After the nurse describes fetal circulation to a pregnant woman, the woman asks why her fetus has a different circulation pattern than hers. In planning a response, the nurse integrates understanding of which of the following? A ) Fetal blood is thicker than that of adults and needs different pathways. B ) Fetal circulation carries highly oxygenated blood to vital areas first. C Fetal blood has a higher oxygen saturation and circulates more slowly. ) D Fetal heart rates are rapid and circulation time is double that of adults. ) 1 3 . When describing genetic disorders to a group of childbearing couples, the nurse would identify which as an example of an autosomal dominant inheritance disorder? A ) Huntingtons disease B ) Sickle cell disease C ) Phenylketonuria D ) Cystic fibrosis 1 4 . Prenatal testing is used to assess for genetic risks and to identify genetic disorders. In explaining to a couple about an elevated alpha-fetoprotein screening test result, the nurse would discuss the need for: A ) Special care needed for a Down syndrome infant B ) A more specific determination of the acidbase status C ) Further, more definitive evaluations to conclude anything D ) Immediate termination of the pregnancy based on results 1 5 . A nursing instructor is preparing a teaching plan for a group of nursing students about the potential for misuse of genetic discoveries and advances. Which the following would the instructor most likely include? A ) Gene replacement therapy for defective genes B ) Individual risk profiling and confidentiality C ) Greater emphasis on the causes of diseases D ) Slower diagnosis of specific diseases 1 6 . After teaching a class on the stages of fetal development, the instructor determines that the teaching was successful when the students identify which of the following as a stage? (Select all that apply.) A ) Placental B ) Preembryonic C ) Umbilical D ) Embryonic E ) Fetal 1 A nurse is discussing fetal development with a pregnant woman. The woman is 7 12 weeks pregnant and asks, Whats happening with my baby? Which of the . following would the nurse integrate into the response? (Select all that apply.) A Continued sexual differentiation ) B Eyebrows forming ) C Startle reflex present ) D Digestive system becoming active ) E Lanugo present on the head ) 1 8 . After teaching a group of students about fetal development, the instructor determines that the teaching was successful when the students identify which of the following as essential for fetal lung development? A ) Umbilical cord B ) Amniotic fluid C ) Placenta D ) Trophoblasts 1 9 . During a hormones include? prenatal class for a group of new mothers, the nurse is describing the produced by the placenta. Which of the following would the nurse (Select all that apply.) A ) Prolactin B ) Estriol C ) Relaxin D ) Progestin E ) Human chorionic somatomammotropin 2 0 . When describing the structures involved in fetal circulation, the nursing instructor describes which structure as the opening between the right and left atrium? A ) Ductus venosus B ) Foramen ovale C ) Ductus arteriosus D ) Umbilical artery 2 1 . A group of students are reviewing information about genetic inheritance. The students demonstrate understanding of the information when they identify which of the following as an example of an autosomal recessive disorder? (Select all that apply.) A ) Cystic fibrosis B ) Phenylketonuria C ) Tay-Sachs disease D ) Polycystic kidney disease E ) Achondroplasia 2 2 . A nurse is assessing a child with Klinefelters syndrome. Which of the following would the nurse expect to assess? (Select all that apply.) A ) Gross mental retardation B ) Long arms C ) Profuse body hair D ) Gynecomastia E ) Enlarged testicles 2 3 . A woman is scheduled to undergo fetal nuchal translucency testing. Which of the following would the nurse include when describing this test? A ) A needle will be inserted directly into the fetuss umbilical vessel. B ) Youll have an intravaginal ultrasound to measure fluid in the fetus. C The doctor will take a sample of fluid from your bag of waters. ) D A small piece of tissue from the fetal part of the placenta is taken. ) Answer Key 1. C 2. A 3. A 4. B 5. A 6. B 7. C 8. B 9. D 10. A 11. D 12. B 13. A 14. C 15. B 16. B, C, E 17. A, D 18. B 19. B, C, D, E 20. B 21. A 22. B, D 23. B Chapter 11 Maternal Adaptation During Pregnancy 1 . During a vaginal exam, the nurse notes that the cervix has a bluish color. The nurse documents this finding as: A ) Hegars sign B ) Goodells sign C ) Chadwicks sign D ) Ortolanis sign 2 . The nurse teaches a primigravida client that lightening occurs about 2 weeks before the onset of labor. The mother will most likely experience which of the following at that time? A ) Dysuria B ) Dyspnea C ) Constipation D Urinary frequency ) 3 . A gravida 2 para 1 client in the 10th week of her pregnancy says to the nurse, Ive never urinated as often as I have for the past three weeks. Which response would be most appropriate for the nurse to make? A ) Having to urinate so often is annoying. I suggest that you watch how much fluid you are drinking and limit it. B ) You shouldnt be urinating this frequently now; it usually stops by the time youre eight weeks pregnant. Is there anything else bothering you? C ) By the time you are 12 weeks pregnant, this frequent urination should no longer be a problem, but it is likely to return toward the end of your pregnancy. D ) Women having their second child generally dont have frequent urination. Are you experiencing any burning sensations? 4 . In a clients seventh month of pregnancy, she reports feeling dizzy, like Im going to pass out, when I lie down flat on my back. The nurse integrates which of the following in to the explanation? A ) Pressure of the gravid uterus on the vena cava B ) A 50% increase in blood volume C ) Physiologic anemia due to hemoglobin decrease D ) Pressure of the presenting fetal part on the diaphragm 5 . A primiparous client is being seen in the clinic for her first prenatal visit. It is determined that she is 11 weeks pregnant. The nurse develops a teaching plan to educate the client about what she will most likely experience during this period. Which of the following would the nurse include? A ) Ankle edema B ) Urinary frequency C ) Backache D ) Hemorrhoids 6 . A pregnant client in her second trimester has a hemoglobin level of 11 g/dL. The nurse interprets this as indicating which of the following? A ) Iron-deficiency anemia B ) A multiple gestation pregnancy C ) Greater-than-expected weight gain D ) Hemodilution of pregnancy 7 . The nurse is discussing the insulin needs of a primaparous client with diab who has been using insulin for the past few years. The nurse informs the that her insulin needs will increase during pregnancy based on the nurses understanding that the placenta produces: etes client A ) hCG, which increases maternal glucose levels B ) hPL, which deceases the effectiveness of insulin C ) Estriol, which interferes with insulin crossing the placenta D ) Relaxin, which decreases the amount of insulin produced 8 . When teaching a pregnant client about the physiologic changes of pregnancy, the nurse reviews the effect of pregnancy on glucose metabolism. Which of the following would the nurse include as the underlying reason for the effect? A ) Pancreatic function is affected by pregnancy. B ) Glucose is utilized more rapidly during a pregnancy. C ) The pregnant woman increases her dietary intake. D ) Glucose moves through the placenta to assist the fetus. 9 . When assessing a woman in her first trimester, which emotional response would the nurse most likely expect to find? A ) Ambivalence B ) Introversion C ) Acceptance D ) Emotional lability 1 0 . The nurse is assessing a pregnant woman in the second trimester. Which of the following tasks would indicate to the nurse that the client is incorporating the maternal role into her personality? A ) The woman demonstrates concern for herself and her fetus as a unit. B ) The client identifies what she must give up to assume her new role. C ) The woman acknowledges the fetus as a separate entity within her. D ) The client demonstrates unconditional acceptance without rejection. 1 1 . A woman comes to the prenatal clinic suspecting that she is pregnant, and assessment reveals probable signs of pregnancy. Which of the following would be included as part of this assessment? (Select all that apply.) A ) Positive pregnancy test B ) Ultrasound visualization of the fetus C ) Auscultation of a fetal heart beat D ) Ballottement E ) Absence of menstruation F ) Softening of the cervix 1 2 . The nurse is teaching a pregnant woman about recomme woman has a prepregnancy body mass index of 26. The the teaching was successful when the woman states that more than which amount during pregnancy? nded weight gain. The nurse determines that she should gain no A ) 35 to 40 pounds B ) 25 to 35 pounds C ) 28 to 40 pounds D ) 15 to 25 pounds 1 3 . A nurse strongly encourages a pregnant client to avoid eating swordfish and tilefish because these fish contain which of the following? A ) Excess folic acid, which could increase the risk for neural tube defects B ) Mercury, which could harm the developing fetus if eaten in large amounts C ) Lactose, which leads to abdominal discomfort, gas, and diarrhea D ) Low-quality protein that does not meet the womans requirements 1 Which of the following changes in the musculoskeletal system would the nurse 4 mention when teaching a group of pregnant women about the physiologic . changes of pregnancy? A Ligament tightening ) B ) Decreased swayback C ) Increased lordosis D ) Joint contraction 1 Assessment of a pregnant woman reveals a pigmented line down the middle of 5 her abdomen. The nurse documents this as which of the following? . A Linea nigra ) B Striae gravidarum ) C Melasma ) D Vascular spiders ) 1 A nurse is assessing a pregnant woman on a routine checkup. When assessing 6 the womans gastrointestinal tract, which of the following would the nurse . expect to find? (Select all that apply.) A Hyperemic gums ) B Increased peristalsis ) C Complaints of bloating ) D Heartburn ) E Nausea ) 1 7 . A woman suspecting she is pregnant asks the nurse about which signs would confirm her pregnancy. The nurse would explain that which of the following would confirm the pregnancy? A ) Absence of menstrual period B ) Abdominal enlargement C ) Palpable fetal movement D ) Morning sickness 1 A nurse is developing a teaching plan about nutrition for a group of pregnant 8 women. Which of the following would the nurse include in the discussion? . (Select all that apply.) A Keep weight gain to 15 lb ) B Eat three meals with snacking ) C Limit the use of salt in cooking ) D Avoid using diuretics ) E Participate in physical activity ) 1 Assessment of a pregnant woman reveals that she compulsively craves ice. The 9 nurse documents this finding as which of the following? . A Quickening ) B Pica ) C Ballottement ) D Linea nigra ) 2 A woman in her second trimester comes for a follow-up visit and says to the 0 nurse, I feel like Im on an emotional roller-coaster. Which response by the nurse . would be most appropriate? A How often has this been happening to you? ) B Maybe you need some medication to level things out. ) C Mood swings are completely normal during pregnancy. ) D Have you been experiencing any thoughts of harming yourself? ) 2 While talking with a woman in her third trimester, which behavior indicates to 1 the nurse that the woman is learning to give of oneself? . A ) Showing concern for self and fetus as a unit B ) Unconditionally accepting the pregnancy without rejection C ) Longing to hold infant D ) Questioning ability to become a good mother 2 2 . A group of students are reviewing the signs of pregnancy. The students demonstrate understanding of the information when they identify which as presumptive signs? (Select all that apply.) A ) Amenorrhea B ) Nausea C ) Abdominal enlargement D ) Braxton-Hicks contractions E ) Fetal heart sounds 2 3 . A nursing insulin, and include as instructor is teaching a class to a group of students about pregnancy, glucose. Which of the following would the instructor least likely opposing insulin? A ) Prolactin B Estrogen ) C Progesterone ) D Cortisol ) 2 4 . A woman is at 20 weeks gestation. The nurse would expect to find the fundus at which of the following? A ) Just above the symphysis pubis B ) Mid-way between the pubis and umbilicus C ) At the level of the umbilicus D ) Mid-way between the umbilicus and xiphoid process 2 5 . A pregnant woman comes to having a whitish vaginal dis based on which assessment the clinic and tells the nurse that she has been charge. The nurse suspects vulvovaginal candidiasis finding? A ) Fever B ) Vaginal itching C ) Urinary frequency D Incontinence ) Answer Key 1. C 2. D 3. C 4. A 5. B 6. D 7. B 8. D 9. A 10. C 11. A, D, F 12. D 13. B 14. C 15. A 16. A, C, D, E 17. C 18. B, D, E 19. B 20. C 21. D 22. A, B 23. D 24. C 25. B Chapter 12 Nursing Management During Pregnancy 1 . A woman in the 34th week of pregnancy says to the nurse, I still feel like having intercourse with my husband. The womans pregnancy has been uneventful. The nurse responds based on the understanding that: A ) It is safe to have intercourse at this time. B ) Intercourse at this time is likely to cause rupture of membranes. C ) There are other ways that the couple can satisfy their needs. D ) Intercourse at this time is likely to result in premature labor. 2 . On the first prenatal visit, examination of the womans internal genitalia reveals a bluish coloration of the cervix and vaginal mucosa. The nurse records this finding as: A ) Hegars sign B ) Goodells sign C Chadwicks sign ) D Homans sign ) 3 . When describing perinatal education to a pregnant woman and her partner, the nurse emphasizes that the primary goal of these classes is to: A ) Equip a couple with the knowledge to experience a pain-free childbirth B ) Provide knowledge and skills to actively participate in birth and parenting C ) Eliminate anxiety so that they can have an uncomplicated birth D ) Empower the couple to totally control the birth process 4 . When assessing a woman at follow-up prenatal visits, the nurse would anticipate which of the following to be performed? A ) Hemoglobin and hematocrit B ) Urine for culture C ) Fetal ultrasound D ) Fundal height measurement 5 . During a routine prenatal visit, a client, 36 weeks pregnant, states she has difficulty breathing and feels like her pulse rate is really fast. The nurse finds her pulse to be 100 beats per minute (increased from baseline readings of 70 to 74 beats per minute. and irregular, with bilateral crackles in the lower lung bases. Which nursing diagnosis would be the priority for this client? A ) Ineffective tissue perfusion related to supine hypotensive syndrome B ) Impaired gas exchange related to pulmonary congestion C ) Activity intolerance related to increased metabolic requirements D ) Anxiety related to fear of pregnancy outcome 6 . When preparing a woman for an amniocentesis, the nurse would instruct her to do which of the following? A ) Shower with an antiseptic scrub. B ) Swallow the preprocedure sedative. C ) Empty her bladder. D ) Lie on her left side. 7 A client who is 4 months pregnant is at the prenatal clinic for her initial visit. . Her history reveals she has 7-year-old twins who were born at 34 weeks gestation, a 2-year old son born at 39 weeks gestation, and a spontaneous abortion 1 year ago at 6 weeks gestation. Using the GTPAL method, the nurse would document her obstetric history as: A 3 2 1 0 3 ) B 3 1 2 2 3 ) C 4 1 1 1 3 ) D 4 2 1 3 1 ) 8 A clients last menstrual period was April 11. Using Nageles rule, her expected . date of birth (EDB. would be: A January 4 ) B January 18 ) C January 25 ) D February 24 ) 9 During a nonstress test, when monitoring the fetal heart rate, the nurse notes that . when the expectant mother reports fetal movement, the heart rate increases 15 beats or more above the baseline. The nurse interprets this as: A Variable decelerations ) B Fetal tachycardia ) C A nonreactive pattern ) D Reactive pattern ) 1 0 . A clients maternal serum alpha-fetoprotein (MSAFP. level was unusually elevated at 17 weeks. The nurse suspects which of the following? A ) Fetal hypoxia B ) Open spinal defects C ) Down syndrome D ) Maternal hypertension 1 1 . When assessing a pregnant woman in her last trimester, which question would be most appropriate to use to gather information about weight gain and fluid retention? A ) Whats your usual dietary intake for a typical day? B ) What size maternity clothes are you wearing now? C ) How puffy does your face look by the end of a day? D ) How swollen do your ankles appear before you go to bed? 1 2 . A pregnant woman in the 36th week of gestation complains that her feet are quite swollen at the end of the day. After careful assessment, the nurse determines that this is an expected finding at this stage of pregnancy. Which intervention would be most appropriate for the nurse to suggest? A ) Limit your intake of fluids. B ) Eliminate salt from your diet. C ) Try elevating your legs when you sit. D ) Wear Spandex-type full-length pants. 1 3 . A pregnant woman needs an update in her immunizations. Which of the following vaccinations would the nurse ensure that the woman receives? A ) Measles B ) Mumps C ) Rubella D ) Hepatitis B 1 A pregnant woman is flying across the country to visit her family. After 4 teaching the woman about traveling during pregnancy, which statement . indicates that the teaching was successful? A Ill sit in a window seat so I can focus on the sky to help relax me. ) B ) I wont drink too much fluid so I dont have to urinate so often. C ) Ill get up and walk around the airplane about every 2 hours. D ) Ill do some upper arm stretches while sitting in my seat. 1 Which of the following would the nurse include when teaching a pregnant 5 woman about chorionic villus sampling? . A The results should be available in about a week. ) B Youll have an ultrasound first and then the test. ) C Afterwards, you can resume your exercise program. ) D This test is very helpful for identifying spinal defects. ) 1 A pregnant woman is scheduled to undergo percutaneous umbilical blood 6 sampling. When discussing this test with the woman, the nurse reviews what . can be evaluated with the specimens collected. Which of the following would the nurse include? (Select all that apply.) A Rh incompatibility ) B Fetal acidbase status ) C Sex-linked disorders ) D Enzyme deficiencies ) E Coagulation studies ) 1 7 . A biophys interprets ical profile has been completed on a pregnant woman. The nurse which score as normal? A ) 9 B ) 7 C ) 5 D ) 3 1 After teaching a group of students about the discomforts of pregnancy, the 8 students demonstrate understanding of the information when they identify . which as common during the first trimester? (Select all that apply.) A Urinary frequency ) B Breast tenderness ) C Cravings ) D Backache ) E Leg cramps ) 1 A nurse is reviewing the medical record of a pregnant woman and notes that she 9 is gravid II. The nurse interprets this to indicate the number of: . A Deliveries ) B Pregnancies ) C Spontaneous abortions ) D Pre-term births ) 2 A nurse measures a pregnant womans fundal height and finds it to be 28 cm. 0 The nurse interprets this to indicate which of the following? . A 14 weeks gestation ) B 20 weeks gestation ) C 28 weeks gestation ) D 36 weeks gestation ) 2 A pregnant woman has a rubella titer drawn on her first prenatal visit. The nurse 1 explains that this test measures which of the following? . A ) Platelet level B ) Rh status C ) Immunity to German measles D ) Red blood cell count 2 2 . A nurse is working with a pregnancy woman to schedule follow-up visits for her pregnancy. Which statement by the woman indicates that she understands the scheduling? A ) I need to make visits every 2 months until Im 36 weeks pregnant. B ) Once I get to 28 weeks, I have to come twice a month. C ) From now until Im 28 weeks, Ill be coming once a month. D ) Ill make sure to get a day off every 2 weeks to make my visits. 2 3 . A nursing instructor is describing the various childbirth methods. Which of the following would the instructor include as part of the Lamaze method? A ) Focus on the pleasurable sensations of childbirth B ) Concentration on sensations while turning on to own bodies C Interruption of the fear-tension-pain cycle ) D Use of specific breathing and relaxation techniques ) 2 4 . After teaching a group of students about the different perinatal education methods, the instructor determines that the teaching was successful when the students identify which of the following as the Bradley method? A ) Psychoprophylactic method B ) Partner-coached method C ) Natural childbirth method D ) Mind prevention method 2 5 . A pregnant woman in her second trimester tells the nurse, Ive been passing a lot of gas and feel bloated. Which of the following suggestions would be helpful for the woman? A ) Watch how much beans and onions you eat. B ) Limit the amount of fluid you drink with meals C ) Try exercising a little more. D ) Some say that eating mints can help. E Cut down on your intake of cheeses. ) Answer Key 1. A 2. C 3. B 4. D 5. B 6. C 7. C 8. B 9. D 10. B 11. D 12. C 13. D 14. C 15. B 16. A, B, E 17. A 18. A, B, C 19. B 20. C 21. C 22. C 23. D 24. B 25. A, C, D Chapter 13 Labor and Birth Process 1 . A woman in her 40th week of pregnancy calls the nurse at the clinic and says shes not sure whether she is in true or false labor. Which statement by the client would lead the nurse to suspect that the woman is experiencing false labor? A ) Im feeling contractions mostly in my back. B ) My contractions are about 6 minutes apart and regular. C ) The contractions slow down when I walk around. D ) If I try to talk to my partner during a contraction, I cant. 2 Which of the following would indicate to the nurse that the placenta is . separating? A Uterus becomes globular ) B Fetal head is at vaginal opening ) C Umbilical cord shortens ) D Mucous plug is expelled ) 3 . When assessing cervical effacement of a client in labor, the nurse assesses which of the following characteristics? A ) Extent of opening to its widest diameter B ) Degree of thinning C ) Passage of the mucous plug D ) Fetal presenting part 4 . A woman calls the health care facility stating that she is in labor. The nurse would urge the client to come to the facility if the client reports which of the following? A ) Increased energy level with alternating strong and weak contractions B ) Moderately strong contractions every 4 minutes, lasting about 1 minute C ) Contractions noted in the front of abdomen that stop when she walks D ) Pink-tinged vaginal secretions and irregular contractions lasting about 30 seconds 5 A woman is in the first stage of labor. The nurse would encourage her to assume . which position to facilitate the progress of labor? A Supine ) B Lithotomy ) C Upright ) D Kneechest ) 6 . A client has not received any medication during her labor. She is having frequent contractions every 1 to 2 minutes and has become irritable with her coach and no longer will allow the nurse to palpate her fundus during contractions. Her cervix is 8 cm dilated and 90% effaced. The nurse interprets these findings as indicating: A ) Latent phase of the first stage of labor B ) Active phase of the first stage of labor C ) Transition phase of the first stage of labor D ) Pelvic phase of the second stage of labor 7 . The fetus of a nulliparous woman is in a shoulder presentation. The nurse would most likely prepare the client for which type of birth? A ) Cesarean B ) Vaginal C Forceps-assisted ) D Vacuum extraction ) 8 . Assessment of a woman in labor reveals cervical dilation of 3 cm, cervical effacement of 30%, and contractions occurring every 7 to 8 minutes, lasting about 40 seconds. The nurse determines that this client is in: A ) Latent phase of the first stage B ) Active phase of the first stage C ) Transition phase of the first stage D ) Perineal phase of the second stage 9 . A client is admitted to the labor and birthing suite in early labor. On review of her medical record, the nurse determines that the clients pelvic shape as identified in the antepartal progress notes is the most favorable one for a vaginal delivery. Which pelvic shape would the nurse have noted? A ) Platypelloid B ) Gynecoid C ) Android D ) Anthropoid 1 0 . A woman telephones her health care provider and reports that her water just broke. Which suggestion by the nurse would be most appropriate? A ) Call us back when you start having contractions. B ) Come to the clinic or emergency department for an evaluation. C ) Drink 3 to 4 glasses of water and lie down. D ) Come in as soon as you feel the urge to push. 1 1 . After teaching a group of students about the maternal bony pelvis, which statement by the group indicates that the teaching was successful? A ) The bony pelvis plays a lesser role during labor than soft tissue. B ) The pelvic outlet is associated with the true pelvis. C ) The false pelvis lies below the imaginary linea terminalis. D ) The false pelvis is the passageway through which the fetus travels. 1 A fetus is assessed at 2 cm above the ischial spines. The nurse would document 2 fetal station as: . A +4 ) B +2 ) C 0 ) D 2 ) 1 3 . Assessment of a fetus identifies the buttocks as the presenting part, with the legs extended upward. The nurse identifies this as which type of breech presentation? A ) Frank B ) Full C ) Complete D ) Footling 1 4 . A woman in her third trimester comes to the clinic for a prenatal visit. During assessment the woman reports that her breathing has become much easier in the last week but she has noticed increased pelvic pressure , cramping and lower back pain. The nurse determines that which of the following has most likely occurred? A ) Cervical dilation B ) Lightening C ) Bloody show D Braxton-Hicks contractions ) 1 5 . After teaching a group of students about the factors affecting the labor process, the instructor determines that the teaching was successful when the group identifies which of the following as a component of the true pelvis? (Select all that apply.) A ) Pelvic inlet B ) Cervix C ) Mid pelvis D ) Pelvic outlet E ) Vagina F ) Pelvic floor muscles 1 6 . A nurse is documenting fetal lie of a woman in labor. Which term would the nurse most likely use? A ) Flexion B ) Extension C ) Longitudinal D Cephalic ) 1 7 . The nurse is reviewing the medical record of a woman in labor and notes that the fetal position is documented as LSA. The nurse interprets this information as indicating which of the following is the presenting part? A ) Occiput B ) Face C ) Buttocks D ) Shoulder 1 8 . A nurse is preparing a class for pregnant women about labor and birth. When describing the typical movements that the fetus goes through as it travels through the passageway, which of the following would the nurse most likely include? (Select all that apply.) A ) Internal rotation B ) Abduction C ) Descent D ) Pronation E ) Flexion 1 9 . The nurse is reviewing the monitoring strip of a woman in labor who is experiencing a contraction. The nurse notes the time the contraction takes from its onset to reach its highest intensity. The nurse interprets this time as which of the following? A ) Increment B ) Acme C ) Peak D ) Decrement 2 0 . A nurse is assessing a woman in labor. Which finding would the nurse identify as a cause for concern during a contraction? A ) Heart rate increase from 76 bpm to 90 bpm B ) Blood pressure rise from 110/60 mm Hg to 120/74 C ) White blood cell count of 12,000 cells/mm3 D ) Respiratory rate of 10 breaths /minute 2 When describing the stages of labor to a pregnant woman, which of the 1 following would the nurse identify as the major change occurring during the . first stage? A Regular contractions ) B ) Cervical dilation C ) Fetal movement through the birth canal D ) Placental separation 2 A nurse is caring for several women in labor. The nurse determines that which 2 woman is in the transition phase of labor? . A Contractions every 5 minutes, cervical dilation 3 cm ) B Contractions every 3 minutes, cervical dilation 5 cm ) C Contractions every 2 minutes, cervical dilation 7 cm ) D Contractions every 1 minute, cervical dilation 9 cm ) 2 A nurse is preparing a presentation for a group of pregnant women about the 3 labor experience. Which of the following would the nurse most likely include . when discussing measures to promote coping for a positive labor experience? (Select all that apply.) A Presence of a support partner ) B View of birth as a stressor ) C Low anxiety level ) D Fear of loss of control ) E Participation in a pregnancy exercise program ) 2 During a follow-up prenatal visit, a pregnant woman asks the nurse, How long 4 do you think I will be in labor? Which response by the nurse would be most . appropriate? A Its difficult to predict how your labor will progress, but well be there for you the ) entire time. B Since this is your first pregnancy, you can estimate it will be about 10 hours. ) C It will depend on how big the baby is when you go into labor. ) D Time isnt important; your health and the babys health are key. ) 2 A nurse is describing how the fetus moves through the birth canal. Which of the 5 following would the nurse identify as being most important in allowing the fetal . head to move through the pelvis? A Sutures ) B Fontanelles ) C Frontal bones ) D Biparietal diameter ) 2 Assessment of a pregnant woman reveals that the presenting part of the fetus is 6 at the level of the maternal ischial spines. The nurse documents this as which . station? A 2 ) B 1 ) C 0 ) D +1 ) Answer Key 1. C 2. A 3. B 4. B 5. C 6. C 7. A 8. A 9. B 10. B 11. B 12. D 13. A 14. B 15. A, C, D 16. C 17. C 18. A, C, E 19. A 20. D 21. B 22. D 23. A, C, E 24. A 25. A 26. C Chapter 14 Nursing Management During Labor and Birth 1 . A woman in labor who received an opioid for pain relief develops respiratory depression. The nurse would expect which agent to be administered? A ) Butorphanol B ) Fentanyl C ) Naloxone D ) Promethazine 2 . A clients membranes spontaneously ruptured, as evidenced by a gush of clear fluid with a contraction. Which of the following would the nurse do next? A ) Check the fetal heart rate. B ) Perform a vaginal exam. C ) Notify the physician immediately. D ) Change the linen saver pad. 3 . A woman has just entered the second stage of labor. The nurse would focus care on which of the following? A ) Encouraging the woman to push when she has a strong desire to do so B ) Alleviating perineal discomfort with the application of ice packs C ) Palpating the womans fundus for position and firmness D ) Completing the identification process of the newborn with the mother 4 . The nurse notes persistent early decelerations on the fetal monitoring strip. Which of the following would the nurse do next? A ) Continue to monitor the FHR because this pattern is benign. B ) Perform a vaginal exam to assess cervical dilation and effacement. C Stay with the client while reporting the finding to the physician. ) D Administer oxygen after turning the client on her left side. ) 5 . A woman is admitted to the labor and birthing suite. Vaginal examination reveals that the presenting part is approximately 2 cm above the ischial spines. The nurse documents this finding as: A ) +2 station B ) 0 station C ) 2 station D ) Crowning 6 . The nurse is performing Leopolds maneuvers to determine fetal presentation, position, and lie. Which action would the nurse do first? A ) Feel for the fetal buttocks or head while palpating the abdomen. B ) Feel for the fetal back and limbs as the hands move laterally on the abdomen. C ) Palpate for the presenting part in the area just above the symphysis pubis. D ) Determine flexion by pressing downward toward the symphysis pubis. 7 . A client states, I think my waters broke! I felt this gush of fluid between my legs. The nurse tests the fluid with Nitrazine paper and confirms membrane rupture if the paper turns: A ) Yellow B ) Olive green C ) Pink D ) Blue 8 . A woman in labor is to receive continuous internal electronic fetal monitoring. The nurse reviews the womans medical record to ensure which of the following as being required? A ) Intact membranes B ) Cervical dilation of 2 cm or more C ) Floating presenting fetal part D ) A neonatologist to insert the electrode 9 When assessing fetal heart rate, the nurse finds a heart rate of 175 bpm, . accompanied by a decrease in variability and late decelerations. Which of the following would the nurse do next? A Have the woman change her position. ) B ) Administer oxygen. C ) Notify the health care provider. D ) Continue to monitor the pattern every 15 minutes. 1 0 . A woman in labor has chosen to use hydrotherapy as a method of pain relief. Which statement by the woman would lead the nurse to suspect that the woman needs additional teaching? A ) The warmth and buoyancy of the water has a nice relaxing effect. B ) I can stay in the bath for as long as I feel comfortable. C ) My cervix should be dilated more than 5 cm before I try using this method. D ) The temperature of the water should be at least 105 F. 1 1 . A woman in labor received an opioid close to the time of birth. The nurse would assess the newborn for which of the following? A ) Respiratory depression B ) Urinary retention C ) Abdominal distention D Hyperreflexia ) 1 2 . When applying the ultrasound transducers for continuous external electronic fetal monitoring, at which location would the nurse place the transducer to record the FHR? A ) Over the uterine fundus where contractions are most intense B ) Above the umbilicus toward the right side of the diaphragm C ) Between the umbilicus and the symphysis pubis D ) Between the xiphoid process and umbilicus 1 3 . After describing continuous internal electronic fetal monitoring to a laboring woman and her partner, which of the following would indicate the need for additional teaching? A ) This type of monitoring is the most accurate method for our baby. B ) Unfortunately, Im going to have to stay quite still in bed while it is in place. C ) This type of monitoring can only be used after my membranes rupture. D ) Youll be inserting a special electrode into my babys scalp. 1 When planning the care of a woman in the active phase of labor, the nurse 4 would anticipate assessing the fetal heart rate at which interval? . A ) Every 2 to 4 hours B ) Every 45 to 60 minutes C ) Every 15 to 30 minutes D ) Every 10 to 15 minutes 1 5 . Which of the following is a priority when caring for a woman during the fourth stage of labor? A ) Assessing the uterine fundus B ) Offering fluids as indicated C ) Encouraging the woman to void D ) Assisting with perineal care 1 6 . When palpating the fundus during a contraction, the nurse notes that it feels like a chin. The nurse interprets this finding as indicating which type of contraction? A ) Intense B ) Strong C Moderate ) D Mild ) 1 7 . A nurse palpates a womans fundus to determine contraction intensity. Which of the following would be most appropriate for the nurse to use for palpation? A ) Finger pads B ) Palm of the hand C ) Finger tips D ) Back of the hand 1 8 . A womans amniotic fluid is noted to be cloudy. The nurse interprets this finding as which of the following? A ) Normal B ) Possible infection C ) Meconium passage D ) Transient fetal hypoxia 1 9 . After teaching a group of students about fetal heart rate patterns, the instructor determines the need for additional teaching when the students identify which of the following as indicating normal fetal acidbase status? (Select all that apply.) A ) Sinusoidal pattern B ) Recurrent variable decelerations C ) Fetal bradycardia D ) Absence of late decelerations E ) Moderate baseline variability 2 0 . A nurse is reviewing the fetal heart rate pattern and observes abrupt decreases in FHR below the baseline, appearing as a U-shape. The nurse interprets these changes as reflecting which of the following? A ) Early decelerations B ) Variable decelerations C ) Prolonged decelerations D ) Late decelerations 2 A nurse is explaining the use of therapeutic touch as a pain relief measure 1 during labor. Which of the following would the nurse include in the . explanation? A This technique focuses on manipulating body tissues. ) B The technique requires focusing on a specific stimulus. ) C This technique redirects energy fields that lead to pain. ) D The technique involves light stroking of the abdomen with breathing. ) 2 A group of nursing students are reviewing the various medications used for pain 2 relief during labor. The students demonstrate understanding of the information . when they identify which agent as the most commonly used opioid? A Butorphanol ) B Nalbuphine ) C Fentanyl ) D Meperidine ) 2 3 . A nurse is describing the different types of regional analgesia and anesthesia for labor to a group of pregnant women. Which statement by the group indicates that the teaching was successful? A ) We can get up and walk around after receiving combined spinalepidural analgesia. B ) Higher anesthetic doses are needed for patient-controlled epidural analgesia. C A pudendal nerve block is highly effective for pain relief in the first stage of ) labor. D Local infiltration using lidocaine is an appropriate method for controlling ) contraction pain. 2 4 . A nurse is completing the assessment of a woman admitted to the labor and birth suite. Which of the following would the nurse expect to include as part of the physical assessment? (Select all that apply.) A ) Current pregnancy history B ) Fundal height measurement C ) Support system D ) Estimated date of birth E ) Membrane status F ) Contraction pattern 2 5 . A pregnant woman admitted to the labor and birth suite undergoes rapid HIV testing and is found to be HIV-positive. Which of the following would the nurse expect to include when developing a plan of care for this women? (Select all that apply.) A ) Administration of penicillin G at the onset of labor B ) Avoidance of scalp electrodes for fetal monitoring C ) Refraining from obtaining fetal scalp blood for pH testing D ) Adminstering zidovudine at the onset of labor. E ) Electing for the use of forceps-assisted delivery 2 6 . Which position would be most appropriate for the nurse to suggest as a comfort measure to a woman who is in the first stage of labor? (Select all that apply.) A ) Walking with partner support B ) Straddling with forward leaning over a chair C ) Closed kneechest position D ) Rocking back and forth with foot on chair E ) Supine with legs raised at a 90-degree angle 2 7 . Which of the following would be most appropriate for the nurse to suggest about pushing to a woman in the second stage of labor? A ) Lying flat with your head elevated on two pillows makes pushing easier. B ) Choose whatever method you feel most comfortable with for pushing. C Let me help you decide when it is time to start pushing. ) D Bear down like youre having a bowel movement with every contraction. ) 2 8 . A nurse is assessing a woman after birth and notes a second-degree laceration. The nurse interprets this as indicating that the tear extends through which of the following? A ) Skin B ) Muscles of perineal body C ) Anal sphincter D ) Anterior rectal wall 2 9 . A nurse is assisting with the delivery of a newborn. The fetal head has just emerged. Which of the following would be done next? A ) Suctioning of the mouth and nose B ) Clamping of the umbilical cord C ) Checking for the cord around the neck D ) Drying of the newborn 3 0 . A nurse is providing care to a woman during the third stage of labor. Which of the following would alert the nurse that the placenta is separating? (Select all that apply.) A ) Boggy, soft uterus B ) Uterus becoming discoid shaped C ) Sudden gush of dark blood from the vagina D ) Shortening of the umbilical cord Answer Key 1. C 2. A 3. A 4. A 5. C 6. A 7. D 8. B 9. C 10. D 11. A 12. C 13. B 14. C 15. A 16. C 17. A 18. B 19. A, B, C 20. B 21. C 22. D 23. A 24. B, E, F 25. B, C, D 26. A, B, D 27. B 28. B 29. C 30. C Chapter 15 Postpartum Adaptations 1 . A primipara client gave birth vaginally to a healt The nurse palpates the clients fundus, expecting hy newborn girl 48 hours ago. it to be at which location? A ) Two fingerbreadths above the umbilicus B ) At the level of the umbilicus C ) Two fingerbreadths below the umbilicus D ) Four fingerbreadths below the umbilicus 2 . When caring for a mother who has had a cesarean birth, the nurse would expect the clients lochia to be: A ) Greater than after a vaginal delivery B ) About the same as after a vaginal delivery C ) Less than after a vaginal delivery D ) Saturated with clots and mucus 3 . The nurse is developing a teaching plan for a client who has decided to bottle feed her newborn. Which of the following would the nurse include in the teaching plan to facilitate suppression of lactation? A ) Encouraging the woman to manually express milk B ) Suggesting that she take frequent warm showers to soothe her breasts C ) Telling her to limit the amount of fluids that she drinks D ) Instructing her to apply ice packs to both breasts every other hour 4 . The nurse is making a follow-up home visit to a woman who is 12 days postpartum. Which of the following would the nurse expect to find when assessing the clients fundus? A ) Cannot be palpated B ) 2 cm below the umbilicus C ) 6 cm below the umbilicus D ) 10 cm below the umbilicus 5 A client who is breast-feeding her newborn tells the nurse, I notice that when I . feed him, I feel fairly strong contraction-like pain. Labor is over. Why am I having contractions now? Which response by the nurse would be most appropriate? A ) Your uterus is still shrinking in size; thats why youre feeling this pain. B ) Let me check your vaginal discharge just to make sure everything is fine. C ) Your body is responding to the events of labor, just like after a tough workout. D ) The babys sucking releases a hormone that causes the uterus to contract. 6 When the nurse is assessing a postpartum client approximately 6 hours after . delivery, which finding would warrant further investigation? A Deep red, fleshy-smelling lochia ) B ) Voiding of 350 cc C ) Heart rate of 120 beats/minute D ) Profuse sweating 7 A postpartum client who is bottle feeding her newborn asks, When should my . period return? Which response by the nurse would be most appropriate? A Its difficult to say, but it will probably return in about 2 to 3 weeks. ) B It varies, but you can estimate it returning in about 7 to 9 weeks. ) C You wont have to worry about it returning for at least 3 months. ) D You dont have to worry about that now. Itll be quite a while. ) 8 . The nurse interprets which of the following as evidence that a client is in the taking-in phase? A ) Client states, He has my eyes and nose. B ) Client shows interest in caring for the newborn. C ) Client performs self-care independently. D ) Client confidently cares for the newborn. 9 . Which of the following would the nurse interpret as being least indicative of paternal engrossment? A ) Demonstrating pleasure when touching or holding the newborn B ) Identifying imperfections in the newborns appearance C ) Being able to distinguish his newborn from others in the nursery D ) Showing feelings of pride with the birth of the newborn 1 0 . A postpartum client comes to the clinic for her 6-week postpartum checkup. When assessing the clients cervix, the nurse would expect the external cervical os to appear: A ) Shapeless B ) Circular C ) Triangular D ) Slit-like 1 The nurse develops a teaching plan for a postpartum client and includes 1 teaching about how to perform Kegel exercises. The nurse includes this . information for which reason? A Reduce lochia ) B ) Promote uterine involution C ) Improve pelvic floor tone D ) Alleviate perineal pain 1 A father of a newborn tells the nurse, I may not know everything about being a 2 dad, but Im going to do the best I can for my son. The nurse interprets this as . indicating the father is in which stage of adaptation? A Expectations ) B Transition to mastery ) C Reality ) D Taking-in ) 1 A postpartum client is experiencing subinvolution. When reviewing the womans 3 labor and birth history, which of the following would the nurse identify as being . least significant to this condition? A Early ambulation ) B Prolonged labor ) C Large fetus ) D Use of anesthetics ) 1 Which of the following would lead the nurse to suspect that a postpartum 4 woman is experiencing a problem? . A Elevated white blood cell count ) B Acute decrease in hematocrit ) C Increased levels of clotting factors ) D Pulse rate of 60 beats/minute ) 1 A woman who gave birth 24 hours ago tells the nurse, Ive been urinating so 5 much over the past several hours. Which response by the nurse would be most . appropriate? A You must have an infection, so let me get a urine specimen. ) B Your body is undergoing many changes that cause your bladder to fill quickly. ) C Your uterus is not contracting as quickly as it should. ) D The anesthesia that you received is wearing off and your bladder is working ) again. 1 A group of students are reviewing the process of breast milk production. The 6 students demonstrate understanding when they identify which hormone as . responsible for milk let-down? A Prolactin ) B Estrogen ) C Progesterone ) D Oxytocin ) 1 7 . A nurse is making a home visit to a postpartum woman who delivered a healthy newborn 4 days ago. The womans breasts are swollen, hard, and tender to the touch. The nurse documents this finding as which of the following? A ) Involution B ) Engorgement C ) Mastitis D ) Engrossment 1 8 . A nurse is assessing a postpartum woman Which of the following would the nurse s adjustment to her maternal role. expect to occur first? A ) Reestablishing relationships with others B ) Demonstrating increasing confidence in care of the newborn C ) Assuming a passive role in meeting her own needs D ) Becoming preoccupied with the present 1 9 . The partner of a woman who has given birth to a healthy newborn says to the nurse, I want to be involved, but Im not sure that Im able to care for such a little baby. The nurse interprets this as indicating which of the following stages? A ) Expectations B ) Reality C ) Transition to mastery D ) Taking-hold 2 0 . A group of nursing students are reviewing information about maternal and paternal adaptations to the birth of a newborn. The nurse observes the parents interacting with their newborn physically and emotionally. The nurse documents this as which of the following? A ) Puerperium B ) Lactation C ) Attachment D Engrossment ) 2 1 . After teaching a group of nursing students about the process of involution, the instructor determines that additional teaching is needed when the students identify which of the following as being involved? A ) Catabolism B ) Muscle fiber contraction C ) Epithelial regeneration D ) Vasodilation 2 2 . A nurse is visiting a postpartum woman who delivered a healthy newborn 5 days ago. Which of the following would the nurse expect to find? A ) Bright red discharge B ) Pinkish brown discharge C ) Deep red mucus-like discharge D ) Creamy white discharge 2 A nurse teaches a postpartum woman about her risk for thromboembolism. 3 Which of the following would the nurse be least likely to include as a factor . increasing her risk? A ) Increased clotting factors B ) Vessel damage C ) Immobility D ) Increased red blood cell production 2 4 . A nursing student is preparing a class presentation about changes in the various body systems during the postpartum period and their effects. Which of the following would the student include as influencing a postpartum womans ability to void? (Select all that apply.) A ) Use of an opioid anesthetic during labor B ) Generalized swelling of the perineum C ) Decreased bladder tone from regional anesthesia D ) Use of oxytocin to augment labor E ) Need for an episiotomy 2 A postpartum woman who has experienced diastasis recti asks the nurse about 5 what to expect related to this condition. Which response by the nurse would be . most appropriate? A Youll notice that this will fade to silvery lines. ) B ) Exercise will help to improve the muscles. C ) Expect the color to lighten somewhat. D ) Youll notice that your shoe size will increase. 2 A group of nursing students are reviewing respiratory system adaptations that 6 occur during the postpartum period. The students demonstrate understanding of . the information when they identify which of the following as a postpartum adaptation? A Continued shortness of breath ) B Relief of rib aching ) C Diaphragmatic elevation ) D Decrease in respiratory rate ) 2 7 . A woman who delivered a healthy newborn several hours ago asks the nurse, Why am I perspiring so much? The nurse integrates knowledge that a decrease in which hormone plays a role in this occurrence? A ) Estrogen B ) hCG C ) hPL D Progesterone ) Answer Key 1. C 2. C 3. D 4. A 5. D 6. C 7. B 8. A 9. B 10. D 11. C 12. B 13. A 14. B 15. B 16. D 17. B 18. C 19. B 20. C 21. D 22. B 23. D 24. B, C, D 25. B 26. B 27. A Chapter 16 Nursing Management During the Postpartum Period 1 . A woman who is 12 hours postpartum had a pulse rate around 80 beats per minute during pregnancy. Now, the nurse finds a pulse of 60 beats per minute. Which of these actions should the nurse take? A ) Document the finding, as it is a normal finding at this time. B ) Contact the physician, as it indicates early DIC. C ) Contact the physician, as it is a first sign of postpartum eclampsia. D ) Obtain an order for a CBC, as it suggests postpartum anemia. 2 . To decrease the pain assoc which action by the nurse iated with an episiotomy immediately after birth, would be most appropriate? A ) Offer warm blankets. B ) Encourage the woman to void. C ) Apply an ice pack to the site. D ) Offer a warm sitz bath. 3 . A postpartum client has a fourth-degree perineal laceration. The nurse would expect which of the following medications to be ordered? A ) Ferrous sulfate (Feosol) B ) Methylergonovine (Methergine) C ) Docusate (Colace) D ) Bromocriptine (Parlodel) 4 . Which statement would alert the nurse to the potential for impaired bonding between mother and newborn? A ) You have your daddys eyes. B ) He looks like a frog to me. C ) Where did you get all that hair? D ) He seems to sleep a lot. 5 . After a normal labor and birth, a client is discharged from the hospital 12 hours later. When the community health nurse makes a home visit 2 days later, which finding would alert the nurse to the need for further intervention? A ) Presence of lochia serosa B ) Frequent scant voidings C ) Fundus firm, below umbilicus D ) Milk filling in both breasts 6 . A primipara client who is bottle feeding her baby begins to experience breast engorgement on her third postpartum day. Which instruction would be most appropriate to aid in relieving her discomfort? A ) Express some milk from your breasts every so often to relieve the distention. B ) Remove your bra to relieve the pressure on your sensitive nipples and breasts. C ) Apply ice packs to your breasts to reduce the amount of milk being produced. D ) Take several warm showers daily to stimulate the milk let-down reflex. 7 The nurse administers RhoGAM to an Rh-negative client after delivery of an . Rh-positive newborn based on the understanding that this drug will prevent her from: A Becoming Rh positive ) B ) Developing Rh sensitivity C ) Developing AB antigens in her blood D ) Becoming pregnant with an Rh-positive fetus 8 Which of the following factors in a clients history would alert the nurse to an . increased risk for postpartum hemorrhage? A Multiparity, age of mother, operative delivery ) B Size of placenta, small baby, operative delivery ) C Uterine atony, placenta previa, operative procedures ) D Prematurity, infection, length of labor ) 9 After teaching parents about their newborn, the nurse determines that the . teaching was successful when they identify the development of a close emotional attraction to a newborn by parents during the first 30 to 60 minutes after birth as which of the following? A Reciprocity ) B Engrossment ) C Bonding ) D Attachment ) 1 0 . A nurse is working as part of a committee to establish policies to promote bonding and attachment. Which practice would be least effective in achieving this goal? A ) Allowing unlimited visiting hours on maternity units B ) Offering round-the-clock nursery care for all infants C ) Promoting rooming-in D ) Encouraging infant contact immediately after birth 1 1 . When developing the plan of care for the parents of a newborn, the nurse identifies interventions to promote bonding and attachment based on the rationale that bonding and attachment are most supported by which measure? A ) Early parentinfant contact following birth B ) Expert medical care for the labor and birth C ) Good nutrition and prenatal care during pregnancy D ) Grandparent involvement in infant care after birth 1 A postpartum woman is having difficulty voiding for the first time after giving 2 birth. Which of the following would be least effective in helping to stimulate . voiding? A ) Pouring warm water over her perineal area B ) Having her hear the sound of water running nearby C ) Placing her hand in a basin of cool water D ) Standing her in the shower with the warm water on 1 3 . The nurse is assisting a postpartum woman out of bed to the bathroom for a sitz bath. Which of the following would be a priority? A ) Placing the call light within her reach B ) Teaching her how the sitz bath works C ) Telling her to use the sitz bath for 30 minutes D ) Cleaning the perineum with the peri-bottle 1 4 . A nurse is reviewing the medical record of a postpartum client. The nurse identifies that the woman is at risk for a postpartum infection based on which of the following? (Select all that apply.) A ) History of diabetes B ) Labor of 12 hours C ) Rupture of membranes for 16 hours D ) Hemoglobin level 10 mg/dL E ) Placenta requiring manual extraction 1 5 . A nurse is completing a postpartum assessment. Which finding would alert the nurse to a potential problem? A ) Lochia rubra with a fleshy odor B ) Respiratory rate of 16 breaths per minute C ) Temperature of 101 F D ) Pain rating of 2 on a scale from 0 to 10 1 6 . The nurse is assessing a postpartum clients lochia and finds that there is about 4-inch stain on the perineal pad. The nurse documents this finding as which of the following? a A ) Scant B ) Light C ) Moderate D Large ) 1 7 . When reviewing the medical record of a postpartum client, the nurse notes that the client has experienced a third-degree laceration. The nurse understands that the laceration extends to which of the following? A ) Superficial structures above the muscle B ) Through the perineal muscles C ) Through the anal sphincter muscle D ) Through the anterior rectal wall 1 8 . A nurse is observing a postpartum client interacting with her newborn and notes that the mother is engaging with the newborn in the en face position. Which of the following would the nurse be observing? A ) Mother placing the newborn next to bare breast. B ) Mother making eye-to-eye contact with the newborn C ) Mother gently stroking the newborns face D ) Mother holding the newborn upright at the shoulder 1 After teaching a group of students about risk factors associated with postpartum 9 hemorrhage, the instructor determines that the teaching was successful when the . students identify which of the following as a risk factor? (Select all that apply.) A ) Prolonged labor B ) Placenta previa C ) Null parity D ) Hydramnios E ) Labor augmentation 2 0 . A postpartum woman who is breast-feeding tells the nurse that she is experiencing nipple pain. Which of the following would be least appropriate for the nurse to suggest? A ) Use of a mild analgesic about 1 hour before breast-feeding B ) Application of expressed breast milk to the nipples C ) Application of glycerin-based gel to the nipples D ) Reinstruction about proper latching-on technique 2 A nurse is developing a teaching plan for a postpartum woman who is breast1 feeding about sexuality and contraception. Which of the following would the . nurse most likely include? (Select all that apply.) A Resumption of sexual intercourse about two weeks after delivery ) B Possible experience of fluctuations in sexual interest ) C Use of a water-based lubricant to ease vaginal discomfort ) D Use of combined hormonal contraceptives for the first three weeks ) E Possibility of increased breast sensitivity during sexual activity ) 2 After teaching a postpartum woman about breast-feeding, the nurse determines 2 that the teaching was successful when the woman states which of the following? . A I should notice a decrease in abdominal cramping during breast-feeding. ) B I should wash my hands before starting to breast-feed. ) C The baby can be awake or sleepy when I start to feed him. ) D The babys mouth will open up once I put him to my breast. ) 2 3 . A postpartum woman who is bottle-feeding her newborn asks the nurse, About how much should my newborn drink at each feeding? The nurse responds by saying that to feel satisfied, the newborn needs which amount at each feeding? A ) 1 to 2 ounces B ) 2 to 4 ounces C 4 to 6 ounces ) D 6 to 8 ounces ) 2 4 . A nurse is observing a postpartum woman and her partner interact with the their newborn. The nurse determines that the parents are developing parental attachment with their newborn when they demonstrate which of the following? (Select all that apply.) A ) Frequently ask for the newborn to be taken from the room B ) Identify common features between themselves and the newborn C ) Refer to the newborn as having a monkey-face D ) Make direct eye contact with the newborn E ) Refrain from checking out the newborns features 2 5 . After reviewing information about postpartum blues, a group of students demonstrate understanding when they state which of the following about this condition? A ) Postpartum blues is a long-term emotional disturbance. B ) Sleep usually helps to resolve the blues. C ) The mother loses contact with reality. D Extended psychotherapy is needed for treatment. ) Answer Key 1. A 2. C 3. C 4. B 5. B 6. C 7. B 8. C 9. C 10. B 11. A 12. C 13. A 14. A, D, E 15. C 16. B 17. C 18. B 19. B, D, E 20. A 21. B, C, E 22. B 23. B 24. B, D 25. B Chapter 17 Newborn Transitioning 1 . When explaining how a newborn adapts to extrauterine life, the nurse would describe which body systems as undergoing the most rapid changes? A ) Gastrointestinal and hepatic B ) Urinary and hematologic C ) Respiratory and cardiovascular D ) Neurological and integumentary 2 . A new mother reports that her newborn often spits up after feeding. Assessment reveals regurgitation. The nurse responds integrating understanding that this most likely is due to which of the following? A ) Placing the newborn prone after feeding B ) Limited ability of digestive enzymes C ) Underdeveloped pyloric sphincter D Relaxed cardiac sphincter ) 3 . After teaching a class about hepatic system adaptations after birth, the instructor determines that the teaching was successful when the class identifies which of the following as the process of changing bilirubin from a fat-soluble product to a water-soluble product? A ) Hemolysis B ) Conjugation C ) Jaundice D ) Hyperbilirubinemia 4 . Twenty minutes after birth, a baby begins to move his head from side to side, making eye contact with the mother, and pushes his tongue out several times. The nurse interprets this as indicating which of the following? A ) A good time to initiate breast-feeding B ) The period of decreased responsiveness preceding sleep C ) The need to be alert for gagging and vomiting D ) Evidence that the newborn is becoming chilled 5 . The nurse institutes measure to maintain thermoregulation based on the understanding that newborns have limited ability to regulate body temperature because they: A ) Have a smaller body surface compared to body mass B ) Lose more body heat when they sweat than adults C ) Have an abundant amount of subcutaneous fat all over D ) Are unable to shiver effectively to increase heat production 6 . A new mother is changing the diaper of her 20-hour-old newborn and asks why the stool is almost black. Which response by the nurse would be most appropriate? A ) You probably took iron during your pregnancy. B ) This is meconium stool, normal for a newborn. C ) Ill take a sample and check it for possible bleeding. D ) This is unusual and I need to report this. 7 A client expresses concern that her 2-hour-old newborn is sleepy and difficult to . awaken. The nurse explains that this behavior indicates which of the following? A Normal progression of behavior ) B ) Probable hypoglycemia C ) Physiological abnormality D ) Inadequate oxygenation 8 . After the birth of a newborn, which of the following would the nurse do first to assist in thermoregulation? A ) Dry the newborn thoroughly. B ) Put a hat on the newborns head. C ) Check the newborns temperature. D ) Wrap the newborn in a blanket. 9 Assessment of a newborn reveals rhythmic spontaneous movements. The nurse . interprets this as indicating: A Habituation ) B Motor maturity ) C Orientation ) D Social behaviors ) 1 0 . After teaching new parents about the sensory capabilities of their newborn, the nurse determines that the teaching was successful when they identify which sense as being the least mature? A ) Hearing B ) Touch C ) Taste D ) Vision 1 1 . The nurse places a warmed blanket on the scale when weighing a newborn. The nurse does so to minimize heat loss via which mechanism? A ) Evaporation B ) Conduction C ) Convection D ) Radiation 1 Which of the following would alert the nurse to the possibility of respiratory 2 distress in a newborn? . A Symmetrical chest movements ) B ) Periodic breathing C ) Respirations of 40 breaths/minute D ) Sternal retractions 1 3 . A nur The nu milk se is counseling a mother about the immunologic properties of breast milk. rse integrates knowledge of immunoglobulins, emphasizing that breast is a major source of which immunoglobulin? A ) IgA B ) IgG C ) IgM D ) IgE 1 4 . The nurse is teaching a group of students about the similarities and differences between newborn skin and adult skin. Which statement by the group indicates that additional teaching is needed? A ) The newborns skin and that of an adult are similar in thickness. B ) The lipid composition of the skin of a newborn and adult is about the same. C ) Skin development in the newborn is complete at birth. D The newborn has more fibrils connecting the dermis and epidermis. ) 1 5 . A nurse is developing a teaching plan for the parents of a newborn. When describing the neurologic development of a newborn to his parents, the nurse would explain that the development occurs in which fashion? A ) Head-to-toe B ) Lateral-to-medial C ) Outward-to-inward D ) Distal-to-caudal 1 6 . The nurse is assessing the respirations of several newborns. The nurse would notify the health care provider for the newborn with which respiratory rate at rest? A ) 38 breaths per minute B ) 46 breaths per minute C ) 54 breaths per minute D ) 68 breaths per minute 1 A new mother asks the nurse, Why has my baby lost weight since he was born? 7 The nurse integrates knowledge of which of the following when responding to . the new mother? A ) Insufficient calorie intake B ) Shift of water from extracellular space to intracellular space C ) Increase in stool passage D ) Overproduction of bilirubin 1 8 . The nurse observes the stool of a newborn who has begun to breast-feed. Which of the following would the nurse expect to find? A ) Greenish black, tarry stool B ) Yellowish-brown, seedy stool C ) Yellow-gold, stringy stool D ) Yellowish-green, pasty stool 1 9 . A nurse is assessing a newborn who is about 4 hours old. The nurse would expect this newborn to exhibit which of the following? (Select all that apply.) A ) Sleeping B ) Interest in environmental stimuli C ) Passage of meconium D ) Difficulty arousing the newborn E ) Spontaneous Moro reflexes 2 0 . A nurse is assessing a newborn and observes the newborn moving his head and eyes toward a loud sound. The nurse interprets this as which of the following? A ) Habituation B ) Motor maturity C ) Social behavior D ) Orientation 2 1 . A newborn is experiencing cold stress. Which of the following would the nurse expect to assess? (Select all that apply.) A ) Respiratory distress B ) Decreased oxygen needs C ) Hypoglycemia D Metabolic alkalosis ) E Jaundice ) 2 2 . A group of nursing students are reviewing the changes in the newborns lungs that must occur to maintain respiratory function. The students demonstrate understanding of this information when they identify which of the following as the first event? A ) Expansion of the lungs B ) Increased pulmonary blood flow C ) Initiation of respiratory movement D ) Redistribution of cardiac output 2 3 . A nurse is reviewing the laboratory test results of a newborn. Which result would the nurse identify as a cause for concern? A ) Hemoglobin 19 g/dL B ) Platelets 75,000/uL C ) White blood cells 20,000/mm3 D ) Hematocrit 52% 2 4 . A nursing instructor is preparing a class on newborn adaptations. When describing the change from fetal to newborn circulation, which of the following would the instructor most likely include? (Select all that apply.) A ) Decrease in right atrial pressure leads to closure of the foramen ovale. B ) Increase in oxygen levels leads to a decrease in systemic vascular resistance. C ) Onset of respirations leads to a decrease in pulmonary vascular resistance. D ) Increase in pressure in the left atrium results from increases in pulmonary blood flow. E ) Closure of the ductus venosus eventually forces closure of the ductus arteriosus. 2 5 . A nursing student is preparing a presentation on minimizing heat loss in the newborn. Which of the following would the student include as a measure to prevent heat loss through convection? A ) Placing a cap on a newborns head B ) Working inside an isolette as much as possible. C ) Placing the newborn skin-to-skin with the mother D ) Using a radiant warmer to transport a newborn 2 After teaching a group of nursing students about a neutral thermal environment, 6 the instructor determines that the teaching was successful when the students . identify which of the following as the newborns primary method of heat production? A Convection ) B Nonshivering thermogenesis ) C Cold stress ) D Bilirubin conjugation ) 2 7 . While observing the interaction between a newborn and his mother, the nurse notes the newborn nestling into the arms of his mother. The nurse identifies this behavior as which of the following? A ) Habituation B ) Self-quieting ability C ) Social behaviors D ) Orientation Answer Key 1. C 2. D 3. B 4. A 5. D 6. B 7. A 8. A 9. B 10. D 11. B 12. D 13. A 14. C 15. A 16. D 17. A 18. B 19. B, C 20. D 21. A, C, E 22. C 23. B 24. A, C, D, E 25. B 26. B 27. C Chapter 18 Nursing Management of the Newborn 1 . Prior to discharging a 24-hour-old newborn, the nurse assesses her respiratory status. Which of the following would the nurse expect to assess? A ) Respiratory rate 45, irregular B ) Costal breathing pattern C ) Nasal flaring, rate 65 D ) Crackles on auscultation 2 . The nurse encourages the mother of a healthy newborn to put the newborn to the breast immediately after birth for which reason? A ) To aid in maturing the newborns sucking reflex B ) To encourage the development of maternal antibodies C ) To facilitate maternalinfant bonding D ) To enhance the clearing of the newborns respiratory passages 3 . When making a home visit, the nurse observes a newborn sleeping on his in a bassinet. In one corner of the bassinet is a soft stuffed animal and at other end is a bulb syringe. The nurse determines that the mother needs additional teaching because of which of the following? back the A ) The newborn should not be sleeping on his back. B ) Stuffed animals should not be in areas where infants sleep. C ) The bulb syringe should not be kept in the bassinet. D ) This newborn should be sleeping in a crib. 4 . Assessment of a newborn reveals a heart rate of 180 beats/minute. To determine whether this finding is a common variation rather than a sign of distress, what else does the nurse need to know? A ) How many hours old is this newborn? B ) How long ago did this newborn eat? C ) What was the newborns birth weight? D ) Is acrocyanosis present? 5 . Just after delivery, a newborns axillary temperature is 94 C. What action would be most appropriate? A ) Assess the newborns gestational age. B ) Rewarm the newborn gradually. C ) Observe the newborn every hour. D ) Notify the physician if the temperature goes lower. 6 . The parents of a newborn become concerned when they notice that their baby seems to stop breathing for a few seconds. After confirming the parents findings by observing the newborn, which of the following actions would be most appropriate? A ) Notify the health care provider immediately. B ) Assess the newborn for signs of respiratory distress. C ) Reassure the parents that this is an expected pattern. D ) Tell the parents not to worry since his color is fine. 7 . When assessing a newborn 1 hour after birth, the nurse measures an axillary temperature of 95.8 F, an apical pulse of 114 beats/minute, and a respiratory rate of 60 breaths/minute. Which nursing diagnosis takes highest priority? A ) Hypothermia related to heat loss during birthing process B ) Impaired parenting related to addition of new family member C ) Risk for deficient fluid volume related to insensible fluid loss D ) Risk for infection related to transition to extrauterine environment 8 . The nurse places a nursery to achieve newborn with jaundice under the phototherapy lights in the which goal? A ) Prevent cold stress B ) Increase surfactant levels in the lungs C ) Promote respiratory stability D ) Decrease the serum bilirubin level 9 . The nurse completes the initial assessment of a newborn. Which finding would lead the nurse to suspect that the newborn is experiencing difficulty with oxygenation? A ) Respiratory rate of 54 breaths/minute B ) Abdominal breathing C ) Nasal flaring D ) Acrocyanosis 1 0 . During a physical assessment of a newborn, the nurse observes bluish markings across the newborns lower back. The nurse documents this finding as which of the following? A ) Milia B ) Mongolian spots C ) Stork bites D Birth trauma ) 1 1 . While making rounds in the nursery, the nurse sees a 6-hour-old baby girl gagging and turning bluish. Which of the following would the nurse do first? A ) Alert the physician stat and turn the newborn to her right side. B ) Administer oxygen via facial mask by positive pressure. C ) Lower the newborns head to stimulate crying. D ) Aspirate the oral and nasal pharynx with a bulb syringe. 1 2 . While performing a physical assessment of a newborn boy, the nurse notes diffuse edema of the soft tissues of his scalp that crosses suture lines. The nurse documents this finding as: A ) Molding B ) Microcephaly C ) Caput succedaneum D ) Cephalhematoma 1 Assessment of a newborn reveals uneven gluteal (buttocks. skin creases and a 3 clunk when Ortolanis maneuver is performed. Which of the following would the . nurse suspect? A ) Slipping of the periosteal joint B ) Developmental hip dysplasia C ) Normal newborn variation D ) Overriding of the pelvic bone 1 4 . The nurse strokes the lateral sole of the newborns foot from the heel to the ball of the foot when evaluating which reflex? A ) Babinski B ) Tonic neck C ) Stepping D ) Plantar grasp 1 5 . The nurse administers vitamin K intramuscularly to the newborn based on which of the following rationales? A ) Stop Rh sensitization B ) Increase erythopoiesis C Enhance bilirubin breakdown ) D Promote blood clotting ) 1 6 . The nurse is assessing the skin of a newborn and notes a rash on the newborns face, and chest. The rash consists of small papules and is scattered with no pattern. The nurse interprets this finding as which of the following? A ) Harlequin sign B ) Nevus flammeus C ) Erythema toxicum D ) Port wine stain 1 7 . After teaching a group of nursing students about variations in newborn head size and appearance, the instructor determines that the teaching was successful when the students identify which of the following as a normal variation? (Select all that apply.) A ) Cephalhematoma B ) Molding C ) Closed fontanels D ) Caput succedaneum E Posterior fontanel diameter 1.5 cm ) 1 8 . The nurse is assessing a newborns eyes. Which of the following would the nurse identify as normal? (Select all that apply.) A ) Slow blink response B ) Able to track object to midline C ) Transient deviation of the eyes D ) Involuntary repetitive eye movement E ) Absent red reflex 1 9 . Assessment of a newborns head circumference reveals that it is 34 cm. The nurse would suspect that this newborns chest circumference would be: A ) 30 cm B ) 32 cm C ) 34 cm D ) 36 cm 2 0 . The nurse is auscultating a newborns heart and places the stethoscope at the point of maximal impulse at which location? A ) Just superior to the nipple, at the midsternum B ) Lateral to the midclavicular line at the fourth intercostal space C ) At the fifth intercostal space to the left of the sternum D ) Directly adjacent to the sternum at the second intercostals space 2 1 . The nurse is inspecting the external genitalia of a male newborn. Which of the following would alert the nurse to a possible problem? A ) Limited rugae B ) Large scrotum C ) Palpable testes in scrotal sac D ) Absence of engorgement 2 2 . When assessing a newborns reflex the newborn turns toward the side nurse documents which reflex as es, the nurse strokes the newborns cheek and that was stroked and begins sucking. The being positive? A ) Palmar grasp reflex B ) Tonic neck reflex C ) Moro reflex D ) Rooting reflex 2 3 . A nurse is teaching new parents about bathing their newborn. The nurse determines that the teaching was successful when the parents state which of the following? A ) We can put a tiny bit of lotion on his skin and then rub it in gently. B ) We should avoid using any kind of baby powder. C ) We need to bathe him at least four to five times a week. D ) We should clean his eyes after washing his face and hair. 2 4 . A new mother who is breast-feeding her newborn asks the nurse, How will I know if my baby is drinking enough? Which response by the nurse would be most appropriate? A ) If he seems content after feeding, that should be a sign. B ) Make sure he drinks at least 5 minutes on each breast. C ) He should wet between 6 to 12 diapers each day. D If his lips are moist, then hes okay. ) 2 5 . A nurse is teaching postpartum client and her partner about caring for their newborns umbilical cord site. Which statement by the parents indicates a need for additional teaching? A ) We can put him in the tub to bathe him once the cord falls off and is healed. B ) The cord stump should change from brown to yellow. C ) Exposing the stump to the air helps it to dry. D ) We need to call the doctor if we notice a funny odor. 2 6 . While changing a female newborns diaper, the nurse observes a mucus-like, slightly bloody vaginal discharge. Which of the following would the nurse do next? A ) Document this as pseudomenstruation B ) Notify the practitioner immediately C ) Obtain a culture of the discharge D ) Inspect for engorgement 2 A nursing instructor is describing the advantages and disadvantages associated 7 with newborn circumcision to a group of nursing students. Which statement by . the students indicates effective teaching? A ) Sexually transmitted infections are more common in circumcised males. B ) The rate of penile cancer is less for circumcised males. C ) Urinary tract infections are more easily treated in circumcised males. D ) Circumcision is a risk factor for acquiring HIV infection. 2 8 . A newborn is scheduled to undergo a screening test for phenylketonuria (PKU). The nurse prepares to obtain the blood sample from the newborns: A ) Finger B ) Heel C ) Scalp vein D ) Umbilical vein 2 9 . Assessment of a newborn reveals transient tachypnea. The nurse reviews the newborns medical record. Which of the following would the nurse be least likely to identify as a risk factor for this condition? A ) Cesarean birth B ) Shortened labor C Central nervous system depressant during labor ) D Maternal asthma ) 3 0 . A nurse is providing teaching to a new mother about her newborns nutritional needs. Which of the following would the nurse be most likely to include in the teaching? (Select all that apply.) A ) Supplementing with iron if the woman is breast-feeding B ) Providing supplemental water intake with feedings C ) Feeding the newborn every 2 to 4 hours during the day D ) Burping the newborns frequently throughout each feeding E ) Using feeding time for promoting closeness Answer Key 1. A 2. C 3. B 4. A 5. B 6. B 7. A 8. D 9. C 10. B 11. D 12. C 13. B 14. A 15. D 16. C 17. A, B, D 18. B, C, D 19. B 20. B 21. A 22. D 23. B 24. C 25. B 26. A 27. B 28. B 29. B 30. C, D, E Chapter 19 Nursing Management of Pregnancy at Risk: Pregnancy-Related Complications 1 . After teaching a woman who has had an evacuation for a hydatidiform mole (molar pregnancy. about her condition, which of the following statements indicates that the nurses teaching was successful? A ) I will be sure to avoid getting pregnant for at least 1 year. B ) My intake of iron will have to be closely monitored for 6 months. C ) My blood pressure will continue to be increased for about 6 more months. D ) I wont use my birth control pills for at least a year or two. 2 . Which of the following findings on a prenatal visit at 10 weeks might lead the nurse to suspect a hydatidiform mole? A ) Complaint of frequent mild nausea B ) Blood pressure of 120/84 mm Hg C ) History of bright red spotting 6 weeks ago D ) Fundal height measurement of 18 cm 3 A client is diagnosed with gestational hypertension and is receiving magnesium . sulfate. Which finding would the nurse interpret as indicating a therapeutic level of medication? A Urinary output of 20 mL per hour ) B Respiratory rate of 10 breaths/minute ) C Deep tendons reflexes 2+ ) D Difficulty in arousing ) 4 Upon entering the room of a client who has had a spontaneous abortion, the . nurse observes the client crying. Which of the following responses by the nurse would be most appropriate? A Why are you crying? ) B Will a pill help your pain? ) C Im sorry you lost your baby. ) D A baby still wasnt formed in your uterus. ) 5 Which of the following data on a clients health history would the nurse identify . as contributing to the clients risk for an ectopic pregnancy? A Use of oral contraceptives for 5 years ) B Ovarian cyst 2 years ago ) C Recurrent pelvic infections ) D Heavy, irregular menses ) 6 In a woman who is suspected of having a ruptured ectopic pregnancy, the nurse . would expect to assess for which of the following as a priority? A Hemorrhage ) B Jaundice ) C Edema ) D Infection ) 7 Which of the following findings would the nurse interpret as suggesting a . diagnosis of gestational trophoblastic disease? A Elevated hCG levels, enlarged abdomen, quickening ) B Vaginal bleeding, absence of FHR, decreased hPL levels ) C Visible fetal skeleton on ultrasound, absence of quickening, enlarged abdomen ) D Gestational hypertension, hyperemesis gravidarum, absence of FHR ) 8 It is determined that a clients blood Rh is negative and her partners is positive. . To help prevent Rh isoimmunization, the nurse anticipates that the client will receive RhoGAM at which time? A ) At 34 weeks gestation and immediately before discharge B ) 24 hours before delivery and 24 hours after delivery C ) In the first trimester and within 2 hours of delivery D ) At 28 weeks gestation and again within 72 hours after delivery 9 . The nurse is developing a plan of care for a woman who is pregnant with twins. The nurse includes interventions focusing on which of the following because of the womans increased risk? A ) Oligohydramnios B ) Preeclampsia C ) Post-term labor D ) Chorioamnionitis 1 0 . A woman hospitalized with severe preeclampsia is being treated with hydralazine to control blood pressure. Which of the following would the lead the nurse to suspect that the client is having an adverse effect associated with this drug? A ) Gastrointestinal bleeding B ) Blurred vision C Tachycardia ) D Sweating ) 1 1 . After reviewing a clients history, which factor would the nurse identify as placing her at risk for gestational hypertension? A ) Mother had gestational hypertension during pregnancy. B ) Client has a twin sister. C ) Sister-in-law had gestational hypertension. D ) This is the clients second pregnancy. 1 2 . A client with hyperemesis gravidarum is admitted to the facility after being cared for at home without success. Which of the following would the nurse expect to include in the clients plan of care? A ) Clear liquid diet B ) Total parenteral nutrition C ) Nothing by mouth D ) Administration of labetalol 1 3 . The nurse is reviewing the laboratory test results of a pregnant client. Which one of the following findings would alert the nurse to the development of HELLP syndrome? A ) Hyperglycemia B ) Elevated platelet count C ) Leukocytosis D ) Elevated liver enzymes 1 4 . Which of the following would the nurse have readily available for a client who is receiving magnesium sulfate to treat severe preeclampsia? A ) Calcium gluconate B ) Potassium chloride C ) Ferrous sulfate D ) Calcium carbonate 1 Which assessment finding would lead the nurse to suspect infection as the 5 cause of a clients PROM? . A Yellow-green fluid ) B ) Blue color on Nitrazine testing C ) Ferning D ) Foul odor 1 6 . While assessing a pregnant woman, the nurse suspects that the client may be at risk for hydramnios based on which of the following? (Select all that apply.) A ) History of diabetes B ) Complaints of shortness of breath C ) Identifiable fetal parts on abdominal palpation D ) Difficulty obtaining fetal heart rate E ) Fundal height below that for expected gestataional age 1 7 . After teaching a group of nursing students about the possible causes of spontaneous abortion, the instructor determines that the teaching was successful when the students identify which of the following as the most common cause of first trimester abortions? A ) Maternal disease B ) Cervical insufficiency C Fetal genetic abnormalities ) D Uterine fibroids ) 1 8 . A pregnant woman is admitted with premature rupture of the membranes. The nurse is assessing the woman closely for possible infection. Which of the following would lead the nurse to suspect that the woman is developing an infection? (Select all that apply.) A ) Fetal bradycardia B ) Abdominal tenderness C ) Elevated maternal pulse rate D ) Decreased C-reactive protein levels E ) Cloudy malodorous fluid 1 9 . A nurse is teaching a pregnant woman with preterm premature rupture of membranes who is about to be discharged home about caring for herself. Which statement by the woman indicates a need for additional teaching? A ) I need to keep a close eye on how active my baby is each day. B ) I need to call my doctor if my temperature increases. C ) Its okay for my husband and me to have sexual intercourse. D I can shower but I shouldnt take a tub bath. ) 2 0 . A nurse is assessing a pregnant woman with gestational hypertension. Which of the following would lead the nurse to suspect that the client has developed severe preeclampsia? A ) Urine protein 300 mg/24 hours B ) Blood pressure 150/96 mm Hg C ) Mild facial edema D ) Hyperreflexia 2 1 . A nurse suspects that a pregnant client may be experiencing abruption placenta based on assessment of which of the following? (Select all that apply.) A ) Dark red vaginal bleeding B ) Insidious onset C ) Absence of pain D ) Rigid uterus E ) Absent fetal heart tones 2 2 . The health care provider orders PGE2 for a woman to help evacuate the uterus following a spontaneous abortion. Which of the following would be most important for the nurse to do? A ) Use clean technique to administer the drug. B ) Keep the gel cool until ready to use. C ) Maintain the client for hour after administration. D ) Administer intramuscularly into the deltoid area. 2 3 . A nursing student is reviewing an article about preterm premature rupture of membranes. Which of the following would the student expect to find as factor placing a woman at high risk for this condition? (Select all that apply.) A ) High body mass index B ) Urinary tract infection C ) Low socioeconomic status D ) Single gestations E ) Smoking 2 A woman with placenta previa is being treated with expectant management. The 4 woman and fetus are stable. The nurse is assessing the woman for possible . discharge home. Which statement by the woman would suggest to the nurse that home care might be inappropriate? A My mother lives next door and can drive me here if necessary. ) B I have a toddler and preschooler at home who need my attention. ) C I know to call my health care provider right away if I start to bleed again. ) D I realize the importance of following the instructions for my care. ) 2 A woman with hyperemesis gravidarum asks the nurse about suggestions to 5 minimize nausea and vomiting. Which suggestion would be most appropriate . for the nurse to make? A Make sure that anything around your waist is quite snug. ) B Try to eat three large meals a day with less snacking. ) C Drink fluids in between meals rather than with meals. ) D Lie down for about an hour after you eat ) 2 6 . A woman with gestational hypertension experiences a seizure. Which of the following would be the priority? A ) Fluid replacement B ) Oxygenation C Control of hypertension ) D Delivery of the fetus ) 2 7 . A woman is preeclampsia Which level receiving magnesium sulfate as part of her treatment for severe . The nurse is monitoring the womans serum magnesium levels. would the nurse identify as therapeutic? A ) 3.3 mEq/L B ) 6.1 mEq/L C ) 8.4 mEq/L D ) 10.8 mEq/L Answer Key 1. A 2. D 3. C 4. C 5. C 6. A 7. D 8. D 9. B 10. C 11. A 12. C 13. D 14. A 15. D 16. A, B, D 17. C 18. B, C, E 19. C 20. D 21. A, D, E 22. C 23. B, C, E 24. B 25. C 26. B 27. B Chapter 20 Nursing Management Pregnancy at Risk: Health Conditions & Vulnerable Populations 1 The nurse is teaching a pregnant woman with type 1 diabetes about her diet . during pregnancy. Which client statement indicates that the nurses teaching was successful? A Ill basically follow the same diet that I was following before I became pregnant. ) B Because I need extra protein, Ill have to increase my intake of milk and meat. ) C Pregnancy affects insulin production, so Ill need to make adjustments in my diet. ) D Ill adjust my diet and insulin based on the results of my urine tests for glucose. ) 2 A nurse is developing a program for pregnant women with diabetes about . reducing complications. Which factor would the nurse identify as being most important in helping to reduce the maternal/fetal/neonatal complications associated with pregnancy and diabetes? A Stability of the womans emotional and psychological status ) B Degree of glycemic control achieved during the pregnancy ) C Evaluation of retinopathy by an ophthalmologist ) D Blood urea nitrogen level (BUN. within normal limits ) 3 . Because a pregnant clients diabetes has been poorly controlled throughout her pregnancy, the nurse would be alert for which of the following in the neonate at birth? A ) Macrosomia B ) Hyperglycemia C Low birth weight ) D Hypobilirubinemia ) 4 . A woman with diabetes is considering becoming pregnant. She asks the nurse whether she will be able to take oral hypoglycemics when she is pregnant. The nurses response is based on the understanding that oral hypoglycemics: A ) Can be used as long as they control serum glucose levels B ) Can be taken until the degeneration of the placenta occurs C ) Are usually suggested primarily for women who develop gestational diabetes D ) Show promising results but more studies are needed to confirm their effectiveness 5 . A 10-week pregnant woman with diabetes has a glycosylated hemoglobin (HbA1C. level of 13%. At this time the nurse should be most concerned about which of the following possible fetal outcomes? A ) Congenital anomalies B ) Incompetent cervix C ) Placenta previa D ) Abruptio placentae 6 . After teaching a group of students about the use of antiretroviral agents in pregnant women who are HIV-positive, the instructor determines that the teaching was successful when the group identifies which of the following as the underlying rationale? A ) Reduction in viral loads in the blood B ) Treatment of opportunistic infections C ) Adjunct therapy to radiation and chemotherapy D ) Can cure acute HIV/AIDS infections 7 . Assessment of a pregnant woman and her fetus reveals tachycardia and hypertension. There is also evidence suggesting vasoconstriction. The nurse would question the woman about use of which substance? A ) Marijuana B ) Alcohol C ) Heroin D ) Cocaine 8 When teaching a class of pregnant women about the effects of substance abuse . during pregnancy, which of the following would the nurse most likely include? A Low-birth-weight infants ) B ) Excessive weight gain C ) Higher pain tolerance D ) Longer gestational periods 9 A client who is HIV-positive is in her second trimester and remains . asymptomatic. She voices concern about her newborns risk for the infection. Which of the following statements by the nurse would be most appropriate? A Youll probably have a cesarean birth to prevent exposing your newborn. ) B Antibodies cross the placenta and provide immunity to the newborn. ) C Wait until after the infant is born and then something can be done. ) D Antiretroviral medications are available to help reduce the risk of transmission. ) 1 0 . When assessing a pregnant woman with heart disease throughout the antepartal period, the nurse would be especially alert for signs and symptoms of cardiac decompensation at which time? A ) 16 to 20 weeks gestation B ) 20 to 24 weeks gestation C ) 24 to 28 weeks gestation D 28 to 32 weeks gestation ) 1 1 . When preparing a schedule of follow-up visits for a pregnant woman with chronic hypertension, which of the following would be most appropriate? A ) Monthly visits until 32 weeks, then bi-monthly visits B ) Bi-monthly visits until 28 weeks, then weekly visits C ) Monthly visits until 20 weeks, then bi-monthly visits D ) Bi-monthly visits until 36 weeks, then weekly visits 1 2 . Which medication would the nurse question if ordered to control a pregnant womans asthma? A ) Budesonide B ) Albuterol C ) Salmeterol D ) Oral prednisone 1 After teaching a pregnant woman with iron deficiency anemia about her 3 prescribed iron supplement, which statement indicates successful teaching? . A ) I should take my iron with milk. B ) I should avoid drinking orange juice. C ) I need to eat foods high in fiber. D ) Ill call the doctor if my stool is black and tarry. 1 4 . A nurse is providing care to several pregnant women at the clinic. The nurse would screen for group B streptococcus infection in a client at: A ) 16 weeks gestation B ) 28 week gestation C ) 32 weeks gestation D ) 36 weeks gestation 1 5 . The nurse is assessing a newborn of a woman who is suspected of abusing alcohol. Which newborn finding would provide additional evidence to support this suspicion? A ) Wide large eyes B ) Thin upper lip C Protruding jaw ) D Elongated nose ) 1 6 . After teaching a group of nursing students about the impact of pregnancy on the older woman, the instructor determines that the teaching was successful when the students state which of the following? A ) The majority of women who become pregnant over age 35 experience complications. B ) Women over the age of 35 who become pregnant require a specialized type of assessment. C ) Women over age 35 and are pregnant have an increased risk for spontaneous abortions. D ) Women over age 35 are more likely to have substance abuse problems. 1 A group of students are reviewing information about sexually transmitted 7 infections and their effect on pregnancy. The students demonstrate . understanding of the information when they identify which infection as being responsible for ophthalmia neonatorum? A Syphilis ) B Gonorrhea ) C Chlamydia ) D HPV ) 1 8 . A nurse is preparing a presentation for a group of young adult pregnant women about common infections and their effect on pregnancy. When describing the infections, which infection would the nurse include as the most common congenital and perinatal viral infection in the world? A ) Rubella B ) Hepatitis B C ) Cytomegalovirus D ) Parvovirus B19 1 9 . A pregnant woman asks the nurse, Im a big coffee drinker. Will the caffeine in my coffee hurt my baby? Which response by the nurse would be most appropriate? A ) The caffeine in coffee has been linked to birth defects. B ) Caffeine has been shown to cause growth restriction in the fetus. C ) Caffeine is a stimulant and needs to be avoided completely. D ) If you keep your intake to less than 300 mg/day, you should be okay. 2 A neonate born to a mother who was abusing heroin is exhibiting signs and 0 symptoms of withdrawal. Which of the following would the nurse assess? . (Select all that apply.) A Low whimpering cry ) B Hypertonicity ) C Lethargy ) D Excessive sneezing ) E Overly vigorous sucking ) F Tremors ) 2 A nurse has been invited to speak at a local high school about adolescent 1 pregnancy. When developing the presentation, the nurse would incorporate . information related to which of the following? (Select all that apply.) A Peer pressure to become sexually active ) B Rise in teen birth rates over the years. ) C Latinas as having the highest teen birth rate ) D Loss of self-esteem as a major impact ) E Majority of teen pregnancies in the 1517-year-old age group ) 2 A nurse is counseling a pregnant woman with rheumatoid arthritis about 2 medications that can be used during pregnancy. Which drug would the nurse . emphasize as being contraindicated at this time? A Hydroxychloroquine ) B Nonsteroidal anti-inflammatory drug ) C Glucocorticoid ) D Methotrexate ) 2 A nurse is preparing a teaching program for a group of pregnant women about 3 preventing infections during pregnancy. When describing measures for . preventing cytomegalovirus infection, which of the following would the nurse most likely include? A Frequent handwashing ) B Immunization ) C Prenatal screening ) D Antibody titer screening ) 2 4 . A pregnant woman tests positive for HBV. Which of the following would the nurse expect to administer? A ) HBV immune globulin B ) HBV vaccine C Acylcovir ) D Valacyclovir ) 2 After teaching a pregnant woman with iron deficiency anemia about nutrition, 5 the nurse determines that the teaching was successful when the woman . identifies which of the following as being good sources of iron in her diet? (Select all that apply.) A Dried fruits ) B Peanut butter ) C Meats ) D Milk ) E White bread ) 2 6 . A group of nursing students are preparing a presentation for their class about measures to prevent toxoplasmosis. Which of the following would the students be least likely to include? Select all that apply. A ) Washing raw fruits and vegetables before eating them B ) Cooking all meat to an internal temperature of 140 F C ) Wearing gardening gloves when working in the soil D Avoiding contact with a cats litter box. ) 2 7 . A pregnant woman wit blood glucose level. Wh which result indicates h gestational diabetes comes to the clinic for a fasting en reviewing the results, the nurse determines that good glucose control? A ) 90 mg/dL B ) 100 mg/dL C ) 110 mg /dL D ) 120 mg/dL Answer Key 1. C 2. B 3. A 4. D 5. A 6. A 7. D 8. A 9. D 10. D 11. B 12. D 13. C 14. D 15. B 16. C 17. B 18. C 19. D 20. B, D, F 21. A, C, D 22. D 23. A 24. A 25. A, B, C 26. B 27. A Chapter 21 Nursing Management of Labor and Birth at Risk 1 . After spontaneous rupture of membranes, the nurse notices a prolapsed cord. The nurse immediately places the woman in which position? A ) Supine B ) Side-lying C Sitting ) D Kneechest ) 2 . A primigravida whose labor was initially progressing normally is now experiencing a decrease in the frequency and intensity of her contractions. The nurse would assess the woman for which condition? A ) A low-lying placenta B ) Fetopelvic disproportion C ) Contraction ring D ) Uterine bleeding 3 . The nurse would be alert for possible placental abruption during labor when assessment reveals which of the following? A ) Macrosomia B ) Gestational hypertension C ) Gestational diabetes D ) Low parity 4 . Assessment of a woman in labor who is experiencing hypertonic uterine dysfunction would reveal contractions that are: A ) Well coordinated B ) Poor in quality C ) Rapidly occurring D ) Erratic 5 . A woman in labor is experiencing hypotonic uterine dysfunction. Assessment reveals no fetopelvic disproportion. Which group of medications would the nurse expect to administer? A ) Sedatives B ) Tocolytics C ) Oxytocins D ) Corticosteroids 6 The fetus of a woman in labor is determined to be in persistent occiput posterior . position. Which of the following would the nurse identify as the priority intervention? A Position changes ) B ) Pain relief measures C ) Immediate cesarean birth D ) Oxytocin administration 7 A woman gave birth to a newborn via vaginal delivery with the use of a vacuum . extractor. The nurse would be alert for which of the following in the newborn? A Asphyxia ) B Clavicular fracture ) C Caput succedaneum ) D Central nervous system injury ) 8 A pregnant client undergoing labor induction is receiving an oxytocin infusion. . Which of the following findings would require immediate intervention? A Fetal heart rate of 150 beats/minute ) B Contractions every 2 minutes, lasting 45 seconds ) C Uterine resting tone of 14 mm Hg ) D Urine output of 20 mL/hour ) 9 A woman with a history of crack cocaine abuse is admitted to the labor and birth . area. While caring for the client, the nurse notes a sudden onset of fetal bradycardia. Inspection of the abdomen reveals an irregular wall contour. The client also complains of acute abdominal pain that is continuous. Which of the following would the nurse suspect? A Amniotic fluid embolism ) B Shoulder dystocia ) C Uterine rupture ) D Umbilical cord prolapse ) 1 0 . When assessing several women for possible VBAC, which woman would the nurse identify as being the best candidate? A ) One who has undergone a previous myomectomy B ) One who had a previous cesarean birth via a low transverse incision C ) One who has a history of a contracted pelvis D ) One who has a vertical incision from a previous cesarean birth 1 A woman is to undergo an amnioinfusion. Which statement would be most 1 appropriate to include when teaching the woman about this procedure? . A Youll need to stay in bed while youre having this procedure. ) B Well give you an analgesic to help reduce the pain. ) C After the infusion, youll be scheduled for a cesarean birth. ) D A suction cup is placed on your babys head to help bring it out. ) 1 Which finding would indicate to the nurse that a womans cervix is ripe in 2 preparation for labor induction? . A Posterior position ) B Firm ) C Closed ) D Shortened ) 1 3 . A woman with preterm labor is receiving magnesium sulfate. Which finding would require the nurse to intervene immediately? A ) Respiratory rate of 16 breaths per minute B ) Diminished deep tendon reflexes C Urine output of 45 mL/hour ) D Alert level of consciousness ) 1 4 . A woman who is 42 weeks pregnant comes to the clinic. Which of the following would be most important? A ) Determining an accurate gestational age B ) Asking her about the occurrence of contractions C ) Checking for spontaneous rupture of membranes D ) Measuring the height of the fundus 1 5 . After teaching a couple about what to expect with their planned cesarean birth, which statement indicates the need for additional teaching? A ) Holding a pillow against my incision will help me when I cough. B ) Im going to have to wait a few days before I can start breast-feeding. C ) I guess the nurses will be getting me up and out of bed rather quickly. D ) Ill probably have a tube in my bladder for about 24 hours or so. 1 6 . The nur for labo having se is providing care to several pregnant women who may be scheduled r induction. The nurse identifies the woman with which Bishop score as the best chance for a successful induction and vaginal birth? A ) 11 B ) 8 C ) 6 D ) 3 1 7 . After teaching a group of nursing students about risk factors associated with dystocia, the instructor determines that the teaching was successful when the students identify which of the following as increasing the risk? (Select all that apply.) A ) Pudendal block anesthetic use B ) Multiparity C ) Short maternal stature D ) Maternal age over 35 E ) Breech fetal presentation 1 A nurse is preparing an inservice education program for a group of nurses about 8 dystocia involving problems with the passenger. Which of the following would . the nurse most likely include as the most common problem? A Macrosomia ) B Breech presentation ) C Persistent occiput posterior position ) D Multifetal pregnancy ) 1 After teaching a group of nursing students about tocolytic therapy, the instructor 9 determines that the teaching was successful when they identify which drug as . being used for tocolysis? (Select all that apply.) A Nifedipine ) B Terbutaline ) C Dinoprostone ) D Misoprostol ) E Indomethacin ) 2 A nurse is assessing a pregnant woman who has come to the clinic. The woman 0 reports that she feels some heaviness in her thighs since yesterday. The nurse . suspects that the woman may be experiencing preterm labor based on which additional assessment findings? A Dull low backache ) B ) Malodorous vaginal discharge C ) Dysuria D ) Constipation 2 A nurse is teaching a pregnant woman at risk for preterm labor about what to do 1 if she experiences signs and symptoms. The nurse determines that the teaching . was successful when the woman states that if she experiences any symptoms, she will do which of the following? A Ill sit down to rest for 30 minutes. ) B Ill try to move my bowels. ) C Ill lie down with my legs raised. ) D Ill drink several glasses of water. ) 2 A nurse is describing the risks associated with prolonged pregnancies as part of 2 an inservice presentation. Which of the following would the nurse be least likely . to incorporate in the discussion as an underlying reason for problems in the fetus? A Aging of the placenta ) B Increased amniotic fluid volume ) C Meconium aspiration ) D Cord compression ) 2 A group of nursing students are reviewing information about methods used for 3 cervical ripening. The students demonstrate understanding of the information . when they identify which of the following as a mechanical method? A Herbal agents ) B Laminaria ) C Membrane stripping ) D Amniotomy ) 2 The nurse notifies the obstetrical team immediately because the nurse suspects 4 that the pregnant woman may be exhibiting signs and symptoms of amniotic . fluid embolism. Which findings would the nurse most likely assess? (Select all that apply.) A Significant difficulty breathing ) B Hypertension ) C Tachycardia ) D Pulmonary edema ) E Bleeding with bruising ) 2 A group of nursing students are reviewing information about cesarean birth. The 5 students demonstrate understanding of the information when they identify . which of the following as an appropriate indication? (Select all that apply.. A Active genital herpes infection ) B Placenta previa ) C Previous cesarean birth ) D Prolonged labor ) E Fetal distress ) 2 A pregnant woman is receiving misoprostol to ripen her cervix and induce 6 labor. The nurse assesses the woman closely for which of the following? . A Uterine hyperstimulation ) B Headache ) C Blurred vision ) D Hypotension ) Answer Key 1. D 2. B 3. B 4. D 5. C 6. B 7. C 8. D 9. C 10. B 11. A 12. D 13. B 14. A 15. B 16. A 17. C, D, E 18. C 19. A, B, E 20. C 21. D 22. B 23. B 24. A, C, D, E 25. A, B, C, E 26. A Chapter 22 Nursing Management of the Postpartum Woman at Risk 1 . Review of a primiparous womans labor and birth record reveals a prolonged second stage of labor and extended time in the stirrups. Based on an interpretation of these findings, the nurse would be especially alert for which of the following? A ) Retained placental fragments B ) Hypertension C ) Thrombophlebitis D ) Uterine subinvolution 2 . As part of an inservice program, a nurse is describing a transient, self-limiting mood disorder that affects mothers after childbirth. The nurse correctly identifies this as postpartum: A ) Depression B ) Psychosis C ) Bipolar disorder D ) Blues 3 . A woman who is 2 weeks postpartum calls the clinic and says, My left breast hurts. After further assessment on the phone, the nurse suspects the woman has mastitis. In addition to pain, the nurse would assess for which of the following? A ) An inverted nipple on the affected breast B ) No breast milk in the affected breast C ) An ecchymotic area on the affected breast D ) Hardening of an area in the affected breast 4 . A group of students are reviewing the causes of postpartum hemorrhage. The students demonstrate understanding of the information when they identify which of the following as the most common cause? A ) Labor augmentation B ) Uterine atony C ) Cervical or vaginal lacerations D ) Uterine inversion 5 After presenting a class on measures to prevent postpartum hemorrhage, the . presenter determines that the teaching was successful when the class states which of the following as an important measure to prevent postpartum hemorrhage due to retained placental fragments? A Administering broad-spectrum antibiotics ) B ) Inspecting the placenta after delivery for intactness C ) Manually removing the placenta at delivery D ) Applying pressure to the umbilical cord to remove the placenta 6 A multipara client develops thrombophlebitis after delivery. Which of the . following would alert the nurse to the need for immediate intervention? A Dyspnea, diaphoresis, hypotension, and chest pain ) B Dyspnea, bradycardia, hypertension, and confusion ) C Weakness, anorexia, change in level of consciousness, and coma ) D Pallor, tachycardia, seizures, and jaundice ) 7 A client experienced prolonged labor with prolonged premature rupture of . membranes. The nurse would be alert for which of the following in the mother and the newborn? A Infection ) B Hemorrhage ) C Trauma ) D Hypovolemia ) 8 When assessing the postpartum woman, the nurse uses indicators other than . pulse rate and blood pressure for postpartum hemorrhage based on the knowledge that: A These measurements may not change until after the blood loss is large ) B The bodys compensatory mechanisms activate and prevent any changes ) C They relate more to change in condition than to the amount of blood lost ) D Maternal anxiety adversely affects these vital signs ) 9 The nurse is assessing a woman with abruption placentae who has just given . birth. The nurse would be alert for which of the following? A Severe uterine pain ) B Board-like abdomen ) C Appearance of petechiae ) D Inversion of the uterus ) 1 A nurse is assessing a postpartum woman. Which finding would cause the nurse 0 to be most concerned? . A Leg pain on ambulation with mild ankle edema ) B ) Calf pain with dorsiflexion of the foot. C ) Perineal pain with swelling along the episiotomy D ) Sharp stabbing chest pain with shortness of breath 1 A woman experiencing postpartum hemorrhage is ordered to receive a 1 uterotonic agent. Which of the following would the nurse least expect to . administer? A Oxytocin ) B Methylergonovine ) C Carboprost ) D Terbutaline ) 1 2 . Which of the following would be most appropriate when massaging a womans fundus? A ) Place the hands on the sides of the abdomen to grasp the uterus. B ) Use an up-and-down motion to massage the uterus. C ) Wait until the uterus is firm to express clots. D Continue massaging the uterus for at least 5 minutes. ) 1 3 . After teaching a woman with a postpartum infection about care after discharge, which client statement indicates the need for additional teaching? A ) I need to call my doctor if my temperature goes above 100.4 F. B ) When I put on a new pad, Ill start at the back and go forward. C ) If I have chills or my discharge has a strange odor, Ill call my doctor. D ) Ill point the spray of the peribottle so the water flows front to back. 1 4 . A nurse suspects that a postpartum client is experiencing postpartum psychosis. Which of the following would most likely lead the nurse to suspect this condition? A ) Delirium B ) Feelings of anxiety C ) Sadness D ) Insomnia 1 A postpartum woman is diagnosed with metritis. The nurse interprets this as an 5 infection involving which of the following? (Select all that apply.) . A Endometrium ) B Decidua ) C Myometrium ) D Broad ligament ) E Ovaries ) F Fallopian tubes ) 1 6 . A group of nursing causes. The studen identify which of students are reviewing information about mastitis and its ts demonstrate understanding of the information when they the following as the most common cause? A ) E. coli B ) S. aureus C ) Proteus D ) Klebsiella 1 A home health care nurse is assessing a postpartum woman who was discharged 7 2 days ago. The woman tells the nurse that she has a low-grade fever and feels . lousy. Which of the following findings would lead the nurse to suspect metritis? (Select all that apply.) A Lower abdominal tenderness ) B Urgency ) C Flank pain ) D Breast tenderness ) E Anorexia ) 1 8 . A postpartum client comes to the clinic for her routine 6-week visit. The nurse assesses the client and suspects that she is experiencing subinvolution based on which of the following? A ) Nonpalpable fundus B ) Moderate lochia serosa C ) Bruising on arms and legs D ) Fever 1 Assessment of a postpartum client reveals a firm uterus with bright-red bleeding 9 and a localized bluish bulging area just under the skin at the perineum. The . woman also is complaining of significant pelvic pain and is experiencing problems with voiding. The nurse suspects which of the following? A Hematoma ) B ) Laceration C ) Bladder distention D ) Uterine atony 2 A postpartum woman is ordered to receive oxytocin to stimulate the uterus to 0 contract. Which of the following would be most important for the nurse to do? . A Administer the drug as an IV bolus injection. ) B Give as a vaginal or rectal suppository. ) C Piggyback the IV infusion into a primary line. ) D Withhold the drug if the woman is hypertensive. ) 2 Assessment of a postpartum woman experiencing postpartum hemorrhage 1 reveals mild shock. Which of the following would the nurse expect to assess? . (Select all that apply.) A Diaphoresis ) B Tachycardia ) C Oliguria ) D Cool extremities ) E Confusion ) 2 A group of students are reviewing risk factors associated with postpartum 2 hemorrhage. The students demonstrate understanding of the information when . they identify which of the following as associated with uterine tone? (Select all that apply.) A Rapid labor ) B Retained blood clots ) C Hydramnios ) D Operative birth ) E Fetal malpostion ) 2 A nurse is massaging a postpartum clients fundus and places the nondominant 3 hand on the area above the symphysis pubis based on the understanding that this . action: A Determines that the procedure is effective ) B Helps support the lower uterine segment ) C Aids in expressing accumulated clots ) D Prevents uterine muscle fatigue ) 2 A nurse is developing a plan of care for a woman who is at risk for 4 thromboembolism. Which of the following would the nurse include as the most . cost-effective method for prevention? A Prophylactic heparin administration ) B Compression stocking ) C Early ambulation ) D Warm compresses ) 2 A postpartum woman who developed deep vein thrombosis is being discharged 5 on anticoagulant therapy. After teaching the woman about this treatment, the . nurse determines that additional teaching is needed when the woman states which of the following? A I will use a soft toothbrush to brush my teeth. ) B I can take ibuprofen if I have any pain. ) C I need to avoid drinking any alcohol. ) D I will call my health care provider if my stools are black and tarry. ) 2 The nurse is developing a discharge teaching plan for a postpartum woman who 6 has developed a postpartum infection. Which of the following would the nurse . most likely include in this teaching plan? (Select all that apply.) A Taking the prescribed antibiotic until it is finished ) B Checking temperature once a week ) C Washing hands before and after perineal care ) D Handling perineal pads by the edges ) E Directing peribottle to flow from back to front ) 2 7 . A nurse is assessing a postpartum client who is at home. Which statement by the client would lead the nurse to suspect that the client may be developing postpartum depression? A ) I just feel so overwhelmed and tired. B ) Im feeling so guilty and worthless lately. C ) Its strange, one minute Im happy, the next Im sad. D ) I keep hearing voices telling me to take my baby to the river. Answer Key 1. C 2. D 3. D 4. B 5. B 6. A 7. A 8. A 9. C 10. D 11. D 12. C 13. B 14. A 15. A, B, C 16. B 17. A, E 18. B 19. A 20. C 21. A, D 22. A, C 23. B 24. C 25. B 26. A, C, D 27. B Chapter 23 Nursing Care of the Newborn with Special Needs 1 . The nurse is teaching a group of students about the differences between a fullterm newborn and a preterm newborn. The nurse determines that the teaching is effective when the students state that the preterm newborn has: A ) Fewer visible blood vessels through the skin B ) More subcutaneous fat in the neck and abdomen C ) Well-developed flexor muscles in the extremities D ) Greater surface area in proportion to weight 2 . When assessing a postterm newborn, which of the following would the nurse correlate with this gestational age variation? A ) Moist, supple, plum skin appearance B ) Abundant lanugo and vernix C ) Thin umbilical cord D ) Absence of sole creases 3 . The parents of a preterm newborn being cared for in the neonatal intensive care unit (NICU. are coming to visit for the first time. The newborn is receiving mechanical ventilation and intravenous fluids and medications and is being monitored electronically by various devices. Which action by the nurse would be most appropriate? A ) Suggest that the parents stay for just a few minutes to reduce their anxiety. B ) Reassure them that their newborn is progressing well. C ) Encourage the parents to touch their preterm newborn. D ) Discuss the care they will be giving the newborn upon discharge. 4 . When performing newborn resuscitation, which action would the nurse do first? A ) Intubate with an appropriate-sized endotracheal tube. B ) Give chest compressions at a rate of 80 times per minute. C ) Administer epinephrine intravenously. D ) Suction the mouth and then the nose. 5 The nurse frequently assesses the respiratory status of a preterm newborn based . on the understanding that the newborn is at increased risk for respiratory distress syndrome because of which of the following? A Inability to clear fluids ) B ) Immature respiratory control center C ) Deficiency of surfactant D ) Smaller respiratory passages 6 The nurse prepares to assess a newborn who is considered to be large for . gestational age (LGA). Which of the following would the nurse correlate with this gestational age variation? A Strong, brisk motor skills ) B Difficulty in arousing to a quiet alert state ) C Birth weight of 7 lb 14 oz ) D Wasted appearance of extremities ) 7 An LGA newborn has a blood glucose level of 30 mg/dL and is exhibiting . symptoms of hypoglycemia. Which of the following would the nurse do next? A Administer intravenous glucose immediately. ) B Feed the newborn 2 ounces of formula. ) C Initiate blow-by oxygen therapy. ) D Place the newborn under a radiant warmer. ) 8 A group of pregnant women are discussing high-risk newborn conditions as part . of a prenatal class. When describing the complications that can occur in these newborns to the group, which would the nurse include as being at lowest risk? A Small-for-gestational-age (SGA. newborns ) B Large-for-gestational-age (LGA. newborns ) C Appropriate-for-gestational-age (AGA. newborns ) D Low-birth-weight newborns ) 9 While caring for a preterm newborn receiving oxygen therapy, the nurse . monitors the oxygen therapy duration closely based on the understanding that the newborn is at risk for which of the following? A Retinopathy of prematurity ) B Metabolic acidosis ) C Infection ) D Cold stress ) 1 When planning the care for an SGA newborn, which action would the nurse 0 determine as a priority? . A Preventing hypoglycemia with early feedings ) B ) Observing for respiratory distress syndrome C ) Promoting bonding between the parents and the newborn D ) Monitoring vital signs every 2 hours 1 1 . A woman gives birth to a newborn at 36 weeks gestation. She tells the nurse, Im so glad that my baby isnt premature. Which response by the nurse would be most appropriate? A ) You are lucky to have given birth to a term newborn. B ) We still need to monitor him closely for problems. C ) How do you feel about delivering your baby at 36 weeks? D ) Your baby is premature and needs monitoring in the NICU. 1 2 . Which of the following would be most appropriate for the nurse to do when assisting parents who have experienced the loss of their preterm newborn? A ) Avoid using the terms death or dying. B ) Provide opportunities for them to hold the newborn. C ) Refrain from initiating conversations with the parents. D Quickly refocus the parents to a more pleasant topic. ) 1 3 . Which of the following, if noted in the maternal history, would the nurse identify as possibly contributing to the birth of an LGA newborn? A ) Drug abuse B ) Diabetes C ) Preeclampsia D ) Infection 1 4 . Which of the following would alert the nurse to suspect that a preterm newborn is in pain? A ) Bradycardia B ) Oxygen saturation level of 94% C ) Decreased muscle tone D ) Sudden high-pitched cry 1 When describing newborns with birth-weight variations to a group of nursing 5 students, the instructor identifies which variation if the newborn weighs 5.2 lb at . any gestational age? A ) Small for gestational age B ) Low birth weight C ) Very low birth weight D ) Extremely low birth weight 1 6 . A nurse is assessing a newborn who has been classified as small for gestational age. Which of the following would the nurse expect to find? (Select all that apply.) A ) Wasted extremity appearance B ) Increased amount of breast tissue C ) Sunken abdomen D ) Adequate muscle tone over buttocks E ) Narrow skull sutures 1 7 . The nurse is rev nurse notes that would classify iewing the medical record of a newborn born 2 hours ago. The the newborn was delivered at 35 weeks gestation. The nurse this newborn as which of the following? A ) Preterm B Late preterm ) C Full term ) D Postterm ) 1 8 . A nursing instructor is describing common problems associated with preterm birth. When describing the preterm newborns risk for perinatal asphyxia, the instructor includes which of the following as contributing to the newborns risk? (Select all that apply.) A ) Surfactant deficiency B ) Placental deprivation C ) Immaturity of the respiratory control centers D ) Decreased amounts of brown fat E ) Depleted glycogen stores 1 9 . After determining that a newborn is in need of resuscitation, which of the following would the nurse do first? A ) Dry the newborn thoroughly B ) Suction the airway C Administer ventilations ) D Give volume expanders ) 2 0 . A nurse is developing a plan of care for a preterm infant experiencing respiratory distress. Which of the following would the nurse be least likely to include in this plan? A ) Stimulate the infant with frequent handling. B ) Keep the newborn in a warmed isolette. C ) Administer oxygen using a oxygen hood. D ) Give gavage or continous tube feedings. 2 1 . A nurse suspects that a preterm newborn is having problems with thermal regulation. Which of the following would support the nurses suspicion? (Select all that apply.) A ) Shallow, slow respirations B ) Cyanotic hands and feet C ) Irritability D ) Hypertonicity E Feeble cry ) 2 2 . The nurse is assessing a preterm newborns fluid and hydration status. Which of the following would alert the nurse to possible overhydration? A ) Decreased urine output B ) Tachypnea C ) Bulging fontanels D ) Elevated temperature 2 3 . The nurse is assessing a preterm newborn who is in the neonatal intensive care unit (NICU. for signs and symptoms of overstimulation. Which of the following would the nurse be least likely to assess? A ) Increased respirations B ) Flaying hands C ) Periods of apnea D ) Decreased heart rate 2 A group of nursing students are reviewing the literature in preparation for a 4 class presentation on newborn pain prevention and management. Which of the . following would the students be most likely to find about this topic? A ) Newborn pain is frequently recognized and treated B ) Newborns rarely experience pain with procedures C ) Pain is frequently mistaken for irritability or agitation D ) Newborns may be less sensitive to pain than adult. 2 5 . A nurse is developing a plan of care for a preterm newborn to address the nursing diagnosis of risk for delayed development. Which of the following would the nurse include? (Select all that apply.) A ) Clustering care to promote rest B ) Positioning newborn in extension C ) Using kangaroo care D ) Loosely covering the newborn with blankets E ) Providing nonnutritive sucking 2 6 . A nurse is assisting the anxious parents of a preterm newborn to cope situation. Which statement by the nurse would be least appropriate? with the A ) Ill be here to help you all along the way. B ) What has helped you to deal with stressful situations in the past? C ) Let me tell you about what you will see when you visit your baby. D ) Forget about whats happened in the past and focus on the now. Answer Key 1. D 2. C 3. C 4. D 5. C 6. B 7. A 8. C 9. A 10. A 11. B 12. B 13. B 14. D 15. B 16. A, C, E 17. B 18. A, C 19. A 20. C 21. A, B, E 22. C 23. A 24. C 25. A, C, E 26. D Chapter 24 Nursing Management of the Newborn at Risk: Acquired & Congenital Newborn Conditions 1 . A newborn with severe meconium aspiration syndrome (MAS. is not responding to conventional treatment. Which of the following would the nurse anticipate as possibly necessary for this newborn? A ) Extracorporeal membrane oxygenation (ECMO) B ) Respiratory support with a ventilator C ) Insertion of a laryngoscope for deep suctioning D ) Replacement of an endotracheal tube via x-ray 2 Which of the following would the nurse expect to assess in a newborn who . develops sepsis? A Increased urinary output ) B Interest in feeding ) C Hypothermia ) D Wakefulness ) 3 . Which of the following would the nurse include in the plan of care for a newborn receiving phototherapy? A ) Keeping the newborn in the supine position B ) Covering the newborns eyes while under the bililights C ) Ensuring that the newborn is covered or clothed D ) Reducing the amount of fluid intake to 8 ounces daily 4 . A newborn has been diagnosed with a Group B streptococcal infection shortly after birth. The nurse understands that the newborn most likely acquired this infection from which of the following? A ) Improper handwashing B ) Contaminated formula C ) Nonsterile catheter insertion D Mothers birth canal ) 5 . Which action would be most appropriate for the nurse to take when a newborn has an unexpected anomaly at birth? A ) Show the newborn to the parents as soon as possible while explaining the defect. B ) Remove the newborn from the birthing area immediately. C ) Inform the parents that there is nothing wrong at the moment. D ) Tell the parents that the newborn must go to the nursery immediately. 6 . The nurse prepares to administer a gavage feeding for a newborn with transient tachypnea based on the understanding that this type of feeding is necessary for which reason? A ) Lactase enzymatic activity is not adequate. B ) Oxygen demands need to be reduced. C ) Renal solute lead must be considered. D ) Hyperbilirubinemia is likely to develop. 7 Which of the following would the nurse include when teaching a new mother . about the difference between pathologic and physiologic jaundice? A ) Physiologic jaundice results in kernicterus. B ) Pathologic jaundice appears within 24 hours after birth. C ) Both are treated with exchange transfusions of maternal O- blood. D ) Physiologic jaundice requires transfer to the NICU. 8 . When planning the care of a newborn addicted to cocaine who is experiencing withdrawal, which of the following would be least appropriate to include? A ) Wrapping the newborn snugly in a blanket B ) Waking the newborn every hour C ) Checking the newborns fontanels D ) Offering a pacifier 9 . A newborn is suspected of having fetal alcohol syndrome. Which of the following would the nurse expect to assess? A ) Bradypnea B ) Hydrocephaly C ) Flattened maxilla D Hypoactivity ) 1 0 . After teaching the parents of a newborn with periventricular hemorrhage about the disorder and treatment, which statement by the parents indicates that the teaching was successful? A ) Well make sure to cover both of his eyes to protect them. B ) Our newborn could develop a learning disability later on. C ) Once the bleeding ceases, there wont be any more worries. D ) We need to get family members to donate blood for transfusion. 1 1 . A newborn has an Apgar score of 6 at 5 minutes. Which of the following is the priority? A ) Initiating IV fluid therapy B ) Beginning resuscitative measures C ) Promoting kangaroo care D ) Obtaining a blood culture 1 While reviewing a newborns medical record, the nurse notes that the chest x-ray 2 shows a ground glass pattern. The nurse interprets this as indicative of: . A ) Respiratory distress syndrome B ) Transient tachypnea of the newborn C ) Asphyxia D ) Persistent pulmonary hypertension 1 3 . A newborn is suspected of developing persistent pulmonary hypertension. The nurse would expect to prepare the newborn for which of the following to confirm the suspicion? A ) Chest x-ray B ) Blood cultures C ) Echocardiogram D ) Stool for occult blood 1 4 . Which of the following would alert the nurse to suspect that a newborn has developed NEC? A ) Irritability B ) Sunken abdomen C Clay-colored stools ) D Bilious vomiting ) 1 5 . Which of the following would not be considered a risk factor for bronchopulmonary dysplasia (chronic lung disease)? A ) Preterm birth (less than 32 weeks) B ) Female gender C ) White race D ) Sepsis 1 6 . A group of nursing students are reviewing the different types of congenital heart disease in infants. The students demonstrate a need for additional review when they identify which of the following as an example of increased pulmonary blood flow (left-to-right shunting)? A ) Atrial septal defect B ) Tetralogy of Fallot C ) Ventricular septal defect D ) Patent ductus arteriosus 1 7 . After teaching the parents of a newborn with retinopathy of prematurity (ROP. about the disorder and treatment, which statement by the parents indicates that the teaching was successful? A ) Can we schedule follow-up eye examinations with the pediatric ophthalmologist now? B ) We can fix the problem with surgery. C ) Well make sure to administer eye drops each day for the next few weeks. D ) Im sure the baby will grow out of it. 1 8 . The nurse is assessing the newborn of a mother who had gestational diabetes. Which of the following would the nurse expect to find? (Select all that apply.) A ) Pale skin color B ) Buffalo hump C ) Distended upper abdomen D ) Excessive subcutaneous fat E ) Long slender neck 1 The nurse is assessing a newborn who is large for gestational age. The newborn 9 was born breech. The nurse suspects that the newborn may have experienced . trauma to the upper brachial plexus based on which assessment findings? A Absent grasp reflex ) B Hand weakness ) C Absent Moro reflex ) D Facial asymmetry ) 2 The nurse is assessing a newborn and suspects that the newborn was exposed to 0 drugs in utero because the newborn is exhibiting signs of neonatal abstinence . syndrome. Which of the following would the nurse expect to assess? (Select all that apply.) A Tremors ) B Diminished sucking ) C Regurgitation ) D Shrill, high-pitched cry ) E Hypothermia ) F Frequent sneezing ) 2 A nurse is developing a plan of care for a newborn with omphalocele. Which of 1 the following would the nurse include? . A Placing the newborn into a sterile drawstring bowel bag ) B Using clean technique for dressing changes ) C Preparing the newborn for incision and drainage ) D Instituting gavage feedings ) 2 A nurse is explaining to the parents of a child with bladder exstrophy about the 2 care their infant requires. Which of the following would the nurse include in the . explanation? (Select all that apply.) A Covering the area with a sterile, clear, nonadherent dressing ) B Irrigating the surface with sterile saline twice a day ) C Monitoring drainage through the suprapubic catheter ) D Administering prescribed antibiotic therapy ) E Preparing for surgical intervention in about 2 weeks ) 2 A nursing student is preparing a presentation for the class on clubfoot. The 3 student determines that the presentation was successful when the class states . which of the following? A Clubfoot is a common genetic disorder. ) B ) The condition affects girls more often than boys. C ) The exact cause of clubfoot is not known. D ) The intrinsic form can be manually reduced. 2 Assessment of newborn reveals a large protruding tongue, slow reflexes, 4 distended abdomen, poor feeding, hoarse cry, goiter and dry skin. Which of the . following would the nurse suspect? A Phenylketonuria ) B Galactosemia ) C Congenital hypothyroidism ) D Maple syrup urine disease ) 2 A group of students are reviewing information about the effects of substances 5 on the newborn. The students demonstrate understanding of the information . when they identify which drug as not being associated with teratogenic effects on the fetus? A Alcohol ) B Nicotine ) C Marijuana ) D Cocaine ) 2 A nurse is teaching the mother of a newborn diagnosed with galactosemia about 6 dietary restrictions. The nurse determines that the mother has understood the . teaching when she identifies which of the following as needing to be restricted? A Phenylalanine ) B Protein ) C Lactose ) D Iodine ) 2 7 . A newborn was diagnosed with a congenital heart defect and will undergo surgery at a later time. The nurse is teaching the parents about signs and symptoms that need to be reported. The nurse determines that the parents have understood the instructions when they state that they will report which of the following? (Select all that apply.) A ) Weight loss B ) Pale skin C ) Fever D ) Absence of edema E ) Increased respiratory rate 2 8 . When developing the plan of care for a newborn with an acquired condition, which of the following would the nurse include to promote participation by the parents? A ) Use verbal instructions primarily for explanations B ) Assist with decision making process C ) Provide personal views about their decisions D ) Encourage them to refrain from showing emotions 2 9 . A nurse is assisting in the resuscitation of a newborn. The nurse would expect to stop resuscitation efforts when the newborn has no heartbeat and respiratory effort after which time frame? A ) 5 minutes B ) 10 minutes C ) 15 minutes D ) 20 minutes 3 A newborn is diagnosed with meconium aspiration syndrome. When assessing 0 this newborn, which of the following would the nurse expect to find? (Select all . that apply.) A Pigeon chest ) B Prolonged tachypnea ) C Intercostal retractions ) D High blood pH level ) E Coarse crackles on auscultation ) Answer Key 1. A 2. C 3. B 4. D 5. A 6. B 7. B 8. B 9. C 10. B 11. B 12. A 13. C 14. D 15. B 16. B 17. A 18. B, C, D 19. C 20. A, C, D, F 21. A 22. A, C, D 23. C 24. C 25. C 26. C 27. A, C, E 28. B 29. B 30. B, C, E Chapter 25 Growth and Development of the Newborn and Infant 1. A mother calls the pediatricians office because her infant is colicky. The helpful measure the nurse would suggest to the parent is: a . Sing songs to the infant in a soft voice. b . Place the infant in a well-lit room. c . Walk around and massage the infants back. d Rock the fussy infant slowly and gently. . ANS: D One technique the nurse can offer parents of a fussy infant is to rock the infant gently and slowly while being careful to avoid sudden movements. DIF: Cognitive Level: Application REF: Text Reference: 390 OBJ: Objective: 11 TOP: Topic: Health Maintenance KEY: Nursing Process Step: Implementation MSC: NCLEX: Physiological Integrity: Basic Care and Comfort 2. The nurse is aware that the age at which the posterior fontanelle closes is: a 2 to 3 months . b 3 to 6 months . c 6 to 9 months . d 9 to 12 months . ANS: A The posterior fontanel closes between 2 and 3 months of age. DIF: Cognitive Level: Knowledge REF: Text Reference: 384, Table 16-1 OBJ: Objective: 2 TOP: Topic: Development and Care KEY: Nursing Process Step: Assessment MSC: NCLEX: Health Promotion and Maintenance: Growth and Development 3. The nurse knows that an infants birthweight should be tripled by: a 9 months . b 1 year . c 18 months . d 2 years . ANS: B The infant usually triples his or her birth weight by about 12 months of age. DIF: Cognitive Level: Knowledge REF: Text Reference: 386, Table 16-1 OBJ: Objective: 2 TOP: Topic: Development and Care KEY: Nursing Process Step: Assessment MSC: NCLEX: Health Promotion and Maintenance: Growth and Development 4. The nurse is aware that the age at which an infant is able to sit steadily alone is: a 4 months . b 5 months . c 8 months . d 15 months . ANS: C The infant can sit alone without support at about 8 months of age. DIF: Cognitive Level: Knowledge REF: Text Reference: 382, Figure 16-3 OBJ: Objective: 2 TOP: Topic: Development and Care KEY: Nursing Process Step: Assessment MSC: NCLEX: Health Promotion and Maintenance: Growth and Development 5. The infant should be able to walk independently by the age of: a 8-10 months . b 12-15 months . c 15-18 months . d 18-21 months . ANS: B For the majority of children, the milestone of walking alone is achieved between 12 and 15 months. DIF: Cognitive Level: Knowledge REF: Text Reference: 383, Table 16-3 OBJ: Objective: 2 TOP: Topic: Development and Care KEY: Nursing Process Step: Assessment MSC: NCLEX: Health Promotion and Maintenance: Growth and Development 6. The parent of a 3-month-old infant asks the nurse, At what age do infants usually begin drinking from a cup? The nurse would reply: a 5 months . b 9 months . c 1 year . d 2 years . ANS: A The infant can usually drink from a cup when it is offered at about 5 months. DIF: Cognitive Level: Comprehension REF: Text Reference: 386, Table 16-1 OBJ: Objective: 7 TOP: Topic: Nutrition Counseling KEY: Nursing Process Step: Implementation MSC: NCLEX: Health Promotion and Maintenance: Growth and Development 7. The nurse would expect a 4-month-old to be able to: a Hold a cup . b Stand with assistance . c Lift head and shoulders . d Sit with back straight . ANS: C Because development is cephalocaudal, of these choices, sitting is the one that the infant learns to do first. The infant can usually sit with support at about 5 months of age and can sit alone at about 8 months. DIF: Cognitive Level: Analysis REF: Text Reference: 381, Table 16-1 OBJ: Objective: 2 TOP: Topic: Development and Care KEY: Nursing Process Step: Assessment MSC: NCLEX: Health Promotion and Maintenance: Growth and Development 8. The abnormal finding in an evaluation of growth and development for a 6-month-old infant would be: a Weight gain of 4-7 ounces per week . b Length increase of 1 inch in 2 months . c Head lag present . d Can sit alone for a few seconds . ANS: C The infant should be holding the head up well by 5 months of age. If head lag is present at 6 months, the child should undergo further evaluation. DIF: Cognitive Level: Analysis REF: Text Reference: 386, Table 16-1 OBJ: Objective: 2 TOP: Topic: Development and Care KEY: Nursing Process Step: Assessment MSC: NCLEX: Health Promotion and Maintenance: Growth and Development 9. A parent brings a 6-month-old infant to the pediatric clinic for her well-child examination. Her birthweight was 8 pounds, 2 ounces. The nurse weighing the infant today would expect her weight to be at least: a 12 pounds . b 16 pounds . c 20 pounds . d 24 pounds . ANS: B Birth weight is usually doubled by 6 months of age. DIF: Cognitive Level: Application REF: Text Reference: 386, Table 16-1 OBJ: Objective: 8 TOP: Topic: Development and Care KEY: Nursing Process Step: Assessment MSC: NCLEX: Health Promotion and Maintenance: Growth and Development 10. The nurse would advise a parent when introducing solid foods to: a Begin with one tablespoon of the food. . b Mix foods together. . c Eliminate a refused food from the diet. . d Introduce each new food 4 to 7 days apart. . ANS: D Only one new food is offered in a 4- to 7-day period to determine tolerance. DIF: Cognitive Level: Comprehension REF: Text Reference: 394 OBJ: Objective: 5 TOP: Topic: Nutrition Counseling KEY: Nursing Process Step: Implementation MSC: NCLEX: Health Promotion and Maintenance: Growth and Development 11. When talking with a parent about tooth eruption, the nurse explains that the first deciduous teeth to erupt are the: a Lower central incisors . b Upper central incisors . c Lower lateral incisors . d Upper lateral incisors . ANS: A The first teeth to erupt, usually at about 7 months, are the lower central incisors. DIF: Cognitive Level: Knowledge REF: Text Reference: 387, Table 16-1 OBJ: Objective: 8 TOP: Topic: Development and Care KEY: Nursing Process Step: Implementation MSC: NCLEX: Health Promotion and Maintenance: Growth and Development 12. When assessing development in a 9-month-old infant, the nurse would expect to observe the infant: a . Sitting if supported b . Grasping objects with the palm c . Imitating sounds such as da-da d Beginning to use a spoon rather sloppily . ANS: C The 9-month-old tries to imitate sounds such as da-da or ba-ba. DIF: Cognitive Level: Analysis REF: Text Reference: 388, Table 16-1 OBJ: Objective: 2 TOP: Topic: Development and Care KEY: Nursing Process Step: Assessment MSC: NCLEX: Health Promotion and Maintenance: Growth and Development 13. The statement made by a parent that indicates correct understanding of infant feeding is: a . Ive been mixing rice cereal and formula in the babys bottle. b . I switched the baby to low-fat milk at 9 months. c . The baby really likes little pieces of chocolate. d . I give the baby any new foods before he takes his bottle. ANS: D New solid foods should be introduced before formula or breast milk to encourage the infant to try new foods. DIF: Cognitive Level: Analysis REF: Text Reference: 394 OBJ: Objective: 5 TOP: Topic: Nutrition Counseling KEY: Nursing Process Step: Evaluation MSC: NCLEX: Health Promotion and Maintenance: Growth and Development 14. The nurse would advise a mother who is concerned because her 10-month-old is lethargic, to: a . Keep the babys room well-lit. b . Rub the babys soles vigorously. c . Offer the baby a pacifier. d . Handle the infant slowly and gently. ANS: D Some infants respond to stimulating environments by shutting down. Move and handle infants slowly and gently. DIF: Cognitive Level: Application REF: Text Reference: 390 OBJ: Objective: 11, 14 TOP: Topic: Health Maintenance KEY: Nursing Process Step: Implementation MSC: NCLEX: Physiological Integrity: Basic Care and Comfort 15. The nurse discusses safety-proofing the home with the mother of a 9-month-old. The statement made by the mother that indicates an unsafe behavior is: a . I put covers on all of the electrical outlets. b . In the car, she rides in a front-facing car seat. c . There are locks on all of the cabinets in the house. d . I have a gate at the top and bottom of the stairs. ANS: B A rear-facing infant car seat should be used for infants under 1 year of age. DIF: Cognitive Level: Analysis REF: Text Reference: 396 OBJ: Objective: 13 TOP: Topic: Infant Safety KEY: Nursing Process Step: Evaluation MSC: NCLEX: Safe, Effective Care Environment: Safety and Infection Control 16. The nurse observes a 10-month-old infant using her index finger and thumb to pick up Cheerios. This behavior is evidence that the infant has developed the: a . Pincer grasp b . Grasp reflex c . Prehension ability d . Parachute reflex ANS: A By 1 year, the pincer-grasp coordination of index finger and thumb is well established. DIF: Cognitive Level: Analysis REF: Text Reference: 382, Figure16-3 OBJ: Objective: 2 TOP: Topic: General Characteristics KEY: Nursing Process Step: Assessment MSC: NCLEX: Health Promotion and Maintenance: Growth and Development 17. A parent is concerned because her infant has a diaper rash. The nurse would advise the parent to: a . Use commercial diaper wipes to clean the area. b . Apply a protective ointment on the area. c . Change the babys diaper less frequently. d . Keep the diaper area covered all of the time. ANS: B A protective ointment can be applied when the skin in the diaper area appears pink and irritated. DIF: Cognitive Level: Application: Basic Care and Comfort REF: Text Reference: 390 OBJ: Objective: 10 TOP: Topic: Community-Based Care KEY: Nursing Process Step: Implementation MSC: NCLEX: Physiological Integrity 18. The mother of an infant born prematurely tells the nurse, The baby is irritable. He cries during diaper changes and feedings. Can you make some suggestions about what I should do to soothe him? The most appropriate recommendation to help this parent would be: a . Play the radio or TV while you feed the baby. b . Put the baby in a room with sunlight. c . Cover the baby snugly when you hold him. d . Change the babys position quickly. ANS: C A strategy that may be helpful is to swaddle the infant snugly in a light blanket with extremities flexed and hands near the face. DIF: Cognitive Level: Application REF: Text Reference: 383 OBJ: Objective: 11 TOP: Topic: Community-Based Care KEY: Nursing Process Step: Implementation MSC: NCLEX: Physiological Integrity: Basic Care and Comfort 19. The most appropriate activity to recommend to parents to promote sensorimotor stimulation for a 1-year-old would be: a Ride a tricycle. . b . Spend time in an infant swing. c . Play with push-pull toys. d . Read large picture books. ANS: C Push-pull toys are appropriate to promote sensorimotor stimulation for a 1-year-old child. DIF: Cognitive Level: Analysis REF: Text Reference: 397, Table 16-4 OBJ: Objective: 12 TOP: Topic: Infant Safety KEY: Nursing Process Step: Implementation MSC: NCLEX: Health Promotion and Maintenance: Growth and Development 20. The statement that indicates the mother of an 8-month-old understands infant sleep patterns is: a I put the baby in my bed until she falls asleep, then I put her in her crib. . b I let the baby skip an afternoon nap so she will fall asleep earlier. . c I put the pacifier in the crib so she can find it when she wakes up. . d I rock the baby back to sleep if she wakes up at night. . ANS: C The parent should assist the infant to develop self-soothing behaviors so the infant can get back to sleep on her own. DIF: Cognitive Level: Analysis REF: Text Reference: 390 OBJ: Objective: 14 TOP: Topic: Health Maintenance KEY: Nursing Process Step: Evaluation MSC: NCLEX: Health Promotion and Maintenance: Growth and Development MULTIPLE RESPONSE 1. The nurse is aware that the 7-month-old can signal feeding readiness by: Select all that apply. a Pulling spoon toward mouth . b Biting at spoon with upper and lower incisors . c Pointing to food bowl . d Bouncing up and down with excitement at sight of food . e Manipulating finger foods . ANS: A, E The 7-month-old pulls the spoon toward its mouth, and can manipulate finger foods. The 7- month-old does not have upper incisors and has not developed adequately to recognize the food container or exhibit excitement related to the sight of food. Chapter 26 Growth and Development of the Toddler 1. Which of these behaviors reported by a parent of an 18-month-old toddler would the nurse report to the pediatrician as a cause for concern? a The child has temper tantrums. . b The child feeds himself sloppily. . c The child walks by holding onto furniture. . d The child speaks in short sentences. . ANS: C By 18 months, a toddler should have been walking alone for several months. The toddler who walks holding onto furniture should be evaluated by a developmental specialist. DIF: Cognitive Level: Analysis REF: Text Reference: 400, Table 17-1 OBJ: Objective: 2 TOP: Topic: Development KEY: Nursing Process Step: Implementation MSC: NCLEX: Health Promotion and Maintenance: Growth and Development 2. The nurse assessing growth and development of a 2-year-old child would expect to find: a That the child jumps with both feet . b That 20 deciduous teeth have erupted . c That the child can hop on one foot . d A vocabulary of 900 words . ANS: A The 2-year-old can jump with both feet. The remaining achievements occur after 2 years of age. DIF: Cognitive Level: Analysis REF: Text Reference: 400, Table 17-1 OBJ: Objective: 2 TOP: Topic: Physical Development KEY: Nursing Process Step: Assessment MSC: NCLEX: Health Promotion and Maintenance: Growth and Development 3. A parent remarks, My 18-month-old daughter carries her blanket around everywhere. Is this normal? The nurse who has an understanding of toddler development might explain that: a She carries her blanket because she is ritualistic. . b Carrying her favorite blanket is self-consoling behavior. . c This behavior can be discouraged by offering new toys to the child. . d This could be indicative of emotional distress. . ANS: B Favorite possessions and repetitive rituals are self-consoling behaviors for the toddler. DIF: Cognitive Level: Application REF: Text Reference: 403 OBJ: Objective: 6 TOP: Topic: Guidance and Discipline KEY: Nursing Process Step: Implementation MSC: NCLEX: Health Promotion and Maintenance: Growth and Development 4. The nurse observed three toddlers playing side by side with dolls. Closer observation revealed that the children were not interacting with one another. This type of play would be characterized as: a Solitary . b Parallel . c Associative . d Cooperative . ANS: B Toddlers engage in parallel play. Children play next to, but not with, each other. DIF: Cognitive Level: Analysis REF: Text Reference: 411 OBJ: Objective: 9 TOP: Topic: Play KEY: Nursing Process Step: Assessment MSC: NCLEX: Health Promotion and Maintenance: Growth and Development 5. The nurse planning anticipatory guidance for parents of a toddler would include which of the following instructions? a Adhere to a rigid schedule because the toddler is ritualistic. . b Limit setting should include praise. . c Shoes should fit snugly at the toe and arch. . d Dress the toddler in pants with a zipper so he or she can learn to zip and unzip . clothes. ANS: B Limit-setting should include praise as well as disapproval for undesired behavior. DIF: Cognitive Level: Application REF: Text Reference: 403 OBJ: Objective: 2 TOP: Topic: Daily Care KEY: Nursing Process Step: Planning MSC: NCLEX: Health Promotion and Maintenance: Growth and Development 6. The best advice the nurse can offer a parent who is concerned because her 2-year-old is very active and does not eat much is: a . Insist that the child eat one food on the plate. b . Help the child to wind down with a quiet activity before mealtime. c . Maintain a consistent eating schedule for the family. d Serve the meal with a variety of interesting plates, cups, and utensils. . ANS: B Quiet time before meals provides an opportunity for the active toddler to wind down. DIF: Cognitive Level: Application REF: Text Reference: 406 OBJ: Objective: 11 TOP: Topic: Nutrition Counseling KEY: Nursing Process Step: Implementation MSC: NCLEX: Health Promotion and Maintenance: Growth and Development 7. How would the nurse advise a parent who states, I never know how much food to feed my child? a . Serving sizes should not exceed 1 teaspoon of each type of food. b . Food quantities must be carefully measured to avoid overfeeding. c . Use 1 tablespoon of each food for each year of age as a guideline. d . A toddler should eat three balanced meals. Snacks are not necessary. ANS: C A tablespoon of each type of food for each year of age is a good guideline to follow when determining serving sizes. DIF: Cognitive Level: Application REF: Text Reference: 406 OBJ: Objective: 11 TOP: Topic: Nutrition Counseling KEY: Nursing Process Step: Implementation MSC: NCLEX: Health Promotion and Maintenance: Growth and Development 8. The nurse discussing toilet training with parents would identify which of the following as an indicator of readiness? The child is: a . Willing to sit on the potty for 15 to 20 minutes b . Dry in the daytime for 4-hour periods c . Able to communicate that he or she is wet d . Curious about bathroom activities ANS: C Children are ready for toilet training when they can communicate in some fashion that they are wet or need to urinate or defecate. DIF: Cognitive Level: Comprehension REF: Text Reference: 405 OBJ: Objective: 7 TOP: Topic: Toilet Independence KEY: Nursing Process Step: Implementation MSC: NCLEX: Health Promotion and Maintenance: Growth and Development 9. The nurse selects the most appropriate toy for a normal 2-year-old child, which is: a . A bicycle with training wheels b . A dump truck c . Wind-up toys d . Legos ANS: B The 2-year-old enjoys playing with objects that can be pushed or pulled. DIF: Cognitive Level: Application REF: Text Reference: 410 OBJ: Objective: 9 TOP: Topic: Toys and Play KEY: Nursing Process Step: Assessment MSC: NCLEX: Health Promotion and Maintenance: Growth and Development 10. To encourage a toddler to practice independence, the nurse would recommend that the childs mother: a . Offer a variety of items to choose from to stimulate his mind. b . Allow the child to determine his own daily routine. c . Offer him a choice between two items. d . Set the routine herself, but discuss with her toddler how he or she would have done it differently. ANS: C The toddler can be allowed to make choices as the situation warrants, but the number of choices should be limited because too many confuse the toddler. DIF: Cognitive Level: Application REF: Text Reference: 399 OBJ: Objective: 3 TOP: Topic: General Characteristics KEY: Nursing Process Step: Implementation MSC: NCLEX: Health Promotion and Maintenance: Growth and Development 11. On a home visit, the nurse notes each of the following. The observation that requires teaching intervention to protect the 15-month-old child who lives there is: a . The fireplace has a screen. b . The dining room table has a tablecloth on it. c . There are paintings on the wall. d . The kitchen floor is clean but not shiny. ANS: B A tablecloth presents a safety hazard because the curious toddler will reach up and pull on it. The toddler could be injured if items on the table are moved when the tablecloth is pulled. DIF: Cognitive Level: Analysis REF: Text Reference: 409, Table 17-6 OBJ: Objective: 8 TOP: Topic: Injury Prevention KEY: Nursing Process Step: Assessment MSC: NCLEX: Safe, Effective Care Environment: Safety and Infection Control 12. The nurse considers the appropriate snack for a 2-year-old child would be: a Hot dog sections . b Grapes . c Popcorn . d Applesauce . ANS: D Applesauce is a healthy and safe snack food for the toddler. The toddler risks choking on such foods as grapes, hot dogs, and popcorn. DIF: Cognitive Level: Analysis REF: Text Reference: 407 OBJ: Objective: 8 TOP: Topic: Injury Prevention KEY: Nursing Process Step: Planning MSC: NCLEX: Health Promotion and Maintenance: Growth and Development 13. The nurse assessing vital signs on a 2-year-old would be concerned about the finding of: a Temperature 98.8 F . b Pulse 100 beats/min . c Respirations 36 breaths/min . d Blood pressure 90/60 mm Hg . ANS: C In the toddler period, the respiratory rate decreases to 25 breaths per minute. DIF: Cognitive Level: Analysis REF: Text Reference: 400 OBJ: Objective: 2 TOP: Topic: Physical Development KEY: Nursing Process Step: Assessment MSC: NCLEX: Physiological Integrity: Physiological Adaptation 14. When assessing language development in a 2-year-old, an expected finding would be: a A 900-word vocabulary . b Use of two-word sentences . c Use of pronouns and prepositions . d 100% of speech is understandable . ANS: B The 2-year-old should be using two-word sentences. DIF: Cognitive Level: Analysis REF: Text Reference: 403 OBJ: Objective: 5 TOP: Topic: Speech Development KEY: Nursing Process Step: Assessment MSC: NCLEX: Health Promotion and Maintenance: Growth and Development 15. The nurse has explained the use of time-outs to the parent of a 3-year-old. The nurse determines the parent understands the information when she states an appropriate period for a time-out is: a 3 minutes . b 6 minutes . c 10 minutes . d 15 minutes . ANS: A Timing for time out is usually based on 1 minute per year of age. DIF: Cognitive Level: Application REF: Text Reference: 403 OBJ: Objective: 10 TOP: Topic: Guidance and Discipline KEY: Nursing Process Step: Evaluation MSC: NCLEX: Health Promotion and Maintenance: Growth and Development 16. The parent of a toddler tells the nurse, My daughters appetite has decreased. Thank goodness she loves to drink milk. The most appropriate response for the nurse to make is: a Has your daughter been sick recently? . b How much milk does she drink in a day? . c Has she become a fussy eater, too? . d Have you tried offering her finger foods? . ANS: B Milk should be limited to 24 ounces a day. Too few solid foods can lead to dietary deficiencies of iron. DIF: Cognitive Level: Analysis REF: Text Reference: 406 OBJ: Objective: 11 TOP: Topic: Nutrition Counseling KEY: Nursing Process Step: Assessment MSC: NCLEX: Physiological Integrity 17. The nurse suggests that bladder training should start when the toddler can stay dry for hours. a 1 . b 2 . c 3 . d 4 . ANS: B If the toddler is mature enough to retain urine for 2 hours, bladder training can be effective. Chapter 27 Growth and Development of the Preschooler MULTIPLE CHOICE 1. Which of the following statements best describes the 3-year-old child? a Boisterous, tattles on others . b Aggressive, shows off . c Helpful, wants to assist with chores . d Talkative, inquisitive about the environment . ANS: C Three-year-old children are helpful and can assist in simple household chores. DIF: Cognitive Level: Analysis REF: Text Reference: 418 OBJ: Objective: 3 TOP: Topic: Development KEY: Nursing Process Step: Assessment MSC: NCLEX: Health Promotion and Maintenance: Growth and Development 2. The parents of a 4-year-old boy are concerned because they have noticed him frequently touching his penis. The nurse would base a response on the knowledge that: a This behavior indicates a normal curiosity about sexuality. . b Masturbation suggests the boy has an excessive fear of castration. . c It is usually a result of discomfort from a penile rash or irritation. . d The behavior is abnormal and the child should be referred for counseling. . ANS: A Masturbation at this age is common and indicates that the preschooler has a normal curiosity about sexuality. DIF: Cognitive Level: Comprehension REF: Text Reference: 418 OBJ: Objective: 17 TOP: Topic: Guidance KEY: Nursing Process Step: Implementation MSC: NCLEX: Health Promotion and Maintenance: Growth and Development 3. A preschool-age child is asked, Why do trees have leaves? Which of the following responses would be an example of animism? a So I can have shade over my sandbox. . b Because God made them that way. . c To hide behind when they are scared. . d For the squirrels to play in. . ANS: C Animism describes the tendency of preschool children to attribute human characteristics to nonhuman objects. DIF: Cognitive Level: Comprehension REF: Text Reference: 415 OBJ: Objective: 4 TOP: Topic: Cognitive Development KEY: Nursing Process Step: Assessment MSC: NCLEX: Health Promotion and Maintenance 4. The tasks that would be appropriate to expect of a 5-year-old would be: a Setting the table with paper plates . b Washing the dirty knives . c Carrying glasses from the table to the sink . d Scrubbing out the sink with cleanser . ANS: A Parents must consider developmental level and safety when asking the 5-year-old child to help with chores. DIF: Cognitive Level: Analysis REF: Text Reference: 419 OBJ: Objective: 3 TOP: Topic: Development-Safety KEY: Nursing Process Step: Assessment MSC: NCLEX: Health Promotion and Maintenance: Growth and Development 5. A 3-year-old child, while playing with her favorite toy in the playroom of the pediatric unit, is approached by another child who also wants to play with the same toy. The nurse anticipates that the 3-year-old will: a Play well with the other child . b Give the toy up and then not play any more . c Become angry and a physical response might ensue . d Ignore the toy and go on to something else . ANS: C The 3-year-old child is egocentric and likely will become angry when others attempt to take his or her possessions. DIF: Cognitive Level: Analysis REF: Text Reference: 419 OBJ: Objective: 6 TOP: Topic: Play KEY: Nursing Process Step: Assessment MSC: NCLEX: Health Promotion and Maintenance: Growth and Development 6. A parent is concerned about her childrens reaction should their grandmother die. In planning a response, the nurse is guided by the understanding that: a Children are unlikely to notice their grandmothers absence if no one reminds . them. b Young children often understand that other people die, but do not equate it with . themselves. c The childrens response will depend entirely on whether they have been . acquainted with death before this. d Children can understand the concept of a higher being much like adults can. . ANS: B Between 3 and 4 years of age, the child becomes curious about death and dying. They may realize that others die, but they do not relate death to themselves. DIF: Cognitive Level: Comprehension REF: Text Reference: 419 OBJ: Objective: 13 TOP: Topic: Concept of Death KEY: Nursing Process Step: Planning MSC: NCLEX: Health Promotion and Maintenance: Growth and Development 7. The intervention that is most effective in dealing with occasional aggression in a 4-year-old child is: a Have the child take a time-out in the corner for 4 minutes. . b . Spank the child at the time of the incident. c . Take away television privileges for the day. d . Send the child to his room for 30 minutes. ANS: A Time-out periods, usually lasting 1 minute per year of age, with the child sitting in a chair or corner, are considered an effective disciplinary technique. DIF: Cognitive Level: Application REF: Text Reference: 420 OBJ: Objective: 9 TOP: Topic: Discipline and Limit Setting KEY: Nursing Process Step: Implementation MSC: NCLEX: Health Promotion and Maintenance: Growth and Development 8. A parent is concerned about how to make his preschool-age child stop sucking his thumb and asks the nurse for suggestions. The nurses most helpful response would be: a Most children will stop thumb-sucking naturally by school age. . b Over-the-counter treatments that give a bad taste can be placed on the thumb to . discourage the practice. c Consistently touching the childs fingers whenever he sucks his thumb is most . effective. d Thumb-sucking is detrimental to the eruption of the childs teeth and must be . stopped as soon as possible. ANS: A Most children give up the habit of thumb-sucking by the time they reach school. DIF: Cognitive Level: Comprehension REF: Text Reference: 422 OBJ: Objective: 12 TOP: Topic: Thumb-Sucking KEY: Nursing Process Step: Implementation MSC: NCLEX: Health Promotion and Maintenance: Growth and Development 9. The nurse characterizes the play of 5-year-old children as: a Rough and tumble play . b Well-organized games . c Following rules . d Prefer inside activities . ANS: C The 5-year-old wants to play by the rules but cannot accept losing. The rules may be very strict or change as the game progresses. DIF: Cognitive Level: Comprehension REF: Text Reference: 419 OBJ: Objective: 6 TOP: Topic: Play KEY: Nursing Process Step: Assessment MSC: NCLEX: Health Promotion and Maintenance: Growth and Development 10. When discussing preschoolers sexual curiosity with the parent, the nurse determines that the parent understands the information when she states she would: a Make up funny words for body parts. . b Distract my child with a toy if she asks about sex. . c Answer her questions when she asks. . d Tell her to ask me again when she is 6 years old. . ANS: C Parents should provide sex education at the time the child asks about sex. DIF: Cognitive Level: Analysis REF: Text Reference: 426 OBJ: Objective: 12 TOP: Topic: Sexual Curiosity KEY: Nursing Process Step: Evaluation MSC: NCLEX: Health Promotion and Maintenance 11. In planning care for a moderately retarded child, the type of play most appropriate is: a Play should exercise leg and arm muscles. . b Play should be educationally oriented to make up for lost time. . c Play should be adjusted to her mental age rather than her chronological age. . d Play is not a necessary component of the care of a mentally retarded child. . ANS: C The nurse must consider the childs mental age rather than her chronological age when selecting toys for play. DIF: Cognitive Level: Analysis REF: Text Reference: 425 OBJ: Objective: 6 TOP: Topic: Play KEY: Nursing Process Step: Planning MSC: NCLEX: Health Promotion and Maintenance: Growth and Development 12. The nurses best advice to a parent about a preschoolers imaginary friend would be that: a Imaginary friends is a sign that the child has a low self-esteem. . b It is common for preschoolers to have imaginary friends. . c The preschooler invents an imaginary friend when he feels overwhelmed. . d The best approach to dealing with an imaginary friend is to ignore them. . ANS: B Imaginary friends are common and normal during the preschool period and serve many purposes, such as relief from loneliness, mastery of feats, and scapegoat. DIF: Cognitive Level: Comprehension REF: Text Reference: 418 OBJ: Objective: 3 TOP: Topic: Play KEY: Nursing Process Step: Implementation MSC: NCLEX: Health Promotion and Maintenance: Growth and Development 13. The nurse suggests measures that might be helpful for the child with enuresis, such as: a Apply an electric pad that gently shocks the child. . b Wake the child several times during the night to urinate. . c Decrease fluid intake after the evening meal. . d Increase dietary fiber intake. . ANS: C If a child is experiencing enuresis, liquids after dinner should be limited and the child should routinely void before going to bed. DIF: Cognitive Level: Application REF: Text Reference: 413 OBJ: Objective: 12 TOP: Topic: Enuresis KEY: Nursing Process Step: Implementation MSC: NCLEX: Health Promotion and Maintenance: Growth and Development 14. The nurse suggests that the most appropriate toy choice for a 3-year-old would be: a A board game . b A small pet, such as a goldfish . c A large construction set . d Push-pull toys . ANS: C Large construction sets are suitable toys for the preschool-age child. DIF: Cognitive Level: Application REF: Text Reference: 425 OBJ: Objective: 12 TOP: Topic: Play KEY: Nursing Process Step: Implementation MSC: NCLEX: Health Promotion and Maintenance: Growth and Development 15. The parent of a 3-year-old child tells the nurse, My daughter points whenever she wants me to get something for her but she understands me when I ask her to do something. Based on the parents comment, the nurse recognizes that: a The childs language development is age-appropriate. . b The child may have expressive language delay. . c The child has a receptive language delay. . d The child should have her hearing tested. . ANS: B An expressive language delay is suspected when the child understands spoken language but is not talking. DIF: Cognitive Level: Analysis REF: Text Reference: 416, Table 18-1 OBJ: Objective: 3 TOP: Topic: Language Development KEY: Nursing Process Step: Assessment MSC: NCLEX: Health Promotion and Maintenance: Growth and Development 16. The parent of a 4-year-old child tells the nurse, Bedtime is difficult. I cant get my child to bed at night. The nurse and the childs mother discuss options and decide that the best choice would be to: a Allow the child to put himself to bed when he is tired. . b Let the child read in his room until he falls asleep. . c Establish a bedtime routine and use it consistently. . d Tire him out with physical activity before bedtime. . ANS: C Parents should engage the child in quiet activities before bedtime and establish a ritual that signals readiness for bedtime. DIF: Cognitive Level: Application REF: Text Reference: 418 OBJ: Objective: 11 TOP: Topic: Bedtime Habits KEY: Nursing Process Step: Implementation MSC: NCLEX: Health Promotion and Maintenance: Growth and Development 17. The nurse understands that a fear unique to the preschool period is: a Fear of water . b Fear of animals . c Fear of bodily harm . d Fear of death . ANS: C The fear of bodily harm, particularly the loss of body parts, is unique to this stage. DIF: Cognitive Level: Knowledge REF: Text Reference: 419 OBJ: Objective: 3 TOP: Topic: Development KEY: Nursing Process Step: Assessment MSC: NCLEX: Health Promotion and Maintenance: Growth and Development 18. A 4-year-old child tells the nurse that he will not eat peas because they are green. This is an example of: a Egocentrism . b Artificialism . c Animism . d Centering . ANS: D The tendency to concentrate on a single outstanding characteristic of an object while excluding other features is known as centering. DIF: Cognitive Level: Application REF: Text Reference: 415 OBJ: Objective: 4 TOP: Topic: Cognitive Development KEY: Nursing Process Step: Assessment MSC: NCLEX: Health Promotion and Maintenance: Growth and Development 19. A 4-year-old child insists that he has more money with a nickel than his father has with a dime. The nurse is aware that this perception is described in Piagets theory as: a Egocentrism . b Artificialism . c Animism . d Centering . ANS: D The intuitive stage, as described by Piaget, is prelogical thinking that is based on the outside appearance of objects. A nickel is larger than a dime, and therefore more valuable Chapter 28 Growth and Development of the School-Age Child 1. The nurse is aware that, in general, the school-age child will: a Grow 3 to 6 inches/year . b Gain 5 to 7 pounds/year . c Increase head circumference by 1 inch/year . d Reach a visual acuity of 20/20 by 9 years of age . ANS: B During the school-age period, the average weight gain per year is generally 5.5 to 7 pounds. DIF: Cognitive Level: Knowledge REF: Text Reference: 429 OBJ: Objective: 2 TOP: Topic: Physical Growth KEY: Nursing Process Step: Assessment MSC: NCLEX: Health Promotion and Maintenance: Growth and Development 2. The nurse, planning to teach a class on nutrition to fourth-grade students, would keep in mind that school-age children: a Can concentrate on only one aspect of a situation . b Can think abstractly . c Are egocentric in their thinking . d Think logically and concretely . ANS: D Piaget refers to the thought process of this period as concrete operations, which involves logical thinking and an understanding of cause and effect. DIF: Cognitive Level: Analysis REF: Text Reference: 429 OBJ: Objective: 2 TOP: Topic: Cognitive Development KEY: Nursing Process Step: Planning MSC: NCLEX: Health Promotion and Maintenance: Growth and Development 3. The nurse explains that the preferred social interaction for the school-age child is based on relationships that are: a Heterosexual interest groups . b Association with one best friend . c Organized groups like Boy Scouts . d Same-sex peer groups . ANS: D The preferred social interaction of the school-age child is in same-sex peer groups or cliques. DIF: Cognitive Level: Analysis REF: Text Reference: 429 OBJ: Objective: 2 TOP: Topic: Social Development, Play KEY: Nursing Process Step: Implementation MSC: NCLEX: Health Promotion and Maintenance: Growth and Development 4. The nurse advises the parents of a 10-year-old boy that, according to Eriksons theory, the most developmentally supportive experience for him would be: a Constant variety of activities . b Successful performance in Little League . c Feeling healthy and strong . d Having a girl friend . ANS: B The child who is successful in activities will feel positively about himself or herself. DIF: Cognitive Level: Analysis REF: Text Reference: 429 OBJ: Objective: 2 TOP: Topic: Psychosocial Development KEY: Nursing Process Step: Implementation MSC: NCLEX: Health Promotion and Maintenance: Growth and Development 5. The parents of an 8-year-old tell the nurse the child wakes the household crying out during his frequent nightmares. The nurses most helpful response is to explain that nightmares are: a A normal extension of the childs fear of mutilation . b An abnormal response to repressed feelings . c A common result of latent sexuality . d A side effect of overactivity and stimulation . ANS: A The nightmares experienced by an 8-year-old are an extension of their characteristic fear of mutilation. DIF: Cognitive Level: Comprehension REF: Text Reference: 438, Table 19-3 OBJ: Objective: 2 TOP: Topic: Eight-Year-Old Nightmares KEY: Nursing Process Step: Implementation MSC: NCLEX: Health Promotion and Maintenance: Growth and Development 6. The nurse suggests an appropriate toy for a hospitalized 6-year-old boy would be a: a Game Boy game . b . Compact disc player c . Adventure book d . Jigsaw puzzle ANS: A The 6-year-old child can perform numerous feats that require muscle coordination. At this age, the Gameboy toy will offer nonaggressive competition. DIF: Cognitive Level: Analysis REF: Text Reference: 433 OBJ: Objective: 2 TOP: Topic: Six-Year-Old KEY: Nursing Process Step: N/A MSC: NCLEX: Health Promotion and Maintenance: Growth and Development 7. The nurse discusses preparation for school with the parents of a 6-year-old girl who will soon be starting first grade. The nurse determines that the parents understood the information when the girls father states: a We should put a stop to her thumb-sucking. . b Well have a talk about what school is like. . c We will let her walk to the bus stop by herself. . d Well have her meet some children who will be in her class. . ANS: D To prepare a child for school, parents can arrange for the child to meet other children who will be entering school with her. DIF: Cognitive Level: Application REF: Text Reference: 443, Box 19-2 OBJ: Objective: 3 TOP: Topic: Parental Guidance for Starting School KEY: Nursing Process Step: Evaluation MSC: NCLEX: Health Promotion and Maintenance: Growth and Development 8. A 9-year-old boy is often cranky and irritable and his school performance has declined. All the options are true about the child. The possible factor causing this behavior is that he: a Sleeps only 6 to 7 hours a night . b Eats eggs every day . c Has a new dog . d Plays about 1 to 3 hours each evening . ANS: A The 9-year-old child requires about 10 hours of sleep per night. DIF: Cognitive Level: Analysis REF: Text Reference: 436 OBJ: Objective: 2 TOP: Topic: Nine-Year-Old Child KEY: Nursing Process Step: Assessment MSC: NCLEX: Health Promotion and Maintenance: Growth and Development 9. A parent asked the nurse, At what age are children capable of assuming more responsibility for personal belongings? Based on a knowledge of growth and development, the nurse would respond: a 6 years . b 7 years . c 9 years . d 12 years . ANS: C The 9-year-old is dependable and assumes more responsibility for personal belongings. DIF: Cognitive Level: Comprehension REF: Text Reference: 436 OBJ: Objective: 2 TOP: Topic: Development KEY: Nursing Process Step: Implementation MSC: NCLEX: Health Promotion and Maintenance: Growth and Development 10. The school nurse who is preserving a tooth that was knocked out on the school yard will be especially careful to: a Wrap the tooth loosely in a clean cloth. . b Rinse the tooth with alcohol. . c Handle the tooth only by the crown. . d Place the tooth in a warm environment. . ANS: C When a permanent tooth is avulsed, the tooth should be picked up by the crown to prevent any further damage to the root and placed in milk until the child can be examined by a dentist. DIF: Cognitive Level: Application REF: Text Reference: 431 OBJ: Objective: 6 TOP: Topic: Safety KEY: Nursing Process Step: Implementation MSC: NCLEX: Physiological Integrity: Reduction of Risk 11. A parent states, My 7-year-old really wants a dog. His 10-year-old brother has allergies to animal dander. I dont know what to do. The nurse could advise this parent to: a Choose a small breed of dog because the large dogs produce more allergens. . b An older unneutered dog produces fewer allergens than a younger one. . c A cat may be a good choice since it requires less care and is less allergenic. . d Poodles do not shed, making this dog a good choice for people with allergies. . ANS: D The poodle breed of dog does not have a shed cycle and so it may be the least offensive pet for the allergic child. DIF: Cognitive Level: Analysis REF: Text Reference: 441 OBJ: Objective: 7 TOP: Topic: Pet Ownership KEY: Nursing Process Step: Implementation MSC: NCLEX: Physiological Integrity: Physiological Adaptation 12. When asked about her activities, a 10-year-old girl responded, I like school. I play the flute in the school band and I take tennis lessons. The nurse knows these activities will help this child develop a sense of: a Initiative . b Industry . c Identity . d Intimacy . ANS: B The school-age period is referred to by Erikson as the stage of industry. Successful participation in activities facilitates the childs sense of industry. DIF: Cognitive Level: Analysis REF: Text Reference: 438, Table 19-3 OBJ: Objective: 2 TOP: Topic: Psychosocial Development KEY: Nursing Process Step: Assessment MSC: NCLEX: Health Promotion and Maintenance: Growth and Development 13. A mother reports that she has a new job and her 12-year-old child is home alone for a time after school. The statement made by the parent, indicating a potentially unsafe situation for this child, is: a I told him that he could invite a few friends after school. . b I put a list of emergency numbers next to the telephone. . c Last week we made a first aid kit together. . d There is a neighbor available in case of an emergency. . ANS: A Latchkey children are subject to a higher rate of accidents. Permitting school-age children and their friends to be home alone in an unsupervised environment is an unsafe situation. DIF: Cognitive Level: Analysis REF: Text Reference: 439 OBJ: Objective: 3 TOP: Topic: Latchkey Children KEY: Nursing Process Step: Evaluation MSC: NCLEX: Health Promotion and Maintenance: Growth and Development 14. A mother is concerned because her 9-year-old has developed the habit of twitching his eyes and flipping his hair while communicating with anyone. The best nursing response to this parent is: a This may indicate that he needs eyeglasses. . b Children sometimes do these things for attention. . c This behavior suggests low self-esteem. . d Tics appear when a child is under stress. . ANS: D The child cannot help such actions and should not be scolded for them because they are mainly a result of tension. DIF: Cognitive Level: Analysis REF: Text Reference: 436 OBJ: Objective: 2 TOP: Topic: Nine-Year-Old Child KEY: Nursing Process Step: Implementation MSC: NCLEX: Health Promotion and Maintenance: Growth and Development 15. A seventh-grade girl tells the school nurse that her art teacher, also a female, is her hero. The most appropriate interpretation of the girls comment is: a . The student may be exploring her career options. b . The comment is cause for concern about sexual abuse. c . The child may have difficulty interacting with her peers. d Hero worship is a normal phenomenon. . ANS: D School-age children tend to admire their teachers and adult companions. For the 11- to 12-yearold, hero worship is a normal phenomenon. DIF: Cognitive Level: Analysis REF: Text Reference: 429 OBJ: Objective: 2 TOP: Topic: Social Development KEY: Nursing Process Step: Assessment MSC: NCLEX: Health Promotion and Maintenance: Growth and Development 16. According to Piaget, a 9-year-old child is in which stage of cognitive development? a . Formal operations b . Preoperational c . Concrete operations d . Sensorimotor ANS: C School-age children are in the concrete operations stage of cognitive development. DIF: Cognitive Level: Knowledge REF: Text Reference: 429 OBJ: Objective: 2 TOP: Topic: Cognitive Development KEY: Nursing Process Step: Assessment MSC: NCLEX: Health Promotion and Maintenance: Growth and Development 17. The nurse assesses that the 11-year-old has moved from the mind set of egocentrism when he says: a . I am a member of the best Cub Scout group in the world. b . I must do my homework before I can play. c . My dad can do anything! d . Im sorry. I bet that hurt your feelings. ANS: D The ability to see anothers point of view indicates moving away from egocentrism into a more altruistic mind set. DIF: Cognitive Level: Analysis REF: Text Reference: 429 OBJ: Objective: 5 TOP: Topic: Increasing Understanding KEY: Nursing Process Step: Assessment MSC: NCLEX: Health Promotion and Maintenance: Growth and Development 18. When the school-age child becomes frustrated with a school assignment and says, I cant do this!, the parent should: a . Ask, What is it that is so difficult? b . Allow the child to quit the effort. c . Call in older siblings to help. d . Finish the project for him. ANS: A Helping the child focus on the problem that is keeping him from mastery can limit frustration. Quitting or having someone else finish is detrimental to the development of industry. DIF: Cognitive Level: Analysis REF: Text Reference: 429 OBJ: Objective: 2 TOP: Topic: Industry KEY: Nursing Process Step: N/A MSC: NCLEX: Health Promotion and Maintenance: Growth and Development MULTIPLE RESPONSE 1. The nurse, in attempting to help a 7-year-old girl express her feelings about being in a new school, would prompt the child with basic feeling words, such as: Select all that apply. a Mad . b Glad . c Sad . d Scared . e Jealous . ANS: A, B, C, D The words mad, glad, sad, and scared are basic feeling words that can prompt a young child to better express his or her feelings. DIF: Cognitive Level: Application REF: Text Reference: 440 OBJ: Objective: 2 TOP: Topic: Expression of Feelings KEY: Nursing Process Step: Implementation MSC: NCLEX: Psychosocial Integrity: Psychosocial Adaptation COMPLETION 1. The nurse advises the parents of a 6-year-old to try and ensure at least hours of sleep daily. ANS: 11 DIF: Cognitive Level: Comprehension REF: Text Reference: 434 OBJ: Objective: 2 TOP: Topic: Sleep Needs KEY: Nursing Process Step: Implementation MSC: NCLEX: Health Promotion and Maintenance: Growth and Development NOT: Rationale: The 6-year-old school-age child needs at least 11 hours of sleep. 2. The nurse reminds the parents who are trying to select a dog for their allergic child that the best selection would be a female dog that is and . ANS: young, neutered DIF: Cognitive Level: Comprehension REF: Text Reference: 441 OBJ: Objective: 7 TOP: Topic: Pet Selection for Allergic Child KEY: Nursing Process Step: Implementation MSC: NCLEX: Health Promotion and Maintenance: Prevention and Early Detection of Disease NOT: Rationale: Young, neutered female dogs produced less allergens. 3. When the fifth-grade class collected geckos in a special aquarium in the classroom, the school nurse cautioned the teacher to be alert for symptoms of that can be carried by the reptiles. ANS: Salmonella DIF: Cognitive Level: Comprehension REF: Text Reference: 440, Table 19-14 OBJ: Objective: 7 TOP: Topic: Salmonella KEY: Nursing Process Step: Implementation MSC: NCLEX: Health Promotion and Maintenance: Prevention and Early Detection of Disease NOT: Rationale: Geckos can infect humans with Salmonella. 4. The pediatric nurse assesses the child who has been diagnosed with diabetes to ensure that he does to come to believe that his disease is a form of . ANS: punishment Chapter 29 Growth and Development of the Adolescent 1. When assessing a 13-year-old boy, the nurse would keep in mind physical changes in the pubertal male, beginning with: a . Development of axillary and facial hair b . Enlargement of pectoral muscles c . Enlargement of testicles d . Voice changes ANS: C In boys, pubertal changes begin with enlargement of the testicles and internal structures. DIF: Cognitive Level: Comprehension REF: Text Reference: 447 OBJ: Objective: 4 TOP: Topic: Physical Development KEY: Nursing Process Step: Assessment MSC: NCLEX: Health Promotion and Maintenance: Growth and Development 2. A 13-year-old boy states, The girls in my class tower over me. The nurses most informative response would be: a . It may seem that way because girls have a growth spurt 2 years earlier than boys. b . Perhaps your parents are not exceptionally tall. c . Boys usually experience a growth spurt 1 year earlier than girls. d . You may feel short, but you are actually average height for your age. ANS: A Although the age for growth spurts during puberty varies, growth spurts occur 2 years earlier for girls than for boys. DIF: Cognitive Level: Application REF: Text Reference: 445 OBJ: Objective: 4 TOP: Topic: Physical Development KEY: Nursing Process Step: Implementation MSC: NCLEX: Health Promotion and Maintenance: Growth and Development 3. A parent comments that her adolescent daughter seems to be daydreaming a lot these days. The nurse understands that this behavior indicates she is: a Bored . b Not getting enough rest . c Trying to block out stress and anxiety . d Mentally preparing for real situations . ANS: D Daydreaming allows adolescents to act out in their imaginations what will be said or done in certain situations. This helps them to prepare for and cope with interactions with others. DIF: Cognitive Level: Analysis REF: Text Reference: 453 OBJ: Objective: 4 TOP: Topic: Development-Daydreams KEY: Nursing Process Step: Assessment MSC: NCLEX: Health Promotion and Maintenance: Growth and Development 4. The nurse planning a safety program for high school students should understand that most accidental deaths in adolescence are related to: a Firearms . b Automobiles . c Drowning . d Diving injuries . ANS: B The chief safety hazard for the adolescent is automobiles. DIF: Cognitive Level: Knowledge REF: Text Reference: 458 OBJ: Objective: 11 TOP: Topic: Safety KEY: Nursing Process Step: Planning MSC: NCLEX: Health Promotion and Maintenance: Growth and Development 5. A 16-year-old excitedly tells his parents that he was offered a part-time job. Which response represents an effective problem-solving approach for his parents? a Your studies are too important for you to have a part-time job. . b When we went to high school, academics were the teenagers priority. . c We want you to put your earnings in a savings account. . d How do you think you will manage your school work and a job? . ANS: D An effective approach to help adolescents learn to solve problems is for parents to guide them in exploring alternatives. DIF: Cognitive Level: Application REF: Text Reference: 454 OBJ: Objective: 11 TOP: Topic: Parenting KEY: Nursing Process Step: Evaluation MSC: NCLEX: Health Promotion and Maintenance: Growth and Development 6. One psychosocial task of adolescence on which the nurse must focus when planning care, is the development of a sense of: a Initiative . b Industry . c Identity . d Involvement . ANS: C Psychosocial milestones that must be accomplished during adolescence include the five Isimage of self, identity, independence, interpersonal relationships, and intellectual maturity. DIF: Cognitive Level: Knowledge REF: Text Reference: 445, Box 20-1 OBJ: Objective: 2 TOP: Topic: Psychosocial Development KEY: Nursing Process Step: Planning MSC: NCLEX: Health Promotion and Maintenance: Growth and Development 7. A 13-year-old female tells the school nurse that she is getting fat, especially in her hips and legs. The understanding by the nurse that would best guide the response is: a Many teenagers are unaware of proper nutrition. . b Teenagers of this age become less active and should eat fewer calories. . c Puberty is often preceded by fat deposits in these areas. . d As soon as menarche occurs, she will lose this excess weight. . ANS: C Secondary sexual characteristics become apparent before menarche. Fat is deposited in the hips, thighs, and breasts, causing them to enlarge. DIF: Cognitive Level: Analysis REF: Text Reference: 449, Box 20-2 OBJ: Objective: 4 TOP: Topic: Physical Development KEY: Nursing Process Step: Planning MSC: NCLEX: Health Promotion and Maintenance: Growth and Development 8. The school nurse is planning a program for girls about the physical changes of puberty; this program should be directed to girls of the age: a 16 years . b 14 years . c 12 years . d 10 years . ANS: D Because puberty can occur in girls as early as age 10 years, instruction must be given by that age. DIF: Cognitive Level: Comprehension REF: Text Reference: 454 OBJ: Objective: 4 TOP: Topic: Physical Development KEY: Nursing Process Step: Planning MSC: NCLEX: Health Promotion and Maintenance: Growth and Development 9. The statement made by a parent indicating understanding about helping a 13-year-old manage allowance money is: a I set amounts he can earn for particular chores. . b I give him a certain amount of money for each day. . c I put money into his bank account each month. . d I told him to ask me when he needs money. . ANS: A If money is simply handed out as requested, it is difficult to develop responsibility for finances and money management. The older adolescent is able to get a job. The younger teen can earn money by doing particular chores. DIF: Cognitive Level: Application REF: Text Reference: 453 OBJ: Objective: 4 TOP: Topic: Development-Responsibility KEY: Nursing Process Step: Evaluation MSC: NCLEX: Health Promotion and Maintenance: Growth and Development 10. The nurse suggests a good dietary source of zinc for an adolescent who is a vegetarian would be: a Green, leafy vegetables . b Citrus fruits . c Nuts . d Enriched breads . ANS: C Zinc is essential for growth and sexual maturation in adolescence. Good vegetable sources include nuts, legumes, and wheat germ. DIF: Cognitive Level: Comprehension REF: Text Reference: 457 OBJ: Objective: 9 TOP: Topic: Nutrition KEY: Nursing Process Step: Assessment MSC: NCLEX: Health Promotion and Maintenance: Growth and Development 11. An adolescents parent comments, My son seems so preoccupied with his appearance these days. Is this normal? The nurses best response would be: a It is his attempt to express his individualism. . b His preoccupation with his looks is quite normal. . c He is probably troubled with his physical changes. . d This shows that he has a positive self-image. . ANS: B Preoccupation with self-image is normal and accounts for the constant primping of teenagers. DIF: Cognitive Level: Application REF: Text Reference: 445 OBJ: Objective: 4 TOP: Topic: Development KEY: Nursing Process Step: Planning MSC: NCLEX: Health Promotion and Maintenance: Growth and Development 12. Foods that would be a healthy choice for an adolescent who just finished playing in a strenuous game would be: a Cheeseburger and soda . b Hot fudge sundae . c Two Egg McMuffins and orange juice . d Bagel and skim milk . ANS: D A bagel provides a rapid supply of carbohydrates to the muscles, and skim milk provides a slow release of carbohydrates to the muscles. DIF: Cognitive Level: Application REF: Text Reference: 457 OBJ: Objective: 9 TOP: Topic: Nutrition KEY: Nursing Process Step: Assessment MSC: NCLEX: Health Promotion and Maintenance: Growth and Development 13. When planning to answer a 16-year-old girls questions about menstruation, the nurse must consider cognitive development. According to Piaget, the cognitive aspect that is developed during adolescence is the ability to: a View a situation from multiple perspectives . b Focus more on the past than present situations . c Exercise concrete reasoning . d Consider hypothetical situations . ANS: D According to Piaget, in the formal operations stage adolescents have the ability to think abstractly. DIF: Cognitive Level: Comprehension REF: Text Reference: 445, Box 20-1 OBJ: Objective: 4 TOP: Topic: Cognitive Development KEY: Nursing Process Step: Planning MSC: NCLEX: Health Promotion and Maintenance: Growth and Development 14. A girl tells the nurse that she and her best friend belong to the popular clique. She states, I love Britney Spears and I want to be a singer. The nurse recognizes the girls statement as characteristic of peer relationships in: a Early adolescence . b Middle adolescence . c Late adolescence . d Entire adolescent period . ANS: A Cliques of unisex friends, having a best friend, and hero worship are characteristics of the early adolescent. DIF: Cognitive Level: Comprehension REF: Text Reference: 452 OBJ: Objective: 4 TOP: Topic: Social Development KEY: Nursing Process Step: Assessment MSC: NCLEX: Health Promotion and Maintenance: Growth and Development 15. The nurse is leading a discussion group with parents of adolescents. One parent comments, My son cant do anything without checking with his friends first. My opinion doesnt count anymore. The nurse would formulate a response on the knowledge that this behavior is: a Unusual for adolescent boys . b . Often more apparent in boys than girls c . A normal phenomenon during adolescence d . Suggestive of feelings of low self-worth ANS: A Parents may need help understanding that the teenagers exaggerated conformity is necessary for moving away from dependence and obtaining approval from persons outside the nuclear family. DIF: Cognitive Level: Application REF: Text Reference: 452 OBJ: Objective: 4 TOP: Topic: Peer Relationships KEY: Nursing Process Step: Planning MSC: NCLEX: Health Promotion and Maintenance: Growth and Development 16. The nurse points out to a group of parents that the most positive developmental significance of a peer group to the adolescent is that the group serves as: a A social outlet . b An association to blur personal identity . c A platform for group think . d An initial separation from family . ANS: D Being a member of a peer group and communicating with and seeking approval from this group are the first separation from the family. DIF: Cognitive Level: Analysis REF: Text Reference: 461 OBJ: Objective: 10 TOP: Topic: Peer Groups KEY: Nursing Process Step: N/A MSC: NCLEX: Health Promotion and Maintenance: Growth and Development 17. The nurse understands that the adolescents avid sexual orientation to be based on Freuds theory, which describes adolescence as the stage. a Conceptual . b Genital . c Glandular . d Pubertal . ANS: B Freud describes the adolescent period as genital. Chapter 30 Atraumatic Care of Children and Families 1. The nurse assessing a newborn recognizes a sign of hypoglycemia, which is: a Increased respiratory rate . b Increased temperature . c Active muscle tone . d High-pitched cry . ANS: D There are many signs of hypoglycemia in the newborn. One is a high-pitched cry. DIF: Cognitive Level: Comprehension REF: Text Reference: 222 OBJ: Objective: 7 TOP: Topic: Care of the Newborn KEY: Nursing Process Step: Assessment MSC: NCLEX: Physiological Integrity: Reduction of Risk 2. The nurse assessing the fundus of the uterus immediately after delivery would expect to find the uterus: a Well-contracted with its upper border at or just below the umbilicus . b Well-contracted with its upper border three or four fingerbreadths above the . umbilicus c Relaxed with its upper border level with the umbilicus . d Relaxed with its upper border two or three fingerbreadths below the umbilicus . ANS: A Immediately after the placenta is expelled, the uterine fundus can be felt as a firm mass, about the size of a grapefruit, at the level of the umbilicus. DIF: Cognitive Level: Analysis REF: Text Reference: 200 OBJ: Objective: 4 TOP: Topic: Postpartum Assessment KEY: Nursing Process Step: Assessment MSC: NCLEX: Physiological Integrity: Physiological Adaptation 3. The statement made by a new mother that indicates she needs additional information about breastfeeding is: a I let the baby nurse 10 to 15 minutes on the first breast and then switch to the . other breast. b The baby needs to nurse at least 5 minutes on the breast to get the hindmilk. . c The baby has been nursing every 2 to 3 hours. . d If the baby gets fussy between feedings, I give her a bottle of water. . ANS: D Supplemental feedings of formula or water should not be offered to a healthy newborn who is breastfeeding. DIF: Cognitive Level: Analysis REF: Text Reference: 228 OBJ: Objective: 8 TOP: Topic: Breastfeeding KEY: Nursing Process Step: Evaluation MSC: NCLEX: Physiological Integrity: Physiological Adaptation 4. Following delivery, the nurses assessment reveals a soft, boggy uterus located above the level of the umbilicus. The appropriate intervention is to: a Notify the doctor . b Massage the fundus . c Initiate measures that encourage voiding . d Position the patient flat . ANS: B A poorly contracted uterus should be massaged until firm to prevent hemorrhage. DIF: Cognitive Level: Application REF: Text Reference: 201 OBJ: Objective: 4 TOP: Topic: Postpartum Assessment KEY: Nursing Process Step: Implementation MSC: NCLEX: Physiological Integrity: Physiological Adaptation 5. The nurse assesses the initial lochia postdelivery, which is: a Serosa . b Rubra . c Alba . d Vaginalis . ANS: B The initial vaginal discharge after delivery is called lochia rubra. It is red and moderately heavy. Lochia rubra lasts for up to 3 days postpartum. DIF: Cognitive Level: Knowledge REF: Text Reference: 201 OBJ: Objective: 4 TOP: Topic: Postpartum Changes-Reproductive System KEY: Nursing Process Step: Implementation MSC: NCLEX: Physiological Integrity: Physiological Adaptation 6. A woman will be discharged 48 hours after a vaginal delivery. When planning discharge teaching, the information the nurse would include about lochia is: a Lochia should disappear 2 to 4 weeks postpartum. . b It is normal for the lochia to have a slightly foul odor. . c A change in lochia from pink to bright red should be reported. . d A decrease in flow will be noticed with ambulation and activity. . ANS: C A return to bright red lochia rubra may indicate a late postpartum hemorrhage. DIF: Cognitive Level: Application REF: Text Reference: 202 OBJ: Objective: 4 TOP: Topic: Postpartum Changes-Reproductive System KEY: Nursing Process Step: Planning MSC: NCLEX: Physiological Integrity: Physiological Adaptation 7. The nurse should teach the postpartum woman about perineal self-care by instructing her to: a Perform perineal self-care at least twice a day . b Cleanse with warm water in a squeeze bottle from front to back . c Remove perineal pads from the rectal area toward the vagina . d Use cool water to decrease edema of the perineum . ANS: B Cleansing from front to back prevents contamination from the rectal area. DIF: Cognitive Level: Application REF: Text Reference: 205 OBJ: Objective: 4 TOP: Topic: Postpartum Changes-Reproductive System KEY: Nursing Process Step: Implementation MSC: NCLEX: Health Promotion and Maintenance: Prevention and Early Detection 8. The nurse can expect which of the following interventions to be ordered if the postpartum woman is not immune to rubella? a The rubella virus vaccine should be administered before discharge. . b The woman should receive the rubella virus vaccine at her 6-week postpartum . checkup. c The woman should be instructed not to get pregnant until she receives the . rubella vaccine. d No intervention is indicated at this time because the woman is not at risk for . rubella. ANS: A The woman who is not immune to rubella is immunized in the immediate postpartum period because there is no danger of her being pregnant. DIF: Cognitive Level: Analysis REF: Text Reference: 208 OBJ: Objective: 4 TOP: Topic: Postpartum Changes-Immune System KEY: Nursing Process Step: Planning MSC: NCLEX: Health Promotion and Maintenance: Prevention and Early Detection 9. The statement that indicates the new mother is breastfeeding correctly is: a . I will put the baby first on the breast that she took last in the previous feeding. b . I keep the baby on a 4-hour feeding schedule. c I let the baby stay on the first breast for 20 minutes. . d I put only the nipple in the babys mouth when I am breastfeeding. . ANS: A Alternating breasts increases milk production, particularly hindmilk, which has a higher protein and fat content. DIF: Cognitive Level: Analysis REF: Text Reference: 227 OBJ: Objective: 8 TOP: Topic: Breastfeeding KEY: Nursing Process Step: Evaluation MSC: NCLEX: Physiological Integrity: Physiological Adaptation 10. The nurse counseling a lactating mother about diet would include instructions to: a . Consume 500 more calories than her usual prepregnancy diet. b . Eat less meat and more fruits and vegetables. c . Drink 3 to 4 tall glasses of fluid daily. d . Eat 1,000 more calories than her usual prepregnancy diet. ANS: A To maintain nutrient stores while breastfeeding, the mother needs 500 additional calories each day over her prepregnancy diet. DIF: Cognitive Level: Comprehension REF: Text Reference: 230 OBJ: Objective: 8 TOP: Topic: Breastfeeding-Maternal Nutrition KEY: Nursing Process Step: Implementation MSC: NCLEX: Physiological Integrity: Physiological Adaptation 11. When a woman asks about resumption of her menstrual cycle after childbirth, the nurse responds that: a . A woman will not ovulate in the absence of menstrual flow. b . Most nonlactating women resume menstruation about 2 months postpartum. c . Generally, a woman does not ovulate in the first few cycles after childbirth. d . The return of menstruation is delayed when a woman does not breastfeed. ANS: B Menstrual periods resume in about 6 to 8 weeks if the woman is not breastfeeding. DIF: Cognitive Level: Comprehension REF: Text Reference: 204 OBJ: Objective: 4 TOP: Topic: Postpartum Changes KEY: Nursing Process Step: Implementation MSC: NCLEX: Physiological Integrity: Physiological Adaptation 12. The nurse explains that the physician will order RhoGAM in the event that a/an: a . Unsensitized Rh-negative mother has an Rh-positive pregnancy. b . Rh-negative mother becomes sensitized. c . Sensitized infant has a rising bilirubin level. d . Unsensitized infant exhibits no outward signs. ANS: A The Rh-negative woman should receive RhoGAM within 72 hours after the birth of an Rhpositive infant. DIF: Cognitive Level: Analysis REF: Text Reference: 208 OBJ: Objective: 4 TOP: Topic: Postpartum Changes-Immune System KEY: Nursing Process Step: Implementation MSC: NCLEX: Health Promotion and Maintenance: Prevention and Early Detection of Disease 13. After birth, the nurse quickly dries and wraps the newborn in a blanket to prevent heat loss by: a Conduction . b Radiation . c Evaporation . d Convection . ANS: C Newborns lose heat quickly after birth as fluid evaporates from their bodies. DIF: Cognitive Level: Comprehension REF: Text Reference: 215 OBJ: Objective: 7 TOP: Topic: Newborn Care KEY: Nursing Process Step: Implementation MSC: NCLEX: Physiological Integrity: Physiological Adaptation 14. The nurses instructions for a new mother to care for the babys umbilical cord will include: a The area should be kept covered with a sterile dressing. . b Clean the stump with alcohol at every diaper change. . c Keep the clamp on until the cord falls off. . d Give the newborn a daily tub bath until the cord falls off. . ANS: B Alcohol is applied at each diaper change to promote drying of the cord, thus preventing infection. DIF: Cognitive Level: Application REF: Text Reference: 219 OBJ: Objective: 10 TOP: Topic: Newborn Care KEY: Nursing Process Step: Implementation MSC: NCLEX: Health Promotion and Maintenance: Prevention and Early Detection of Disease 15. A new mother states her preference to formula-feed her newborn. The nurse planning discharge instructions would tell her about a measure to help suppress lactation and promote comfort, which is: a Wear a well-fitting bra continuously for several days. . b Stand in a warm shower, letting the water spray over the breasts. . c Express small amounts of milk from the breasts several times a day. . d Massage the breasts when they ache. . ANS: A When a mother does not wish to breastfeed, a snug bra worn around the clock can help alleviate discomfort from engorgement. DIF: Cognitive Level: Application REF: Text Reference: 204-205 OBJ: Objective: 4 TOP: Topic: Postpartum Changes-Reproductive System KEY: Nursing Process Step: Planning MSC: NCLEX: Health Promotion and Maintenance 16. On the second postpartum day, a mother bathed her newborn for the first time. She tells the nurse, I dont think I did it right. Based on the mothers comment, she is most likely in the postpartum psychological stage of: a Taking in . b Taking hold . c Letting go . d Settling down . ANS: B In phase 2, taking hold, the mother begins to initiate action and becomes interested in caring for the baby. In doing so, she may become critical of her performance. DIF: Cognitive Level: Analysis REF: Text Reference: 211 OBJ: Objective: 6 TOP: Topic: Postpartum Changes-Emotional Care KEY: Nursing Process Step: Assessment MSC: NCLEX: Psychosocial Integrity: Physiological Adaptation 17. A primipara tells the nurse, My afterpains get worse when I am breastfeeding. The most appropriate nursing response would be: a . Ill get you some aspirin to relieve the cramping that you feel. b . Afterpains are more intense with your first baby. c . Breastfeeding releases a hormone that causes your uterus to contract. d A change of position when youre breastfeeding might help. . ANS: C Breastfeeding mothers may have more afterpains because infant suckling causes the posterior pituitary to release oxytocin, which is a hormone that contracts the uterus. DIF: Cognitive Level: Analysis REF: Text Reference: 201 OBJ: Objective: 4 TOP: Topic: Postpartum Changes-Reproductive System KEY: Nursing Process Step: Implementation MSC: NCLEX: Physiological Integrity: Physiological Adaptation 18. A new mother has decided not to breastfeed her newborn. The nurse planning to teach the mother about formula feeding would include: a . Position the bottle so that the nipple is full of formula during the entire feeding. b . Infant formula can be heated safely in a microwave. c . Burp the baby after 4 ounces and again when the bottle is empty. d . Do not prop a bottle for a feeding until the baby is older. ANS: A The nipple of the bottle should be kept full of formula to reduce the amount of air the baby swallows. DIF: Cognitive Level: Comprehension REF: Text Reference: 232 OBJ: Objective: 8 TOP: Topic: Formula Feeding KEY: Nursing Process Step: Planning MSC: NCLEX: Physiological Integrity: Reduction of Risk 19. In the recovery room, the nurse checks the newly delivered womans fundus following a cesarean section. How would the nurse proceed with this assessment? a . Palpate from the midline to the side of body b . Palpate from the symphysis to the umbilicus c . Palpate from the side of the uterus to the midline d . Massage the abdomen in a circular motion ANS: C The fundus is checked gently by walking the fingers from the side of the uterus to the midline. DIF: Cognitive Level: Application REF: Text Reference: 211 OBJ: Objective: 5 TOP: Topic: Postpartum Assessment-Cesarean Birth KEY: Nursing Process Step: Implementation MSC: NCLEX: Health Promotion and Maintenance: Prevention and Early Detection of Disease 20. The nurse instructed a postpartum woman about storing and freezing breast milk. The nurse determines that the teaching was effective when the woman says: a . I can thaw frozen breast milk in the microwave. b . Ill put enough breast milk for one day in a container. c . Breast milk can be stored in glass containers. d . Breast milk can be kept in the refrigerator for up to 3 months. ANS: C Breast milk can be safely stored in glass or clear hard plastic containers. DIF: Cognitive Level: Analysis REF: Text Reference: 230 OBJ: Objective: 8 TOP: Topic: Breastfeeding KEY: Nursing Process Step: Evaluation MSC: NCLEX: Safe, Effective Care Environment: Safety and Infection Control MULTIPLE RESPONSE 1. While instructing a new mother on formula preparation, the nurse will include information about formula choices, such as: Select all that apply. a Ready-to-feed formula . b Concentrated liquid formula . c . Powdered formula d . Cows milk e . Canned evaporated milk ANS: A, B, C Formula choices are ready-to-use, concentrated liquid formula that will be diluted according to the babys needs, and powdered formula that is mixed as needed. Cows milk and canned evaporated milk are unsuitable because they are nutritionally inadequate and stress the kidneys. DIF: Cognitive Level: Comprehension REF: Text Reference: 231 OBJ: Objective: 8 TOP: Topic: Formula Choices KEY: Nursing Process Step: Implementation MSC: NCLEX: Health Promotion and Maintenance: Growth and Development COMPLETION 1. The nurse assesses a 6-inch stain of rubra lochia on a pad that was worn for 2 hours. The nurse would document this as a amount of lochia. ANS: moderate DIF: Cognitive Level: Application REF: Text Reference: 202, Skill 9-1 OBJ: Objective: 4 TOP: Topic: Estimating Lochia Discharge KEY: Nursing Process Step: Assessment MSC: NCLEX: Physiological Integrity: Reduced Risk NOT: Rationale: A 6-inch stain on a pad worn 2 hours is regarded as a moderate amount of lochia discharge. 2. The nurse explains that the only absolute contraindication for a mother to breastfeed her child is infection. ANS: HIV Chapter 31 Health Supervision & Chapter 32 Health Assessment of Children MULTIPLE CHOICE 1. The nurse percussing over an empty stomach expects to hear which sound? a Tympany . b Resonance . c Flatness . d Dullness . ANS: A Feedback A Tympany is a high-pitched, loud-intensity sound heard over air-filled body parts such as the stomach and bowel. B Resonance is a low-pitched, low-intensity sound elicited over hollow organs such as the lungs. C Flatness is a high-pitched, soft-intensity sound elicited by percussing over solid masses such as bone or muscle. D Dullness is a medium-pitched, medium-intensity sound elicited when percussing over high-density structures such as the liver. PTS: 1 DIF: Cognitive Level: Application REF: p. 806 | Box 33-1 OBJ: Nursing Process: Assessment MSC: Client Needs: Health Promotion and Maintenance 2. You are the nurse admitting a toddler to the pediatric infectious disease unit. What is the single most important component of the childs physical examination? a . Assessment of heart and lungs b . Measurement of height and weight c . Documentation of parental concerns d . Obtaining an accurate history ANS: D Feedback A Heart and lung assessment is not as important as an accurate history. B A single measurement of height and weight is not as significant as determining growth over time. The childs growth pattern can be elicited from the history. C Documentation of parental concerns is not as relevant to the physical examination as an accurate history. D An accurate history is most helpful in identifying problems and potential problems. PTS: 1 DIF: Cognitive Level: Application REF: p. 807 OBJ: Nursing Process: Assessment MSC: Client Needs: Health Promotion and Maintenance 3. In which section of the health history should the nurse record that the parent brought the infant to the clinic today because of frequent diarrhea? a . Review of systems b . Chief complaint c . Lifestyle and life patterns d . Health history ANS: B Feedback A The review of systems includes past health functions of body systems. B The chief complaint is documented using the childs or parents words for the reason the child was brought to the health care center. C Lifestyle and life patterns include the childs interaction with the social, psychological, physical, and cultural environment. D Health history includes birth history, growth and development, common childhood illnesses, immunizations, hospitalizations, injuries, and allergies. PTS: 1 DIF: Cognitive Level: Comprehension REF: p. 808 | Box 33-4 OBJ: Nursing Process: Implementation MSC: Client Needs: Health Promotion and Maintenance 4. Which choice includes the components of a complete pediatric history? a . Statistical information, client profile, health history, family history, review of systems, lifestyle and life patterns b . Vital signs, chief complaint, and list of previous problems c . Chief complaint, including body location, quality, quantity, timeframe, and alleviating and aggravating factors d . Pertinent developmental and family information ANS: A Feedback A The identified items are included in a complete pediatric history. B Vital signs, chief complaint, and list of previous problems do not constitute a complete history. C A problem-oriented history includes specific information about the chief complaint. D Pertinent developmental and family information are part of the complete history. PTS: 1 DIF: Cognitive Level: Application REF: p. 807 | Box 33-3 OBJ: Nursing Process: Assessment MSC: Client Needs: Health Promotion and Maintenance 5. The nurse is performing a comprehensive physical examination on a young child in the hospital. At what age can the nurse expect a childs head and chest circumferences to be almost equal? a Birth . b 6 months . c 1 year . d 3 years . ANS: C Feedback A Head circumference is larger than chest circumference until approximately 12 months of age. B Chest circumference is smaller than head circumference until approximately 1 year of age. C Head and chest measurements are almost equal at 1 year of age. D By 3 years of age, the chest circumference exceeds the head circumference. PTS: 1 DIF: Cognitive Level: Application REF: p. 811 OBJ: Nursing Process: Assessment MSC: Client Needs: Health Promotion and Maintenance 6. An 8-year-old girl asks the nurse how the blood pressure apparatus works. The most appropriate nursing action is to a . Ask her why she wants to know. b . Determine why she is so anxious. c . Explain in simple terms how it works. d Tell her she will see how it works as it is used. . ANS: C Feedback A The nurse should respond positively for requests for information about procedures and health information. By not responding, the nurse may be limiting communication with the child. B The child is not exhibiting anxiety, just requesting clarification of what will be occurring. C School-age children require explanations and reasons for everything. They are interested in the functional aspect of all procedures, objects, and activities. It is appropriate for the nurse to explain how equipment works and what will happen to the child. D The nurse must explain how the blood pressure cuff works so that the child can then observe during the procedure. PTS: 1 DIF: Cognitive Level: Comprehension REF: p. 805 OBJ: Nursing Process: Implementation MSC: Client Needs: Health Promotion and Maintenance 7. Which chart should the nurse use to assess the visual acuity of an 8-year-old child? a Lea chart . b Snellen chart . c HOTV chart . d Tumbling E chart . ANS: B Feedback A The Lea chart tests vision using four different symbols designed for use with preschool children. B The Snellen chart is used to assess the vision of children older than 6 years of age. C The HOTV chart tests vision by using graduated letters and is designed for use with children ages 3 to 6 years. D The tumbling E chart uses the letter E in various directions and is designed for use with children ages 3 to 6 years. PTS: 1 DIF: Cognitive Level: Comprehension REF: p. 817 | Box 33-8 OBJ: Nursing Process: Implementation MSC: Client Needs: Health Promotion and Maintenance 8. Which action is appropriate when the nurse is assessing breath sounds of an 18-month-old crying child? a Ask the parent to quiet the child so the nurse can listen. . b Auscultate breath sounds and chart that the child was crying. . c Encourage the child to play with the stethoscope to distract and to calm down . before auscultating. d Document that data are not available because of noncompliance. . ANS: C Feedback A Asking a parent to quiet the child may or may not work. B Auscultating while the child is crying typically results in less than optimal data. C Distracting the child with an interesting activity can assist the child to calm down so an accurate assessment can be made. D Documenting that the child is not compliant is not appropriate. An assessment needs to be completed. PTS: 1 DIF: Cognitive Level: Application REF: p. 805 OBJ: Nursing Process: Implementation MSC: Client Needs: Health Promotion and Maintenance 9. The nurse is obtaining vital signs on a 1-year-old child. What is the most appropriate site for assessing the pulse rate? a Apical . b Radial . c Carotid . d Femoral . ANS: A Feedback A Apical pulse rates are taken in children younger than 2 years. B Radial pulse rates may be taken in children older than 2 years. C It is difficult to palpate the carotid pulse in an infant. D The femoral pulse is palpated when comparing peripheral pulses, but it is not used to measure an infants pulse rate. PTS: 1 DIF: Cognitive Level: Application REF: p. 809 OBJ: Nursing Process: Assessment MSC: Client Needs: Health Promotion and Maintenance 10. A nurse is reviewing pediatric physical assessment techniques. Which statement about performing a pediatric physical assessment is correct? a . Physical examinations proceed systematically from head to toe unless developmental considerations dictate otherwise. b . The physical examination should be done with parents in the examining room for children of any age. c . Measurement of head circumference is done until the child is 5 years old. d . The physical examination is done only when the child is cooperative. ANS: A Feedback A Physical assessment usually proceeds from head to toe; however, developmental considerations with infants and toddlers dictate that the least threatening assessments be done first to obtain accurate data. B Having parents in the examining room with adolescents is not appropriate. C Head circumference is routinely measured until 36 months of age. D Children will not always be cooperative during the physical examination. The examiner will need to incorporate communication and play techniques to facilitate cooperation. PTS: 1 DIF: Cognitive Level: Comprehension REF: p. 804 OBJ: Nursing Process: Evaluation MSC: Client Needs: Health Promotion and Maintenance 11. What term should be used in the nurses documentation to describe auscultation of breath sounds that are short, popping, and discontinuous on inspiration? a . Pleural friction rub b . Bronchovesicular sounds c . Crackles d . Wheeze ANS: C Feedback A A pleural friction rub has a grating, coarse, low-pitched sound. B Bronchovesicular sounds are auscultated over mainstem bronchi. They are clear, without any adventitious sounds. C Crackles are short, popping, discontinuous sounds heard on inspiration. D Wheezes are musical, high-pitched, predominant sounds heard on expiration. PTS: 1 DIF: Cognitive Level: Comprehension REF: p. 825 | Table 33-2 OBJ: Nursing Process: Assessment MSC: Client Needs: Health Promotion and Maintenance 12. Which strategy is the best approach when initiating the physical examination of a 9-monthold male infant? a . Undress the infant and do a head-to-toe examination. b . Have the parent hold the child on his or her lap. c . Put the infant on the examination table and begin assessments at the head. d Ask the parent to leave because the infant will be upset. . ANS: B Feedback A The head-to-toe approach needs to be modified for the infant. Uncomfortable procedures, such as the otoscopic examination, should be left until last. B Infants 6 months and older feel stranger anxiety. It is easier to do most of the examination on the parents lap to lessen anxiety. C The infant may feel less fearful if placed in the parents lap or with the parent within visual range if placed on the examining table. The head-to-toe approach is modified for the infant. D There is no reason to ask a parent to leave when an infant is being examined. Having the parent with the infant will make the experience less upsetting for the infant. PTS: 1 DIF: Cognitive Level: Comprehension REF: pp. 804-805 OBJ: Nursing Process: Implementation MSC: Client Needs: Health Promotion and Maintenance 13. Which strategy is not always appropriate for pediatric physical examination? a . Take the history in a quiet, private place. b . Examine the child from head to toe. c . Exhibit sensitivity to cultural needs and differences. d . Perform frightening procedures last. ANS: B Feedback A The nurse should collect the childs health history in a quiet, private area. B The classic approach to physical examination is to begin at the head and proceed through the entire body to the toes. When examining a child, however, the examiner must tailor the physical assessment to the childs age and developmental level. C The nurse should always be sensitive to cultural needs and differences among children. D When examining children, painful or frightening procedures should be left to the end of the examination. PTS: 1 DIF: Cognitive Level: Comprehension REF: p. 804 OBJ: Nursing Process: Implementation MSC: Client Needs: Health Promotion and Maintenance 14. Which assessment should the nurse perform last when examining a 5-year-old child? a Heart . b Lungs . c Abdomen . d Throat . ANS: D Feedback A The nurse may proceed from head to toe with preschool-age children. More invasive procedures should be saved until the end of the examination. Assessment of the heart is considered noninvasive. B For preschool children, invasive procedures should be left to the end of the examination. Assessment of the lungs is not considered to be frightening. C For preschool children, invasive procedures should be left to the end of the examination. Assessment of the abdomen is not considered to be frightening. D Examination of the mouth and throat is considered to be more invasive than other parts of a physical examination. For preschool children, invasive procedures should be left to the end of the examination. PTS: 1 DIF: Cognitive Level: Application REF: p. 805 OBJ: Nursing Process: Implementation MSC: Client Needs: Health Promotion and Maintenance 15. When is the most appropriate time to inspect the genital area during a well-child examination of a 14-year-old girl? a . It is not necessary to inspect the genital area. b . Examine the genital area first. c . After the abdominal assessment. d . Do the genital inspection last. ANS: C Feedback A A visual inspection of all areas of the body is included in a physical examination. B Examination of the genital area can be embarrassing. It is not be appropriate to begin the examination of this area. C It is best to incorporate the genital assessment into the middle of the examination. This allows ample time for questions and discussion. If possible, proceed from the abdominal area to the genital area. D Assessing the genital area earlier in the examination allows more time for the adolescent to ask questions and engage in discussion. PTS: 1 DIF: Cognitive Level: Application REF: p. 805 OBJ: Nursing Process: Implementation MSC: Client Needs: Health Promotion and Maintenance 16. Which measurement is not indicated for a 4-year-old well-child examination? a . Blood pressure b . Weight c . Height d . Head circumference ANS: D Feedback A Blood pressure measurements are taken on all children at every ambulatory visit. B Weight is measured at every well-child examination. C Height is measured at every well-child examination. D Head circumference is measured on all children from birth to 3 years. Children older than 3 years of age with questionable head size or a history of megalocephaly, hydrocephalus, or microcephaly should have their head circumference assessed at every visit. A 4-year-old without a history of these problems does not need his or her head circumference measured. PTS: 1 DIF: Cognitive Level: Comprehension REF: p. 810 OBJ: Nursing Process: Assessment MSC: Client Needs: Health Promotion and Maintenance 17. The nurse inspecting the skin of a dark-skinned child notices an area that is a dusky red or violet color. This skin coloration is associated with what? a Cyanosis . b Erythema . c Vitiligo . d Nevi . ANS: B Feedback A Cyanosis in a dark-skinned child appears as a black coloration of the skin. B In dark-skinned children, erythema appears as dusky red or violet skin coloration. C Vitiligo refers to areas of depigmentation. D Nevi are areas of increased pigmentation. PTS: 1 DIF: Cognitive Level: Comprehension REF: p. 812 OBJ: Nursing Process: Assessment MSC: Client Needs: Health Promotion and Maintenance 18. The nurse palpated the anterior fontanel of a 14-month-old infant and found that it was closed. What does this finding indicate? a . This is a normal finding. b . This finding indicates premature closure of cranial sutures. c . This is abnormal and the child should have a developmental evaluation. d . This is an abnormal finding and the child should have a neurologic evaluation. ANS: A Feedback A The anterior fontanel should be completely closed by 12 to 18 months of age. B A closed anterior fontanel at 14 months of age does not indicate premature closure of cranial sutures. C This finding is not abnormal and does not necessitate a developmental evaluation. D This finding is not abnormal and does not indicate the need for a neurologic examination. PTS: 1 DIF: Cognitive Level: Analysis REF: p. 814 OBJ: Nursing Process: Assessment MSC: Client Needs: Health Promotion and Maintenance 19. The nurse is assessing a 4-year-old childs visual acuity. He is planning to attend preschool next week. The results indicate a visual acuity of 20/40 in both eyes. The childs father asks the nurse about his sons results. Which response, if made by the nurse, is correct? a . Your child will need a referral to the ophthalmologist before he can attend preschool next week. b . Your childs visual acuity is normal for his age. c . The results of this test indicate your child may be color blind. d . Your child did not pass the screening test. He will need to return within the next few weeks to be reevaluated. ANS: B Feedback A This is within the normal range for visual acuity at 4 years of age. The 4-yearolds acuity is usually 20/30 to 20/40. There is no need for evaluation by an ophthalmologist at this time. B This is the correct response. C The childs visual acuity is within normal range for his age. Color vision is evaluated by different methods than visual acuity. D This is within the normal range for visual acuity at 4 years of age. The 4-yearolds acuity is usually 20/30 to 20/40. There is no need for further evaluation at this time. PTS: 1 DIF: Cognitive Level: Application REF: p. 817 OBJ: Nursing Process: Evaluation MSC: Client Needs: Health Promotion and Maintenance 20. When interviewing the mother of a 3-year-old child, the nurse asks about developmental milestones such as the age of walking without assistance. This should be considered a . Unnecessary information, because the child is 3 years old b . An important part of the family history c An important part of the childs past growth and development . d An important part of the childs review of systems . ANS: C Feedback A Developmental milestones provide important information about the childs physical, social, and neurologic health. B The developmental milestones are specific to this child. If pertinent, attainment of milestones by siblings should be included in the family history. C Information about the attainment of developmental milestones is important to obtain. It provides data about the childs growth and development that should be included in the history. D The review of systems does not include the developmental milestones. PTS: 1 DIF: Cognitive Level: Comprehension REF: p. 807 | Box 33-3 OBJ: Nursing Process: Assessment MSC: Client Needs: Health Promotion and Maintenance 21. Which cranial nerve is assessed when the child is asked to imitate the examiners wrinkled frown, wrinkled forehead, smile, and raised eyebrow? a Accessory . b Hypoglossal . c Trigeminal . d Facial . ANS: D Feedback A To assess the accessory nerve, the examiner palpates and notes the strength of the trapezius and sternocleidomastoid muscles against resistance. B To assess the hypoglossal nerve, the examiner asks the child to stick out the tongue. C To assess the trigeminal nerve, the child is asked to identify a wisp of cotton on the face. The corneal reflex and temporal and masseter muscle strength are evaluated. D The facial nerve is assessed as described in the question. PTS: 1 DIF: Cognitive Level: Comprehension REF: p. 834 | Tables 33-4, 33-6 OBJ: Nursing Process: Assessment MSC: Client Needs: Health Promotion and Maintenance 22. Which assessment finding is considered a neurologic soft sign in a 7-year-old child? a Plantar reflex . b Poor muscle coordination . c Stereognostic function . d Graphesthesia . ANS: B Feedback A The plantar reflex is a normal response. When the lateral aspect of the sole of the foot is stroked in a movement curving medially from the heel to the ball, the response will be plantar flexion of the toes. B Poor muscle coordination is a neurologic soft sign. C Stereognostic function refers to the ability to identify familiar objects placed in each hand. D Graphesthesia is the ability to identify letters or numbers traced on the palm or back of the hand with a blunt point. PTS: 1 DIF: Cognitive Level: Application REF: p. 838 | Box 33-13 OBJ: Nursing Process: Assessment MSC: Client Needs: Health Promotion and Maintenance 23. Which parameter correlates best with measurements of the bodys total muscle-mass to fat ratio? a . Height b . Weight c . Skin-fold thickness d . Mid arm circumference ANS: D Feedback A Height is reflective of past nutritional status. B Weight is indicative of current nutritional status. C Skin-fold thickness is a measurement of the bodys fat content. D Mid arm circumference is correlated with measurements of total muscle mass. Muscle serves as the bodys major protein reserve and is considered an index of the bodys protein stores. PTS: 1 DIF: Cognitive Level: Comprehension REF: p. 811 OBJ: Nursing Process: Assessment MSC: Client Needs: Health Promotion and Maintenance 24. Which tool measures body fat most accurately? a . Stadiometer b . Calipers c . Cloth tape measure d . Paper or metal tape measure ANS: B Feedback A Stadiometers are used to measure height. B Calipers are used to measure skin-fold thickness, which is an indicator of body fat content. C Cloth tape measures should not be used because they can stretch. D Paper or metal tape measures can be used for recumbent lengths and other body measurements that must be made. PTS: 1 DIF: Cognitive Level: Comprehension REF: p. 811 OBJ: Nursing Process: Assessment MSC: Client Needs: Health Promotion and Maintenance 25. When palpating the childs cervical lymph nodes, the nurse notes that they are tender, enlarged, and warm. What is the best explanation for this? a . Some form of cancer b . Local scalp infection common in children c . Infection or inflammation distal to the site d . Infection or inflammation close to the site ANS: D Feedback A Tender lymph nodes are not usually indicative of cancer. B A scalp infection usually does not cause inflamed lymph nodes. C The lymph nodes close to the site of inflammation or infection would be inflamed. D Small nontender nodes are normal. Tender, enlarged, and warm lymph nodes may indicate infection or inflammation close to their location. PTS: 1 DIF: Cognitive Level: Analysis REF: p. 812 OBJ: Nursing Process: Assessment MSC: Client Needs: Health Promotion and Maintenance 26. What heart sound is produced by vibrations within the heart chambers or in the major arteries from the back-and-forth flow of blood? a S1, S2 . b S3, S4 . c Murmur . d Physiologic splitting . ANS: C Feedback A These are normal heart sounds. S1 is the closure of the tricuspid and mitral valves, and S2 is the closure of the pulmonic and aortic valves. B S3 is a normal heart sound sometimes heard in children. S4 is rarely heard as a normal heart sound. If heard, medical evaluation is required. C Murmurs are the sounds that are produced in the heart chambers or major arteries from the purulence of blood flow. Murmurs create a blowing and swooshing sound. D Physiologic splitting is the distinction of the two sounds in S2, which widens on inspiration. It is a significant normal finding. PTS: 1 DIF: Cognitive Level: Comprehension REF: p. 824 OBJ: Nursing Process: Assessment MSC: Client Needs: Health Promotion and Maintenance 27. Examination of the abdomen is performed correctly by the nurse in which order? a Inspection, palpation, and auscultation . b Palpation, inspection, and auscultation . c Palpation, auscultation, and inspection . d Inspection, auscultation, and palpation . ANS: D Feedback A Palpation is always performed last because it may distort the normal abdominal sounds. B Palpation is always performed last because it may distort the normal abdominal sounds. C Palpation is always performed last because it may distort the normal abdominal sounds. D The correct order of abdominal examination is inspection, auscultation, and palpation. PTS: 1 DIF: Cognitive Level: Comprehension REF: p. 828 OBJ: Nursing Process: Assessment MSC: Client Needs: Health Promotion and Maintenance 28. The nurse has a 2-year-old boy sit in tailor position during palpation for the testes. What is the rationale for this position? a It prevents cremasteric reflex. . b Undescended testes can be palpated. . c This tests the child for an inguinal hernia. . d The child does not yet have a need for privacy. . ANS: A Feedback A The tailor position stretches the muscle responsible for the cremasteric reflex. This prevents its contraction, which pulls the testes into the pelvic cavity. B Undescended testes cannot be predictably palpated. C Inguinal hernias are not detected by this method. This position is used for inhibiting the cremasteric reflex. D Privacy should always be provided for children. PTS: 1 DIF: Cognitive Level: Comprehension REF: p. 829 OBJ: Nursing Process: Assessment MSC: Client Needs: Health Promotion and Maintenance 29. During examination of a toddlers extremities, the nurse notes that the child is bowlegged. The nurse should recognize that this finding is a . Abnormal, requiring further investigation b . Abnormal unless it occurs in conjunction with knock-knee c . Normal if the condition is unilateral or asymmetric d . Normal, because the lower back and leg muscles are not yet well developed ANS: D Feedback A This is an expected finding in toddlers. B This is an expected finding in toddlers. C Further evaluation is needed if it persists beyond age 2 to 3 years, especially in African-American children. D Genu varum (bowlegged) is common in toddlers when they begin to walk. It usually persists until all of their lower back and leg muscles are well developed. PTS: 1 DIF: Cognitive Level: Comprehension REF: p. 831 OBJ: Nursing Process: Diagnosis MSC: Client Needs: Health Promotion and Maintenance 30. Kimberly is having a checkup before starting kindergarten. The nurse asks her to do the finger-to-nose test. The nurse is testing for a . Deep tendon reflexes b . Cerebellar function c . Sensory discrimination d . Ability to follow directions ANS: B Feedback A Each deep tendon reflex is tested separately. B The finger-to-nose-test is an indication of cerebellar function. This test checks balance and coordination. C Each sense is tested separately. D Although this test enables the nurse to evaluate the childs ability to follow directions, it is used primarily for cerebellar function. Chapter 33 Caring for Children in Diverse Settings MULTIPLE CHOICE 1. Which situation poses the greatest challenge to the nurse working with a child and family? a . Twenty-four-hour observation b . Emergency hospitalization c . Outpatient admission d . Rehabilitation admission ANS: B Feedback A Although preparation time may be limited with a 24-hour observation, this situation does not usually involve the acuteness of the situation and the high levels of anxiety associated with emergency admission. B Emergency hospitalization involves (1) limited time for preparation both for the child and family, (2) situations that cause fear for the family that the child may die or be permanently disabled, and (3) a high level of activity, which can foster further anxiety. C Outpatient admission generally involves preparation time for the family and child. Because of the lower level of acuteness in these settings, anxiety levels are not as high. D Rehabilitation admission follows a serious illness or disease. This type of unit may resemble a home environment, which decreases the childs and familys anxiety. PTS: 1 DIF: Cognitive Level: Comprehension REF: p. 875 OBJ: Nursing Process: Planning MSC: Client Needs: Safe and Effective Care Environment 2. What is the primary disadvantage associated with outpatient and day facility care? a . Increased cost b . Increased risk of infection c . Lack of physical connection to the hospital d . Longer separation of the child from family ANS: C Feedback A This type of care decreases cost. B This type of care decreases risk of infection. C Outpatient and day facility care do not provide extended care; therefore a child requiring extended care should be transferred to the hospital, causing increased stress to the child and parents. D This type of care minimizes separation of the child from family. PTS: 1 DIF: Cognitive Level: Comprehension REF: p. 875 OBJ: Nursing Process: Implementation MSC: Client Needs: Safe and Effective Care Environment 3. Based on concepts related to the normal growth and development of children, which child would have the most difficulty with separation from family during hospitalization? a A 5-month-old infant . b A 15-month-old toddler . c A 4-year-old child . d A 7-year-old child . ANS: B Feedback A Infants younger than 6 months of age will generally adapt to hospitalization if their basic needs for food, warmth, and comfort are met. B Separation is the major stressor for children hospitalized between ages 6 and 30 months. C Although separation anxiety occurs in hospitalized preschoolers, it is usually less obvious and less serious than that experienced by the toddler. D The school-age child is accustomed to separation from parents. Although hospitalization is a stressor, the 7-year-old child will have less separation anxiety than a 15-month-old toddler. PTS: 1 DIF: Cognitive Level: Application REF: pp. 878-879 OBJ: Nursing Process: Assessment MSC: Client Needs: Psychosocial Integrity 4. What is the best explanation for a 2-year-old child who is quiet and withdrawn on the fourth day of a hospital admission? a The child is protesting her separation from her caregivers. . b The child has adjusted to the hospitalization. . c The child is experiencing the despair stage of separation. . d The child has reached the stage of detachment. . ANS: C Feedback A In the protest stage, the child would be agitated, crying, resistant to caregivers, and inconsolable. B Toddlers do not readily adjust to hospitalization and separation from caregivers. C In the despair stage of separation, the child exhibits signs of hopelessness and becomes quiet, withdrawn, and apathetic. D The detachment stage occurs after prolonged separation. During this phase, the child becomes interested in the environment and begins to play. PTS: 1 DIF: Cognitive Level: Comprehension REF: p. 878 | Box 35-1 OBJ: Nursing Process: Implementation MSC: Client Needs: Psychosocial Integrity 5. A 3-year-old child cries, kicks, and clings to the father when the parents try to leave the hospital room. What is the nurses best response to the parents about this behavior? a Your child is showing a normal response to the stress of hospitalization. . b Your child is not coping effectively with hospitalization. Well need to get a . consult from the doctor due to this behavioral problem. c It is helpful for parents to stay with children during hospitalization. . d You can avoid this if you leave after your child falls asleep. . ANS: A Feedback A The child is exhibiting a healthy attachment to the father. B The childs behavior represents the protest stage of separation and does not represent maladaptive behavior. C This response places undue stress and guilt on the parents. D This response fosters the childs mistrust. PTS: 1 DIF: Cognitive Level: Application REF: p. 880 OBJ: Nursing Process: Implementation MSC: Client Needs: Psychosocial Integrity 6. Which is the most developmentally appropriate intervention when working with the hospitalized adolescent? a . Encourage peers to call and visit when the adolescents condition allows. b . Encourage the adolescents friends to continue with their daily activities; the adolescent has concrete thinking and will understand. c . Discourage questions and concerns about the effects of the illness on the adolescents appearance. d . Ask the parents how the adolescent usually copes in new situations. ANS: A Feedback A The peer group is important to the adolescents sense of belonging and identity; therefore separation from friends is a major source of anxiety for the hospitalized adolescent. B Adolescents should have advanced beyond concrete thinking. In addition, hospitalized adolescents may be upset if their friends continue with daily activities without them. Communication, interacting, and meeting with friends will be important. C Questions and concerns should be encouraged regarding the adolescents appearance and the effects of illness on appearance. D How the adolescent copes should be asked directly of the adolescent. PTS: 1 DIF: Cognitive Level: Application REF: p. 881 OBJ: Nursing Process: Implementation MSC: Client Needs: Psychosocial Integrity 7. The nurse is discussing toddler development with the mother of a 2 1/2-year-old child. Which statement by the mother indicates she has an understanding of how to help her daughter succeed in a developmental task while hospitalized? a . I always help my daughter complete tasks to help her achieve a sense of accomplishment. b . I provide many opportunities for my daughter to play with other children her age. c . I consistently stress the difference between right and wrong to my daughter. d . I encourage my daughter to do things for herself when she can. ANS: D Feedback A Toddlers should be encouraged to do what they can for themselves. B Toddlers participate in parallel play. They play next to rather than with age mates. C Excessive stress on the differences between right and wrong can stifle autonomy in the toddler and foster shame and doubt. D The toddlers developmental task is to achieve autonomy. Encouraging toddlers to do things for themselves assists with this developmental task (i.e. feeding self, putting on own socks.) PTS: 1 DIF: Cognitive Level: Application REF: p. 879 OBJ: Nursing Process: Evaluation MSC: Client Needs: Health Promotion and Maintenance 8. Which intervention helps a hospitalized toddler feel a sense of control? a . Assign the same nurses to care for the child. b . Put a cover over the childs crib. c . Require parents to stay with the child. d . Follow the childs usual routines for feeding and bedtime. ANS: D Feedback A Providing consistent caregivers is most applicable for the very young child, such as the neonate and infant. B Placing a cover over the childs crib may increase feelings of loss of control. C Parents are encouraged, rather than expected, to stay with the child during hospitalization. D Familiar rituals and routines are important to toddlers and give the child a sense of control. Following the childs usual routines during hospitalization minimizes feelings of loss of control. PTS: 1 DIF: Cognitive Level: Comprehension REF: p. 879 OBJ: Nursing Process: Implementation MSC: Client Needs: Psychosocial Integrity 9. Why is observation for 24 hours in an acute-care setting often appropriate for children? a . Longer hospital stays are more costly. b . Children become ill quickly and recover quickly. c . Children feel less separation anxiety when hospitalized for 24 hours. d . Families experience less disruption during short hospital stays. ANS: B Feedback A A childs state of wellness, rather than cost, determines the length of stay. B Children become ill quickly and recover quickly; therefore they can require acute care for a shorter period of time. C Separation anxiety is primarily a factor of the stage of development, not the length of hospital stay. D Family disruption is a secondary outcome of a childs hospitalization; it does not determine length of stay. PTS: 1 DIF: Cognitive Level: Comprehension REF: p. 875 OBJ: Nursing Process: Assessment MSC: Client Needs: Physiologic Integrity 10. In which age-group does the childs active imagination during unfamiliar experiences increase the stress of hospitalization? a . Toddlers b . Preschoolers c . School-age children d Adolescents . ANS: B Feedback A A toddlers primary response to hospitalization is separation anxiety. B Active imagination is a primary characteristic of preschoolers. C School-age children experience stress with loss of control. D Adolescents experience stress from separation from their peers. PTS: 1 DIF: Cognitive Level: Application REF: p. 880 OBJ: Nursing Process: Planning MSC: Client Needs: Health Promotion and Maintenance 11. Having explanations for all procedures and selecting their own meals from hospital menus is an important coping mechanism for which age-group? a . Toddlers b . Preschoolers c . School-age children d . Adolescents ANS: C Feedback A Toddlers need routine and parental involvement for coping. B Preschoolers need simple explanations of procedures. C School-age children are developmentally ready to accept detailed explanations. School-age children can select their own menus and become actively involved in other areas of their care. D Detailed explanations and support of peers help adolescents cope. PTS: 1 DIF: Cognitive Level: Application REF: pp. 880-881 OBJ: Nursing Process: Implementation MSC: Client Needs: Psychosocial Integrity 12. What is the best action for the nurse to take when a 5-year-old child who requires another 2 days of IV antibiotics cries, screams, and resists having the IV restarted? a . Exit the room and leave the child alone until he stops crying. b . Tell the child big boys and girls dont cry. c . Let the child decide which color arm board to use with the IV. d . Administer a narcotic analgesic for pain to quiet the child. ANS: C Feedback A Leaving the child alone robs the child of support when a coping difficulty exists. B Crying is a normal response to stress. C Giving the preschooler some choice and control, while maintaining boundaries of treatment, supports the childs coping skills. D The child needs time to adjust and support to cope with unfamiliar and painful procedures during hospitalization. Although administration of a topical analgesic is indicated before restarting the childs IV, a narcotic analgesic is not indicated. PTS: 1 DIF: Cognitive Level: Application REF: p. 880 OBJ: Nursing Process: Implementation MSC: Client Needs: Psychosocial Integrity 13. What is the best nursing response to the mother of a 4-year-old child who asks what she can do to help the child cope with a siblings repeated hospitalizations? a . Recommend that the child be sent to visit the grandmother until the sibling returns home. b . Inform the parent that the child is too young to visit the hospital. c . Assume the child understands that the sibling will soon be discharged because the child asks no questions. d . Help the mother give the child a simple explanation of the treatment, and encourage the mother to have the child visit the hospitalized sibling. ANS: D Feedback A Separation from family and home may intensify fear and anxiety. B Parents are experts on their children and need to determine when their child can visit a hospital. C Children may have difficulty expressing questions and fears and need the support of parents and other caregivers. D Needs of a sibling will be better met with factual information and contact with the ill child. PTS: 1 DIF: Cognitive Level: Application REF: p. 891 OBJ: Nursing Process: Implementation MSC: Client Needs: Psychosocial Integrity 14. How should the nurse advise parents whose preschooler used to sleep through the night and now awakens at intervals after a short hospitalization? a . Regressive behavior after a hospitalization is normal and usually short term. b . The child is probably expressing anger. c . Egocentric behavior often manifests itself when the child is left alone to sleep. d . The child is probably feeling pain and needs further evaluation. ANS: A Feedback A Regression is manifested in a variety of ways, is normal, and usually is short term. B Nighttime waking is not associated with anger. C Egocentric behavior is not an explanation for nighttime waking. D More information is needed before assessment of pain can be made. PTS: 1 DIF: Cognitive Level: Application REF: p. 880 OBJ: Nursing Process: Implementation MSC: Client Needs: Physiologic Integrity 15. Which is an appropriate nursing intervention for the hospitalized neonate? a . Assign the neonate to a room with other neonates. b . Provide play activities in the hospital room. c . Offer the neonate a pacifier between feedings. d . Request that parents bring a security object from home. ANS: C Feedback A The neonate is not aware of other children. The choice of roommate will not affect the neonate socially. It is important for older children to room with similar-age children. B Formal play activities are not relevant for the neonate. C The neonate needs opportunities for nonnutritive sucking and oral stimulation with a pacifier. D Having parents bring a security object from home is applicable to older children. PTS: 1 DIF: Cognitive Level: Comprehension REF: p. 883 | Box 35-2 OBJ: Nursing Process: Implementation MSC: Client Needs: Physiologic Integrity 16. Which therapeutic approach will best help a 7-year-old child cope with a lengthy course of intravenous antibiotic therapy? a Arrange for the child to go to the playroom daily. . b Ask the child to draw you a picture of himself or herself. . c Allow the child to participate in injection play. . d Give the child stickers for cooperative behavior. . ANS: C Feedback A The hospitalized child should have opportunities to go to the playroom each day if the childs condition warrants. This free play does not have any specific therapeutic purpose. B Children can express their thoughts and beliefs through drawing. Asking the child to draw a picture of himself or herself may not elicit the childs feelings about the treatment. C Injection play is an appropriate intervention for the child who has to undergo frequent blood work, injections, intravenous therapy, or any other therapy involving syringes and needles. D Rewards such as stickers may enhance cooperative behavior. They will not address coping with painful treatments. PTS: 1 DIF: Cognitive Level: Application REF: p. 887 OBJ: Nursing Process: Implementation MSC: Client Needs: Psychosocial Integrity 17. A preschool-aged child tells the nurse I was bad, thats why I got sick. What is the best rationale for this childs statement? a . The child has a fear that mutilation will lead to death. b . The childs imagination is very active, and he may believe the illness is a result of something he did. c . The child has a general understanding of body integrity at this age. d The child will not have fear related to an IV catheter initiation but will have fear . of an impending surgery. ANS: B Feedback A The child has imaginative thoughts at this stage of growth and development. B The child may believe that an illness occurred as a result of some personal deed or thought or perhaps because he touched something or someone. C Preschoolers do not have the cognitive ability to connect mutilation to death and do not have a sound understanding of body integrity. D The preschooler fears all types of intrusive procedures whether undergoing a simple procedure such as an IV start or something more invasive such as surgery. PTS: 1 DIF: Cognitive Level: Analysis REF: p. 880 OBJ: Nursing Process: Evaluation MSC: Client Needs: Health Promotion and Maintenance 18. A 3 1/2-year-old child who is toilet trained has had several accidents since hospital admission. What is the nurses best action in this situation? a . Find out how long the child has been toilet trained at home. b . Encourage the parents to scold the child. c . Explain how to use a bedpan and place it close to the child. d . Follow home routines of elimination. ANS: D Feedback A Some regression to previous behaviors is normal during hospitalization, even when the child has been practicing the skill for some time. B Hospitalization is a stressful experience. If the incontinence is caused by anxiety, scolding is not indicated and may increase the anxiety. C Developmentally, the 3 1/2-year-old child cannot use a bedpan independently. D Cooperation will increase and anxiety will decrease if the childs normal routine and rituals are maintained. Chapter 34 Caring for the Special Needs Child MULTIPLE CHOICE 1. The parents of a school-age child are told that their child is diagnosed with leukemia. As the nurse caring for this child, what is the expected first response of the parents to the diagnosis of chronic illness in their child? a . Anger and resentment b . Sorrow and depression c . Shock and disbelief d . Acceptance and adjustment ANS: C Feedback A Feelings of anger and resentment are part of the grieving process, although not usually the initial response. B Feelings of sadness and depression are part of the grieving process, although not usually the initial response. C According to Kbler-Ross, denial is the initial stage of the grieving process when an individual reacts with shock and disbelief to the diagnosis of chronic illness. D Acceptance is the final stage of the grieving process, not the first response. PTS: 1 DIF: Cognitive Level: Application REF: p. 896 OBJ: Nursing Process: Assessment MSC: Client Needs: Psychosocial Integrity 2. What is the primary concern for the parents of a dying child? a Pain . b Safety . c Food intake . d Fluid intake . ANS: A Feedback A The primary concern of all parents of dying children is the possibility of their child feeling pain. B Although safety is a concern of all parents, it is not the priority concern. C Although eating is important, it is not the priority concern. D Although hydration is a concern, it is not the priority concern. PTS: 1 DIF: Cognitive Level: Application REF: p. 909 OBJ: Nursing Process: Assessment MSC: Client Needs: Physiologic Integrity 3. An important nursing goal in caring for the hospitalized child is to minimize the negative effects of illness and hospitalization. On what should the nurse focus while caring for a hospitalized infant? a . Bodily injury and pain b . Separation from caregivers and fear of strangers c . Loss of control and altered body image d . The unknown and being left alone ANS: B Feedback A Bodily injury and pain are fears of preschool and school-age children. B The major fear of infants during illness and hospitalization are separation from caregivers and fear of strangers. C Loss of control is a fear of children from the preschool period through adolescence. Altered body image applies to adolescents. D Fear of the unknown and being left alone are applicable to preschoolers. PTS: 1 DIF: Cognitive Level: Application REF: p. 898 | Box 36-2 OBJ: Nursing Process: Planning MSC: Client Needs: Psychosocial Integrity 4. What corresponds to a 5-year-old childs understanding of death? a . Loss of a caretaker b . Reversible and temporary c . Permanent d . Inevitable ANS: B Feedback A This is the infant/toddler understanding of death. B Children in early childhood (2 to 7 years old) view death as reversible and temporary. C The school-age child and adolescent understand that death is permanent. D The adolescent understands death not only as permanent, but also inevitable. PTS: 1 DIF: Cognitive Level: Application REF: p. 905 | Table 36-1 OBJ: Nursing Process: Assessment MSC: Client Needs: Psychosocial Integrity 5. You are counseling the family of a 12-month-old child who has lost his mother in a car accident. How should you explain to the father what the childs understanding of death is, related to theories of growth and development? a . Temporary b . Permanent c . Loss of caretaker d Punishment . ANS: C Feedback A The preschool-age child views death as temporary. B The school-age child and adolescent understand the permanence of death. C This is the infant/toddler understanding of death. D The preschool-age child facing impending death may view his or her condition as punishment for behaviors or thoughts. PTS: 1 DIF: Cognitive Level: Application REF: p. 905 | Table 36-1 OBJ: Nursing Process: Implementation MSC: Client Needs: Psychosocial Integrity 6. How can chronic illness and frequent hospitalizations affect the psychosocial development of a toddler? a . They can create a distortion or differentiation of self from parent. b . They can interfere with the development of autonomy. c . They can interfere with the acquisition of language, fine motor, and self-care skills. d . They can create feelings of inadequacy. ANS: B Feedback A The infant with a chronic illness may have distortion of differentiation of self from parents. B Chronic illness may interfere in the development of autonomy, which is the major psychosocial task of the toddler. C Chronic illness with frequent hospitalizations can inhibit the acquisition of language, motor, and self-care skills in the preschool-age child. D Feelings of inadequacy and inferiority can occur if independence is compromised by chronic illness in the school-age child. PTS: 1 DIF: Cognitive Level: Comprehension REF: p. 898 | Box 36-2 OBJ: Nursing Process: Assessment MSC: Client Needs: Psychosocial Integrity 7. How can chronic illness and frequent hospitalizations affect the psychosocial development of an adolescent? a . They can lead to feelings of inadequacy. b . They can interfere with parental attachment. c . They can block the development of identity. d . They can prevent the development of imagination. ANS: C Feedback A Inadequacy and inferiority refer to the school-age period. B Parental attachment is a task of the infant. C Development of identity is the task of the adolescent. D Development of imagination occurs in the preschool period. PTS: 1 DIF: Cognitive Level: Comprehension REF: p. 898 | Box 36-2 OBJ: Nursing Process: Assessment MSC: Client Needs: Psychosocial Integrity 8. What is an important focus of nursing care for the dying child and his or her family? a . Nursing care should be organized to minimize contact with the child. b . Adequate oral intake is crucial to the dying child. c . Families should be made aware that hearing is the last sense to stop functioning before death. d . It is best for the family if nursing care takes place during periods when the child is alert. ANS: C Feedback A Nursing care should minimize disruptions but not contact. B When a child is dying, fluids should be based on the childs requests, with a focus on comfort and preventing a dry mouth. C Families should be encouraged to talk to the child because verbal communication and physical touch are important both for the family and child. D The times when the child is alert should be devoted to family contacts. PTS: 1 DIF: Cognitive Level: Application REF: p. 911 OBJ: Nursing Process: Planning MSC: Client Needs: Psychosocial Integrity 9. What is the most appropriate response to a school-age child who asks if she can talk to her dying sister? a . You need to talk loudly so she can hear you. b . Holding her hand would be better because at this point she cant hear you. c . Although she cant hear you, she can feel your presence so sit close to her. d . Even though she will probably not answer you, she can still hear what you say to her. ANS: D Feedback A There is no evidence that the dying process decreases hearing acuity. B The sense of hearing is intact before death. The sibling should be encouraged to speak to the child, as well as hold the childs hand. C Hearing is the last sense to cease before death. The sibling should be encouraged to sit close and speak to the dying child. D Hearing is the last sense to cease before death. Talking to the dying child is important both for the child and the family. PTS: 1 DIF: Cognitive Level: Application REF: p. 911 OBJ: Nursing Process: Implementation MSC: Client Needs: Psychosocial Integrity 10. What is the priority goal for the child with a chronic illness? a . To maintain the intactness of the family b . To eliminate all stressors c . To achieve complete wellness d To obtain the highest level of wellness . ANS: D Feedback A This is a goal for the family, not specifically the child. B This is not a realistic goal because life will continue to present stressors. C This is unrealistic because chronic illness by definition is a long-term condition either without a cure or with residual limitations. D To obtain the highest level of health and function possible is the priority goal of nursing children with a chronic illness. PTS: 1 DIF: Cognitive Level: Application REF: p. 899 OBJ: Nursing Process: Planning MSC: Client Needs: Health Promotion and Maintenance 11. What is the predominant trait of the resilient family associated with chronic illness? a . Social separation b . Family flexibility c . Family cohesiveness d . Clear family boundaries ANS: C Feedback A Maintaining social integration is one of the traits of a resilient family system. B Family flexibility is a trait of the resilient family, but not the predominant one. C Family cohesiveness is the predominant trait of the resilient family. D Clear family boundaries are a trait of the resilient family, but not the predominant one. PTS: 1 DIF: Cognitive Level: Comprehension REF: p. 895 OBJ: Nursing Process: Assessment MSC: Client Needs: Psychosocial Integrity 12. Many parents who have children diagnosed with a chronic illness experience recurrent feelings of grief, loss, and fear related to the childs condition and loss of the ideal healthy child. The nurse recognizes this process as a . Anticipatory grieving b . Chronic sorrow c . Bereavement d . Illness trajectory ANS: B Feedback A Anticipatory grieving is the process of mourning, coping, interacting, planning, and psychosocial reorganization that is begun as a response to the impending loss of a loved one. B The stated recurrent feelings define chronic sorrow, which is considered a normal process involving grief that may never be resolved. C Bereavement is defined as the objective condition or state of loss. D Illness trajectory is defined as the impact of the disease or condition on all family members, physiologic unfolding of the disease, and work organization done by the family to cope. PTS: 1 DIF: Cognitive Level: Knowledge REF: p. 897 OBJ: Nursing Process: Assessment MSC: Client Needs: Psychosocial Integrity 13. What is a priority nursing diagnosis for the preschool child with chronic illness? a . Risk for delayed growth and development related to chronic illness or disability b . Chronic pain related to frequent injections c . Anticipatory grieving related to impending death d . Anxiety related to frequent hospitalizations ANS: A Feedback A This is the priority nursing diagnosis that is appropriate for the majority of chronic illnesses. B Pain is not associated with the majority of chronic illnesses. C A chronic illness is one that does not have a cure. It does not mean the child will die prematurely. D Frequent hospitalizations are not required for all chronic illnesses. PTS: 1 DIF: Cognitive Level: Application REF: p. 903 OBJ: Nursing Process: Diagnosis MSC: Client Needs: Health Promotion and Maintenance 14. Identify the most appropriate response for the nurse when parents say, Living with this disease is really hard; its not fair. a . Tell me about what is hard for you. b . I know exactly how you must feel. c . I know a local support group for families. d . I am going to ask the grief counselor to meet with you. ANS: A Feedback A The first step in supporting families and helping them deal with chronic sorrow is to listen to and recognize their pain. B This comment does not encourage parents to talk about their feelings. Each individuals perception of a situation is different. A nurse can never know exactlyhow parents feel about having a child with a chronic illness. C This comment does not address the parents immediate feelings. D This response does not address the parents immediate feelings. PTS: 1 DIF: Cognitive Level: Application REF: p. 897 OBJ: Nursing Process: Implementation MSC: Client Needs: Psychosocial Integrity 15. Identify the most appropriate nursing response to a parent who tells the nurse, I dont want my child to know she is dying. a I shall respect your decision. I wont say anything to your child. . b . Dont you think she has a right to know about her condition? c . Would you like me to arrange for the physician to speak with your child? d . Ill answer any questions she asks me as honestly as I can. ANS: D Feedback A As the caregiver and advocate, the nurse should first meet the childs needs. B This is a judgmental response and could affect the nurses relationship with the childs parents. C This response does not address the parents statement. D Nurses can inform parents that they will not initiate any discussion with the child but that they intend to respond openly and honestly if and when the child initiates such a discussion. PTS: 1 DIF: Cognitive Level: Application REF: p. 908 OBJ: Nursing Process: Implementation MSC: Client Needs: Psychosocial Integrity 16. Which activity should the nurse implement for the toddler hospitalized with a chronic illness to promote autonomy? a Playing with a push-pull toy . b Putting together a puzzle . c Playing a simple card game . d Watching cartoons on television . ANS: A Feedback A The developmentally appropriate activity for the hospitalized toddler is to play with a push-pull toy. Chronic illness may interfere in the development of autonomy, which is the major psychosocial task of the toddler. B Putting together a puzzle could frustrate the toddler and is appropriate for a preschool or school-age child. C Playing a simple card game could frustrate the toddler and is appropriate for a preschool or school-age child. D Watching cartoons on television is passive and will not promote autonomy. PTS: 1 DIF: Cognitive Level: Application REF: p. 898 | Box 36-2 OBJ: Nursing Process: Implementation MSC: Client Needs: Psychosocial Integrity 17. The nurse case manager is planning a care conference about a young child who has complex health care needs and will soon be discharged home. Who should the nurse invite to the conference? a Family and nursing staff . b Social worker, nursing staff, and primary care physician . c Family and key health professionals involved in the childs care . d Primary care physician and key health professionals involved in the childs care . ANS: C Feedback A The nursing staff can address the nursing care needs of the child with the family, but other involved disciplines must be included. B The family must be included in the discharge conferences, which allow them to determine what education they will require and the resources needed at home. C A multidisciplinary conference is necessary for coordination of care for children with complex health needs. The family is involved as well as key health professionals who are involved in the childs care. D A member of the nursing staff must be included to review the nursing needs of the child. PTS: 1 DIF: Cognitive Level: Analysis REF: p. 902 OBJ: Nursing Process: Planning MSC: Client Needs: Safe and Effective Care Environment 18. Families progress through various stages of reactions when a child is diagnosed with a chronic illness or disability. After the shock phase, a period of adjustment usually follows. This is often characterized by which response? a Denial . b Guilt and anger . c Social reintegration . d Acceptance of childs limitations . ANS: B Feedback A The initial diagnosis of a chronic illness or disability often is often met with intense emotion and characterized by shock and denial. B For most families, the adjustment phase is accompanied by several responses that are normally part of the adjustment process. Guilt, self-accusation, bitterness, and anger are common reactions. C Social reintegration and acceptance of the childs limitations is the culmination of the adjustment process. D Social reintegration and acceptance of the childs limitations is the culmination of the adjustment process. PTS: 1 DIF: Cognitive Level: Comprehension REF: p. 896 OBJ: Nursing Process: Planning MSC: Client Needs: Psychosocial Integrity 19. The nurse comes into the room of a child who was just diagnosed with a chronic disability. The childs parents begin to yell at the nurse about a variety of concerns. The nurses best response is a . What is really wrong? b . Being angry is only natural. c . Yelling at me will not change things. d . I will come back when you settle down. ANS: B Feedback A These responses, although possible, are not the likely reason for this anger. B Parental anger after the diagnosis of a child with a chronic disability is a common response. One of the most common targets for parental anger is members of the staff. The nurse should recognize the common response of anger to the diagnosis and allow the family to ventilate. C These responses, although possible, are not the likely reason for this anger. D These responses, although possible, are not the likely reason for this anger. PTS: 1 DIF: Cognitive Level: Comprehension REF: p. 896 OBJ: Nursing Process: Assessment MSC: Client Needs: Psychosocial Integrity 20. The feeling of guilt that the child caused the disability or illness is especially critical in which child? a . Toddler b . Preschooler c . School-age child d . Adolescent ANS: B Feedback A Toddlers are focused on establishing their autonomy. The illness will foster dependency. B Preschoolers are most likely to be affected by feelings of guilt that they caused the illness/disability or are being punished for wrongdoings. C The school-age child will have limited opportunities for achievement and may not be able to understand limitations. D Adolescents are faced with the task of incorporating their disabilities into their changing self-concept. PTS: 1 DIF: Cognitive Level: Comprehension REF: p. 905 OBJ: Nursing Process: Assessment MSC: Client Needs: Psychosocial Integrity 21. The nurse is providing support to a family who is experiencing anticipatory grief related to their childs imminent death. An appropriate nursing intervention is to a . Be available to family. b . Attempt to lighten the mood. c . Suggest activities to cheer up the family. d . Discourage crying until actual time of death. ANS: A Feedback A When death is imminent, care should be limited to interventions for palliative care. B Music may be used to provide comfort to the child. C Vital signs do not need to be measured frequently. D The nurse should speak to the child in a clear, distinct voice. Chapter 35 Key Pediatric Nursing Interventions MULTIPLE CHOICE 1. What is the most appropriate statement for the nurse to make to a 5-year-old child who is undergoing a venipuncture? a . You must hold still or Ill have someone hold you down. This is not going to hurt. b . This will hurt like a pinch. Ill get someone to help hold your arm still so it will be over fast and hurt less. c . Be a big boy and hold still. This will be over in just a second. d . Im sending your mother out so she wont be scared. You are big, so hold still and this will be over soon. ANS: B Feedback A Honesty is always best and a venipuncture may hurt. B Honesty is the best approach. Children should be told what sensation they will feel during a procedure. A 5-year-old child should not be expected to hold still, and assistance ensures safety to everyone. C This statement is not supportive or honest. D Parents should be encouraged to remain with the child unless they are extremely uncomfortable doing so. PTS: 1 DIF: Cognitive Level: Application REF: p. 917 | Box 37-1 OBJ: Nursing Process: Implementation MSC: Client Needs: Physiologic Integrity 2. Which nursing diagnosis is appropriate for the 5-year-old child in isolation because of immunosuppression? a Spiritual distress . b . Social isolation c . Deficient diversional activity d . Sleep deprivation ANS: C Feedback A A 5-year-old child is not developmentally advanced enough to feel spiritual distress. B The main social system for a 5-year-old child is the family, who should be allowed liberal visitation. C Children in isolation need extra attention to avoid boredom. D Sleep deprivation may occur during hospitalization but is not specific to isolation. PTS: 1 DIF: Cognitive Level: Application REF: p. 920 OBJ: Nursing Process: Diagnosis MSC: Client Needs: Safe and Effective Care Environment 3. What should the nurse consider when having consent forms signed for surgery and procedures on children? a . Only a parent or legal guardian can give consent. b . The person giving consent must be at least 18 years old. c . The risks and benefits of a procedure are part of the consent process. d A mental age of 7 years or older is required for a consent to be considered . informed. ANS: C Feedback A In special circumstances, such as emancipated minors, the consent can be given by someone younger than 18 years without the parent or legal guardian. B In special circumstances, such as emancipated minors, the consent can be given by someone younger than 18 years without the parent or legal guardian. C The informed consent must include the nature of the procedure, benefits and risks, and alternatives to the procedure. D A mental age of 7 years is too young for consent to be informed. PTS: 1 DIF: Cognitive Level: Comprehension REF: p. 918 OBJ: Nursing Process: Planning MSC: Client Needs: Safe and Effective Care Environment 4. The nurse is planning how to prepare a 4-year-old child for some diagnostic procedures. Guidelines for preparing this preschooler should include a . Planning for a short teaching session of about 30 minutes b . Telling the child that procedures are never a form of punishment c . Keeping equipment out of the childs view d . Using correct scientific and medical terminology in explanations ANS: B Feedback A Teaching sessions for this age-group should be 10 to 15 minutes in length. B Illness and hospitalization may be viewed as punishment in preschoolers. Always state directly that procedures are never a form of punishment. C Demonstrate the use of equipment, and allow the child to play with miniature or actual equipment. D Explain the procedure in simple terms and how it affects the child. PTS: 1 DIF: Cognitive Level: Application REF: p. 918 OBJ: Nursing Process: Implementation MSC: Client Needs: Health Promotion and Maintenance 5. Which nursing action is most appropriate when treating a child who has a fever of 102.5 F? a . Restrict fluid intake. b . Administer an aspirin. c . Administer an antipyretic such as acetaminophen. d . Bathe the child in tepid water. ANS: C Feedback A Dehydration can occur from insensible water loss. Offer the child fluids frequently and evaluate the need for IV therapy. B Aspirin is avoided because of the potential association with Reyes syndrome. C Treatment of a fever can include administration of an antipyretic. D A sponge or tub bath with tepid water to reduce fever can cause shivering and ultimately increase the childs temperature. PTS: 1 DIF: Cognitive Level: Application REF: p. 927 OBJ: Nursing Process: Implementation MSC: Client Needs: Physiologic Integrity 6. What is the best response for a nurse to make to a parent who has asked, When should I start dental care for my child? a . The recommendation is for children to have a dental examination no later than 2.5 years. b . Children should see a dentist at least one time before kindergarten. c . The recommendation is for children to have a dental examination before first grade. d . A dental examination by 1 year of age is the current recommendation. ANS: A Feedback A Children should be examined by a dentist between the time the first teeth erupt and primary dentition is complete at 2.5 years of age. B Children require regular dental examinations well before kindergarten. C Six years of age is too late to begin regular dental examinations. D Children should be examined by a dentist between the time the first teeth erupt and the time primary dentition is complete at 2.5 years of age. PTS: 1 DIF: Cognitive Level: Application REF: p. 922 OBJ: Nursing Process: Implementation MSC: Client Needs: Physiologic Integrity 7. Which action is appropriate to promote a toddlers nutrition during hospitalization? a . Allow the child to walk around during meals. b . Require the child to empty his or her plate. c . Ask the childs parents to bring a cup and utensils from home. d . Select new foods for the child from the menu. ANS: C Feedback A For safety reasons, roaming while eating should not be permitted. The child should be seated during meals. B Toddlers often use food as a source of control. Forcing a toddler to eat only increases the childs sense of powerlessness. Toddlers also experience food jags, a normal phenomenon when they will only eat certain foods. C Using familiar items during mealtimes increases the toddlers sense of security and control. D Hospitalization is a stressful experience for the toddler. It is not the time to introduce the child to new foods. PTS: 1 DIF: Cognitive Level: Application REF: p. 923 OBJ: Nursing Process: Implementation MSC: Client Needs: Physiologic Integrity 8. The nurse knows that measuring temperature is an integral part of assessment. Which concept is important for the nurse to know when taking a childs temperature? a . The method used should be consistent. b . Rectal temperatures should always be taken on infants. c . Oral temperatures can be taken on all children older than 5 years of age. d . Axillary temperatures should be taken at night. ANS: A Feedback A The method that is determined most appropriate for the child should be used consistentlythe same site and device to maintain consistency and allow reliable comparison and tracking of temperatures over time. B Because of the risk of rectal perforation and the intrusive nature of the procedure, rectal temperatures are measured only when no other route can be used or when it is necessary to obtain a core body temperature. C Oral temperatures can be used on most children older than 6 years of age but may be inaccurate because of oral intake, oral surgery, oxygen therapy, nebulizer treatments, or crying. D The method of measuring temperature should be consistent, including at night. PTS: 1 DIF: Cognitive Level: Application REF: p. 924 OBJ: Nursing Process: Assessment MSC: Client Needs: Physiologic Integrity 9. A parent calls the pediatricians office because her 1-year-old child has a 100 F temperature. What is the most appropriate initial nursing response to make to the parent? a Did you feel your childs forehead? . b . Tell me about the childs behavior. c . Has anyone in your home been sick lately? d . There is no need for concern if the childs temperature is less than 101 F. ANS: B Feedback A Feeling a childs forehead can give clues related to whether the childs temperature should be measured; if it has already been measured, this is unnecessary because it does not give accurate information about the childs body temperature. B In general, the height of the fever is not an indication of the seriousness of the illness. It is more important to note changes in the childs behavior. If a child has a low-grade temperature and acts sick, he or she should be assessed further. C This question will yield relevant information for the nurse to use in advising the parent, but it is not the best initial response. D Although the height of the temperature is not an indication of the seriousness of the childs illness, it is incorrect to tell a parent to be unconcerned about temperatures less than 101 F. PTS: 1 DIF: Cognitive Level: Application REF: p. 927 OBJ: Nursing Process: Implementation MSC: Client Needs: Physiologic Integrity 10. What nursing action is appropriate for specimen collection? a Follow sterile technique for specimen collection. . b Sterile gloves are worn if the nurse plans to touch the specimen. . c Use Standard Precautions when handling body fluids. . d Avoid wearing gloves in front of the child and family. . ANS: C Feedback A Specimen collection is not always a sterile procedure. B Gloves should be worn if there is a chance the nurse will be contaminated. The choice of sterile or clean gloves will vary according to the procedure or specimen. C Standard Precautions should always be used when handling body fluids. D The child and family should be educated in the purpose of glove use, including the fact that gloves are used with every patient, so that they will not be offended or frightened. PTS: 1 DIF: Cognitive Level: Application REF: p. 928 OBJ: Nursing Process: Implementation MSC: Client Needs: Physiologic Integrity 11. What information should the nurse include in teaching parents how to care for a childs gastrostomy tube at home? a Never turn the gastrostomy button. . b Clean around the insertion site daily with soap and water. . c Expect some leakage around the button. . d Remove the tube for cleaning once a week. . ANS: B Feedback A The gastrostomy button should be rotated in a full circle during cleaning. B The skin around the tube insertion site should be cleaned with soap and water once or twice daily. C Leakage around the tube should be reported to the physician. D A gastrostomy tube is placed surgically. It is not removed for cleaning. PTS: 1 DIF: Cognitive Level: Application REF: p. 937 OBJ: Nursing Process: Implementation MSC: Client Needs: Physiologic Integrity 12. Which nursing action is the most appropriate when applying a face mask to a child for oxygen therapy? a The oxygen flow rate should be less than 6 L/min. . b Make sure the mask fits properly. . c Keep the child warm. . d Remove the mask for 5 minutes every hour. . ANS: B Feedback A The oxygen flow rate should be greater than 6 L/min to prevent rebreathing of exhaled carbon dioxide. B A properly fitting face mask is essential for adequate oxygen delivery. C Oxygen delivery through a face mask does not affect body temperature. D A face mask used for oxygen therapy is not routinely removed. PTS: 1 DIF: Cognitive Level: Comprehension REF: p. 939 OBJ: Nursing Process: Implementation MSC: Client Needs: Physiologic Integrity 13. What is appropriate to include in the care plan for a family of a child with a tracheostomy? a . Suction of the tracheostomy every 2 to 4 hours or as needed b . Application of powder around the stoma to decrease irritation c . Suction catheter insertion limited to less than 30 seconds d . Hygiene that includes showers, not baths ANS: A Feedback A To maintain a patent airway in a child with a tracheostomy, assessing respiratory status and suctioning every 2 to 4 hours or as needed using Standard Precautions is an important intervention to teach families. B Talc powder should be avoided because of the risk of inhalation injury from breathing the powder particles. C Catheter insertion for suctioning should be less than 5 seconds to prevent hypoxia. D The family should be taught to avoid getting water in the tracheostomy during bath time. Showers should be discouraged. PTS: 1 DIF: Cognitive Level: Application REF: p. 942 OBJ: Nursing Process: Planning MSC: Client Needs: Physiologic Integrity 14. Which action by the nurse indicates that the correct procedure has been used to measure vital signs in a toddler? a . Measuring oral temperature for 5 minutes b . Counting apical heart rate for 60 seconds c . Observing chest movement for respiratory rate d . Recording blood pressure as P/80 ANS: B Feedback A A child younger than 6 years may not be able to hold a thermometer under the tongue. B Apical pulse measurement when the child is quiet for 1 full minute is the preferred method for measuring vital signs in infants and children ages 2 years and younger. C The respiratory rate in infants and young children can be measured by watching abdominal movement. D It may be difficult to auscultate blood pressure in infants and toddlers. Systolic pressure can be palpated and should be recorded as systolic pressure over pulse (e.g., 80/P). PTS: 1 DIF: Cognitive Level: Analysis REF: p. 925 OBJ: Nursing Process: Implementation MSC: Client Needs: Physiologic Integrity 15. Which action by the nurse is appropriate when preparing a child for a procedure? a . Discourage the child from crying during the procedure. b . Use professional terms so the child will understand what is happening. c . Give the child choices whenever possible. d . Discourage the parents from staying in the room during the procedure. ANS: C Feedback A Children (and adults) should be given permission to cry. B Age-appropriate language should always be used. C Allowing children to make choices gives them a sense of control. D Parents should be encouraged to stay in the room and give support to the child. PTS: 1 DIF: Cognitive Level: Comprehension REF: p. 917 OBJ: Nursing Process: Implementation MSC: Client Needs: Physiologic Integrity 16. The nurse is preparing a 12-year-old girl for a bone marrow aspiration. She tells the nurse she wants her mother with her like before. The most appropriate nursing action is to a Grant her request. . b Explain why this is not possible. . c Identify an appropriate substitute for her mother. . d Offer to provide support to her during the procedure. . ANS: A Feedback A The parents preferences for assisting, observing, or waiting outside the room should be assessed as well as the childs preference for parental presence. The childs choice should be respected. B If the mother and child are agreeable, then the mother is welcome to stay. C An appropriate substitute for the mother is necessary only if the mother does not wish to stay. D Support is offered to the child regardless of parental presence. PTS: 1 DIF: Cognitive Level: Application REF: p. 917 OBJ: Nursing Process: Implementation MSC: Client Needs: Health Promotion and Maintenance 17. The nurse wore gloves during a dressing change. When the gloves are removed, the nurse should a Wash hands thoroughly. . b Check the gloves for leaks. . c Use an alcohol-based hand rub. . d Apply new gloves before touching the next patient. . ANS: C Feedback A When gloves are worn, the hands can be cleaned using an alcohol-based hand rub. If hands are visibly soiled they should be washed with soap and water. B Gloves should be disposed of after use. C Evidence-based research has demonstrated that alcohol-based rubs are more effective for eliminating organisms. D Hands should be thoroughly cleaned before new gloves are applied. PTS: 1 DIF: Cognitive Level: Comprehension REF: p. 920 OBJ: Nursing Process: Implementation MSC: Client Needs: Safe and Effective Care Environment 18. An important nursing consideration when performing a bladder catheterization on a young boy is to a Use clean technique, not Standard Precautions. . b Insert 2% lidocaine lubricant into the urethra. . c Lubricate catheter with water-soluble lubricant such as K-Y Jelly. . d Delay catheterization for 20 minutes while anesthetic lubricant is absorbed. . ANS: B Feedback A Catheterization is a sterile procedure, and Standard Precautions for bodysubstance protection should be followed. B The anxiety, fear, and discomfort experienced during catheterization can be significantly decreased by preparation of the child and parents, by selection of the correct catheter, and by appropriate technique of insertion. Generous lubrication of the urethra before catheterization and use of lubricant containing 2% lidocaine may reduce or eliminate the burning and discomfort associated with this procedure. C Water-soluble lubricants do not provide appropriate local anesthesia. D Catheterization should be delayed only 2 to 3 minutes. This provides sufficient local anesthesia for the procedure. Chapter 36 Pain Management in Children MULTIPLE CHOICE 1. Childbirth preparation can be considered successful if the outcome is described as a . Labor was pain-free. b . Birth experiences of friends and families were discredited. c . The woman rehearsed labor and practiced skills to master pain. d . Only nonpharmacologic methods for pain control were used. ANS: C Feedback A Childbirth preparation does not guarantee a pain-free labor. A woman should be prepared for pain and anesthesia/analgesia realistically. B Friends and families can be an important source of support if they convey realistic information about labor pain. C Preparation allows the woman to rehearse for labor and to learn new skills to cope with the pain of labor and the expected behavioral changes. D Women will not always achieve their desired level of pain control by using nonpharmacologic methods alone. PTS: 1 DIF: Cognitive Level: Analysis REF: p. 388 OBJ: Nursing Process: Evaluation MSC: Client Needs: Psychosocial Integrity 2. In order to help patients manage discomfort and pain during labor, nurses should be aware that a . The predominant pain of the first stage of labor is the visceral pain located in the lower portion of the abdomen. b . Somatic pain is the extreme discomfort between contractions. c . The somatic pain of the second stage of labor is more generalized and related to fatigue. d . Pain during the third stage is a somewhat milder version of the second stage. ANS: A Feedback A This pain comes from cervical changes, distention of the lower uterine segment, and uterine ischemia. B Somatic pain is a faster, sharp pain. Somatic pain is most prominent during late first-stage labor and during second-stage labor as the descending fetus puts direct pressure on maternal tissues. C Second-stage labor pain is intense, sharp, burning, and localized. D Third-stage labor pain is similar to that of the first stage. PTS: 1 DIF: Cognitive Level: Knowledge REF: p. 389 OBJ: Nursing Process: Assessment MSC: Client Needs: Health Promotion and Maintenance 3. The nurse caring for women in labor understands that childbirth pain is different from other types of pain in that it is a . More responsive to pharmacologic management b . Associated with a physiologic process c . Designed to make one withdraw from the stimulus d . Less intense ANS: B Feedback A Pain management during labor may affect the course and length of labor. B Childbirth pain is part of a normal process, whereas other types of pain usually signify an injury or illness. C The pain with childbirth is a normal process; it is not caused by the type of injury when withdrawal from the stimuli is seen. D Childbirth pain is not less intense than other types of pain. PTS: 1 DIF: Cognitive Level: Comprehension REF: p. 388 OBJ: Nursing Process: Assessment MSC: Client Needs: Physiologic Integrity 4. Excessive anxiety in labor heightens the womans sensitivity to pain by increasing a . Muscle tension b . Blood flow to the uterus c . The pain threshold d . Rest time between contractions ANS: A Feedback A Anxiety and fear increase muscle tension, diverting oxygenated blood to the womans brain and skeletal muscles. Prolonged tension results in general fatigue, increased pain perception, and reduced ability to use coping skills. B Anxiety can decrease blood flow to the uterus. C Anxiety will decrease the pain threshold. D Anxiety will decrease the amount of rest the mother gets between contractions. PTS: 1 DIF: Cognitive Level: Knowledge REF: p. 390 OBJ: Nursing Process: Assessment MSC: Client Needs: Psychosocial Integrity 5. When providing labor support, the nurse knows that which fetal position might cause the laboring woman more back discomfort? a . Right occiput anterior b . Left occiput anterior c . Right occiput transverse d Left occiput posterior . ANS: D Feedback A Back labor is seen mostly when the fetus is in the posterior position. B Back labor is seen mostly when the fetus is in the posterior position. C Back labor is seen mostly when the fetus is in the posterior position. D In the left occiput posterior position, each contraction pushes the fetal head against the mothers sacrum, which results in intense back discomfort. PTS: 1 DIF: Cognitive Level: Comprehension REF: p. 390 OBJ: Nursing Process: Assessment MSC: Client Needs: Health Promotion and Maintenance 6. It is important for the nurse to develop a realistic birth plan with the pregnant woman. The nurse can explain that a major advantage of nonpharmacologic pain management is that a . More complete pain relief is possible. b . No side effects or risks to the fetus are involved. c . The woman remains fully alert at all times. d . A more rapid labor is likely. ANS: B Feedback A There is less pain relief with nonpharmacologic pain management during childbirth. B Because nonpharmacologic pain management does not include analgesics, adjunct drugs, or anesthesia, it is harmless to the mother and the fetus. C The womans alertness is not altered by medication, but the increase in pain will decrease alertness. D Pain management may or may not alter the length of labor. At times when pain is decreased, the mother relaxes and labor progresses at a quicker pace. PTS: 1 DIF: Cognitive Level: Knowledge REF: p. 391 OBJ: Nursing Process: Assessment MSC: Client Needs: Physiologic Integrity 7. The best time to teach nonpharmacologic pain control methods to an unprepared laboring woman is during which phase? a . Latent phase b . Active phase c . Transition phase d . Second stage ANS: A Feedback A The latent phase of labor is the best time for intrapartum teaching, because the woman is usually anxious enough to be attentive, yet comfortable enough to understand the teaching. B During the active phase, the woman is focused internally and unable to concentrate on teaching. C During transition, the woman is focused on keeping control; she is unable to focus on anyone else or learn at this time. D During the second stage, the woman is focused on pushing. She normally handles the pain better at this point because she is active in doing something to hasten the delivery. PTS: 1 DIF: Cognitive Level: Comprehension REF: p. 391 OBJ: Nursing Process: Planning MSC: Client Needs: Health Promotion and Maintenance 8. The nurse providing newborn stabilization must be aware that the primary side effect of maternal narcotic analgesia in the newborn is a . Respiratory depression b . Bradycardia c . Acrocyanosis d . Tachypnea ANS: A Feedback A An infant delivered within 1 to 4 hours of maternal analgesic administration is at risk for respiratory depression from the sedative effects of the narcotic. B Bradycardia is not the anticipated side effect of maternal analgesics. C Acrocyanosis is an expected finding in a newborn and is not related to maternal analgesics. D The infant who is having a side effect to maternal analgesics normally would have a decrease in respirations, not an increase. PTS: 1 DIF: Cognitive Level: Knowledge REF: p. 399 | Table 18-1 OBJ: Nursing Process: Assessment MSC: Client Needs: Physiologic Integrity 9. A woman received 50 mcg of Fentanyl intravenously 1 hour before delivery. What drug should the nurse have readily available? a . Promethazine (Phenergan) b . Nalbuphine (Nubain) c . Butorphanol (Stadol) d . Naloxone (Narcan) ANS: D Feedback A Phenergan is normally given for nausea. B Nubain is an analgesic given to women in labor. C Stadol is an analgesic given to women in labor. D Naloxone (Narcan) reverses narcotic-induced respiratory depression, which may occur with administration of narcotic analgesia. PTS: 1 DIF: Cognitive Level: Comprehension REF: p. 408 | Table 18-2 OBJ: Nursing Process: Planning MSC: Client Needs: Physiologic Integrity 10. The nerve block used in labor that provides anesthesia to the lower vagina and perineum is called a(n) a Epidural . b Pudendal . c Local . d Spinal block . ANS: B Feedback A An epidural provides anesthesia for the uterus, perineum, and legs. B A pudendal block anesthetizes the lower vagina and perineum to provide anesthesia for an episiotomy and use of low forceps if needed. C A local provides anesthesia for the perineum at the site of the episiotomy. D A spinal block provides anesthesia for the uterus, perineum, and down the legs. PTS: 1 DIF: Cognitive Level: Knowledge REF: p. 408 | Table 18-2 OBJ: Nursing Process: Assessment MSC: Client Needs: Physiologic Integrity 11. The most important nursing intervention after the injection of epidural anesthesia is monitoring a . Urinary output b . Contractions c . Maternal blood pressure d . Intravenous infusion rate ANS: C Feedback A The mother is prone to bladder distention and this needs to be monitored. However, this does not become a problem until 1 to 3 hours later. B Continuing monitoring of the contractions is important, because they may be altered by the epidural. However, this is not the most important nursing intervention after the epidural is administered. C Epidural anesthesia may produce maternal hypotension due to vasodilation. D The intravenous infusion needs to be maintained before, during, and after the epidural. However, it is not the most important nursing intervention at this point. PTS: 1 DIF: Cognitive Level: Comprehension REF: p. 409 | Table 18-2 OBJ: Nursing Process: Implementation MSC: Client Needs: Physiologic Integrity 12. Which statement is true about the physiologic effects of pain in labor? a . It usually results in a more rapid labor. b . It is considered to be a normal occurrence. c . It may result in decreased placental perfusion. d . It has no effect on the outcome of labor. ANS: C Feedback A Excessive pain may prolong the labor due to increase anxiety in the woman. B Pain is considered normal for labor. However, excessive pain may be an indication of other problems and must be assessed. C When experiencing excessive pain, the woman may react with a stress response that diverts blood flow from the uterus and the fetus. D It may affect the outcome of the labor depending on the cause and the effect on the woman. PTS: 1 DIF: Cognitive Level: Comprehension REF: p. 389 OBJ: Nursing Process: Assessment MSC: Client Needs: Physiologic Integrity 13. Which woman will most likely have increased anxiety and tension during her labor? a . Gravida 1 who did not attend prepared childbirth classes b . Gravida 2 who refused any medication c . Gravida 2 who delivered a stillborn baby last year d . Gravida 3 who has two children younger than 3 years ANS: C Feedback A The woman is not prepared for labor and will have increased anxiety during labor. However, the woman with a poor previous outcome is more likely to experience more anxiety. B A gravida 2 has previous experience and can anticipate what to expect. By refusing any medication, she is taking control over her situation and will have less anxiety. C If a previous pregnancy had a poor outcome, the woman will probably be more anxious during labor and delivery. D This gravida 3 has previous experience and is aware of what to expect. PTS: 1 DIF: Cognitive Level: Comprehension REF: pp. 390-391 OBJ: Nursing Process: Assessment MSC: Client Needs: Psychosocial Integrity 14. Which method of pain management is safest for a gravida 3 para 2 admitted at 8 cm cervical dilation? a . Epidural anesthesia b . Narcotics c . Spinal block d . Breathing and relaxation techniques ANS: D Feedback A Probably not enough time remains to administer epidural anesthesia. B A narcotic given at this time may reach its peak about the time of birth and result in respiratory depression in the newborn. C Probably not enough time remains to administer spinal anesthesia. D Nonpharmacologic methods of pain management may be the best option for a woman in advanced labor. PTS: 1 DIF: Cognitive Level: Application REF: p. 392 OBJ: Nursing Process: Assessment MSC: Client Needs: Physiologic Integrity 15. The laboring woman who imagines her body opening to let the baby out is using a mental technique called a Dissociation . b Effleurage . c Imagery . d Distraction . ANS: C Feedback A Dissociation helps the woman learn to relax all muscles except those that are working. B Effleurage is self-massage. C Imagery is a technique of visualizing images that will assist the woman in coping with labor. D Distraction can be used in the early latent phase by having the woman involved in another activity. PTS: 1 DIF: Cognitive Level: Comprehension REF: p. 393 OBJ: Nursing Process: Assessment MSC: Client Needs: Psychosocial Integrity 16. When giving a narcotic to a laboring woman, the nurse should inject the medication at the beginning of a contraction so that a . Full benefit of the medication is received during that contraction. b . Less medication will be transferred to the fetus. c . The medication will be rapidly circulated. d . The maternal vital signs will not be adversely affected. ANS: B Feedback A The full benefit will be received by the woman; however, it will decrease the amount reaching the fetus. B Injecting at the beginning of a contraction, when blood flow to the placenta is normally reduced, limits transfer to the fetus. C It will not increase the circulation of the medication. D It will not alter the vital signs any more than giving it at another time. PTS: 1 DIF: Cognitive Level: Application REF: p. 401 OBJ: Nursing Process: Implementation MSC: Client Needs: Physiologic Integrity 17. The method of anesthesia in labor considered the safest for the fetus is the a Pudendal block . b Epidural block . c Spinal (subarachnoid) block . d Local infiltration . ANS: D Feedback A The fetus can be affected by maternal side effects. B The fetus can be affected by maternal hypotension. C The fetus can be affected by maternal side effects. D Local infiltration of the perineum rarely has any adverse effects on either the mother or the fetus. PTS: 1 DIF: Cognitive Level: Comprehension REF: p. 402 OBJ: Nursing Process: Assessment MSC: Client Needs: Physiologic Integrity 18. To improve placental blood flow immediately after the injection of an epidural anesthetic, the nurse should a Turn the woman to the right side. . b Place a wedge under the womans right hip. . c Give the woman oxygen. . d Decrease the intravenous infusion rate. . ANS: B Feedback A The woman needs to maintain the supine position for proper dispersal of the medication. However, placing a wedge under the hip will relieve compression of the vena cava. B Tilting the womans pelvis to the left side relieves compression of the vena cava and compensates for a lower blood pressure without interfering with dispersal of the epidural medication. C Oxygen administration will not improve placental blood flow. D The intravenous infusion rate needs to be increased to prevent hypotension. PTS: 1 DIF: Cognitive Level: Application REF: p. 409 | Table 18-2 OBJ: Nursing Process: Implementation MSC: Client Needs: Physiologic Integrity 19. The most important nursing intervention for the patient who has received an epidural narcotic is a Monitoring respiratory rate hourly . b Administering analgesics as needed . c Monitoring blood pressure every 4 hours . d Assessing the level of anesthesia . ANS: A Feedback A The possibility of respiratory depression exists for up to 24 hours after administration of an epidural narcotic. B Epidural narcotic should be enough pain relief that further medication is not necessary. Administering any other narcotic may cause an overdose. C The patients blood pressure needs to be monitored. However, that is not the major concern with this medication. D The epidural narcotic should provide pain relief, but not anesthesia. PTS: 1 DIF: Cognitive Level: Comprehension REF: p. 409 | Table 18-2 OBJ: Nursing Process: Implementation MSC: Client Needs: Physiologic Integrity 20. One of the greatest risks to the mother during administration of general anesthesia is a . Respiratory depression b . Uterine relaxation c . Inadequate muscle relaxation d . Aspiration of stomach contents ANS: D Feedback A Respirations can be altered during general anesthesia, and the anesthesiologist will take precautions to maintain proper oxygenation. B Uterine relaxation can occur with some anesthesia, but this can be monitored and prevented. C Inadequate muscle relaxation can be altered. This is not the greatest risk for the mother. D Aspiration of acidic gastric contents and possible airway obstruction is a potentially fatal complication of general anesthesia. PTS: 1 DIF: Cognitive Level: Comprehension REF: p. 407 OBJ: Nursing Process: Assessment MSC: Client Needs: Physiologic Integrity 21. To provide optimal care to the intrapartum woman, the nurse understands that the least favorable maternal position for labor is a Supine . b Sitting . c Lying on the side . d Standing . ANS: A Feedback A The supine position allows the heavy uterus to compress the inferior vena cava and can reduce placental blood flow, compromising fetal oxygen supply. B The sitting position allows gravity to assist in the descent of the fetus. C The side-lying position is comfortable for most mothers, and it allows for adequate placental blood flow. D Standing and walking may be comfortable for some women. This does not prevent adequate placental blood flow and can be encouraged if there are no other contraindications. PTS: 1 DIF: Cognitive Level: Comprehension REF: p. 404 OBJ: Nursing Process: Implementation MSC: Client Needs: Health Promotion and Maintenance 22. A newborn infant weighing 8 lb (3632 g) needs naloxone (Narcan). This infant should receive approximately mg. a 0.36 . b 3.6 . c 0.03 . d 0.3 . ANS: A Feedback A The recommended neonatal dose of naloxone is 0.1 mg/kg. B This dose is too large for this infant. C This dose is too small and would not be effective. D This dose is too small. PTS: 1 DIF: Cognitive Level: Application REF: p. 399 | Table 18-1 OBJ: Nursing Process: Implementation MSC: Client Needs: Physiologic Integrity 23. To assist the woman after delivery of the infant, the nurse knows that the blood patch is used after spinal anesthesia to relieve a . Hypotension b . Headache c . Neonatal respiratory depression d Loss of movement . ANS: B Feedback A Hypotension is prevented by increasing fluid volume before the procedure. B The subarachnoid block may cause a postspinal headache due to loss of cerebrospinal fluid from the puncture in the dura. When blood is injected into the epidural space in the area of the dural puncture, it forms a seal over the hole to stop leaking of cerebrospinal fluid. C Neonatal respiratory depression is not an expected outcome with spinal anesthesia. D Loss of movement is an expected outcome of spinal anesthesia. PTS: 1 DIF: Cognitive Level: Knowledge REF: p. 409 | Table 18-2 OBJ: Nursing Process: Assessment MSC: Client Needs: Physiologic Integrity 24. What is the first type of breathing technique used in labor? a . Slow-paced b . Modified-paced c . Patterned-paced d . Pant-blow ANS: A Feedback A Breathing for the first stage of labor consists of a cleansing breath and various breathing techniques known as paced breathing. The first type used in labor is the slow-paced. B Modified-paced breathing is used when the slow-paced breathing is no longer effective. C Patterned-paced breathing is used later in the labor and has the woman focusing on a pattern of breathing. D Pant-blow breathing can be used to prevent pushing before the cervix is completely dilated. PTS: 1 DIF: Cognitive Level: Knowledge REF: p. 394 OBJ: Nursing Process: Assessment MSC: Client Needs: Health Promotion and Maintenance 25. When instructing the woman in early labor, the nurse teaches her that an important aspect of proper breathing technique is a . Breathing no more than three times the normal rate b . Beginning and ending with a cleansing breath c . Holding the breath no longer than 10 seconds d . Adhering exactly to the techniques as they were taught ANS: B Feedback A It is important to prevent hyperventilation; however, the cleansing breaths are the most important aspect of the breathing techniques. B The cleansing breath helps the woman clear her mind to focus on relaxing and signals the coach that the contraction is beginning or ending. C The woman should hold her breath for no more than 6 to 8 seconds. D The woman needs to be flexible and change her breathing techniques as needed to keep her comfortable. Chapter 37 Nursing Care of the Child With an Infectious or Communicable Disorder MULTIPLE CHOICE 1. The nurse takes into consideration that the child most susceptible to an opportunistic infection is the one taking: a . Anticonvulsants b . A beta-adrenergic agent c . An antibiotic d . Corticosteroids ANS: D Steroids are immunosuppressive drugs that make the child very susceptible to opportunistic infections. DIF: Cognitive Level: Analysis REF: Text Reference: 717 OBJ: Objective: N/A TOP: Topic: Host Resistance KEY: Nursing Process Step: Assessment MSC: NCLEX: Safe, Effective Care Environment: Safety and Infection Control 2. When the 8-year-old asks the nurse how she got the antibodies that kept her from getting whooping cough, the nurse explains that those shots: a . Were borrowed antibodies from another person who had whooping cough b . Gave her a tiny case of whooping cough and then she made her own antibodies c . Strengthened antibodies she was born with d . Are only temporary borrowed antibodies and she needs to have another shot every 5 years ANS: B Vaccines contain live weakened or dead organisms not strong enough to cause disease but they stimulate the body to develop an immune reaction and antibodies. This is active acquired immunity. DIF: Cognitive Level: Knowledge REF: Text Reference: 717 OBJ: Objective: 3 TOP: Topic: Types of Immunity KEY: Nursing Process Step: Assessment MSC: NCLEX: Physiological Integrity: Physiological Adaptation 3. The nurse would document a rash that has erythematous circular raised lesions as: a Macular . b Papular . c Vesicular . d Pustular . ANS: B A papule is a circular, reddened elevated area on the skin. DIF: Cognitive Level: Knowledge REF: Text Reference: 718 OBJ: Objective: 6 TOP: Topic: Rashes KEY: Nursing Process Step: Assessment MSC: NCLEX: Physiological Integrity: Physiological Adaptation 4. The nurse would delay the administration of DTaP when the mother says that her infant: a Has diarrhea . b Had a temperature of 105 F from the previous inoculation . c Is teething . d Is traveling with her to Europe in a week . ANS: B A contraindication to giving the DTaP vaccine is a 105 F temperature following the previous vaccination. DIF: Cognitive Level: Analysis REF: Text Reference: 721 OBJ: Objective: 4 TOP: Topic: Immunizations KEY: Nursing Process Step: Implementation MSC: NCLEX: Safe, Effective Care Environment: Safety and Infection Control 5. The type of precaution that is necessary when caring for a toddler with varicella is: a Contact . b Protective . c Airborne infection . d Large droplet infection . ANS: C Airborne-infection precautions are used for patients with conditions such as tuberculosis, varicella, and rubeola. Small airborne particles caught on floating dust in the room can be inhaled from anywhere in the room. DIF: Cognitive Level: Application REF: Text Reference: 718 OBJ: Objective: 2 TOP: Topic: Medical Asepsis and Standard Precautions KEY: Nursing Process Step: Implementation MSC: NCLEX: Safe, Effective Care Environment: Safety and Infection Control 6. A parent is concerned because her son was exposed to varicella at preschool. The nurse would tell this parent that the incubation period for varicella is: a 2 to 10 days . b 4 to 14 days . c 3 to 32 days . d 14 to 21 days . ANS: D The incubation period for varicella is 2 to 3 weeks, usually 13 to 17 days. DIF: Cognitive Level: Knowledge REF: Text Reference: 713, Table 31-1 OBJ: Objective: 5 TOP: Topic: Common Childhood Communicable Diseases KEY: Nursing Process Step: Implementation MSC: NCLEX: Safe, Effective Care Environment: Safety and Infection Control 7. The nurse can be assured that parents understand how long a child who has varicella is contagious when they state: a . My child should stay home from school for 6 days after the pox appear. b . My child can return to school when the rash fades. c . My child must stay away from other children until all of the lesions have healed. d . My child is contagious as long as he has a fever. ANS: A The child with varicella is contagious for 6 days after the appearance of the rash. DIF: Cognitive Level: Application REF: Text Reference: 713, Table 31-1 OBJ: Objective: 5 TOP: Topic: Common Childhood Communicable Diseases KEY: Nursing Process Step: Evaluation MSC: NCLEX: Safe, Effective Care Environment: Safety and Infection Control 8. The statement made by a sexually active adolescent girl indicating an understanding of the prevention of sexually transmitted diseases is: a . I always douche after intercourse. b . I think you can get a vaccination for STDs now. c . I insist that my partner wear a condom. d . I am protected because I take the pill. ANS: C The use of condoms to prevent STDs is not considered 100% effective but is recommended for sexual intercourse. DIF: Cognitive Level: Application REF: Text Reference: 725 OBJ: Objective: 8 TOP: Topic: Sexually Transmitted Diseases KEY: Nursing Process Step: Evaluation MSC: NCLEX: Safe, Effective Care Environment: Safety and Infection Control 9. The priority nursing diagnosis for a hospitalized infant who is HIV-positive would be: a . Risk for injury b . Altered nutrition c . Impaired skin integrity d . Risk for infection ANS: D The infant who is HIV-positive has impaired immunologic functioning and is at high risk for infection. DIF: Cognitive Level: Analysis REF: Text Reference: 725 OBJ: Objective: 7 TOP: Topic: Human Immunodeficiency Virus KEY: Nursing Process Step: Nursing Diagnosis MSC: NCLEX: Safe, Effective Care Environment: Safety and Infection Control 10. A parent of a newborn asked the nurse, When will my baby get the hepatitis B vaccine? The nurse bases a response on the knowledge that the first dose of Comvax should be given to infants born to a hepatitis B-negative mother at: a 2 months . b 4 months . c 6 months . d 1 year . ANS: B The American Academy of Pediatrics recommends that Comvax, the only thimerosal-free hepatitis B vaccine, should be used for infants born to HBsAg-negative mothers beginning at the 2-month well-child visit. DIF: Cognitive Level: Knowledge REF: Text Reference: 722, Figure 31-6 OBJ: Objective: 4 TOP: Topic: Immunization Schedule KEY: Nursing Process Step: Implementation MSC: NCLEX: Safe, Effective Care Environment: Safety and Infection Control 11. The nurse would base a response to a parent about how his child got hepatitis A on the information that the child: a Came in contact with infected blood . b Came in contact with droplets in the air . c Was bitten by a mosquito or a tick . d Ate shrimp while they were in Mexico . ANS: D Hepatitis A results from ingestion of contaminated water or shellfish. DIF: Cognitive Level: Comprehension REF: Text Reference: 714, Table 31-1 OBJ: Objective: 5 TOP: Topic: Common Childhood Communicable Diseases KEY: Nursing Process Step: Implementation MSC: NCLEX: Safe, Effective Care Environment: Safety and Infection Control 12. An infant is hospitalized for RSV bronchiolitis. Which type of precautions would the nurse use when caring for the infant? a Large-droplet infection precautions . b Airborne-infection precautions . c Contact precautions . d Protective precautions . ANS: C Contact precautions are used when the condition transmits organisms via skin-to-skin contact or indirect touch of a contaminated fomite. DIF: Cognitive Level: Application REF: Text Reference: 718 OBJ: Objective: 2 TOP: Topic: Medical Asepsis and Standard Precautions KEY: Nursing Process Step: Implementation MSC: NCLEX: Safe, Effective Care Environment: Safety and Infection Control 13. A 9-year-old child hospitalized for neutropenia is placed in protective isolation. What is the most appropriate response for the nurse to make when the child asks, Why do you have to wear a gown and mask when you are in my room? a Nurses and doctors wear gowns and masks because you have a condition that . could be spread to others. b The gown and mask are to protect you because you could get an infection very . easily. c Im wearing this because there are a lot of bacteria in the hospital. . d I might look scary but you wont need this after you have had medication for 24 . hours. ANS: B Protective isolation is used for patients who are not communicable but have a lowered resistance and are highly susceptible to infection. DIF: Cognitive Level: Application REF: Text Reference: 718 OBJ: Objective: 2 TOP: Topic: Protective Isolation KEY: Nursing Process Step: Implementation MSC: NCLEX: Safe, Effective Care Environment: Safety and Infection Control 14. The nurse is planning to administer immunizations at a well-child visit when a parent reports the 18-month-old child is allergic to eggs. The vaccine that would be contraindicated is: a Influenza . b Inactivated polio vaccine . c Diphtheria, tetanus, acellular pertussis . d Hepatitis B . ANS: A The influenza vaccine should not be given to children who have an allergy to eggs. DIF: Cognitive Level: Analysis REF: Text Reference: 720 OBJ: Objective: 3 TOP: Topic: Nurses Role in Immunizations-Allergy KEY: Nursing Process Step: Planning MSC: NCLEX: Safe, Effective Care Environment: Safety and Infection Control 15. The nurse would choose to administer the immunization injection of: a DTaP subcutaneously . b Hib vaccine prepared in a separate syringe . c Varicella intramuscularly . d Varicella 1 week after the MMR vaccine . ANS: B Hib vaccine must be given in a separate syringe from other vaccines administered at the same time. DIF: Cognitive Level: Analysis REF: Text Reference: 722, Figure 31-6 OBJ: Objective: 3 TOP: Topic: Nurses Role in Immunizations KEY: Nursing Process Step: Evaluation MSC: NCLEX: Physiological Integrity: Basic Care and Comfort 16. A child was sent to the school nurse because of a rash. The nurse noted the rash was present on the trunk, extremities, and face. The childs cheeks were bright red. The nurse is aware this type of rash is consistent with: a Measles . b Roseola . c Varicella . d Fifth disease . ANS: D When a child has fifth disease, the child has a generalized rash and the cheeks have a slapped cheek appearance. DIF: Cognitive Level: Application REF: Text Reference: 713, Table 31-1 OBJ: Objective: 6 TOP: Topic: Common Childhood Communicable Diseases KEY: Nursing Process Step: Assessment MSC: NCLEX: Safe, Effective Care Environment: Safety and Infection Control 17. The nurse determined the parent understood the information when he stated: a Ill have my son wear dark clothing on his hike. . b We should all get the Lyme disease vaccine before our trip. . c Ill get a prescription for amoxicillin to take with us. . d We will wear long pants and long-sleeved shirts in the woods. . ANS: D People should keep skin covered by wearing protective clothing in wooded areas to prevent tick bites. DIF: Cognitive Level: Application REF: Text Reference: 715, Table 31-1 OBJ: Objective: 5 TOP: Topic: Common Childhood Communicable Diseases KEY: Nursing Process Step: Evaluation MSC: NCLEX: Safe, Effective Care Environment: Safety and Infection Control 18. An adolescent is taking tetracycline for a sexually transmitted disease. The nurse would stress in the instruction about this medication to: a Finish all of the medication. . b Get plenty of fresh air and sunlight. . c The medication should be taken with food. . d Take an antacid if the medication causes an upset stomach. . ANS: A The nurse would teach the adolescent to take all of the prescribed medication to avoid making the microorganism resistant to tetracyclines. Chapter 38 Nursing Care of the Child With Alteration in Intracranial Regulation/Neurologic Disorder MULTIPLE CHOICE 1. What is a sign of increased intracranial pressure (ICP) in a 10-year-old child? a . Headache b . Bulging fontanel c . Tachypnea d . Increase in head circumference ANS: A Feedback A Headaches are a clinical manifestation of increased ICP in children. A change in the childs normal behavior pattern may be an important early sign of increased ICP. B This is a manifestation of increased ICP in infants. A 10-year-old child would have a closed fontanel. C A change in respiratory pattern is a late sign of increased ICP. Cheyne-Stokes respiration may be evident. This refers to a pattern of increasing rate and depth of respirations followed by a decreasing rate and depth with a pause of variable length. D By 10 years of age, cranial sutures have fused so that head circumference will not increase in the presence of increased ICP. PTS: 1 DIF: Cognitive Level: Application REF: p. 1418 | Box 52-1 OBJ: Nursing Process: Assessment MSC: Client Needs: Physiologic Integrity 2. Which information should the nurse give to a child who is to have magnetic resonance imaging (MRI) of the brain? a . Your head will be restrained during the procedure. b . You will have to drink a special fluid before the test. c . You will have to lie flat after the test is finished. d . You will have electrodes placed on your head with glue. ANS: A Feedback A To reduce fear and enhance cooperation during the MRI, the child should be made aware that the head will be restricted to obtain accurate information. B Drinking fluids is usually done for gastrointestinal procedures. C A child should lie flat after a lumbar puncture, not during an MRI. D Electrodes are attached to the head for an electroencephalogram. PTS: 1 DIF: Cognitive Level: Application REF: p. 1415 OBJ: Nursing Process: Planning MSC: Client Needs: Physiologic Integrity 3. Which term is used when a patient remains in a deep sleep, responsive only to vigorous and repeated stimulation? a . Coma b . Stupor c . Obtundation d . Persistent vegetative state ANS: B Feedback A Coma is the state in which no motor or verbal response occurs to noxious (painful) stimuli. B Stupor exists when the child remains in a deep sleep, responsive only to vigorous and repeated stimulation. C Obtundation describes a level of consciousness in which the child is arousable with stimulation. D Persistent vegetative state describes the permanent loss of function of the cerebral cortex. PTS: 1 DIF: Cognitive Level: Comprehension REF: p. 1419 OBJ: Nursing Process: Assessment MSC: Client Needs: Physiologic Integrity 4. The Glasgow Coma Scale consists of an assessment of a . Pupil reactivity and motor response b . Eye opening and verbal and motor responses c . Level of consciousness and verbal response d . ICP and level of consciousness ANS: B Feedback A Pupil reactivity is not a part of the Glasgow Coma Scale but is included in the pediatric coma scale. B The Glasgow Coma Scale assesses eye opening, and verbal and motor responses. C Level of consciousness is not a part of the Glasgow Coma Scale. D Intracranial pressure and level of consciousness are not part of the Glasgow Coma Scale. PTS: 1 DIF: Cognitive Level: Comprehension REF: p. 1419 | Table 52-1 OBJ: Nursing Process: Assessment MSC: Client Needs: Physiologic Integrity 5. Nursing care of the infant who has had a myelomeningocele repair should include a . Securely fastening the diaper b . Measurement of pupil size c . Measurement of head circumference d . Administration of seizure medications ANS: C Feedback A A diaper should be placed under the infant but not fastened. Keeping the diaper open facilitates frequent cleaning and decreases the risk for skin breakdown. B Pupil size measurement is usually not necessary. C Head circumference measurement is essential because hydrocephalus can develop in these infants. D Seizure medications are not routinely given to infants who do not have seizures. PTS: 1 DIF: Cognitive Level: Application REF: p. 1422 OBJ: Nursing Process: Implementation MSC: Client Needs: Physiologic Integrity 6. The most common problem of children born with a myelomeningocele is a Neurogenic bladder . b . Intellectual impairment c . Respiratory compromise d . Cranioschisis ANS: A Feedback A Myelomeningocele is one of the most common causes of neuropathic (neurogenic) bladder dysfunction among children. B Risk of intellectual impairment is minimized through early intervention and management of hydrocephalus. C Respiratory compromise is not a common problem in myelomeningocele. D Cranioschisis is a skull defect through which various tissues protrude. It is not associated with myelomeningocele. PTS: 1 DIF: Cognitive Level: Comprehension REF: p. 1423 OBJ: Nursing Process: Problem Identification MSC: Client Needs: Physiologic Integrity 7. A recommendation to prevent neural tube defects is the supplementation of a . Vitamin A throughout pregnancy b . Multivitamin preparations as soon as pregnancy is suspected c . Folic acid for all women of childbearing age d Folic acid during the first and second trimesters of pregnancy . ANS: C Feedback A Vitamin A does not have a relation to the prevention of spina bifida. B Folic acid supplementation is recommended for the preconceptual period, as well as during the pregnancy. C The widespread use of folic acid among women of childbearing age is expected to decrease the incidence of spina bifida significantly. D Folic acid supplementation is recommended for the preconceptual period, as well as during the pregnancy. PTS: 1 DIF: Cognitive Level: Analysis REF: p. 1422 OBJ: Nursing Process: Implementation MSC: Client Needs: Physiologic Integrity 8. How much folic acid is recommended for women of childbearing age? a 1.0 mg . b 0.4 mg . c 1.5 mg . d 2.0 mg . ANS: B Feedback A 1.0 mg is too low a dose. B It has been estimated that a daily intake of 0.4 mg of folic acid in women of childbearing age has contributed to a reduction in the number of children with neural tube defects. C 1.5 mg is not the recommended dosage of folic acid. D 2.0 mg is not the recommended dosage of folic acid. PTS: 1 DIF: Cognitive Level: Comprehension REF: p. 1421 OBJ: Nursing Process: Implementation MSC: Client Needs: Physiologic Integrity 9. Latex allergy is suspected in a child with spina bifida. Appropriate nursing interventions include a Avoiding using any latex product . b Using only nonallergenic latex products . c Administering medication for long-term desensitization . d Teaching family about long-term management of asthma . ANS: A Feedback A Care must be taken that individuals who are at high risk for latex allergies do not come in direct or secondary contact with products or equipment containing latex at any time during medical treatment. Latex allergy is estimated to occur in 75% of this patient population. B There are no nonallergic latex products. C At this time, desensitization is not an option. D The child does not have asthma. The parents must be taught about allergy and the risk of anaphylaxis. PTS: 1 DIF: Cognitive Level: Application REF: p. 1422 OBJ: Nursing Process: Intervention MSC: Client Needs: Physiologic Integrity 10. When a 2-week-old infant is seen for irritability, poor appetite, and rapid head growth with observable distended scalp veins, the nurse recognizes these signs as indicative of a . Hydrocephalus b . Syndrome of inappropriate antidiuretic hormone (SIADH) c . Cerebral palsy d . Reyes syndrome ANS: A Feedback A The combination of signs is strongly suggestive of hydrocephalus. B SIADH would not manifest in this way. The child would have decreased urination, hypertension, weight gain, fluid retention, hyponatremia, and increased urine specific gravity. C The manifestations of cerebral palsy vary but may include persistence of primitive reflexes, delayed gross motor development, and lack of progression through developmental milestones. D Reyes syndrome is associated with an antecedent viral infection with symptoms of malaise, nausea, and vomiting. Progressive neurologic deterioration occurs. PTS: 1 DIF: Cognitive Level: Comprehension REF: p. 1423 | Table 52-2 OBJ: Nursing Process: Assessment MSC: Client Needs: Physiologic Integrity 11. What finding should cause the nurse to suspect a diagnosis of spastic cerebral palsy? a . Tremulous movements at rest and with activity b . Sudden jerking movement caused by stimuli c . Writhing, uncontrolled, involuntary movements d . Clumsy, uncoordinated movements ANS: B Feedback A Tremulous movements are characteristic of rigid/tremor/atonic cerebral palsy. B Spastic cerebral palsy, the most common type of cerebral palsy, will manifest with hypertonicity and increased deep tendon reflexes. The childs muscles are very tight and any stimuli may cause a sudden jerking movement. C Slow, writhing, uncontrolled, involuntary movements occur with athetoid or dyskinetic cerebral palsy. D Clumsy movements, loss of coordination, equilibrium, and kinesthetic sense occur in ataxic cerebral palsy. PTS: 1 DIF: Cognitive Level: Application REF: p. 1426 OBJ: Nursing Process: Assessment MSC: Client Needs: Physiologic Integrity 12. Which finding in an analysis of cerebrospinal fluid (CSF) is consistent with a diagnosis of bacterial meningitis? a . CSF appears cloudy. b . CSF pressure is decreased. c . Few leukocytes are present. d . Glucose level is increased compared with blood. ANS: A Feedback A In acute bacterial meningitis, the CSF is cloudy to milky or yellowish in color. B The CSF pressure is usually increased in acute bacterial meningitis. C Many polymorphonuclear cells are present in CSF with acute bacterial meningitis. D The CSF glucose level is usually decreased compared with the serum glucose level. PTS: 1 DIF: Cognitive Level: Analysis REF: p. 1439 OBJ: Nursing Process: Assessment MSC: Client Needs: Physiologic Integrity 13. How should the nurse explain positioning for a lumbar puncture to a 5-year-old child? a You will be on your knees with your head down on the table. . b You will be able to sit up with your chin against your chest. . c You will be on your side with the head of your bed slightly raised. . d You will lie on your side and bend your knees so that they touch your chin. . ANS: D Feedback A The knee-chest position is not appropriate for a lumbar puncture. B An infant can be placed in a sitting position with the infant facing the nurse and the head steadied against the nurses body. C A side-lying position with the head of the bed elevated is not appropriate for a lumbar puncture. D The child should lie on her side with knees bent and chin tucked in to the knees. This position exposes the area of the back for the lumbar puncture. PTS: 1 DIF: Cognitive Level: Application REF: p. 1415 OBJ: Nursing Process: Implementation MSC: Client Needs: Psychosocial Integrity 14. A mother reports that her child has episodes where he appears to be staring into space. This behavior is characteristic of which type of seizure? a Absence . b Atonic . c Tonic-clonic . d Simple partial . ANS: A Feedback A Absence seizures are very brief episodes of altered awareness. The child has a blank expression. B Atonic seizures cause an abrupt loss of postural tone, loss of consciousness, confusion, lethargy, and sleep. C Tonic-clonic seizures involve sustained generalized muscle contractions followed by alternating contraction and relaxation of major muscle groups. D There is no change in level of consciousness with simple partial seizures. Simple partial seizures consist of motor, autonomic, or sensory symptoms. PTS: 1 DIF: Cognitive Level: Comprehension REF: p. 1434 | Box 52-5 OBJ: Nursing Process: Assessment MSC: Client Needs: Physiologic Integrity 15. What is the best response to a father who tells the nurse that his son daydreams at home and his teacher has observed this behavior at school? a Your son must have an active imagination. . b Can you tell me exactly how many times this occurs in one day? . c Tell me about your sons activity when you notice the daydreams. . d He is probably overtired and needs more rest. . ANS: C Feedback A This response does not address the childs symptoms or the fathers concern. B This behavior is consistent with absence seizures, which can occur one after the other several times a day. Determining an exact number of absence seizures is not as useful as learning about behavior before the seizure that might have precipitated seizure activity. C The daydream episodes are suggestive of absence seizures, and data about activity associated with the daydreams should be obtained. D This response ignores both the childs symptoms and the fathers concern about the daydreaming behavior. PTS: 1 DIF: Cognitive Level: Application REF: p. 1434 | Box 52-5 OBJ: Nursing Process: Implementation MSC: Client Needs: Physiologic Integrity 16. The nurse teaches parents to alert their health care provider about which adverse effect when a child receives valproic acid (Depakene) to control generalized seizures? a Weight loss . b Bruising . c Anorexia . d Drowsiness . ANS: B Feedback A Weight gain, not loss, is a side effect of valproic acid. B Thrombocytopenia is an adverse effect of valproic acid. Parents should be alert for any unusual bruising or bleeding. C Drowsiness is not a side effect of valproic acid, although it is associated with other anticonvulsant medications. D Anorexia is not a side effect of valproic acid. PTS: 1 DIF: Cognitive Level: Application REF: p. 1435 | Table 52-3 OBJ: Nursing Process: Implementation MSC: Client Needs: Health Promotion and Maintenance 17. A child with a head injury sleeps unless aroused, and when aroused responds briefly before falling back to sleep. What should the nurse chart for this childs level of consciousness? a Disoriented . b Obtunded . c Lethargic . d Stuporous . ANS: B Feedback A Disoriented refers to lack of ability to recognize place or person. B Obtunded describes an individual who sleeps unless aroused and once aroused has limited interaction with the environment. C An individual is lethargic when he or she awakens easily but exhibits limited responsiveness. D Stupor refers to requiring considerable stimulation to arouse the individual. PTS: 1 DIF: Cognitive Level: Application REF: p. 1419 OBJ: Nursing Process: Assessment MSC: Client Needs: Physiologic Integrity 18. Which type of fractures describes traumatic separation of cranial sutures? a Basilar . b Linear . c Commuted . d Depressed . ANS: C Feedback A A basilar fracture involves the basilar portion of the frontal, ethmoid, sphenoid, temporal, or occipital bone. B A linear fracture includes a straight-line fracture without dura involvement. C Commuted skull fractures include fragmentation of the bone or a multiple fracture line. D A depressed fracture has the bone pushed inward, causing pressure on the brain. Chapter 39 Nursing Care of the Child With Alteration in Sensory Perception/Disorder of Eyes or Ears MULTIPLE CHOICE 1. A parent comments that her infant has had several ear infections in the past few months. The nurse understands that infants are more susceptible to otitis media because: a . Infants are in a supine or prone position most of the time. b . Sucking on a nipple creates middle ear pressure. c . They have increased susceptibility to upper respiratory tract infections. d . The eustachian tube is short, straight, and wide. ANS: D An infants eustachian tubes are shorter, wider, and straighter, allowing microorganisms easy access to the middle ear. DIF: Cognitive Level: Knowledge REF: 511 OBJ: 2 TOP: Otitis Media KEY: Nursing Process Step: Assessment MSC: NCLEX: Physiological Integrity: Physiological Adaptation 2. The nurse determines a mother understands instructions about administering an oral antibiotic for otitis media when the mother verbalizes that she will: a . Continue using the medication until symptoms are relieved. b . Share the medicine with siblings if their symptoms are the same. c . Give the medication with a glass of milk. d . Administer prescribed doses until all the medication is used. ANS: D Antibiotic therapy for otitis media is continued until the prescribed amount has been completed, even if symptoms are alleviated. DIF: Cognitive Level: Application REF: 522 OBJ: 2 TOP: Otitis Media KEY: Nursing Process Step: Evaluation MSC: NCLEX: Safe, Effective Care Environment: Safety and Infection Control 3. The situation in which the nurse would be suspicious about a hearing impairment is: a . A 3-month-old infant with a positive Moro reflex b . A 15-month-old toddler who is babbling c . An 18-month-old toddler who is speaking one-syllable words d . A 24-month-old toddler who communicates by pointing ANS: D The child who is not making verbal attempts by 18 months should undergo a complete physical examination. DIF: Cognitive Level: Analysis REF: 523 OBJ: 3 TOP: Hearing Impairment KEY: Nursing Process Step: Assessment MSC: NCLEX: Health Promotion and Maintenance: Prevention and Early Detection of Disease 4. The best way for the nurse to communicate with a 10-year-old child who has a hearing impairment would be to: a Use gestures and signs as much as possible. . b . Let the childs parents communicate for her. c . Face the child and speak clearly in short sentences. d . Recognize that the childs ability to communicate will be on a 6-year-old level. ANS: C The nurse who faces the child and speaks clearly will help the hearing-impaired child in the hospital to develop a healthy personality. DIF: Cognitive Level: Application REF: 523 OBJ: 3 TOP: Hearing Impairment KEY: Nursing Process Step: Implementation MSC: NCLEX: Physiological Integrity 5. The nurse planning postoperative teaching for a child who has had a tympanostomy with insertion of tubes would include: a Keep the infant flat after feeding. . b Give over-the-counter anticongestants. . c Avoid getting water in the ears. . d Clean the ear canal with cotton-tipped applicators. . ANS: C Following a tympanostomy, care should be taken to avoid getting water in the ears. DIF: Cognitive Level: Comprehension REF: 522 OBJ: 2 TOP: Postoperative Care of Tympanostomy KEY: Nursing Process Step: Planning MSC: NCLEX: Physiological Integrity: Reduction of Risk 6. The school nurse would suspect amblyopia when the child: a Has a reddened sclera in one eye . b Covers one eye to read the board . c Complains of a headache . d Has copious tears while watching TV . ANS: B Indicators of amblyopia include covering one eye to see, tilting the head to see, missing objects in attempts to pick them up. Although headaches may be associated with amblyopia, it is too vague to point suspicion to any disorder. DIF: Cognitive Level: Analysis REF: 526 OBJ: 4 TOP: Amblyopia KEY: Nursing Process Step: Assessment MSC: NCLEX: Health Promotion and Maintenance: Prevention and Early Detection of Disease 7. The nurse explains that a common treatment for amblyopia is: a Patching the good eye to force the brain to use the affected eye . b . Patching the affected eye to allow the refractory muscles to rest c . Using glasses that will slightly blur the image for the good eye d . Using corticosteroids to treat inflammation of the optic nerve ANS: A Early detection and treatment are essential for the child with amblyopia. Treatment includes patching the good eye and using glasses to correct refractive errors. DIF: Cognitive Level: Knowledge REF: 526 OBJ: 4 TOP: Amblyopia KEY: Nursing Process Step: N/A MSC: NCLEX: Health Promotion and Maintenance: Prevention and Early Detection of Disease 8. The school nurse recognizes the cardinal sign of a hyphema when she assesses: a Opacity of the lens . b A yellow-white reflex on the pupil . c A dark-red spot in front of the iris . d Inflamed mucous membranes of the eyelids . ANS: C A dark red spot in front of the iris is blood that has drained into the anterior chamber as the result of an injury. DIF: Cognitive Level: Knowledge REF: 527 OBJ: N/A TOP: Retinoblastoma KEY: Nursing Process Step: Assessment MSC: NCLEX: Health Promotion and Maintenance: Prevention and Early Detection of Disease 9. The nurse is planning to teach parents about prevention of Reyes syndrome. What information would the nurse include in this teaching? a Use aspirin instead of acetaminophen for children with viral illness. . b Advise parents to have their children immunized against Reyes syndrome. . c Avoid giving salicylate-containing medications to a child who has viral . symptoms. d Get the child tested for Reyes syndrome if the child exhibits fever, vomiting, . and lethargy. ANS: C Prevention of Reyes syndrome includes educating parents not to give aspirin-containing medication to children with viral symptoms. DIF: Cognitive Level: Application REF: 529 OBJ: 11 TOP: Reyes Syndrome KEY: Nursing Process Step: Planning MSC: NCLEX: Health Promotion and Maintenance: Prevention and Early Detection of Disease 10. The nurse caring for a 5-month-old with viral influenza suspects the development of Reyes syndrome when the child: a Has respirations drop from 18 to 14 breaths/min . b . Goes to sleep after feeding c . Suddenly vomits d . Develops a macular rash ANS: C A child with a viral infection is at risk for Reyes syndrome, the onset of which is effortless vomiting, lethargy, and a change in LOC. A 5-month-old child that sleeps after eating is normal. DIF: Cognitive Level: Application REF: 529 OBJ: 11 TOP: Reyes Syndrome KEY: Nursing Process Step: Assessment MSC: NCLEX: Health Promotion and Maintenance: Prevention and Early Detection of Disease 11. The nurse explains that febrile seizures: a Occur when the body temperature exceeds 103F . b Can be prevented by anticonvulsant medication . c Usually lead to the development of epilepsy . d Occur when the temperature rises quickly . ANS: D Febrile seizures occur in response to a rapid rise in temperature, often above 102F (38.8C). DIF: Cognitive Level: Comprehension REF: 533 OBJ: 9 TOP: Febrile Seizures KEY: Nursing Process Step: N/A MSC: NCLEX: Physiological Integrity: Physiological Adaptation 12. A parent reports that her child experiences episodes where he appears to be staring into space. This behavior is characteristic of which type of seizure? a Absence . b Akinetic . c Myoclonic . d Complex partial . ANS: A Absence seizures are characterized by transient loss of consciousness where the child appears to stare blankly, and which may last only a few seconds. DIF: Cognitive Level: Analysis REF: 534, Table 23-2 OBJ: 9 TOP: Epilepsy KEY: Nursing Process Step: Assessment MSC: NCLEX: Physiological Integrity: Physiological Adaptation 13. An adolescent has just had a generalized seizure lasting 1 minute. Following the seizure, the nurse should: a . Help the patient to sit upright b . Turn on the side c Offer ice chips . d Assist to ambulate . ANS: B During the tonic phase of a generalized seizure, the head, legs, and back stiffen. DIF: Cognitive Level: Analysis REF: 534, Table 23-2 OBJ: 9 TOP: Epilepsy KEY: Nursing Process Step: Assessment MSC: NCLEX: Physiological Integrity: Basic Care and Comfort 14. When a child is experiencing a generalized tonic-clonic seizure, an appropriate nursing action would be to: a . Guide the child to the floor if the child is standing, and then go for help. b . Move objects out of the childs immediate area. c . Stick a padded tongue blade between the childs teeth. d . Manually restrain the child. ANS: B During a generalized tonic-clonic seizure, the immediate area is cleared to protect the child from injury. DIF: Cognitive Level: Application REF: 534, Table 23-2 OBJ: 9 TOP: Epilepsy KEY: Nursing Process Step: Implementation MSC: NCLEX: Safe, Effective Care Environment: Safety and Infection Control 15. A child had a generalized tonic-clonic seizure that lasted 90 seconds. After a generalized tonic-clonic seizure, the nurse would expect that the child might be: a Restless . b Sleepy . c Nauseated . d Anxious . ANS: B Following a generalized tonic-clonic seizure, the child may have some confusion and may sleep for a time (postictal lethargy) and then return to full consciousness. DIF: Cognitive Level: Analysis REF: 535 OBJ: 9 TOP: Epilepsy KEY: Nursing Process Step: Assessment MSC: NCLEX: Physiological Integrity: Physiological Adaptation 16. The nurse would include in a teaching plan pertinent to the long-term administration of Dilantin that: a . The medication should be given with food to reduce gastrointestinal distress. b . Behavioral changes are a possible side effect. c . Gums should be massaged regularly to prevent hyperplasia. d Blood pressure should be closely monitored. . ANS: C Dilantin can cause gum overgrowth, which can be minimized by regular massaging. DIF: Cognitive Level: Application REF: 536 OBJ: 9 TOP: Epilepsy KEY: Nursing Process Step: Implementation MSC: NCLEX: Physiological Integrity: Pharmacological Therapies 17. The nurse observes that the legs of a child with cerebral palsy cross involuntarily, and the child exhibits jerky movements with his arms as he tries to eat. The nurse recognizes that he has which type of cerebral palsy? a Athetoid . b Ataxic . c Spastic . d Mixed . ANS: C Spasticity is characterized by tension in certain muscle groups, which makes voluntary movements of muscles jerky and uncoordinated. DIF: Cognitive Level: Analysis REF: 536 OBJ: 10 TOP: Cerebral Palsy KEY: Nursing Process Step: Assessment MSC: NCLEX: Physiological Integrity: Physiological Adaptation 18. The assessment finding that should be reported immediately if observed in a child with meningitis is: a Irregular respirations . b Tachycardia . c Slight drop in blood pressure . d Elevated temperature . ANS: A Irregular respirations in conjunction with slowing heart rate and increasing blood pressure are reported immediately because they could indicate increased intracranial pressure. Chapter 40 Nursing Care of the Child With an Alteration in Gas Exchange/Respiratory Disorder MULTIPLE CHOICE 1. A child has had cold symptoms for more than 2 weeks, a headache, nasal congestion with purulent nasal drainage, facial tenderness, and a cough that increases during sleep. The nurse recognizes that these symptoms are characteristic of which respiratory condition? a Allergic rhinitis . b Bronchitis . c Asthma . d Sinusitis . ANS: D Feedback A The classic symptoms of allergic rhinitis are watery rhinorrhea, itchy nose, eyes, ears, and palate, and sneezing. Symptoms occur as long as the child is exposed to the allergen. B Bronchitis is characterized by a gradual onset of rhinitis and a cough that is initially nonproductive but may change to a loose cough. C The manifestations of asthma may vary, with wheezing being a classic sign. The symptoms presented in the question do not suggest asthma. D Sinusitis is characterized by signs and symptoms of a cold that do not improve after 14 days, a low-grade fever, nasal congestion and purulent nasal discharge, headache, tenderness, a feeling of fullness over the affected sinuses, halitosis, and a cough that increases when the child is lying down. PTS: 1 DIF: Cognitive Level: Application REF: p. 1150 OBJ: Nursing Process: Assessment MSC: Client Needs: Physiologic Integrity 2. For which problem should the child with chronic otitis media with effusion be evaluated? a Brain abscess . b Meningitis . c Hearing loss . d Perforation of the tympanic membrane . ANS: C Feedback A The infection of acute otitis media can spread to surrounding tissues, causing a brain abscess. B The infection of acute otitis media can spread to surrounding tissues, causing meningitis. C Chronic otitis media with effusion is the most common cause of hearing loss in children. D Inflammation and pressure from acute otitis media may result in perforation of the tympanic membrane. PTS: 1 DIF: Cognitive Level: Analysis REF: p. 1154 OBJ: Nursing Process: Evaluation MSC: Client Needs: Physiologic Integrity 3. The nurse expects the initial plan of care for a 9-month-old child with an acute otitis media infection to include a symptomatic treatment and observation for 48 to 72 hours after diagnosis . b an oral antibiotic, such as amoxicillin, five times a day for 7 days . c pneumococcal conjugate vaccine . d myringotomy with tympanoplasty tubes . ANS: A Feedback A Select children 6 months of age or older with acute otitis media are treated by initiating symptomatic treatment and observation for 48 to 72 hours. B Acute otitis media may be treated with a 5- to 10-day course of oral antibiotics. When treatment is indicated, amoxicillin at a divided dose of 80 to 90 mg/kg/ day given either every 8 or 12 hours for 5 to 10 days may be ordered. C Pneumococcal conjugate vaccine helps to prevent ear infections but is not included in the initial plan of care for a child with acute otitis media. D Surgical intervention is considered when the child has persistent ear infection despite antibiotic therapy or with otitis media with effusion that persists for more than 3 months and is associated with hearing loss. PTS: 1 DIF: Cognitive Level: Application REF: p. 1152 OBJ: Nursing Process: Implementation MSC: Client Needs: Physiologic Integrity 4. Which statement made by a parent indicates an understanding about treatment of streptococcal pharyngitis? a . I guess my child will need to have his tonsils removed. b . A couple of days of rest and some ibuprofen will take care of this. c . I should give the penicillin three times a day for 10 days. d . I am giving my child prednisone to decrease the swelling of the tonsils. ANS: C Feedback A Surgical removal of the tonsils is a controversial issue. It may be warranted in cases of recurrent tonsillitis. It is not indicated for the treatment of acute tonsillitis. B Comfort measures such as rest and analgesics are indicated, but these will not treat the bacterial infection. C Streptococcal pharyngitis is best treated with oral penicillin two to three times daily for 10 days. D Corticosteroids are not used in the treatment of streptococcal pharyngitis. PTS: 1 DIF: Cognitive Level: Application REF: p. 1156 OBJ: Nursing Process: Evaluation MSC: Client Needs: Health Promotion and Maintenance 5. The father of an infant calls the nurse to his sons room because he is making a strange noise. A diagnosis of laryngomalacia is made. What does the nurse expect to find on assessment? a . Stridor b . High-pitched cry c . Nasal congestion d . Spasmodic cough ANS: A Feedback A Stridor is usually present at birth but may begin as late as 2 months. Symptoms increase when the infant is supine or crying. B High-pitched cries are consistent with neurologic abnormalities and are not usually respiratory in nature. C Nasal congestion is nonspecific in relation to laryngomalacia. D Spasmodic cough is associated with croup; it is not a common symptom of laryngomalacia. PTS: 1 DIF: Cognitive Level: Analysis REF: p. 1158 OBJ: Nursing Process: Assessment MSC: Client Needs: Physiologic Integrity 6. The nurse should assess a child who has had a tonsillectomy for a . Frequent swallowing b . Inspiratory stridor c . Rhonchi d . Elevated white blood cell count ANS: A Feedback A Frequent swallowing is indicative of postoperative bleeding. B Inspiratory stridor is characteristic of croup. C Rhonchi are lower airway sounds indicating pneumonia. D Assessment of blood cell counts is part of a preoperative workup. PTS: 1 DIF: Cognitive Level: Application REF: p. 1157 OBJ: Nursing Process: Assessment MSC: Client Needs: Health Promotion and Maintenance 7. The parent of a toddler calls the nurse, asking about croup. What is a distinguishing manifestation of spasmodic croup? a Wheezing is heard audibly. . b . It has a harsh, barky cough. c . It is bacterial in nature. d . The child has a high fever. ANS: B Feedback A Wheezing is not a distinguishing manifestation of croup. It can accompany conditions such as asthma or bronchiolitis. B Spasmodic croup is viral in origin; is usually preceded by several days of symptoms of upper respiratory tract infection; often begins at night; and is marked by a harsh, metallic, barky cough; sore throat; inspiratory stridor; and hoarseness. C Spasmodic croup is viral in origin. D A high fever is not usually present. PTS: 1 DIF: Cognitive Level: Application REF: p. 1159 OBJ: Nursing Process: Assessment MSC: Client Needs: Physiologic Integrity 8. Which intervention for treating croup at home should be taught to parents? a Have a decongestant available to give the child when an attack occurs. . b Have the child sleep in a dry room. . c Take the child outside. . d Give the child an antibiotic at bedtime. . ANS: C Feedback A Decongestants are inappropriate for croup, which affects the middle airway level. B A dry environment may contribute to symptoms. C Taking the child into the cool, humid, night air may relieve mucosal swelling and improve symptoms. D Croup is caused by a virus. Antibiotic treatment is not indicated. PTS: 1 DIF: Cognitive Level: Application REF: p. 1160 OBJ: Nursing Process: Implementation MSC: Client Needs: Health Promotion and Maintenance 9. A 5-year-old child is brought to the emergency department with copious drooling and a croaking sound on inspiration. Her mother states that the child is very agitated and only wants to sit upright. What should be the nurses first action in this situation? a Prepare intubation equipment and call the physician. . b Examine the childs oropharynx and call the physician. . c Obtain a throat culture for respiratory syncytial virus (RSV). . d Obtain vital signs and listen to breath sounds. . ANS: A Feedback A This child has symptoms of epiglottitis, is acutely ill, and requires emergency measures. B If epiglottitis is suspected, the nurse should not examine the childs throat. Inspection of the epiglottis is only done by a physician, because it could trigger airway obstruction. C A throat culture could precipitate a complete respiratory obstruction. D Vital signs can be assessed after emergency equipment is readied. PTS: 1 DIF: Cognitive Level: Analysis REF: pp. 1163-1164 OBJ: Nursing Process: Implementation MSC: Client Needs: Physiologic Integrity 10. What intervention can be taught to the parents of a 3-year-old child with pneumonia who is not hospitalized? a . Offer the child only cool liquids. b . Offer the child her favorite warm liquid drinks. c . Use a warm mist humidifier. d . Call the physician for a respiratory rate less than 28 breaths/min. ANS: B Feedback A Warm liquids are preferable because they help loosen secretions. B Offering the child fluids that she likes will facilitate oral intake. Warm liquids help loosen secretions. C Cool mist humidifiers are preferred to warm mist. Warm mist is a safety concern and could cause burns if touched by the child. D Typically parents are not taught to count their childrens respirations and report abnormalities to the physician. Even if this were the case, a respiratory rate of less than 28 breaths/min is normal for a 3-year-old child. The expected respiratory rate for a 3-year-old child is 20 to 30 breaths/min. PTS: 1 DIF: Cognitive Level: Application REF: p. 1169 OBJ: Nursing Process: Implementation MSC: Client Needs: Health Promotion and Maintenance 11. What sign is indicative of respiratory distress in infants? a . Nasal flaring b . Respiratory rate of 55 breaths/min c . Irregular respiratory pattern d . Abdominal breathing ANS: A Feedback A Infants have difficulty breathing through their mouths; therefore nasal flaring is usually accompanied by extra respiratory efforts. It also allows more air to enter as the nares flare. B A respiratory rate of 55 breaths/min is a normal assessment for an infant. Tachypnea is a respiratory rate of 60 to 80 breaths/min. C Irregular respirations are normal in the infant. D Abdominal breathing is common because the diaphragm is the neonates major breathing muscle. PTS: 1 DIF: Cognitive Level: Application REF: p. 1166 OBJ: Nursing Process: Assessment MSC: Client Needs: Physiologic Integrity 12. Once an allergen is identified in a child with allergic rhinitis, the treatment of choice about which to educate the parents is a . Using appropriate medications b . Beginning desensitization injections c . Eliminating the allergen d . Removing the adenoids ANS: C Feedback A Medications are not a first-line treatment but can be helpful in controlling allergic rhinitis. B Immunotherapy is usually the final component of controlling allergic rhinitis. C The first priority is to attempt to remove the causative agent from the childs environment. D Adenoids are tissues that can swell with constant rhinitis; however, a surgical procedure is not indicated for allergic rhinitis. Dealing with the cause is the first priority. PTS: 1 DIF: Cognitive Level: Application REF: p. 1149 OBJ: Nursing Process: Planning MSC: Client Needs: Health Promotion and Maintenance 13. Which assessment finding after tonsillectomy should be reported to the physician? a . Vomiting bright red blood b . Pain at surgical site c . Pain on swallowing d . The ability to only take small sips of liquids ANS: A Feedback A Vomiting bright red blood and swallowing frequently are signs of bleeding postoperatively and should be reported to the physician. B It is normal for the child to have pain at the surgical site. C It is normal for the child to have pain on swallowing. D Only clear liquids are offered immediately after surgery, and small sips are preferred. PTS: 1 DIF: Cognitive Level: Comprehension REF: p. 1157 OBJ: Nursing Process: Assessment MSC: Client Needs: Health Promotion and Maintenance 14. Teaching safety precautions with the administration of antihistamines is important because of what common side effect? a Dry mouth . b Excitability . c Drowsiness . d Dry mucous membranes . ANS: C Feedback A A dry mouth is not a safety issue. B Excitability may affect rest or sleep, but drowsiness is the most important safety hazard. C Drowsiness is a safety hazard when alertness is needed, especially with a teenage driver. Nonsedating brands should be used. D Dry mucous membranes are not a safety issue. PTS: 1 DIF: Cognitive Level: Application REF: p. 1149 OBJ: Nursing Process: Implementation MSC: Client Needs: Health Promotion and Maintenance 15. What is an appropriate beverage for the nurse to give to a child who had a tonsillectomy earlier in the day? a Chocolate ice cream . b Orange juice . c Fruit punch . d Apple juice . ANS: D Feedback A The child can have full liquids on the second postoperative day. B Citrus drinks are not offered because they can irritate the throat. C Red liquids are avoided because they give the appearance of blood if vomited. D The child can have clear, cool liquids when fully awake. PTS: 1 DIF: Cognitive Level: Application REF: p. 1158 OBJ: Nursing Process: Implementation MSC: Client Needs: Health Promotion and Maintenance 16. Which type of croup is always considered a medical emergency? a Laryngitis . b Epiglottitis . c Spasmodic croup . d Laryngotracheobronchitis (LTB) . ANS: B Feedback A Laryngitis is a common viral illness in older children and adolescents, with hoarseness and URI symptoms. B Epiglottitis is always a medical emergency that requires antibiotics and airway support for treatment. C Spasmodic croup is treated with humidity. D LTB may progress to a medical emergency in some children. PTS: 1 DIF: Cognitive Level: Comprehension REF: p. 1162 OBJ: Nursing Process: Assessment MSC: Client Needs: Physiologic Integrity 17. What information should the nurse teach workers at a daycare center about RSV? a . RSV is transmitted through particles in the air. b . RSV can live on skin or paper for up to a few seconds after contact. c . RSV can survive on nonporous surfaces for about 60 minutes. d . Frequent handwashing can decrease the spread of the virus. ANS: D Feedback A RSV infection is not airborne. It is acquired mainly through contact with contaminated surfaces. B RSV can live on skin or paper for up to 1 hour. C RSV can live on cribs and other nonporous surfaces for up to 6 hours. D Meticulous handwashing can decrease the spread of organisms. PTS: 1 DIF: Cognitive Level: Application REF: p. 1165 OBJ: Nursing Process: Implementation MSC: Client Needs: Health Promotion and Maintenance 18. Which intervention is appropriate for the infant hospitalized with bronchiolitis? a Position on the side with neck slightly flexed. . b . Administer antibiotics as ordered. c . Restrict oral and parenteral fluids if tachypneic. d . Give cool, humidified oxygen. ANS: D Feedback A The infant should be positioned with the head and chest elevated at a 30- to 40- degree angle and the neck slightly extended to maintain an open airway and decrease pressure on the diaphragm. B The etiology of bronchiolitis is viral. Antibiotics are only given if there is a secondary bacterial infection. C Tachypnea increases insensible fluid loss. If the infant is tachypneic, fluids are given parenterally to prevent dehydration. D Cool, humidified oxygen is given to relieve dyspnea, hypoxemia, and insensible fluid loss from tachypnea. PTS: 1 DIF: Cognitive Level: Application REF: p. 1166 OBJ: Nursing Process: Implementation MSC: Client Needs: Physiologic Integrity 19. A child has a chronic, nonproductive cough and diffuse wheezing during the expiratory phase of respiration. This suggests a Asthma . b Pneumonia . c Bronchiolitis . d Foreign body in trachea . ANS: A Feedback A Children with asthma usually have these chronic symptoms. B Pneumonia appears with an acute onset and fever and general malaise. C Bronchiolitis is an acute condition caused by RSV. D Foreign body in the trachea will occur with an acute respiratory distress or failure and maybe stridor. PTS: 1 DIF: Cognitive Level: Comprehension REF: p. 1176 OBJ: Nursing Process: Diagnosis MSC: Client Needs: Physiologic Integrity 20. The nurse encourages the mother of a toddler with acute LTB to stay at the bedside as much as possible. The nurses rationale for this action is primarily that a Mothers of hospitalized toddlers often experience guilt. . b The mothers presence will reduce anxiety and ease childs respiratory efforts. . c Separation from mother is a major developmental threat at this age. . d The mother can provide constant observations of the childs respiratory efforts. . ANS: B Feedback A This is true, but not the best answer. B The familys presence will decrease the childs distress. C Although true for toddlers, the main reason to keep parents at the childs bedside is to ease anxiety and therefore respiratory effort. D The child should have constant monitoring by cardiorespiratory monitor and noninvasive oxygen saturation monitoring, but the parent should not play this role in the hospital. PTS: 1 DIF: Cognitive Level: Application REF: p. 1161 OBJ: Nursing Process: Diagnosis MSC: Client Needs: Psychosocial Adaptation 21. Which statement indicates that a parent of a toddler needs more education about preventing foreign body aspiration? a . I keep objects with small parts out of reach. b . My toddler loves to play with balloons. c . I wont permit my child to have peanuts. d . I never leave coins where my child could get them. ANS: B Feedback A Keeping toys with small parts and other small objects out of reach can prevent foreign body aspiration. B Latex balloons account for a significant number of deaths from aspiration every year. C Peanuts are just one of the foods that pose a choking risk if given to young children. D Small objects, such as coins, need to be put out of the small childs reach. PTS: 1 DIF: Cognitive Level: Application REF: p. 1169 OBJ: Nursing Process: Evaluation MSC: Client Needs: Health Promotion and Maintenance 22. What is a common trigger for asthma attacks in children? a Febrile episodes . b Dehydration . c Exercise . d Seizures . ANS: C Feedback A Febrile episodes are consistent with other problems, for example, seizures. B Dehydration occurs as a result of diarrhea; it does not trigger asthma attacks. Viral infections are triggers for asthma. C Exercise is one of the most common triggers for asthma attacks, particularly in school-age children. D Seizures can result from a too-rapid intravenous infusion of theophyllinea therapy for asthma. PTS: 1 DIF: Cognitive Level: Application REF: p. 1177 OBJ: Nursing Process: Assessment MSC: Client Needs: Health Promotion and Maintenance 23. Which child requires a Mantoux test? a . The child who has episodes of nighttime wheezing and coughing b . The child who has a history of allergic rhinitis c . The child whose baby-sitter has received a tuberculosis diagnosis d . The premature infant who is being treated for apnea of infancy ANS: C Feedback A Nighttime wheezing and coughing are consistent with a diagnosis of asthma. B Allergic rhinitis requires an allergy workup. C The Mantoux test is the initial screening mechanism for patients exposed to tuberculosis. D This infant requires a sleep study as part of the evaluation. PTS: 1 DIF: Cognitive Level: Analysis REF: p. 1191 | Box 45-5 OBJ: Nursing Process: Assessment MSC: Client Needs: Health Promotion and Maintenance 24. What explanation should the nurse give to the parent of a child with asthma about using a peak flow meter? a . It is used to monitor the childs breathing capacity. b . It measures the childs lung volume. c It will help the medication reach the childs airways. . d It measures the amount of air the child breathes in. . ANS: A Feedback A The peak flow meter is a device used to monitor breathing capacity in the child with asthma. B A child with asthma would have a pulmonary function test to measure lung volume. C A spacer used with a metered-dose inhaler prolongs medication transit so medication reaches the airways. D The peak flow meter measures the flow of air in a forced exhalation in liters per minute. PTS: 1 DIF: Cognitive Level: Application REF: p. 1177 OBJ: Nursing Process: Implementation MSC: Client Needs: Health Promotion and Maintenance 25. What is the best nursing response to the parent of a child with asthma who asks if his child can still participate in sports? a . Children with asthma are usually restricted from physical activities. b . Children can usually play any type of sport if their asthma is well controlled. c . Avoid swimming because breathing underwater is dangerous for people with asthma. d . Even with good asthma control, I would advise limiting the child to one athletic activity per school year. ANS: B Feedback A Children with asthma should not be restricted from physical activity. B Sports that do not require sustained exertion, such as gymnastics, baseball, and weight lifting, are well tolerated. Children can usually play any type of sport if their asthma is well controlled. C Swimming is recommended as the ideal sport for children with asthma because the air is humidified and exhaling underwater prolongs exhalation and increases end-expiratory pressure. D If asthma is well controlled, the child can participate in any type of sport. PTS: 1 DIF: Cognitive Level: Application REF: p. 1177 OBJ: Nursing Process: Implementation MSC: Client Needs: Health Promotion and Maintenance 26. A school-age child had an upper respiratory tract infection for several days and then began having a persistent dry, hacking cough that was worse at night. The cough has become productive in the past 24 hours. This is most suggestive of a . Bronchitis b . Bronchiolitis c . Viral-induced asthma d . Acute spasmodic laryngitis ANS: A Feedback A Bronchitis is characterized by these symptoms and occurs in children older than 6 years. B Bronchiolitis is rare in children older than 2 years. C Asthma is a chronic inflammation of the airways that may be exacerbated by a virus. D Acute spasmodic laryngitis occurs in children between 3 months and 3 years. PTS: 1 DIF: Cognitive Level: Comprehension REF: p. 1164 OBJ: Nursing Process: Diagnosis MSC: Client Needs: Physiologic Integrity 27. Which classification of drugs is used to relieve an acute asthma episode? a . Short-acting beta2-adrenergic agonist b . Inhaled corticosteroids c . Leukotriene blockers d . Long-acting bronchodilators ANS: A Feedback A Short-acting beta2-adrenergic agonist is the first medication administered. Later, systemic corticosteroids decrease airway inflammation in an acute asthma attack. They are given for short courses of 5 to 7 days. B Inhaled corticosteroids are used for long-term, routine control of asthma. C Leukotriene blockers diminish the mediator action of leukotrienes and are used for long-term, routine control of asthma in children older than 12 years. D A long-acting bronchodilator would not relieve acute symptoms. PTS: 1 DIF: Cognitive Level: Application REF: p. 1176 OBJ: Nursing Process: Implementation MSC: Client Needs: Physiologic Integrity 28. The nurse getting an end-of-shift report on a child with status asthmaticus should question which intervention? a . Administer oxygen by nasal cannula to keep oxygen saturation at 100%. b . Assess intravenous (IV) maintenance fluids and site every hour. c . Notify physician for signs of increasing respiratory distress. d . Organize care to allow for uninterrupted rest periods. ANS: A Feedback A Supplemental oxygen should not be administered to maintain oxygen saturation at 100%. Keeping the saturation around 95% is adequate. Administration of too much oxygen to a child may lead to respiratory depression by decreasing the stimulus to breathe, leading to carbon dioxide retention. B When the child cannot take oral fluids because of respiratory distress, IV fluids are administered. The child with a continuous IV infusion must be assessed hourly to prevent complications. C A physician should be notified of any changes indicating increasing respiratory distress. D A child in respiratory distress is easily fatigued. Nursing care should be organized so the child can get needed rest without being disturbed. PTS: 1 DIF: Cognitive Level: Analysis REF: p. 1176 OBJ: Nursing Process: Evaluation MSC: Client Needs: Physiologic Integrity 29. What is the earliest recognizable clinical manifestation(s) of CF? a . Meconium ileus b . History of poor intestinal absorption c . Foul-smelling, frothy, greasy stools d . Recurrent pneumonia and lung infections ANS: A Feedback A The earliest clinical manifestation of CF is a meconium ileus, which is found in about 10% of children with CF. Clinical manifestations include abdominal distention, vomiting, failure to pass stools, and rapid development of dehydration. B History of malabsorption is a later sign that manifests as failure to thrive. C Foul-smelling stools are a later manifestation of CF. D Recurrent respiratory infections are a later sign of CF. PTS: 1 DIF: Cognitive Level: Comprehension REF: p. 1187 OBJ: Nursing Process: Assessment MSC: Client Needs: Physiologic Integrity 30. What should the nurse teach a child about using an albuterol metered-dose inhaler for exercise-induced asthma? a . Take two puffs every 6 hours around the clock. b . Use the inhaler only when the child is short of breath. c . Use the inhaler 30 minutes before exercise. d . Take one to two puffs every morning upon awakening. ANS: C Feedback A This schedule will not relieve exercise-induced asthma. B Waiting until symptoms are severe is too late to begin using a metered-dose inhaler. C The appropriate time to use an inhaled beta2-agonist or cromolyn is before an event that could trigger an attack. D This may be the childs usual schedule for medication. If exercise causes symptoms, additional medication is indicated. PTS: 1 DIF: Cognitive Level: Application REF: p. 1178 OBJ: Nursing Process: Implementation MSC: Client Needs: Health Promotion and Maintenance 31. The infant with bronchopulmonary dysplasia (BPD) who has RSV bronchiolitis is a candidate for which treatment? a Pancreatic enzymes . b . Cool humidified oxygen c . Erythromycin intravenously d . Intermittent positive pressure ventilation ANS: B Feedback A Pancreatic enzymes are used for patients with cystic fibrosis. B Humidified oxygen is delivered if the oxygen saturation level drops to less than 90%. C Antibiotics are ineffective against viral illnesses. Oxygen can be administered by hood, facemask, or nasal cannula. D Assisted ventilation is not necessary in the treatment of RSV infections. PTS: 1 DIF: Cognitive Level: Application REF: p. 1166 OBJ: Nursing Process: Planning MSC: Client Needs: Physiologic Integrity 32. Which statement, if made by parents of a child with cystic fibrosis, indicates that they understood the nurses teaching on pancreatic enzyme replacement? a Enzymes will improve my childs breathing. . b I should give the enzymes 1 hour after meals. . c Enzymes should be given with meals and snacks. . d The enzymes are stopped if my child begins wheezing. . ANS: C Feedback A Pancreatic enzymes do not affect the respiratory system. B Pancreatic enzymes are taken within 30 minutes of eating all meals and snacks. Giving the medication 1 hour after meals is inappropriate and ineffective for absorption of nutrients. C Children with cystic fibrosis need to take enzymes with food for adequate absorption of nutrients. D Wheezing is not a reason to stop taking enzyme replacements. PTS: 1 DIF: Cognitive Level: Application REF: p. 1190 OBJ: Nursing Process: Evaluation MSC: Client Needs: Health Promotion and Maintenance 33. Which vitamin supplements are necessary for children with cystic fibrosis? a Vitamin C and calcium . b Vitamin B6 and B12 . c Magnesium . d Vitamins A, D, E, and K . ANS: D Feedback A Vitamin C and calcium are not fat soluble. B B6 and B12 are not fat-soluble vitamins. C Magnesium is not a vitamin. D Fat-soluble vitamins are poorly absorbed because of deficient pancreatic enzymes in children with cystic fibrosis; therefore supplements are necessary. Chapter 41 Nursing Care of the Child With an Alteration in Perfusion/Cardiovascular Disorder MULTIPLE CHOICE 1. Which postoperative intervention should be questioned for a child after a cardiac catheterization? a . Continue intravenous (IV) fluids until the infant is tolerating oral fluids. b . Check the dressing for bleeding. c . Assess peripheral circulation on the affected extremity. d . Keep the affected leg flexed and elevated. ANS: D Feedback A IV fluid administration continues until the child is taking and retaining adequate amounts of oral fluids. B The insertion site dressing should be observed frequently for bleeding. The nurse should also look under the child to check for pooled blood. C Peripheral perfusion is monitored after catheterization. Distal pulses should be palpable, although they may be weaker than in the contralateral extremity. D The child should be positioned with the affected leg straight for 4 to 6 hours after the procedure. PTS: 1 DIF: Cognitive Level: Analysis REF: pp. 1212-1213 OBJ: Nursing Process: Implementation MSC: Client Needs: Physiologic Integrity 2. Which information should be included in the nurses discharge instructions for a child who underwent a cardiac catheterization earlier in the day? a . Pressure dressing is changed daily for the first week. b . The child may soak in the tub beginning tomorrow. c . Contact sports can be resumed in 2 days. d . The child can return to school on the third day after the procedure. ANS: D Feedback A The day after the cardiac catheterization, the pressure dressing is removed and replaced with a Band-Aid. The catheter insertion site is assessed daily for healing. Any bleeding or sign of infection, such as drainage, must be reported to the cardiologist. B Bathing is limited to a shower, a sponge bath, or a brief tub bath (no soaking) for the first 1 to 3 days after the procedure. C Strenuous exercise such as contact sports, swimming, or climbing trees is avoided for up to 1 week after the procedure. D The child can return to school on the third day after the procedure. It is important to emphasize follow-up with the cardiologist. PTS: 1 DIF: Cognitive Level: Application REF: p. 1213 OBJ: Nursing Process: Implementation MSC: Client Needs: Physiologic Integrity 3. The nurse is admitting a child to the hospital for a cardiac workup. What is the first step in a cardiac assessment? a . Percussion b . Palpation c . Auscultation d . History and inspection ANS: D Feedback A Percussion of the chest is usually deferred. B Palpation can be threatening to the child because it requires a significant amount of physical contact. For this reason it is not the initial step in a cardiac assessment. C Auscultation requires touching the child and is not the initial step in a cardiac assessment. D The assessment should begin with the least threatening interventionsthe history and inspection. Assessment progression includes inspection, auscultation, and palpation because each step includes more touching. PTS: 1 DIF: Cognitive Level: Application REF: p. 1210 | Table 46-2 OBJ: Nursing Process: Assessment MSC: Client Needs: Health Promotion and Maintenance 4. In which situation is there a risk that a newborn infant will have a congenital heart defect (CHD)? a . Trisomy 21 detected on amniocentesis b . Family history of myocardial infarction c . Father has type 1 diabetes mellitus d . Older sibling born with Turner syndrome ANS: A Feedback A The incidence of congenital heart disease is approximately 50% in children with trisomy 21 (Down syndrome). B A family history of congenital heart disease, not acquired heart disease, increases the risk of giving birth to a child with CHD. C Infants born to mothers who are insulin dependent have an increased risk of CHD. D Infants identified as having certain genetic defects, such as Turner syndrome, have a higher incidence of CHD. A family history is not a risk factor. PTS: 1 DIF: Cognitive Level: Application REF: p. 1201 OBJ: Nursing Process: Assessment MSC: Client Needs: Health Promotion and Maintenance 5. Before giving a dose of digoxin (Lanoxin), the nurse checked an infants apical heart rate and it was 114 bpm. What should the nurse do next? a Administer the dose as ordered. . b Hold the medication until the next dose. . c Wait and recheck the apical heart rate in 30 minutes. . d Notify the physician about the infants heart rate. . ANS: A Feedback A The infants heart rate is above the lower limit for which the medication is held. The dose can be given. B A dose of Lanoxin is withheld for a heart rate less than 100 bpm in an infant. C The infants heart rate is acceptable for administering Lanoxin. It is unnecessary to recheck the heart rate at a later time. D The infants heart rate is acceptable. The physician should be notified for a heart rate less than 100 bpm in an infant. PTS: 1 DIF: Cognitive Level: Application REF: p. 1204 OBJ: Nursing Process: Implementation MSC: Client Needs: Physiologic Integrity 6. What intervention should be included in the plan of care for an infant with the nursing diagnosis of Excess Fluid Volume related to congestive heart failure? a Weigh the infant every day on the same scale at the same time. . b Notify the physician when weight gain exceeds more than 20 g/day. . c Put the infant in a car seat to minimize movement. . d Administer digoxin (Lanoxin) as ordered by the physician. . ANS: A Feedback A Excess fluid volume may not be overtly visible. Weight changes may indicate fluid retention. Weighing the infant on the same scale at the same time each day ensures consistency. B An excessive weight gain for an infant is an increase of more than 50 g/day. C With fluid volume excess, skin will be edematous. The infants position should be changed frequently to prevent undesirable pooling of fluid in certain areas. D Lanoxin is used in the treatment of congestive heart failure to improve cardiac function. Diuretics will help the body get rid of excess fluid. PTS: 1 DIF: Cognitive Level: Application REF: p. 1204 OBJ: Nursing Process: Implementation MSC: Client Needs: Physiologic Integrity 7. The nurse assessing a premature newborn infant auscultates a continuous machinery-like murmur. This finding is associated with which congenital heart defect? a Pulmonary stenosis . b Patent ductus arteriosus . c Ventricular septal defect . d Coarctation of the aorta . ANS: B Feedback A A systolic ejection murmur that may be accompanied by a palpable thrill is a manifestation of pulmonary stenosis. B The classic murmur associated with patent ductus arteriosus is a machinery-like one that can be heard throughout both systole and diastole. C The characteristic murmur associated with ventricular septal defect is a loud, harsh, holosystolic murmur. D A systolic murmur that is accompanied by an ejection click may be heard on auscultation when coarctation of the aorta is present. PTS: 1 DIF: Cognitive Level: Application REF: p. 1214 | Table 46-3 OBJ: Nursing Process: Assessment MSC: Client Needs: Physiologic Integrity 8. What is an expected assessment finding in a child with coarctation of the aorta? a . Orthostatic hypotension b . Systolic hypertension in the lower extremities c . Blood pressure higher on the left side of the body d . Disparity in blood pressure between the upper and lower extremities ANS: D Feedback A Orthostatic hypotension is not present with coarctation of the aorta. B Systolic hypertension may be detected in the upper extremities. C The left arm may not accurately reflect systolic hypertension because the left subclavian artery can be involved in the coarctation. D The classic finding in children with coarctation of the aorta is a disparity in pulses and blood pressures between the upper and lower extremities. PTS: 1 DIF: Cognitive Level: Application REF: p. 1217 | Table 46-3 OBJ: Nursing Process: Assessment MSC: Client Needs: Physiologic Integrity 9. A child with pulmonary atresia exhibits cyanosis with feeding. On reviewing this childs laboratory values, the nurse is not surprised to notice which abnormality? a Polycythemia . b Infection . c Dehydration . d Anemia . ANS: A Feedback A Polycythemia is a compensatory response to chronic hypoxia. The body attempts to improve tissue oxygenation by producing additional red blood cells and thereby increases the oxygen-carrying capacity of the blood. B Infection is not a clinical consequence of cyanosis. C Although dehydration can occur in cyanotic heart disease, it is not a compensatory mechanism for chronic hypoxia. It is not a clinical consequence of cyanosis. D Anemia may develop as a result of increased blood viscosity. Anemia is not a clinical consequence of cyanosis. PTS: 1 DIF: Cognitive Level: Analysis REF: p. 1208 OBJ: Nursing Process: Assessment MSC: Client Needs: Physiologic Integrity 10. Which statement made by a parent indicates understanding of restrictions for a child after cardiac surgery? a . My child needs to get extra rest for a few weeks. b . My son is really looking forward to riding his bike next week. c . Im so glad we can attend religious services as a family this coming Sunday. d . I am going to keep my child out of daycare for 6 weeks. ANS: D Feedback A The child should resume his regular bedtime and sleep schedule after discharge. B Activities during which the child could fall, such as riding a bicycle, are avoided for 4 to 6 weeks after discharge. C Large crowds of people should be avoided for 4 to 6 weeks after discharge, including public worship. D Settings where large groups of people are present should be avoided for 4 to 6 weeks after discharge, including day care. PTS: 1 DIF: Cognitive Level: Application REF: p. 1224 OBJ: Nursing Process: Evaluation MSC: Client Needs: Physiologic Integrity 11. A child had an aortic stenosis defect surgically repaired 6 months ago. Which antibiotic prophylaxis is indicated for an upcoming tonsillectomy? a . No antibiotic prophylaxis is necessary. b . Amoxicillin is taken orally 1 hour before the procedure. c . Oral penicillin is given for 7 to 10 days before the procedure. d . Parenteral antibiotics are administered for 5 to 7 days after the procedure. ANS: B Feedback A Antibiotic prophylaxis is indicated for the first 5 months after surgical repair. B The standard prophylactic agent is amoxicillin given orally 1 hour before the procedure. C Antibiotic prophylaxis is not given for this period of time. D The treatment for infective endocarditis involves parenteral antibiotics for 2 to 8 weeks. PTS: 1 DIF: Cognitive Level: Application REF: p. 1225 OBJ: Nursing Process: Planning MSC: Client Needs: Physiologic Integrity 12. The nurse discovers a heart murmur in an infant 1 hour after birth. She is aware that fetal shunts are closed in the neonate at what point? a . When the umbilical cord is cut b . Within several days of birth c . Within a month after birth d By the end of the first year of life . ANS: B Feedback A With the neonates first breath, gas exchange is transferred from the placenta to the lungs. The separation of the fetus from the umbilical cord does not contribute to the establishment of neonatal circulation. B In the normal neonate, fetal shunts functionally close in response to pressure changes in the systemic and pulmonary circulations and to increased oxygen content. This process may take several days to complete. C The fetal shunts normally close within several days of birth. D Fetal shunts normally close soon after birth but may take several days. PTS: 1 DIF: Cognitive Level: Application REF: p. 1199 OBJ: Nursing Process: Assessment MSC: Client Needs: Health Promotion and Maintenance 13. When assessing a child for possible congenital heart defects (CHDs), where should the nurse measure blood pressure? a . The right arm b . The left arm c . All four extremities d . Both arms while the child is crying ANS: C Feedback A Blood pressure measurements for upper and lower extremities are compared during an assessment for CHDs. B Discrepancies in blood pressure between the upper and lower extremities cannot be determined if blood pressure is not measured in all four extremities. C When a CHD is suspected, the blood pressure should be measured in all four extremities while the child is quiet. Discrepancies between upper and lower extremities may indicate cardiac disease. D Blood pressure measurements when the child is crying are likely to be elevated; thus the readings will be inaccurate. Also, all four extremities need to be measured. PTS: 1 DIF: Cognitive Level: Application REF: p. 1210 | Table 46-2 OBJ: Nursing Process: Assessment MSC: Client Needs: Health Promotion and Maintenance 14. What is the nurses first action when planning to teach the parents of an infant with a CHD? a . Assess the parents anxiety level and readiness to learn. b . Gather literature for the parents. c . Secure a quiet place for teaching. d . Discuss the plan with the nursing team. ANS: A Feedback A Any effort to organize the right environment, plan, or literature is of no use if the parents are not ready to learn or have high anxiety. Decreasing level of anxiety is often needed before new information can be processed. B A baseline assessment of prior knowledge should be taken into consideration before developing any teaching plan. C Locating a quiet place for meeting with parents is appropriate; however, an assessment should be done before any teaching is done. D Discussing a teaching plan with the nursing team is appropriate after an assessment of the parents knowledge and readiness. PTS: 1 DIF: Cognitive Level: Application REF: p. 1205 OBJ: Nursing Process: Planning MSC: Client Needs: Health Promotion and Maintenance 15. Before preparing a teaching plan for the parents of an infant with ductus arteriosus, it is important that the nurse understands this condition. Which statement best describes patent ductus arteriosus? a . Patent ductus arteriosus involves a defect that results in a right-to-left shunting of blood in the heart. b . Patent ductus arteriosus involves a defect in which the fetal shunt between the aorta and the pulmonary artery fails to close. c . Patent ductus arteriosus is a stenotic lesion that must be surgically corrected at birth. d . Patent ductus arteriosus causes an abnormal opening between the four chambers of the heart. ANS: B Feedback A Patent ductus arteriosus allows blood to flow from the high-pressure aorta to the low-pressure pulmonary artery, resulting in a left-to-right shunt. B Patent ductus arteriosus is failure of the fetal shunt between the aorta and the pulmonary artery to close. C Patent ductus arteriosus is not a stenotic lesion. Patent ductus arteriosus can be closed both medically and surgically. D Atrioventricular defect occurs when fetal development of the endocardial cushions is disturbed, resulting in abnormalities in the atrial and ventricular septa and the atrioventricular valves. PTS: 1 DIF: Cognitive Level: Application REF: p. 1214 | Table 46-3 OBJ: Nursing Process: Assessment MSC: Client Needs: Physiologic Integrity 16. For what reason might a newborn infant with a cardiac defect, such as coarctation of the aorta, that results in a right-to-left shunt receive prostaglandin E1? a . To decrease inflammation b . To control pain c . To decrease respirations d . To improve oxygenation ANS: D Feedback A Prostaglandin E1 is used to maintain a patent ductus arteriosus, thus increasing pulmonary blood flow. B Prostaglandin E1 is administered to infants with a right-to-left shunt to keep the ductus arteriosus patent, thus increasing pulmonary blood flow. C Prostaglandin E1 is given to infants with a right-to-left shunt to keep the ductus arteriosus patent to increase pulmonary blood flow. D Prostaglandin E1 is given to infants with a right-to-left shunt to keep the ductus arteriosus patent. This will improve oxygenation. PTS: 1 DIF: Cognitive Level: Analysis REF: p. 1217 | Table 46-3 OBJ: Nursing Process: Implementation MSC: Client Needs: Physiologic Integrity 17. Which CHD results in increased pulmonary blood flow? a . Ventricular septal defect b . Coarctation of the aorta c . Tetralogy of Fallot d . Pulmonary stenosis ANS: A Feedback A Ventricular septal defect causes a left-to-right shunting of blood, thus increasing pulmonary blood flow. B Coarctation of the aorta is a stenotic lesion that causes increased resistance to blood flow from the proximal to distal aorta. C The defects associated with tetralogy of Fallot result in a right-to-left shunting of blood, thus decreasing pulmonary blood flow. D Pulmonary stenosis causes obstruction of blood flow from the right ventricle to the pulmonary artery. Pulmonary blood flow is decreased. PTS: 1 DIF: Cognitive Level: Analysis REF: p. 1215 | Table 46-3 OBJ: Nursing Process: Assessment MSC: Client Needs: Physiologic Integrity 18. Which statement suggests that a parent understands how to correctly administer digoxin? a . I measure the amount I am supposed to give with a teaspoon. b . I put the medicine in the babys bottle. c . When she spits up right after I give the medicine, I give her another dose. d . I give the medicine at 8 in the morning and evening every day. ANS: D Feedback A To ensure the correct dosage, the medication should be measured with a syringe. B The medication should not be mixed with formula or food. It is difficult to judge whether the child received the proper dose if the medication is placed in food or formula. C To prevent toxicity, the parent should not repeat the dose without contacting the childs physician. D For maximum effectiveness, the medication should be given at the same time every day. PTS: 1 DIF: Cognitive Level: Application REF: p. 1207 OBJ: Nursing Process: Evaluation MSC: Client Needs: Health Promotion and Maintenance 19. What is the appropriate priority nursing action for the infant with a CHD who has an increased respiratory rate, is sweating, and is not feeding well? a . Recheck the infants blood pressure. b . Alert the physician. c . Withhold oral feeding. d . Increase the oxygen rate. ANS: B Feedback A Although this may be indicated, it is not the priority action. B These are signs of early congestive heart failure, and the physician should be notified. C Withholding the infants feeding is an incomplete response to the problem. D Increasing oxygen may alleviate symptoms, but medications such as digoxin and furosemide are necessary to improve heart function and fluid retention. Notifying the physician is the priority nursing action. PTS: 1 DIF: Cognitive Level: Analysis REF: p. 1205 OBJ: Nursing Process: Implementation MSC: Client Needs: Physiologic Integrity 20. Nursing care for the child in congestive heart failure includes a . Counting the number of saturated diapers b . Putting the infant in the Trendelenburg position c . Removing oxygen while the infant is crying d Organizing care to provide rest periods . ANS: D Feedback A Diapers must be weighed for an accurate record of output. B The head of the bed should be raised to decrease the work of breathing. C Oxygen should be administered during stressful periods such as when the child is crying. D Nursing care should be planned to allow for periods of undisturbed rest. PTS: 1 DIF: Cognitive Level: Application REF: p. 1205 OBJ: Nursing Process: Implementation MSC: Client Needs: Physiologic Integrity 21. Which strategy is appropriate when feeding the infant with congestive heart failure? a . Continue the feeding until a sufficient amount of formula is taken. b . Limit feeding time to no more than 30 minutes. c . Always bottle feed every 4 hours. d . Feed larger volumes of concentrated formula less frequently. ANS: B Feedback A The infant with congestive heart failure may tire easily. If the infant does not consume an adequate amount of formula in 30 minutes, gavage feedings should be considered. B The infant with congestive heart failure may tire easily, so the feeding should not continue beyond 30 minutes. If inadequate amounts of formula are taken, gavage feedings should be considered. C Infants with congestive heart failure may be breastfed. Feedings every 3 hours is a frequently used interval. If the infant were fed less frequently than every 3 hours, more formula would need to be consumed and would tire the infant. D The infant is fed smaller volumes of concentrated formula every 3 hours. PTS: 1 DIF: Cognitive Level: Application REF: p. 1207 OBJ: Nursing Process: Implementation MSC: Client Needs: Physiologic Integrity 22. A nurse is teaching an adolescent about primary hypertension. Which statement made by the adolescent indicates an understanding of primary hypertension? a . Primary hypertension should be treated with diuretics as soon as it is detected. b . Congenital heart defects are the most common cause of primary hypertension. c . Primary hypertension may be treated with weight reduction. d . Primary hypertension is not affected by exercise. ANS: C Feedback A Primary hypertension is usually treated with weight reduction and exercise. If ineffective, pharmacologic intervention may be needed. B Primary hypertension is considered to be an inherited disorder. C Primary hypertension in children may be treated with weight reduction and exercise programs. D An exercise program in conjunction with weight reduction can be effective in managing primary hypertension in children. PTS: 1 DIF: Cognitive Level: Comprehension REF: p. 1234 OBJ: Nursing Process: Evaluation MSC: Client Needs: Health Promotion and Maintenance 23. An adolescent being seen by the nurse practitioner for a sports physical is identified as having hypertension. On further testing, it is discovered the child has a cardiac abnormality. The initial treatment of secondary hypertension initially involves a . Weight control and diet b . Treating the underlying disease c . Administration of digoxin d . Administration of beta-adrenergic receptor blockers ANS: B Feedback A Weight control and diet is a non-pharmacologic treatment for primary hypertension. B Identification of the underlying disease should be the first step in treating secondary hypertension. C Digoxin is indicated in the treatment of congestive heart failure. D Beta-adrenergic receptor blockers are indicated in the treatment of primary hypertension. PTS: 1 DIF: Cognitive Level: Application REF: p. 1235 OBJ: Nursing Process: Implementation MSC: Client Needs: Physiologic Integrity 24. What should the nurse include in discharge teaching as the highest priority for the child with a cardiac dysrhythmia? a . CPR instructions b . Repeating digoxin if the child vomits c . Resting if dizziness occurs d . Checking the childs pulse after digoxin administration ANS: A Feedback A This could potentially be life-saving for the child. The parents and significant others in the childs life should have CPR training. B The digoxin dose is not repeated if the child vomits. C Dizziness is a symptom the child should be taught to report to adults so that the physician can be notified. It is not the priority intervention. D The childs pulse should be counted before the medication is given. The dose is withheld if the pulse is below the parameters set by the physician. PTS: 1 DIF: Cognitive Level: Analysis REF: p. 1228 OBJ: Nursing Process: Planning MSC: Client Needs: Health Promotion and Maintenance 25. A nurse is assigned to care for an infant with an unrepaired tetralogy of Fallot. What should the nurse do first when the baby is crying and becomes severely cyanotic? a . Place the infant in a knee-chest position. b . Administer oxygen. c . Administer morphine sulfate. d . Calm the infant. ANS: D Feedback A Placing the infant in a knee-chest position will decrease venous return so that smaller amounts of highly saturated blood reach the heart. This should be done after calming the infant. B Administering oxygen is indicated after placing the infant in a knee-chest position. C Administering morphine sulfate calms the infant. It may be indicated some time after the infant has been calmed. D Calming the crying infant is the first response. An infant with unrepaired tetralogy of Fallot who is crying and agitated may eventually lose consciousness. PTS: 1 DIF: Cognitive Level: Analysis REF: p. 1209 OBJ: Nursing Process: Implementation MSC: Client Needs: Physiologic Integrity 26. The nurse caring for a child diagnosed with acute rheumatic fever should assess the child for a Sore throat . b . Elevated blood pressure c . Desquamation of the fingers and toes d . Tender, warm, inflamed joints ANS: D Feedback A The child may have had a sore throat previously associated with a group A beta-hemolytic streptococcal infection a few weeks earlier. A sore throat is not a manifestation of rheumatic fever. B Hypertension is not associated with rheumatic fever. C Desquamation of the fingers and toes is a manifestation of Kawasaki syndrome. D Arthritis, characterized by tender, warm, erythematous joints, is one of the major manifestations of acute rheumatic fever in the first 1 to 2 weeks of the illness. Chapter 42 Nursing Care of the Child With Alteration in Bowel Elimination/Gastrointestinal Disorder MULTIPLE CHOICE 1. What is the best response by the nurse to a mother asking about the cause of her infants bilateral cleft lip? a . Did you use alcohol during your pregnancy? b . Do you know of anyone in your family or the babys fathers family who was born with cleft lip or palate problems? c . This defect is associated with intrauterine infection during the second trimester. d The prevalent of cleft lip is higher in Caucasians . ANS: B Feedback A Tobacco during pregnancy has been associated with bilateral cleft lip. B Cleft lip and palate result from embryonic failure resulting from multiple genetic and environmental factors. A genetic pattern or familial risk seems to exist. C The defect occurred at approximately 6 to 8 weeks of gestation. Secondtrimester intrauterine infection is not a known cause of bilateral cleft lip. D The prevalence of cleft lip and palate is higher in Asian and Native American populations. PTS: 1 DIF: Cognitive Level: Application REF: pp. 1069-1070 OBJ: Nursing Process: Implementation MSC: Client Needs: Psychosocial Integrity 2. The postoperative care plan for an infant with surgical repair of a cleft lip includes a . A clear liquid diet for 72 hours b . Nasogastric feedings until the sutures are removed c . Elbow restraints to keep the infants fingers away from the mouth d . Rinsing the mouth after every feeding ANS: C Feedback A The infants diet is advanced from clear liquid to soft foods within 48 hours of surgery. B After surgery, the infant can resume preoperative feeding techniques. C Keeping the infants hands away from the incision reduces potential complications at the surgical site. D Rinsing the mouth after feeding is an inappropriate intervention. Feeding a small amount of water after feedings will help keep the mouth clean. A cleft lip repair site should be cleansed with a wet sterile cotton swab after feedings. PTS: 1 DIF: Cognitive Level: Comprehension REF: p. 1073 OBJ: Nursing Process: Planning MSC: Client Needs: Physiologic Integrity 3. The nurse is caring for a neonate with a suspected tracheoesophageal fistula (TEF). Nursing care should include a . Elevating the head but give nothing by mouth b . Elevating the head for feedings c . Feeding glucose water only d . Avoiding suction unless infant is cyanotic ANS: A Feedback A When a newborn is suspected of having TEF, the most desirable position is supine with the head elevated on an incline plane of at least 30 degrees. It is imperative that any source of aspiration be removed at once; oral feedings are withheld. B Feedings should not be given to infants suspected of having TEF. C Feedings should not be given to infants suspected of having TEF. D The oral pharynx should be kept clear of secretion by oral suctioning. This is to avoid cyanosis that is usually the result of laryngospasm caused by overflow of saliva into the larynx. PTS: 1 DIF: Cognitive Level: Analysis REF: p. 1074 OBJ: Nursing Process: Implementation MSC: Client Needs: Physiologic Integrity 4. A nurse is teaching a group of parents about TEF. Which statement made by the nurse is accurate about TEF? a . This defect results from an embryonal failure of the foregut to differentiate into the trachea and esophagus. b . It is a fistula between the esophagus and stomach that results in the oral intake being refluxed and aspirated. c . An extra connection between the esophagus and trachea develops because of genetic abnormalities. d . The defect occurs in the second trimester of pregnancy. ANS: A Feedback A When the foregut does not differentiate into the trachea and esophagus during the fourth to fifth week of gestation, a TEF occurs. B TEF is an abnormal connection between the esophagus and trachea. C There is no connection between the trachea and esophagus in normal fetal development. D This defect occurs early in pregnancy during the fourth to fifth week of gestation. PTS: 1 DIF: Cognitive Level: Application REF: p. 1071 OBJ: Nursing Process: Implementation MSC: Client Needs: Physiologic Integrity 5. What maternal assessment is related to an infants diagnosis of TEF? a . Maternal age more than 40 years b . First term pregnancy for the mother c . Maternal history of polyhydramnios d . Complicated pregnancy ANS: C Feedback A Advanced maternal age is not a risk factor for TEF. B The first term pregnancy is not a risk factor for an infant with TEF. C A maternal history of polyhydramnios is associated with TEF. D Complicated pregnancy is not a risk factor for TEF. PTS: 1 DIF: Cognitive Level: Comprehension REF: p. 1071 OBJ: Nursing Process: Assessment MSC: Client Needs: Physiologic Integrity 6. What clinical manifestation should a nurse be alert for when suspecting a diagnosis of esophageal atresia? a A radiograph in the prenatal period indicates abnormal development. . b . It is visually identified at the time of delivery. c . A nasogastric tube fails to pass at birth. d . The infant has a low birth weight. ANS: C Feedback A Prenatal radiographs do not provide a definitive diagnosis. B The defect is not externally visible. Bronchoscopy and endoscopy can be used to identify this defect. C Atresia is suspected when a nasogastric tube fails to pass 10 to 11 cm beyond the gum line. Abdominal radiographs will confirm the diagnosis. D Infants with esophageal atresia may have been born prematurely and with a low birth weight, but neither is suggestive of the presence of an esophageal atresia. PTS: 1 DIF: Cognitive Level: Comprehension REF: p. 1071 OBJ: Nursing Process: Assessment MSC: Client Needs: Physiologic Integrity 7. What is the most important information to be included in the discharge planning for an infant with gastroesophageal reflux? a Teach parents to position the infant on the left side. . b Reinforce the parents knowledge of the infants developmental needs. . c Teach the parents how to do infant cardiopulmonary resuscitation (CPR). . d Have the parents keep an accurate record of intake and output. . ANS: C Feedback A Correct positioning minimizes aspiration. The correct position for the infant is on the right side after feeding and supine for sleeping. B Knowledge of developmental needs should be included in discharge planning for all hospitalized infants, but it is not the most important in this case. C Risk of aspiration is a priority nursing diagnosis for the infant with gastroesophageal reflux. The parents must be taught infant CPR. D Keeping a record of intake and output is not a priority and may not be necessary. PTS: 1 DIF: Cognitive Level: Application REF: p. 1081 OBJ: Nursing Process: Planning MSC: Client Needs: Physiologic Integrity 8. What information should the nurse include when teaching the parents of a 5-week-old infant about pyloromyotomy? a The infant will be in the hospital for a week. . b The surgical procedure is routine and no big deal. . c The prognosis for complete correction with surgery is good. . d They will need to ask the physician about home care nursing. . ANS: C Feedback A The infant will remain in the hospital for a day or two postoperatively. B Although the prognosis for surgical correction is good, telling the parents that surgery is no big deal minimizes the infants condition. C Pyloromyotomy is the definitive treatment for pyloric stenosis. Prognosis is good with few complications. These comments reassure parents. D Home care nursing is not necessary after a pyloromyotomy. PTS: 1 DIF: Cognitive Level: Comprehension REF: p. 1095 OBJ: Nursing Process: Planning MSC: Client Needs: Physiologic Integrity 9. A nurse has admitted a child to the hospital with a diagnosis of rule out peptic ulcer disease. Which test will the nurse expect to be ordered to confirm the diagnosis of a peptic ulcer? a . A dietary history b . A positive Hematest result on a stool sample c . A fiberoptic upper endoscopy d . An abdominal ultrasound ANS: C Feedback A Dietary history may yield information suggestive of a peptic ulcer, but the diagnosis is confirmed through endoscopy. B Blood in the stool indicates a gastrointestinal abnormality, but it does not conclusively confirm a diagnosis of peptic ulcer. C Endoscopy provides direct visualization of the stomach lining and confirms the diagnosis of peptic ulcer. D An abdominal ultrasound is used to rule out other gastrointestinal alterations such as gallstones, tumor, or mechanical obstruction. PTS: 1 DIF: Cognitive Level: Comprehension REF: p. 1085 OBJ: Nursing Process: Assessment MSC: Client Needs: Physiologic Integrity 10. What should the nurse teach a school-age child and his parents about the management of ulcer disease? a . Eat a bland, low-fiber diet in small, frequent meals. b . Eat three balanced meals a day with no snacking between meals. c . The child needs to eat alone to avoid stress. d . Do not give antacids 1 hour before or after antiulcer medications. ANS: D Feedback A A bland diet is not indicated for ulcer disease. The diet should be a regular diet that is low in caffeine, and the child should eat a meal or snack every 2 to 3 hours. B The child should eat every 2 to 3 hours. C Eating alone is not indicated. D Antacids can interfere with antiulcer medication if given less than 1 hour before or after antiulcer medications. PTS: 1 DIF: Cognitive Level: Application REF: p. 1086 OBJ: Nursing Process: Planning MSC: Client Needs: Physiologic Integrity 11. What is the major focus of the therapeutic management for a child with lactose intolerance? a . Compliance with the medication regimen. b . Providing emotional support to family members. c . Teaching dietary modifications. d . Administration of daily normal saline enemas. ANS: C Feedback A Medications are not typically ordered in the management of lactose intolerance. B Providing emotional support to family members is not specific to this medical condition. C Simple dietary modifications are effective in management of lactose intolerance. Symptoms of lactose intolerance are usually relieved after instituting a lactose-free diet. D Diarrhea is a manifestation of lactose intolerance. Enemas are contraindicated for this alteration in bowel elimination. PTS: 1 DIF: Cognitive Level: Comprehension REF: p. 1103 OBJ: Nursing Process: Planning MSC: Client Needs: Physiologic Integrity 12. The child with lactose intolerance is most at risk for which electrolyte imbalance? a Hyperkalemia . b . Hypoglycemia c . Hyperglycemia d . Hypocalcemia ANS: D Feedback A The child with lactose intolerance is not at risk for hyperkalemia. B Lactose intolerance does not affect glucose metabolism. C Hyperglycemia does not result from ingestion of a lactose-free diet. D The child between 1 and 10 years requires a minimum of 800 mg of calcium daily. Because high-calcium dairy products containing lactose are restricted from the childs diet, alternative sources such as egg yolk, green leafy vegetables, dried beans, and cauliflower must be provided to prevent hypocalcemia. PTS: 1 DIF: Cognitive Level: Comprehension REF: p. 1103 OBJ: Nursing Process: Assessment MSC: Client Needs: Physiologic Integrity 13. What food choice by the parent of a 2-year-old child with celiac disease indicates a need for further teaching? a Oatmeal . b Rice cake . c Corn muffin . d Meat patty . ANS: A Feedback A The child with celiac disease is unable to fully digest gluten, the protein found in wheat, barley, rye, and oats. Oatmeal contains gluten and is not an appropriate food selection. B Rice is an appropriate choice because it does not contain gluten. C Corn is digestible because it does not contain gluten. D Meats do not contain gluten and can be included in the diet of a child with celiac disease. PTS: 1 DIF: Cognitive Level: Application REF: p. 1105 OBJ: Nursing Process: Evaluation MSC: Client Needs: Physiologic Integrity 14. Which assessment finding should the nurse expect in an infant with Hirschsprung disease? a Currant jelly stools . b Constipation with passage of foul-smelling, ribbon-like stools . c Foul-smelling, fatty stools . d Diarrhea . ANS: B Feedback A Currant jelly stools are associated with intussusception. B Constipation results from absence of ganglion cells in the rectum and colon, and is present since the neonatal period with passage of frequent foul-smelling, ribbon-like, or pellet-like stools. C Foul-smelling, fatty stools are associated with cystic fibrosis and celiac disease. D Diarrhea is not typically associated with Hirschsprung disease but may result from impaction. PTS: 1 DIF: Cognitive Level: Comprehension REF: p. 1099 OBJ: Nursing Process: Assessment MSC: Client Needs: Physiologic Integrity 15. What is an expected outcome for the parents of a child with encopresis? a The parents will give the child an enema daily for 3 to 4 months. . b The family will develop a plan to achieve control over incontinence. . c The parents will have the child launder soiled clothes. . d The parents will supply the child with a low-fiber diet. . ANS: B Feedback A Stool softeners or laxatives, along with dietary changes, are typically used to treat encopresis. Enemas are indicated when a fecal impaction is present. B Parents of the child with encopresis often feel guilty and believe that encopresis is willful on the part of the child. The family functions effectively by openly discussing problems and developing a plan to achieve control over incontinence. C This action is a punishment and will increase the childs shame and embarrassment. The child should not be punished for an action that is not willful. D Increasing fiber in the diet and fluid intake results in greater bulk in the stool, making it easier to pass. PTS: 1 DIF: Cognitive Level: Application REF: p. 1082 OBJ: Nursing Process: Evaluation MSC: Client Needs: Physiologic Integrity 16. Which intervention should be included in the nurses plan of care for a 7-year-old child with encopresis who has cleared the initial impaction? a . Have the child sit on the toilet for 30 minutes when he gets up in the morning and at bedtime. b . Increase sugar in the childs diet to promote bowel elimination. c . Use a Fleets enema daily. d . Give the child a choice of beverage to mix with a laxative. ANS: D Feedback A To facilitate bowel elimination, the child should sit on the toilet for 5 to 10 minutes after breakfast and dinner. B Decreasing the amount of sugar in the diet will help keep stools soft. C Daily Fleets enemas can result in hypernatremia and hyperphosphatemia, and are used only during periods of fecal impaction. D Offering realistic choices is helpful in meeting the school-age childs sense of control. PTS: 1 DIF: Cognitive Level: Application REF: p. 1083 OBJ: Nursing Process: Planning MSC: Client Needs: Physiologic Integrity 17. The nurse is teaching the parents of a child who has been diagnosed with irritable bowel syndrome about the pathophysiology associated with the symptoms their child is experiencing. Which response indicates to the nurse that her teaching has been effective? a . My child has an absence of ganglion cells in the rectum causing alternating diarrhea and constipation. b . The cause of my childs diarrhea and constipation is disorganized intestinal contractility. c . My child has an intestinal obstruction; thats why he has abdominal pain. d . My child has an intolerance to gluten, and this causes him to have abdominal pain. ANS: B Feedback A The absence of ganglion cells in the rectum is associated with Hirschsprung disease. B Disorganized contractility and increased mucus production are precipitating factors of irritable bowel disease. C Intestinal obstruction is associated with pyloric stenosis. D Intolerance to gluten is the underlying cause of celiac disease. PTS: 1 DIF: Cognitive Level: Application REF: p. 1084 OBJ: Nursing Process: Assessment MSC: Client Needs: Physiologic Integrity 18. What is an expected outcome for the child with irritable bowel disease? a . Decreasing symptoms b . Adherence to a low-fiber diet c . Increasing milk products in the diet d . Adapting the lifestyle to the lifelong problems ANS: A Feedback A Management of irritable bowel disease is aimed at identifying and decreasing exposure to triggers and decreasing bowel spasms, which will decrease symptoms. Management includes maintenance of a healthy, well-balanced, moderate-fiber, lower fat diet. B A moderate amount of fiber in the diet is indicated for the child with irritable bowel disease. C No modification in dairy products is necessary unless the child is lactose intolerant. D Irritable bowel syndrome is typically self-limiting and resolves by age 20 years. PTS: 1 DIF: Cognitive Level: Comprehension REF: p. 1084 OBJ: Nursing Process: Evaluation MSC: Client Needs: Physiologic Integrity 19. An infant is born and the nurse notices that the child has herniation of abdominal viscera into the base of the umbilical cord. What will the nurse document on her or his assessment of this condition? a . Diaphragmatic hernia b . Umbilical hernia c . Gastroschisis d . Omphalocele ANS: D Feedback A A diaphragmatic hernia is the protrusion of part of the abdominal organs through an opening in the diaphragm. B An umbilical hernia is a soft skin protrusion of abdominal stricture through the esophageal hiatus. C Gastroschisis is the protrusion of intraabdominal contents through a defect in the abdominal wall lateral to the umbilical ring. There is no peritoneal sac. D Omphalocele is the herniation of the abdominal viscera into the base of the umbilical cord. PTS: 1 DIF: Cognitive Level: Application REF: p. 1078 | Table 43-2 OBJ: Nursing Process: Assessment MSC: Client Needs: Physiologic Integrity 20. What is an appropriate statement for the nurse to make to parents of a child who has had a barium enema to correct an intussusception? a I will call the physician when the baby passes his first stool. . b . I am going to dilate the anal sphincter with a gloved finger to help the baby pass the barium. c . I would like you to save all the soiled diapers so I can inspect them. d . Add cereal to the babys formula to help him pass the barium. ANS: C Feedback A The physician does not need to be notified when the infant passes the first stool. B Dilating the anal sphincter is not appropriate for the child after a barium enema. C The nurse needs to inspect diapers after a barium enema because it is important to document the passage of barium and note the characteristics of the stool. D After reduction, the infant is given clear liquids and the diet is gradually increased. PTS: 1 DIF: Cognitive Level: Application REF: p. 1089 OBJ: Nursing Process: Planning MSC: Client Needs: Physiologic Integrity 21. What is the best response to parents who ask why their infant has a nasogastric tube to intermittent suction before abdominal surgery for hypertrophic pyloric stenosis? a The nasogastric tube decompresses the abdomen and decreases vomiting. . b We can keep a more accurate measure of intake and output with the nasogastric . tube. c The tube is used to decrease postoperative diarrhea. . d Believe it or not, the nasogastric tube makes the baby more comfortable after . surgery. ANS: A Feedback A The nasogastric tube provides decompression and decreases vomiting. B A nursing responsibility when a patient has a nasogastric tube is measurement of accurate intake and output, but this is not why nasogastric tubes are inserted. C Nasogastric tube placement does not decrease diarrhea. D The presence of a nasogastric tube can be perceived as a discomfort by the patient. PTS: 1 DIF: Cognitive Level: Application REF: p. 1096 OBJ: Nursing Process: Assessment MSC: Client Needs: Physiologic Integrity 22. Which description of a stool is characteristic of intussusception? a Ribbon-like stools . b Hard stools positive for guaiac . c Currant jelly stools . d Loose, foul-smelling stools . ANS: C Feedback A Ribbon-like stools are characteristic of Hirschsprung disease. B With intussusception, passage of bloody mucus stools occurs. Stools will not be hard. C Pressure on the bowel from obstruction leads to passage of currant jelly stools. D Loose, foul-smelling stools may indicate infectious gastroenteritis. PTS: 1 DIF: Cognitive Level: Comprehension REF: p. 1097 OBJ: Nursing Process: Assessment MSC: Client Needs: Physiologic Integrity 23. What is a priority concern for a 14-year-old child with inflammatory bowel disease? a . Compliance with antidiarrheal medication therapy b . Long-term complications c . Dealing with the embarrassment and stress of diarrhea d . Home schooling ANS: C Feedback A Antidiarrheal medications are not typically ordered for a child with inflammatory bowel disease. B Long-term complications are not a priority concern for the adolescent with inflammatory bowel disease. C Embarrassment and stress from chronic diarrhea are real concerns for the adolescent with inflammatory bowel disease. D Exacerbations may interfere with school attendance, but home schooling is not a usual consideration for the adolescent with inflammatory bowel disease. PTS: 1 DIF: Cognitive Level: Comprehension REF: pp. 1092-1093 OBJ: Nursing Process: Planning MSC: Client Needs: Physiologic Integrity 24. Which statement about Crohn disease is the most accurate? a . The signs and symptoms of Crohn disease are usually present at birth. b . Signs and symptoms of Crohn disease include abdominal pain, diarrhea, and often a palpable abdominal mass. c . Edema usually accompanies this disease. d . Symptoms of Crohn disease usually disappear by late adolescence. ANS: B Feedback A Signs and symptoms are not usually present at birth. B Crohn disease can occur anywhere in the GI tract from the mouth to the anus and is most common in the terminal ileum. Signs and symptoms include abdominal pain, diarrhea (nonbloody), fever, palpable abdominal mass, anorexia, severe weight loss, fistulas, obstructions, and perianal and anal lesions. C Diarrhea and malabsorption from Crohn disease cause weight loss, anorexia, dehydration, and growth failure. Edema does not accompany this disease. D Crohn disease is a long-term health problem. Symptoms do not typically disappear by adolescence. PTS: 1 DIF: Cognitive Level: Application REF: p. 1092 | Table 43-4 OBJ: Nursing Process: Assessment MSC: Client Needs: Physiologic Integrity 25. Therapeutic management of the child with acute diarrhea and dehydration usually begins with a . Clear liquids b . Adsorbents, such as kaolin and pectin c . Oral rehydration solution (ORS) d . Antidiarrheal medications such as paregoric ANS: C Feedback A Clear liquids are not recommended because they contain too much sugar, which may contribute to diarrhea. B Adsorbents are not recommended. C Orally administered rehydration solution is the first treatment for acute diarrhea. D Antidiarrheals are not recommended because they do not get rid of pathogens. PTS: 1 DIF: Cognitive Level: Comprehension REF: pp. 1088-1089 OBJ: Nursing Process: Assessment MSC: Client Needs: Physiologic Integrity 26. What is the most important action to prevent the spread of gastroenteritis in a daycare setting? a Administering prophylactic medications to children and staff . b Frequent handwashing . c Having parents bring food from home . d Directing the staff to wear gloves at all times . ANS: B Feedback A Prophylactic medications are not helpful in preventing gastroenteritis. B Handwashing is the most the important measure to prevent the spread of infectious diarrhea. C Bringing food from home will not prevent the spread of infectious diarrhea. D Gloves should be worn when changing diapers, soiled clothing, or linens. They do not need to be worn for interactions that do not involve contact with secretions. Handwashing after contact is indicated. PTS: 1 DIF: Cognitive Level: Comprehension REF: p. 1089 OBJ: Nursing Process: Implementation MSC: Client Needs: Physiologic Integrity 27. What is an expected outcome for a 1-month-old infant with biliary atresia? a Correction of the defect with the Kasai procedure . b Adequate nutrition and age-appropriate growth and development . c Adherence to a salt-free diet with vitamin B12 supplementation . d Adequate protein intake . ANS: B Feedback A The goal of the Kasai procedure is to allow for adequate growth until a transplant can be done. It is not a curative procedure. B Adequate nutrition, preventing skin breakdown, adequate growth and development, and family education and support are expected outcomes in an infant with biliary atresia. C Vitamin B12 supplementation is not indicated. A salt-restricted diet is appropriate. D Protein intake may need to be restricted to avoid hepatic encephalopathy. PTS: 1 DIF: Cognitive Level: Comprehension REF: pp. 1110-1111 OBJ: Nursing Process: Planning MSC: Client Needs: Physiologic Integrity 28. Which assessment finding is the most significant to report to the physician for a child with cirrhosis? a Weight loss . b Change in level of consciousness . c Skin with pruritus . d Black, foul-smelling stools . ANS: B Feedback A One complication of cirrhosis is ascites. The child needs to be assessed for increasing abdominal girth and edema. A child who is retaining fluid will not exhibit weight loss. B The child with cirrhosis must be assessed for encephalopathy, which is characterized by a change in level of consciousness. Encephalopathy can result from a buildup of ammonia in the blood from the incomplete breakdown of protein. C Biliary obstruction can lead to pruritus, which is a frequent finding. An alteration in the level of consciousness is of higher priority. D Black, tarry stools may indicate blood in the stool. This needs be reported to the physician. This is not a higher priority than a change in level of consciousness. PTS: 1 DIF: Cognitive Level: Analysis REF: p. 1112 OBJ: Nursing Process: Assessment MSC: Client Needs: Physiologic Integrity 29. Which nursing diagnosis has the highest priority for the toddler with celiac disease? a . Disturbed Body Image related to chronic constipation b . Risk for Disproportionate Growth related to obesity c . Excess Fluid Volume related to celiac crisis d . Imbalanced Nutrition: Less than Body Requirements related to malabsorption ANS: D Feedback A Body Image disturbances are not usually apparent in toddlers. This is more common in adolescents. It is not the priority nursing diagnosis. B Celiac disease causes disproportionate growth and development associated with malnutrition, not obesity. C Celiac crisis causes deficient fluid volume. D Imbalanced Nutrition: Less than Body Requirements is the highest priority nursing diagnosis because celiac disease causes gluten enteropathy, a malabsorption condition. PTS: 1 DIF: Cognitive Level: Application REF: p. 1105 OBJ: Nursing Process: Diagnosis MSC: Client Needs: Physiologic Integrity 30. The nurse notes on assessment that a 1-year-old child is underweight, with abdominal distention, thin legs and arms, and foul-smelling stools. The nurse suspects failure to thrive is associated with a . Celiac disease b . Intussusception c . Irritable bowel syndrome d . Imperforate anus ANS: A Feedback A These are classic symptoms of celiac disease. B Intussusception is not associated with failure to thrive or underweight, thin legs and arms, and foul-smelling stools. Stools are like currant jelly. C Irritable bowel syndrome is characterized by diarrhea and pain, and the child does not typically have thin legs and arms. D Imperforate anus is the incomplete development or absence of the anus in its normal position in the perineum. Symptoms are evident in early infancy. PTS: 1 DIF: Cognitive Level: Comprehension REF: p. 1103 OBJ: Nursing Process: Assessment MSC: Client Needs: Physiologic Integrity 31. The nurse caring for a child with suspected appendicitis should question which order from the physician? a . Keep patient NPO. b . Start IV of D5/0.45 normal saline at 60 mL/hr. c . Apply K-pad to abdomen prn for pain. d . Obtain CBC on admission to nursing unit. ANS: C Feedback A NPO status is appropriate for the potential appendectomy patient. B An IV is appropriate both as a preoperative intervention and to compensate for the short-term NPO status. C A K-pad (moist heat device) is contraindicated for suspected appendicitis because it may contribute to the rupture of the appendix. D Because appendicitis is frequently reflected in an elevated WBC, laboratory data are needed. PTS: 1 DIF: Cognitive Level: Application REF: p. 1090 OBJ: Nursing Process: Evaluation MSC: Client Needs: Physiologic Integrity 32. Which order should the nurse question when caring for a 5-year-old child after surgery for Hirschsprung disease? a . Monitor rectal temperature every 4 hours and report an elevation greater than 38.5 C. b . Assess stools after surgery. c . Keep the child NPO until bowel sounds return. d . Maintain IV fluids at ordered rate. ANS: A Feedback A Rectal temperatures should not be taken after this surgery. Rectal temperatures are generally not the route of choice for children because of the routes traumatic nature. B This is an appropriate intervention postoperatively. Stools should be soft and formed. C This is an appropriate intervention postoperatively. D This is an appropriate postoperative order. PTS: 1 DIF: Cognitive Level: Application REF: p. 1101 OBJ: Nursing Process: Evaluation MSC: Client Needs: Physiologic Integrity 33. Which parasite causes acute diarrhea? a . Shigella organisms b . Salmonella organisms c . Giardia lamblia d Escherichia coli . ANS: C Feedback A Shigella is a bacterial pathogen. B Salmonella is a bacterial pathogen. C Giardiasis a parasite that represents 15% of nondysenteric illness in the United States. D E. coli is a bacterial pathogen. PTS: 1 DIF: Cognitive Level: Comprehension REF: p. 1087 | Table 43-3 OBJ: Nursing Process: Assessment MSC: Client Needs: Physiologic Integrity 34. What goal has the highest priority for a child with malabsorption associated with lactose intolerance? a . The child will experience no abdominal spasms. b . The child will not experience constipation associated with malabsorption syndrome. c . The child will not experience diarrhea associated with malabsorption syndrome. d . The child will receive adequate nutrition as evidenced by a weight gain of 1 kg/ day. ANS: C Feedback A A child usually has abdominal cramping pain and distention rather than spasms. B The child usually has diarrhea, not constipation. C This goal is correct for a child with malabsorption associated with lactose intolerance. D One kilogram a day is too much weight gain with no time parameters. PTS: 1 DIF: Cognitive Level: Application REF: p. 1103 OBJ: Nursing Process: Planning MSC: Client Needs: Physiologic Integrity 35. What should the nurse stress in a teaching plan for the mother of an 11-year-old boy with ulcerative colitis? a . Preventing the spread of illness to others b . Nutritional guidance and preventing constipation c . Teaching daily use of enemas d . Coping with stress and avoiding triggers ANS: D Feedback A Ulcerative colitis is not infectious. B Although nutritional guidance is a priority teaching focus, diarrhea is a problem with ulcerative colitis, not constipation. C This is not part of the therapeutic plan of care. D Coping with the stress of chronic illness and the clinical manifestations associated with ulcerative colitis (diarrhea, pain) are important teaching foci. Avoidance of triggers can help minimize the impact of the disease and its effect on the child. PTS: 1 DIF: Cognitive Level: Application REF: p. 1094 OBJ: Nursing Process: Implementation MSC: Client Needs: Physiologic Integrity 36. An infant with imperforate anus has an anal plasty and temporary colostomy. Which statement by the infants mother indicates that she understands how to care for the infants colostomy at home? a . I will call the doctor right away if my baby starts vomiting. b . Ill call my home health nurse if the colostomy bag needs to be changed. c . Ill call the doctor if I notice that the colostomy stoma is pink. d . Ill have my mother help me with the care of the colostomy. ANS: A Feedback A Parents are taught signs of strangulation; vomiting, pain, and an irreducible mass in the abdomen. The physician should be contacted immediately if strangulation is suspected. B The mother should be taught the basics of colostomy care, including how to change the appliance. C The colostomy stoma should be pink in color, not pale or discolored. D There is no evidence that her mother knows how to care for a colostomy. This also does not indicate the mother has understanding of caring for the infants colostomy. Chapter 43 Nursing Care of the Child With Alteration in Urinary Elimination/Genitourinary Disorder MULTIPLE CHOICE 1. Which statement by a school-age girl indicates the need for further teaching about the prevention of urinary tract infections (UTIs)? a . I always wear cotton underwear. b . I really enjoy taking a bubble bath. c . I go to the bathroom every 3 to 4 hours. d . I drink four to six glasses of fluid every day. ANS: B Feedback A It is desirable to wear cotton rather than nylon underwear. Nylon tends to hold in moisture and promote bacterial growth, whereas cotton absorbs moisture. B Bubble baths should be avoided because they tend to cause urethral irritation, which leads to UTI. C Children should be encouraged to urinate at least four times a day. D An adequate fluid intake prevents the buildup of bacteria in the bladder. PTS: 1 DIF: Cognitive Level: Application REF: p. 1125 OBJ: Nursing Process: Assessment MSC: Client Needs: Health Promotion and Maintenance 2. The nurse assessing a child with acute poststreptococcal glomerulonephritis should be alert for which finding? a . Increased urine output b . Hypotension c . Tea-colored urine d . Weight gain ANS: C Feedback A In acute poststreptococcal glomerulonephritis the urine output may be decreased. B In acute poststreptococcal glomerulonephritis blood pressure may be increased. C Acute poststreptococcal glomerulonephritis is characterized by hematuria, proteinuria, edema, and renal insufficiency. Tea-colored urine is an indication of hematuria. D Edema may be noted around the eyelids and ankles in patients with acute poststreptococcal glomerulonephritis; however, weight gain is associated with nephrotic syndrome. PTS: 1 DIF: Cognitive Level: Application REF: p. 1128 OBJ: Nursing Process: Assessment MSC: Client Needs: Physiologic Integrity 3. The mother of a child who was recently diagnosed with acute glomerulonephritis asks the nurse why the physician keeps talking about casts in the urine. The nurses response is based on the knowledge that the presence of casts in the urine indicates a . Glomerular injury b . Glomerular healing c . Recent streptococcal infection d . Excessive amounts of protein in the urine ANS: A Feedback A The presence of red blood cell casts in the urine indicates glomerular injury. B Casts in the urine are abnormal findings and are indicative of glomerular injury, not glomerular healing. C A urinalysis positive for casts does not confirm a recent streptococcal infection. D Casts in the urine are unrelated to proteinuria. PTS: 1 DIF: Cognitive Level: Application REF: p. 1132 OBJ: Nursing Process: Implementation MSC: Client Needs: Physiologic Integrity 4. Which clinical finding warrants further intervention for the child with acute poststreptococcal glomerulonephritis? a . Weight loss to within 1 lb of the preillness weight b . Urine output of 1 mL/kg/hr c . A positive antistreptolysin O (ASO) titer d Inspiratory crackles . ANS: D Feedback A This is an indication that the child is responding to treatment. B This is an acceptable urine output and indicates that the child is responding to treatment. C A positive ASO titer indicates the presence of antibodies to streptococcal bacteria; it is used to aid in diagnosis of acute poststreptococcal glomerulonephritis. This is an expected finding if the child has this acute illness. D Children with excess fluid volume may have pulmonary edema. Inspiratory crackles indicate fluid in the lungs. Pulmonary edema can be a life-threatening complication. PTS: 1 DIF: Cognitive Level: Application REF: p. 1130 OBJ: Nursing Process: Planning MSC: Client Needs: Physiologic Integrity 5. Which diagnostic finding is present when a child has primary nephrotic syndrome? a . Hyperalbuminemia b . Positive ASO titer c . Leukocytosis d . Proteinuria ANS: D Feedback A Hypoalbuminemia is present because of loss of albumin through the defective glomerulus and the livers inability to synthesize proteins to balance the loss. B ASO titer is negative in a child with primary nephrotic syndrome. C Leukocytosis is not a diagnostic finding in primary nephrotic syndrome. D Large amounts of protein are lost through the urine as a result of an increased permeability of the glomerular basement membrane. PTS: 1 DIF: Cognitive Level: Application REF: p. 1132 OBJ: Nursing Process: Assessment MSC: Client Needs: Physiologic Integrity 6. Which finding indicates that a child receiving prednisone for primary nephrotic syndrome is in remission? a . Urine is negative for casts for 5 days. b . Urine is up to a trace for protein for 5 to 7 days. c . Urine is positive for glucose for 1 week. d . Urine is up to a trace for blood for 1 week. ANS: B Feedback A The absence of casts in the urine gives no indication about the childs response to treatment. The child with primary nephrotic syndrome is considered to be in remission when the urine is negative for protein for 5 to 7 consecutive days. B The child receiving steroids for the treatment of primary nephrotic syndrome is considered in remission when the urine is up to trace for protein for 5 to 7 days. C Remission is achieved when the urine is negative for protein for 5 to 7 consecutive days. It is not unusual for glucose to test positive if the child is taking prednisone. D The presence or absence of hematuria is not used to determine remission in primary nephrotic syndrome. PTS: 1 DIF: Cognitive Level: Application REF: p. 1133 OBJ: Nursing Process: Evaluation MSC: Client Needs: Physiologic Integrity 7. Which statement by a parent of a child with nephrotic syndrome indicates an understanding of a no-added-salt diet? a . I can give my child sweet pickles. b . My child can put ketchup on his hotdog. c . I can let my child have potato chips. d . I do not put any salt in foods when I am cooking. ANS: D Feedback A All types of pickles are high in sodium and should not be served to the child on a no-added-salt diet. B The child should not be allowed to eat hotdogs; they are considered a cured or processed meat and are high in sodium. C Potato chips are a high-sodium food and should not be included in the childs diet when sodium intake is restricted. D A no-added-salt diet means that no salt should be added to foods, either when cooking or before eating. PTS: 1 DIF: Cognitive Level: Comprehension REF: p. 1134 OBJ: Nursing Process: Evaluation MSC: Client Needs: Health Promotion and Maintenance 8. What is an appropriate intervention for a child with nephrotic syndrome who is edematous? a . Teach the child to minimize body movements. b . Change the childs position every 2 hours. c . Avoid the use of skin lotions. d . Bathe every other day. ANS: B Feedback A The child with edema is at risk for impaired skin integrity. It is important for the child to change position frequently to prevent skin breakdown. B Frequent position changes decrease pressure on body parts and help relieve edema in dependent areas. C Applying lotion to the skin helps to increase circulation. D Bathing daily removes irritating body secretions from the skin. PTS: 1 DIF: Cognitive Level: Comprehension REF: p. 1135 OBJ: Nursing Process: Implementation MSC: Client Needs: Physiologic Integrity 9. What should the nurse include in a teaching plan for the parents of a child with vesicoureteral reflux? a . The importance of taking prophylactic antibiotics b . Suggestions for how to maintain fluid restrictions c . The use of bubble baths as an incentive to increase bath time d . The need for the child to hold urine for 6 to 8 hours ANS: A Feedback A Prophylactic antibiotics are used to prevent urinary infection in a child with vesicoureteral reflux, although this treatment plan has become controversial. B Fluids are not restricted when a child has vesicoureteral reflux. In fact, fluid intake should be increased as a measure to prevent UTIs. C Bubble baths should be avoided to prevent urethral irritation and possible UTI. D To prevent UTIs, the child should be taught to void frequently and never resist the urge to urinate. PTS: 1 DIF: Cognitive Level: Application REF: p. 1124 OBJ: Nursing Process: Implementation MSC: Client Needs: Physiologic Integrity 10. Which intervention is appropriate when examining a male infant for cryptorchidism? a Cooling the examiners hands . b Taking a rectal temperature . c Eliciting the cremasteric reflex . d Warming the room . ANS: D Feedback A Examining the infant with cold hands is uncomfortable for the infant and likely to cause the infants testes to retract into the inguinal canal. It may also cause the infant to be uncooperative during the examination. B A rectal temperature yields no information about cryptorchidism. C Testes can retract into the inguinal canal if the infant is upset or cold or if the cremasteric reflex is elicited. This can lead to an incorrect diagnosis. D For the infants comfort, the infant should be examined in a warm room with the examiners hands warmed. Testes can retract into the inguinal canal if the infant is upset or cold. PTS: 1 DIF: Cognitive Level: Application REF: p. 1126 OBJ: Nursing Process: Implementation MSC: Client Needs: Physiologic Integrity 11. Parents ask the nurse when should our childs hypospadias be corrected? The nurse responds based upon the knowledge that correction of hypospadias should be accomplished by the time the child is a . 1 month of age b . 6 to 12 months of age c . School age d Sexually mature . ANS: B Feedback A Surgery to correct hypospadias is not performed when the infant is this young. B The correction of hypospadias should ideally be accomplished by the time the child is 6 to 12 months of age and before toilet training. C It is preferable for hypospadias to be surgically corrected before the child enters school so that the child has normal toileting behaviors in the presence of his peers. D Corrective surgery for hypospadias is done long before sexual maturity. PTS: 1 DIF: Cognitive Level: Application REF: p. 1127 OBJ: Nursing Process: Planning MSC: Client Needs: Physiologic Integrity 12. You are the nurse caring for a 4-year-old child who has developed acute renal failure as a result of hemolytic-uremic syndrome (HUS). Which bacterial infection was most likely the cause of HUS? a . Pseudomonas aeruginosa b . Escherichia coli c . Streptococcus pneumoniae d . Staphylococcus aureus ANS: B Feedback A Pseudomonas aeruginosa is not associated with HUS. B Children with HUS become infected by Escherichia coli, which is usually contracted from eating improperly cooked meat or contaminated dairy products. C Streptococcus pneumoniae is not associated with HUS. D Staphylococcus aureus is not associated with HUS. PTS: 1 DIF: Cognitive Level: Analysis REF: p. 1137 OBJ: Nursing Process: Assessment MSC: Client Needs: Physiologic Integrity 13. Which dietary modification is appropriate for a child with chronic renal failure? a . Decreased protein b . Decreased fat c . Increased potassium d . Increased phosphorus ANS: A Feedback A Protein intake is restricted because of the kidneys inability to remove waste products. B A low-fat diet is not relevant to chronic renal failure. C Potassium intake may be restricted because of the kidneys inability to remove it. D Phosphorus is restricted to help prevent bone disease. PTS: 1 DIF: Cognitive Level: Comprehension REF: p. 1139 OBJ: Nursing Process: Planning MSC: Client Needs: Health Promotion and Maintenance 14. Which condition is characterized by a history of bloody diarrhea, fever, abdominal pain, and low hemoglobin and platelet counts? a . Acute viral gastroenteritis b . Acute glomerulonephritis c . Hemolytic-uremic syndrome d . Acute nephrotic syndrome ANS: C Feedback A Anemia and thrombocytopenia are not associated with acute gastroenteritis. B The symptoms described are not suggestive of acute glomerulonephritis. C Hemolytic-uremic syndrome is an acute disorder characterized by anemia, thrombocytopenia, and acute renal failure. Most affected children have a history of gastrointestinal symptoms, including bloody diarrhea. D The symptoms described are not suggestive of nephrotic syndrome. PTS: 1 DIF: Cognitive Level: Comprehension REF: p. 1137 OBJ: Nursing Process: Assessment MSC: Client Needs: Physiologic Integrity 15. A child with secondary enuresis who complains of dysuria or urgency should be evaluated for which condition? a Hypocalciuria . b Nephrotic syndrome . c Glomerulonephritis . d UTI . ANS: D Feedback A An excessive loss of calcium in the urine (hypercalciuria) can be associated with complaints of painful urination, urgency, frequency, and wetting. B Nephrotic syndrome is not usually associated with complaints of dysuria or urgency. C Glomerulonephritis is not a likely cause of dysuria or urgency. D Complaints of dysuria or urgency from a child with secondary enuresis suggest the possibility of a UTI. PTS: 1 DIF: Cognitive Level: Comprehension REF: p. 1120 OBJ: Nursing Process: Evaluation MSC: Client Needs: Physiologic Integrity 16. A nurse is teaching a class on acute renal failure. The nurse relates that acute renal failure as a result of hemolytic-uremic syndrome (HUS) is classified as a Intrarenal . b Prerenal . c Postrenal . d Chronic . ANS: A Feedback A Intrarenal acute renal failure is the result of damage to kidney tissue. Possible causes of intrarenal acute renal failure are HUS, glomerulonephritis, and pyelonephritis. B Prerenal acute renal failure is the result of decreased perfusion to the kidney. Possible causes include dehydration, septic and hemorrhagic shock, and hypotension. C Postrenal acute renal failure results from obstruction of urine outflow. Conditions causing postrenal failure include ureteropelvic obstruction, ureterovesical obstruction, or neurogenic bladder. Renal failure caused by HUS is of the acute nature. D Chronic renal failure is an irreversible loss of kidney function, which occurs over months or years. PTS: 1 DIF: Cognitive Level: Comprehension REF: p. 1136 OBJ: Nursing Process: Assessment MSC: Client Needs: Physiologic Integrity 17. A true statement describing the differences in the pediatric genitourinary system compared with the adult genitourinary system is that a The young infants kidneys can more effectively concentrate urine than an adults . kidneys. b After 6 years of age, kidney function is nearly like that of an adult. . c Unlike adults, most children do not regain normal kidney function after acute . renal failure. d Young children have shorter urethras, which can predispose them to UTIs. . ANS: D Feedback A The young infants kidneys cannot concentrate urine as efficiently as those of older children and adults because the loops of Henle are not yet long enough to reach the inner medulla, where concentration and reabsorption occur. B By 6 to 12 months of age, kidney function is nearly like that of an adult. C Unlike adults, most children with acute renal failure regain normal function. D Young children have shorter urethras, which can predispose them to UTIs. PTS: 1 DIF: Cognitive Level: Comprehension REF: p. 1117 OBJ: Nursing Process: Assessment MSC: Client Needs: Health Promotion and Maintenance 18. Which factor predisposes the urinary tract to infection? a Increased fluid intake . b Short urethra in young girls . c Prostatic secretions in males . d Frequent emptying of the bladder . ANS: B Feedback A This offers protective measures against UTIs. B The short urethra in females provides a ready pathway for invasions of organisms. C Prostatic secretions have antibacterial properties that inhibit bacteria. D This offers protective measures against UTIs. PTS: 1 DIF: Cognitive Level: Comprehension REF: p. 1122 OBJ: Nursing Process: Assessment MSC: Client Needs: Physiologic Integrity 19. Hypospadias refers to a Absence of a urethral opening . b Penis shorter than usual for age . c Urethral opening along dorsal surface of penis . d Urethral opening along ventral surface of penis . ANS: D Feedback A The urethral opening is present, but not at the glans. B Hypospadias refers to the urethral opening, not to the size of the penis. C This is known as epispadias. D Hypospadias is a congenital condition in which the urethral opening is located anywhere along the ventral surface of the penis. PTS: 1 DIF: Cognitive Level: Comprehension REF: p. 1127 OBJ: Nursing Process: Assessment MSC: Client Needs: Physiologic Integrity 20. The narrowing of preputial opening of foreskin is called a Chordee . b Phimosis . c Epispadias . d Hypospadias . ANS: B Feedback A Chordee is the ventral curvature of the penis. B Phimosis is the narrowing or stenosis of the preputial opening of the foreskin. C Epispadias is the meatal opening on the dorsal surface of the penis. D Hypospadias is a congenital condition in which the urethral opening is located anywhere along the ventral surface of the penis. PTS: 1 DIF: Cognitive Level: Comprehension REF: p. 1129 | Table 44-1 OBJ: Nursing Process: Assessment MSC: Client Needs: Physiologic Integrity 21. The nurse closely monitors the temperature of a child with nephrotic syndrome. The purpose of this is to detect an early sign of which possible complication? a Infection . b Hypertension . c Encephalopathy . d Edema . ANS: A Feedback A An exacerbation of the disease can occur after an infection. B Temperature is not an indication of hypertension or edema. C Encephalopathy is not a complication usually associated with nephrosis. The child will most likely have neurologic signs and symptoms. D Encephalopathy is not a complication usually associated with nephrosis. The child will most likely have neurologic signs and symptoms. PTS: 1 DIF: Cognitive Level: Application REF: p. 1135 OBJ: Nursing Process: Assessment MSC: Client Needs: Physiologic Integrity 22. A child is admitted with acute glomerulonephritis. The nurse expects the urinalysis during this acute phase to show a . Bacteriuria and hematuria b . Hematuria and proteinuria c . Bacteriuria and increased specific gravity d Proteinuria and decreased specific gravity . ANS: B Feedback A Bacteriuria and changes in specific gravity are not usually present during the acute phase. B Urinalysis during the acute phase characteristically shows hematuria and proteinuria. C Bacteriuria and changes in specific gravity are not usually present during the acute phase. D Bacteriuria and changes in specific gravity are not usually present during the acute phase. PTS: 1 DIF: Cognitive Level: Comprehension REF: p. 1129 OBJ: Nursing Process: Assessment MSC: Client Needs: Physiologic Integrity 23. The most appropriate nursing diagnosis for the child with acute glomerulonephritis is a . Risk for Injury related to malignant process and treatment b . Deficient Fluid Volume related to excessive losses c . Risk for Imbalanced Fluid Volume related to a decrease in plasma filtration d . Excess Fluid Volume related to fluid accumulation in tissues and third spaces ANS: C Feedback A No malignant process is involved in acute glomerulonephritis. B Excess fluid volume is found. C Glomerulonephritis has a decreased filtration of plasma. The resulting decrease in plasma filtration results in an excessive accumulation of water and sodium that expands plasma and interstitial fluid volumes, leading to circulatory congestion and edema. D The fluid accumulation is secondary to the decreased plasma filtration. Chapter 44 Nursing Care of Child With Alteration in Mobility/Neuromuscular Musculoskeletal Disorder MULTIPLE CHOICE 1. Which statement is accurate concerning a childs musculoskeletal system and how it may be different from an adults? a . Growth occurs in children as a result of an increase in the number of muscle fibers. b . Infants are at greater risk for fractures because their epiphyseal plates are not fused. c . Because soft tissues are resilient in children, dislocations and sprains are less common than in adults. d . Their bones have less blood flow. ANS: C Feedback A A childs growth occurs because of an increase in size rather than an increase in the number of the muscle fibers. B This is not a true statement. Fractures in children younger than 1 year are unusual because a large amount of force is necessary to fracture their bones. C Because soft tissues are resilient in children, dislocations and sprains are less common than in adults. This is an accurate statement. D A childs bones have greater blood flow than an adults bones. PTS: 1 DIF: Cognitive Level: Comprehension REF: p. 1337 OBJ: Nursing Process: Planning MSC: Client Needs: Physiologic Integrity 2. When infants are seen for fractures, which nursing intervention is a priority? a . No intervention is necessary. It is not uncommon for infants to fracture bones. b . Assess the familys safety practices. Fractures in infants usually result from falls. c . Assess for child abuse. Fractures in infants are often nonaccidental. d . Assess for genetic factors. ANS: C Feedback A Fractures in infancy are not common. B Infants should be cared for in a safe environment and should not be falling. C Fractures in infants warrant further investigation to rule out child abuse. Fractures in children younger than 1 year are unusual because of the cartilaginous quality of the skeleton; a large amount of force is necessary to fracture their bones. D Fractures in infancy are usually nonaccidental rather than related to a genetic factor. PTS: 1 DIF: Cognitive Level: Application REF: p. 1346 OBJ: Nursing Process: Assessment MSC: Client Needs: Physiologic Integrity 3. Which nursing intervention is appropriate to assess for neurovascular competency in a child who fell off the monkey bars at school and hurt his arm? a . The degree of motion and ability to position the extremity b . The length, diameter, and shape of the extremity c . The amount of swelling noted in the extremity and pain intensity d . The skin color, temperature, movement, sensation, and capillary refill of the extremity ANS: D Feedback A The degree of motion in the affected extremity and ability to position the extremity are incomplete assessments of neurovascular competency. B The length, diameter, and shape of the extremity are not assessment criteria in a neurovascular evaluation. C Although the amount of swelling is an important factor in assessing an extremity, it is not a criterion for a neurovascular assessment. D A neurovascular evaluation includes assessing skin color and temperature, ability to move the affected extremity, degree of sensation experienced, and speed of capillary refill in the extremity. PTS: 1 DIF: Cognitive Level: Application REF: p. 1349 OBJ: Nursing Process: Assessment MSC: Client Needs: Physiologic Integrity 4. A mother whose 7-year-old child has been placed in a cast for a fractured right arm reports that he will not stop crying even after taking acetaminophen with codeine. He also will not straighten the fingers on his right arm. The nurse tells the mother to a . Take him to the emergency department. b . Put ice on the injury. c . Avoid letting him get so tired. d . Wait another hour; if he is still crying, call back. ANS: A Feedback A Unrelieved pain and the childs inability to extend his fingers are signs of compartmental syndrome, which requires immediate attention. B Placing ice on the extremity is an inappropriate action for the symptoms. C This is an inappropriate response to give to a mother who is concerned about her child. D A child who has signs and symptoms of compartmental syndrome should be seen immediately. Waiting an hour could compromise the recovery of the child. PTS: 1 DIF: Cognitive Level: Analysis REF: p. 1346 OBJ: Nursing Process: Diagnosis MSC: Client Needs: Physiologic Integrity 5. A 4-year-old child with a long leg cast complains of fire in his cast. The nurse should a . Notify the physician on his next rounds. b . Note the complaint in the nurses notes. c . Notify the physician immediately. d . Report the complaint to the next nurse on duty. ANS: C Feedback A The childs symptom requires immediate attention. Notifying the physician on the next rounds is inappropriate. B Charting the complaint in the nurses notes is an inappropriate action. Careful notation of symptoms is important, but the priority action is to contact the physician. C A burning sensation under the cast is an indication of tissue ischemia. It may be an early indication of serious neurovascular compromise, such as compartmental syndrome, that requires immediate attention. D Communication across shifts is important to the continuing assessment of the child; however, this symptom requires immediate evaluation, and the physician should be contacted. PTS: 1 DIF: Cognitive Level: Analysis REF: p. 1344 OBJ: Nursing Process: Assessment MSC: Client Needs: Physiologic Integrity 6. When a child with a musculoskeletal injury on the foot is assessed, what is most indicative of a fracture? a Increased swelling after the injury is iced . b . The presence of localized tenderness distal to the site c . The presence of an elevated temperature for 24 hours d . The inability of the child to bear weight ANS: D Feedback A Although edema is often present with a fracture, it would be unusual for swelling to increase after application of ice, and this would not be most indicative of a fracture. Swelling after icing does not identify the degree of the injury. B Localized tenderness along with limited joint mobility may indicate serious injury, but inability to bear weight on the extremity is a more reliable sign. Tenderness is not a usual complaint distal to the affected site. C Elevated temperature is associated with infection, but not a fracture. D An inability to bear weight on the affected extremity is indicative of a more serious injury. With a fracture, general manifestations include pain or tenderness at the site, immobility or decreased range of motion, deformity of the extremity, edema, and inability to bear weight. PTS: 1 DIF: Cognitive Level: Application REF: p. 1347 OBJ: Nursing Process: Assessment MSC: Client Needs: Physiologic Integrity 7. A child with osteomyelitis asks the nurse, What is a sed rate? What is the best response for the nurse? a It tells us how you are responding to the treatment. . b It tells us what type of antibiotic you need. . c It tells us whether we need to immobilize your extremity. . d It tells us how your nerves and muscles are doing. . ANS: A Feedback A The erythrocyte sedimentation rate (ESR) indicates the presence of inflammation and infectious process and is one of the best indicators of the childs response to treatment. B Although the ESR indirectly identifies whether an antibiotic is needed, the organism involved dictates the type of antibiotic and the length of treatment. C The ESR does not direct whether the extremity will be immobilized. D An ESR rate will not evaluate neuromuscular status. PTS: 1 DIF: Cognitive Level: Application REF: p. 1352 OBJ: Nursing Process: Evaluation MSC: Client Needs: Physiologic Integrity 8. Which interaction is part of the discharge plan for a school-age child with osteomyelitis who is receiving home antibiotic therapy? a Instructions for a low-calorie diet . b Arrange for tutoring and school work . c Instructions for a high-fat, low-protein diet . d Instructions for the parent to return the child to team sports immediately . ANS: B Feedback A The child with osteomyelitis is on a high-calorie, high-protein diet. B Promoting optimal growth and development in the school-age child is important. It is important to continue school work and arrange for tutoring if indicated. C The child with osteomyelitis is on a high-calorie, high-protein diet. D The child with osteomyelitis may need time for the bone to heal before returning to full activities. PTS: 1 DIF: Cognitive Level: Application REF: p. 1353 OBJ: Nursing Process: Planning MSC: Client Needs: Health Promotion and Maintenance 9. During a 14-year-olds physical examination, the nurse identifies that he plays soccer and football and is complaining of knee pain when he rises from a squatting position, and difficulty with weight bearing. The nurse should suspect a Legg-Calv-Perthes disease . b Osteomyelitis . c Duchenne muscular dystrophy . d Osgood-Schlatter disease . ANS: D Feedback A Pain on activity that decreases with rest is indicative of Legg-Calv-Perthes disease. B Preexisting pain, favoring the affected limb, erythema, and tenderness are associated with osteomyelitis. C Duchenne muscular dystrophy causes progressive generalized weakness and muscle wasting. D Knee pain and tenderness aggravated by activity that requires kneeling, running, climbing stairs, and rising from a squatting position is highly significant for Osgood-Schlatter disease. The cause is believed to be related to repetitive stress from sports-related activities combined with overuse of immature muscles and tendons. PTS: 1 DIF: Cognitive Level: Application REF: p. 1359 | Table 50-5 OBJ: Nursing Process: Assessment MSC: Client Needs: Physiologic Integrity 10. A child is upset because, when the cast is removed from her leg, the skin surface is caked with desquamated skin and sebaceous secretions. What should the nurse suggest to remove this material? a . Wash the area with warm water and soap. b . Vigorously scrub leg. c . Apply powder to absorb material. d . Carefully pick material off leg. ANS: A Feedback A Simple soaking in the bathtub is usually sufficient for the removal of the desquamated skin and sebaceous secretions. B The parents and child should be advised not to scrub the leg vigorously or forcibly remove this material because it may cause excoriation and bleeding. C Oil or lotion, but not powder, may provide comfort for the child. D The parents and child should be advised not to scrub the leg vigorously or forcibly remove this material because it may cause excoriation and bleeding. PTS: 1 DIF: Cognitive Level: Application REF: p. 1346 OBJ: Nursing Process: Implementation MSC: Client Needs: Physiologic Integrity 11. Which factor is important to include in the teaching plan for parents of a child with LeggCalv-Perthes disease? a . It is an acute illness lasting 1 to 2 weeks. b . It affects primarily adolescents. c . There is a disturbance in the blood supply to the femoral epiphysis. d . It is caused by a virus. ANS: C Feedback A The disease process usually lasts between 1 and 2 years and is a disorder of growth. B Legg-Calv-Perthes disease is seen in children between 2 and 12 years of age. Most cases occur between 4 and 9 years of age. C Legg-Calv-Perthes disease is a self-limiting disease that affects the blood supply to the femoral epiphysis. The most serious problem associated is the risk of permanent deformity. D The etiology is unknown. PTS: 1 DIF: Cognitive Level: Application REF: p. 1363 OBJ: Nursing Process: Planning MSC: Client Needs: Health Promotion and Maintenance 12. What is the major concern guiding treatment for the child with Legg-Calv-Perthes disease? a . Avoid permanent deformity. b . Minimize pain. c . Maintain normal activities. d . Encourage new hobbies. ANS: A Feedback A The major concern related to Legg-Calv-Perthes disease is to prevent an arthritic process resulting from the flattening of the femoral head of the femur when it protrudes outside the acetabulum. B The pain associated with Legg-Calv-Perthes disease decreases with increased rest, making activity restriction an important factor for these children. The priority concern for treatment is to prevent deformity. C In Legg-Calv-Perthes disease, the major concern is to prevent deformity through decreased activity. D Prevention of deformity is the major concern for children with Legg-CalvPerthes disease, and rest is a mandatory treatment. Selected hobbies that do not require physical activity are encouraged. PTS: 1 DIF: Cognitive Level: Comprehension REF: p. 1364 OBJ: Nursing Process: Planning MSC: Client Needs: Physiologic Integrity 13. A neonate is born with mild clubfeet. When the parents ask the nurse how this will be corrected, the nurse should explain that a . Traction is tried first. b . Surgical intervention is needed. c . Frequent, serial casting is tried first. d . Children outgrow this condition when they learn to walk. ANS: C Feedback A Serial casting is the preferred treatment. B Surgical intervention is done only if serial casting is not successful. C Serial casting is begun shortly after birth before discharge from the nursery. Successive casts allow for gradual stretching of skin and tight structures on the medial side of the foot. Manipulation and casting of the leg are repeated frequently (every week) to accommodate the rapid growth of early infancy. D Children do not improve without intervention. PTS: 1 DIF: Cognitive Level: Comprehension REF: p. 1366 OBJ: Nursing Process: Implementation MSC: Client Needs: Physiologic Integrity 14. Discharge planning for the child with juvenile arthritis includes the need for a . Routine ophthalmologic examinations to assess for visual problems b . A low-calorie diet to decrease or control weight in the less mobile child c . Avoiding the use of aspirin to decrease gastric irritation d . Immobilizing the painful joints, which is the result of the inflammatory process ANS: A Feedback A The systemic effects of juvenile arthritis can result in visual problems, making routine eye examinations important. B Children with juvenile arthritis do not have problems with increased weight and often are anorexic and in need of high-calorie diets. C Children with arthritis are often treated with aspirin. D Children with arthritis can immobilize their own joints. Range-of-motion exercises are important for maintaining joint flexibility and preventing restricted movement in the affected joints. PTS: 1 DIF: Cognitive Level: Application REF: p. 1369 OBJ: Nursing Process: Planning MSC: Client Needs: Health Promotion and Maintenance 15. During painful episodes of juvenile arthritis, a plan of care should include what nursing intervention? a . A weight-control diet to decrease stress on the joints b . Proper positioning of the affected joints to prevent musculoskeletal complications c . Complete bed rest to decrease stress to joints d . High-resistance exercises to maintain muscular tone in the affected joints ANS: B Feedback A Children in pain often are anorexic and need high-calorie foods. B Proper positioning is important to support and protect affected joints. Isometric exercises and passive range-of-motion exercises will prevent contractures and deformities. C Children with juvenile arthritis need a combination of rest and exercise. D Children with juvenile arthritis need to avoid high-resistance exercises and they benefit from low-resistance exercises, such as swimming. PTS: 1 DIF: Cognitive Level: Application REF: p. 1370 OBJ: Nursing Process: Planning MSC: Client Needs: Physiologic Integrity 16. When assessing a child for an upper extremity fracture, the nurse should know that these fractures most often result from a . Automobile accidents b . Falls c . Physical abuse d Sports injuries . ANS: B Feedback A Automobile accidents result in fractures to any bones. Frequently, the femur is broken. B The major cause of childrens fractures is falls. Because of the protection reflexes, the outstretched arm often receives the full force of the fall. C Physical abuse may result in fractures to any bone. D Sports injuries may result in fractures to any bone. PTS: 1 DIF: Cognitive Level: Comprehension REF: p. 1346 OBJ: Nursing Process: Assessment MSC: Client Needs: Physiologic Integrity 17. In caring for a child with a compound fracture, the nurse should carefully assess for a . Infection b . Osteoarthritis c . Epiphyseal disruption d . Periosteum thickening ANS: A Feedback A Because the skin has been broken, the child is at risk for organisms to enter the wound. B The incidence of osteoarthritis does not increase with a compound fracture. C The chance of epiphyseal disruption is not increased with compound fracture. D Periosteum thickening is part of the healing process and not a complication. PTS: 1 DIF: Cognitive Level: Application REF: p. 1348 OBJ: Nursing Process: Assessment MSC: Client Needs: Physiologic Integrity 18. A nurse is teaching parents the difference between pediatric fractures and adult fractures. Which observation is true about pediatric fractures? a . They seldom are complete breaks. b . They are often compound fractures. c . They are often at the epiphyseal plate. d . They are often the result of decreased mobility of the bones. ANS: A Feedback A Pediatric fractures seldom are complete breaks. Rather, childrens bones tend to bend or buckle. B Compound fractures are no more common than simple fractures in children. C Epiphyseal plate fractures are no more common than any other type of fracture. D Increased mobility of the bones prevents children from having complete fractures. PTS: 1 DIF: Cognitive Level: Comprehension REF: p. 1347 OBJ: Nursing Process: Assessment MSC: Client Needs: Physiologic Integrity 19. Patient and parent education for the child who has a synthetic cast should include a . Applying a heating pad to the cast if the child has swelling in the affected extremity b . Wrapping the outer surface of the cast with an Ace bandage c . Splitting the cast if the child complains of numbness or pain d . Covering the cast with plastic and waterproof tape to keep it dry while bathing or showering ANS: D Feedback A To prevent swelling, elevate the extremity and apply bagged ice to the casted area. B Wrapping the outer surface with an Ace bandage is not indicated. C If the child complains of numbness or pain, the child should return immediately to the clinic or emergency department for an evaluation of neurovascular status. D Damp skin is more susceptible to breakdown. Cast should be kept clean and dry. PTS: 1 DIF: Cognitive Level: Application REF: p. 1346 OBJ: Nursing Process: Planning MSC: Client Needs: Health Promotion and Maintenance 20. A 6-year-old patient who has been placed in skeletal traction has pain, edema, and fever. The nurse should suspect a . Meningitis b . Crepitus c . Osteomyelitis d . Osteochondrosis ANS: C Feedback A The symptoms of meningitis include headache, photophobia, fever, nausea, and vomiting. B Crepitus is the sandy or gravelly feeling noted when a broken bone is palpated. C The most serious complication of skeletal traction is osteomyelitis. Clinical manifestations include complaints of localized pain, swelling, warmth, tenderness, or unusual odor. An elevated temperature may accompany the symptoms. D Osteochondrosis is a disorder of the epiphyses involving an interruption of the blood supply. PTS: 1 DIF: Cognitive Level: Comprehension REF: p. 1342 OBJ: Nursing Process: Assessment MSC: Client Needs: Physiologic Integrity 21. A boy who has fractured his forearm is unable to extend his fingers. The nurse knows that this a Is normal following this type of injury . b . May indicate compartmental syndrome c . May indicate fat embolism d . May indicate damage to the epiphyseal plate ANS: B Feedback A This is not normal and indicates neurovascular compromise of some type. Paresthesia or numbness or loss of feeling can indicate a serious problem and can result in paralysis. B Swelling causes pressure to rise within the immobilizing device leading to compartmental syndrome. Signs include severe pain, often unrelieved by analgesics, and neurovascular impairment. It is not uncommon in the forearm, so the inability to extend the fingers may indicate compartmental syndrome. C The inability to extend the fingers often indicates neurovascular compromise. Fat embolism causes respiratory distress with hypoxia and respiratory acidosis. D This is not related to damage to the epiphyseal plate. PTS: 1 DIF: Cognitive Level: Comprehension REF: p. 1344 OBJ: Nursing Process: Assessment MSC: Client Needs: Physiologic Integrity 22. Which term is used to describe an abnormally increased convex angulation in the curvature of the thoracic spine? a Scoliosis . b Ankylosis . c Lordosis . d Kyphosis . ANS: D Feedback A Scoliosis is a complex spinal deformity usually involving lateral curvature, spinal rotation causing rib asymmetry, and thoracic hypokyphosis. B Ankylosis is the immobility of a joint. C Lordosis is an accentuation of the cervical or lumbar curvature beyond physiologic limits. D Kyphosis is an abnormally increased convex angulation in the curve of the thoracic spine. PTS: 1 DIF: Cognitive Level: Comprehension REF: p. 1356 OBJ: Nursing Process: Assessment MSC: Client Needs: Physiologic Integrity 23. When assessing the child with osteogenesis imperfecta, the nurse should expect to observe a Discolored teeth . b Below-normal intelligence . c Increased muscle tone . d Above-average stature . ANS: A Feedback A Children with osteogenesis imperfecta have incomplete development of bones, teeth, ligaments, and sclerae. Teeth are discolored because of abnormal enamel. B Despite their appearance, children with osteogenesis imperfecta have normal or above-normal intelligence. C The child with osteogenesis imperfecta has weak muscles and decreased muscle tone. D Because of compression fractures of the spine, the child appears short. Chapter 45 Nursing Care of the Child With an Alteration in Tissue Integrity/Integumentary Disorder MULTIPLE CHOICE 1. What should be included in teaching a parent about the management of small red macules and vesicles that become pustules around the childs mouth and cheek? a Keep the child home from school for 24 hours after initiation of antibiotic . treatment. b Clean the rash vigorously with Betadine three times a day. . c Notify the physician for any itching. . d Keep the child home from school until the lesions are healed. . ANS: A Feedback A To prevent the spread of impetigo to others, the child should be kept home from school for 24 hours after treatment is initiated. Good handwashing is imperative in preventing the spread of impetigo. B The lesions should be washed gently with a warm soapy washcloth three times a day. The washcloth should not be shared with other members of the family. C Itching is common and does not necessitate medical treatment. Rather, parents should be taught to clip the childs nails to prevent maceration of the lesions. D The child may return to school 24 hours after initiation of antibiotic treatment. PTS: 1 DIF: Cognitive Level: Application REF: p. 1306 OBJ: Nursing Process: Planning MSC: Client Needs: Health Promotion and Maintenance 2. When taking a history on a child with a possible diagnosis of cellulitis, what should be the priority nursing assessment to help establish a diagnosis? a . Any pain the child is experiencing b . Enlarged, mobile, and nontender lymph nodes c . Childs urinalysis results d . Recent infections or signs of infection ANS: D Feedback A Pain is important, but the history of recent infections is more relevant to the diagnosis. B Lymph nodes may be enlarged (lymphadenitis), but they are not mobile and are nontender. Lymphangitis may be seen with red streaking of the surrounding area. C An abnormal urinalysis result is not usually associated with cellulitis. D Cellulitis may follow an upper respiratory infection, sinusitis, otitis media, or a tooth abscess. The affected area is red, hot, tender, and indurated. PTS: 1 DIF: Cognitive Level: Application REF: p. 1307 OBJ: Nursing Process: Assessment MSC: Client Needs: Health Promotion and Maintenance 3. Which statement made by a parent indicates an understanding about the management of a child with cellulitis? a . I am supposed to continue the antibiotic until the redness and swelling disappear. b . I have been putting ice on my sons arm to relieve the swelling. c . I should call the doctor if the redness disappears. d . I have been putting a warm soak on my sons arm every 4 hours. ANS: D Feedback A The parent should not discontinue antibiotics when signs of infection disappear. To ensure complete healing, the parent should understand that the entire course of antibiotics should be given as prescribed. B A warm soak is indicated for the treatment of cellulitis. Ice will decrease circulation to the affected area and inhibit the healing process. C The disappearance of redness indicates healing and is not a reason to seek medical advice. D Warm soaks applied every 4 hours while the child is awake increase circulation to the infected area, relieve pain, and promote healing. PTS: 1 DIF: Cognitive Level: Application REF: p. 1307 OBJ: Nursing Process: Evaluation MSC: Client Needs: Health Promotion and Maintenance 4. What should the parents of an infant with thrush (oral candidiasis) be taught about medication administration? a . Give nystatin suspension with a syringe without a needle. b . Apply nystatin cream to the affected area twice a day. c . Give nystatin before the infant is fed. d . Swab nystatin suspension onto the oral mucous membranes after feedings. ANS: D Feedback A Medication may not reach the affected areas when it is squirted into the infants mouth. Rubbing the suspension onto the gum ensures contact with the affected areas. B Nystatin cream is used for diaper rash caused by Candida. C To prolong contact with the affected areas, the medication should be administered after a feeding. D It is important to apply the nystatin suspension to the affected areas, which is best accomplished by rubbing it onto the gums and tongue, after feedings, every 6 hours, until 3 to 4 days after symptoms have disappeared. PTS: 1 DIF: Cognitive Level: Application REF: p. 1308 OBJ: Nursing Process: Planning MSC: Client Needs: Health Promotion and Maintenance 5. With what beverage should the parents of a child with ringworm be taught to give griseofulvin? a Water . b A carbonated drink . c Milk . d Fruit juice . ANS: C Feedback A Griseofulvin is insoluble in water. B Carbonated drinks do not contain fat, which aids in the absorption of griseofulvin. C Griseofulvin is insoluble in water. Giving the medication with a high-fat meal or milk increases absorption. D Fruit juice does not contain any fat; fat aids absorption of the medication. PTS: 1 DIF: Cognitive Level: Application REF: p. 1310 OBJ: Nursing Process: Implementation MSC: Client Needs: Health Promotion and Maintenance 6. Which nursing assessment is applicable to the care of a child with herpetic gingivostomatitis? a Comparison of range of motion for the upper and lower extremities . b Urine output, mucous membranes, and skin turgor . c Growth pattern since birth . d Bowel elimination pattern . ANS: B Feedback A An oral herpetic infection does not affect joint function. B The child with herpetic gingivostomatitis is at risk for deficient fluid volume. Painful lesions on the mouth make drinking unpleasant and undesirable, with subsequent dehydration becoming a real danger. C Herpetic gingivostomatitis is not a chronic disorder that would affect the childs long-term growth pattern. D Although constipation could be caused by dehydration, it is more important to assess urine output, skin turgor, and mucous membranes to identify dehydration before constipation is a problem. PTS: 1 DIF: Cognitive Level: Analysis REF: p. 1313 OBJ: Nursing Process: Assessment MSC: Client Needs: Physiologic Integrity 7. Parents of a child with lice infestation should be instructed carefully in the use of antilice products because of which potential side effect? a Nephrotoxicity . b Neurotoxicity . c Ototoxicity . d Bone marrow depression . ANS: B Feedback A Antilice products are not known to be nephrotoxic. B Because of the danger of absorption through the skin and potential for neurotoxicity, antilice treatment must be used with caution. A child with many open lesions can absorb enough to cause seizures. C Antilice products are not ototoxic. D Products that treat lice are not known to cause bone marrow depression. PTS: 1 DIF: Cognitive Level: Application REF: p. 1315 OBJ: Nursing Process: Implementation MSC: Client Needs: Physiologic Integrity 8. When assessing the child with atopic dermatitis, the nurse should ask the parents about a history of a Asthma . b Nephrosis . c Lower respiratory tract infections . d Neurotoxicity . ANS: A Feedback A Most children with atopic dermatitis have a family history of asthma, hay fever, or atopic dermatitis, and up to 80% of children with atopic dermatitis have asthma or allergic rhinitis. B Complications of atopic dermatitis relate to the skin. The renal system is not affected by atopic dermatitis. C There is no link between lower respiratory tract infections and atopic dermatitis. D Atopic dermatitis does not have a relationship to neurotoxicity. PTS: 1 DIF: Cognitive Level: Comprehension REF: p. 1302 OBJ: Nursing Process: Assessment MSC: Client Needs: Physiologic Integrity 9. What should the nurse teach an adolescent who is taking tretinoin (Retin-A) to treat acne? a . The medication should be taken with meals. b . Apply sunscreen before going outdoors. c . Wash with benzoyl peroxide before application. d . The effect of the medication should be evident within 1 week. ANS: B Feedback A Tretinoin is a topical medication. Application is not affected by meals. B Tretinoin causes photosensitivity, and sunscreen should be applied before sun exposure. C If applied together, benzoyl peroxide and tretinoin have reduced effectiveness and a potentially irritant effect. D Optimal results from tretinoin are not achieved for 3 to 5 months. PTS: 1 DIF: Cognitive Level: Application REF: p. 1318 OBJ: Nursing Process: Planning MSC: Client Needs: Health Promotion and Maintenance 10. When changing an infants diaper, the nurse notices small bright red papules with satellite lesions on the perineum, anterior thigh, and lower abdomen. This rash is characteristic of a . Primary candidiasis b . Irritant contact dermatitis c . Intertrigo d . Seborrheic dermatitis ANS: A Feedback A Small red papules with peripheral scaling in a sharply demarcated area involving the anterior thighs, lower abdomen, and perineum are characteristic of primary candidiasis. B A shiny, parchment-like erythematous rash on the buttocks, medial thighs, mons pubis, and scrotum, but not in the folds, is suggestive of irritant contact dermatitis. C Intertrigo is identified by a red macerated area of sharp demarcation in the groin folds. It can also develop in the gluteal and neck folds. D Seborrheic dermatitis is recognized by salmon-colored, greasy lesions with a yellowish scale found primarily in skin-fold areas or on the scalp. PTS: 1 DIF: Cognitive Level: Application REF: pp. 1308-1309 OBJ: Nursing Process: Assessment MSC: Client Needs: Physiologic Integrity 11. The depth of a burn injury may be classified as a . Localized or systemic b . Superficial, superficial partial thickness, deep partial thickness, or full thickness c . Electrical, chemical, or thermal d . Minor, moderate, or major ANS: B Feedback A These terms refer to the effect of the burn injury. For example, is there a reaction in the area of the burn (localized) or throughout the body (systemic)? B The vocabulary to classify the depth of a burn is superficial, partial thickness, or full thickness. C These terms refer to the cause of the burn injury. D These terms refer to the severity of the burn injury. PTS: 1 DIF: Cognitive Level: Comprehension REF: p. 1325 | Table 49-4 OBJ: Nursing Process: Assessment MSC: Client Needs: Physiologic Integrity 12. What best describes a full-thickness (third-degree) burn? a Erythema and pain . b . Skin showing erythema followed by blister formation c . Destruction of all layers of skin evident with extension into subcutaneous tissue d . Destruction injury involving underlying structures such as muscle, fascia, and bone ANS: C Feedback A Erythema and pain are characteristic of a first-degree burn or superficial burn. B Erythema with blister formation is characteristic of a second-degree or partialthickness burn. C A third-degree or full-thickness burn is a serious injury that involves the entire epidermis and dermis and extends into the subcutaneous tissues. D A fourth-degree burn is a full-thickness burn that also involves underlying structures such as muscle, fascia, and bone. PTS: 1 DIF: Cognitive Level: Comprehension REF: p. 1325 | Table 49-4 OBJ: Nursing Process: Assessment MSC: Client Needs: Physiologic Integrity 13. What procedure is contraindicated in the care of a child with a minor partial-thickness burn injury wound? a . Cleaning the affected area with mild soap and water b . Applying antimicrobial ointment to the burn wound c . Changing dressings daily d Leaving all loose tissue or skin intact . ANS: D Feedback A Cleaning with mild soap and water are important to the healing process. B Antimicrobial ointment is used on the burn wound to fight infection. C Clean dressings are applied daily to prevent wound infection. When dressings are changed, the condition of the burn wound can be assessed. D All loose skin and tissue should be debrided, because it can become a breeding ground for infectious organisms. PTS: 1 DIF: Cognitive Level: Application REF: p. 1329 OBJ: Nursing Process: Implementation MSC: Client Needs: Physiologic Integrity 14. The process of burn shock continues until what physiologic mechanism occurs? a . Heart rate returns to normal. b . Airway swelling decreases. c . Body temperature regulation returns to normal. d . Capillaries regain their seal. ANS: D Feedback A The heart rate will be increased throughout the healing process because of increased metabolism. B Airway swelling subsides over a period of 2 to 5 days after injury. C Body temperature regulation will not be normal until healing is well under way. D Within minutes of the burn injury, the capillary seals are lost with a massive fluid leakage into the surrounding tissue, resulting in burn shock. The process of burn shock continues for approximately 24 to 48 hours, when capillary seals are restored. PTS: 1 DIF: Cognitive Level: Comprehension REF: p. 1331 OBJ: Nursing Process: Assessment MSC: Client Needs: Physiologic Integrity 15. To assess the child with severe burns for adequate perfusion, the nurse monitors a . Distal pulses b . Skin turgor c . Urine output d . Mucous membranes ANS: C Feedback A Distal pulses may be affected by many variables. Urine output is the most reliable indicator of end-organ perfusion. B Skin turgor is often difficult to assess on burn patients because the skin is not intact. C Urine output reflects the adequacy of end-organ perfusion. D Mucous membranes do not reflect end-organ perfusion. PTS: 1 DIF: Cognitive Level: Analysis REF: p. 1331 OBJ: Nursing Process: Assessment MSC: Client Needs: Physiologic Integrity 16. What nursing assessment and care holds the highest priority in the initial care of a child with a major burn injury? a . Establishing and maintaining the childs airway b . Establishing and maintaining intravenous access c . Inserting a catheter to monitor hourly urine output d . Inserting a nasogastric tube into the stomach to supply adequate nutrition ANS: A Feedback A Establishing and maintaining the childs airway is always the priority focus for assessment and care. B Establishing intravenous access is the second priority in this situation, after the airway has been established. C Inserting a catheter and monitoring hourly urine output is the third most important nursing intervention. D Nasogastric feedings are not begun initially on a child with major or severe burns. The initial assessment and care focus for a child with major burn injuries is the ABCs. PTS: 1 DIF: Cognitive Level: Analysis REF: p. 1331 OBJ: Nursing Process: Planning MSC: Client Needs: Physiologic Integrity 17. An important nursing consideration when caring for a child with impetigo contagiosa is to a . Apply topical corticosteroids to decrease inflammation. b . Carefully remove dressings so as not to dislodge undermined skin, crusts, and debris. c . Carefully wash hands and maintain cleanliness when caring for an infected child. d . Examine child under a Wood lamp for possible spread of lesions. ANS: C Feedback A Corticosteroids are not indicated in bacterial infections. B Dressings are usually not indicated. The undermined skin, crusts, and debris are carefully removed after softening with moist compresses. C A major nursing consideration related to bacterial skin infections, such as impetigo contagiosa, is to prevent the spread of the infection and complications. This is done by thorough handwashing before and after contact with the affected child. D A Wood lamp is used to detect fluorescent materials in the skin and hair. It is used in certain disease states, such as tinea capitis. PTS: 1 DIF: Cognitive Level: Comprehension REF: p. 1306 OBJ: Nursing Process: Implementation MSC: Client Needs: Physiologic Integrity 18. Impetigo ordinarily results in a . No scarring b . Pigmented spots c . Slightly depressed scars d . Atrophic white scars ANS: A Feedback A Impetigo tends to heal without scarring unless a secondary infection occurs. B Hyperpigmentation may occur; however, only in dark skinned children. C No scarring usually occurs. D No scarring usually occurs. PTS: 1 DIF: Cognitive Level: Comprehension REF: p. 1305 OBJ: Nursing Process: Assessment MSC: Client Needs: Physiologic Integrity 19. The pediatric nurse understands that cellulitis is most often caused by a . Herpes zoster b . Candida albicans c . Human papillomavirus d . Streptococcus or Staphylococcus organisms ANS: D Feedback A Herpes zoster is the virus associated with varicella and shingles. B Candida albicans is associated with candidiasis or thrush. C Human papillomavirus is associated with various types of human warts. D Streptococcus, Staphylococcus, and Haemophilus influenzae are the organisms usually responsible for cellulitis. PTS: 1 DIF: Cognitive Level: Comprehension REF: p. 1307 OBJ: Nursing Process: Assessment MSC: Client Needs: Physiologic Integrity 20. The skin condition commonly known as warts is the result of an infection by which organism? a Bacteria . b Fungus . c Parasite . d Virus . ANS: D Feedback A Infection with these organisms does not result in warts. B Infection with these organisms does not result in warts. C Infection with these organisms does not result in warts. D Human warts are caused by the human papillomavirus. PTS: 1 DIF: Cognitive Level: Comprehension REF: p. 1320 | Table 49-1 OBJ: Nursing Process: Assessment MSC: Client Needs: Physiologic Integrity 21. The primary treatment for warts is a . Vaccination b . Local destruction c . Corticosteroids d . Specific antibiotic therapy ANS: B Feedback A Vaccination is prophylaxis for warts and is not a treatment. B Topical treatments include chemical cautery, which is especially useful for the treatment of warts. Local destructive therapy individualized according to location, type, and number. Surgical removal, electrocautery, curettage, cryotherapy, caustic solutions, x-ray treatment, and laser therapies are used. C These are not effective in the treatment of warts. D These are not effective in the treatment of warts. PTS: 1 DIF: Cognitive Level: Comprehension REF: p. 1320 | Table 49-1 OBJ: Nursing Process: Assessment MSC: Client Needs: Physiologic Integrity 22. Treatment for herpes simplex virus (types 1 or 2) includes a . Corticosteroids b . Oral griseofulvin c . Oral antiviral agent d . Topical and/or systemic antibiotic ANS: C Feedback A Corticosteroids are not effective for viral infections. B Griseofulvin is an antifungal agent and not effective for viral infections. C Oral antiviral agents are effective for viral infections such as herpes simplex. D Antibiotics are not effective in viral diseases. PTS: 1 DIF: Cognitive Level: Comprehension REF: p. 1313 OBJ: Nursing Process: Implementation MSC: Client Needs: Physiologic Integrity 23. Ringworm, frequently found in schoolchildren, is caused by a(n) a Virus . b Fungus . c Allergic reaction . d Bacterial infection . ANS: B Feedback A These are not the causative organisms for ringworm. B Ringworm is caused by a group of closely related filamentous fungi, which invade primarily the stratum corneum, hair, and nails. They are superficial infections that live on, not in, the skin. C Ringworm is not an allergic response. D These are not the causative organisms for ringworm. PTS: 1 DIF: Cognitive Level: Comprehension REF: p. 1310 OBJ: Nursing Process: Assessment MSC: Client Needs: Physiologic Integrity 24. The primary clinical manifestation of scabies is a Edema . b Redness . c Pruritus . d Maceration . ANS: C Feedback A Edema is not observed in scabies. B Redness is not observed in scabies. C Scabies is caused by the scabies mite. The inflammatory response and intense itching occur after the host has become sensitized to the mite. This occurs approximately 30 to 60 days after initial contact. In the previously sensitized person, the response occurs within 48 hours. D Maceration is not observed in scabies. PTS: 1 DIF: Cognitive Level: Comprehension REF: p. 1316 OBJ: Nursing Process: Assessment MSC: Client Needs: Physiologic Integrity 25. The management of a child who has just been stung by a bee or wasp should include the application of a . Cool compresses b . Warm compresses c . Antibiotic cream d . Corticosteroid cream ANS: A Feedback A Bee or wasp stings are initially treated by carefully removing the stinger, cleansing with soap and water, application of cool compresses, and the use of common household agents such as lemon juice or a paste made with aspirin and baking soda. B Warm compresses are avoided. C Antibiotic cream is unnecessary unless a secondary infection occurs. D Corticosteroid cream is not part of the initial therapy. If a severe reaction occurs, systemic corticosteroids may be indicated. Chapter 46 Nursing Care of Child With Alteration Cellular Regulation/Hematologic Neoplastic Disorder MULTIPLE CHOICE 1. What is the best response to a parent who asks the nurse whether her 5-month-old infant can have cows milk? a . You need to wait until she is 8 months old and eating solids well. b . Yes, if you think that she will eat enough meat to get the iron she needs. c . Infants younger than 12 months need iron-rich formula to get the iron they need. d . Try it and see how she tolerates it. ANS: C Feedback A A 5-month-old infant cannot get adequate iron without drinking an ironfortified formula or taking an iron supplement. B The American Academy of Pediatrics recommends beginning solid foods at 4 to 6 months of age. Meats are typically introduced in later infancy. Ironfortified formula is still recommended. C Infants younger than 12 months need iron-fortified formula or breast milk. Infants who drink cows milk do not get adequate iron and are at risk for iron deficiency anemia. D Counseling a parent to give a 5-month-old infant cows milk is inappropriate. PTS: 1 DIF: Cognitive Level: Application REF: p. 1243 OBJ: Nursing Process: Implementation MSC: Client Needs: Health Promotion and Maintenance 2. An assessment of a 7-month-old infant with a hemoglobin level of 6.5 mg/dL is likely to reveal an infant who is a . Lethargic, pale, and irritable b . Thin, energetic, and sleeps little c . Anorexic, vomiting, and has watery stools d . Flushed, fussy, and tired ANS: A Feedback A Pallor, lethargy, irritability, and tachycardia are clinical manifestations of iron deficiency anemia. A child with a hemoglobin level of 6.5 mg/dL has anemia. B A child with a hemoglobin level of 6.5 mg/dL has anemia. Infants with iron deficiency anemia are not typically thin and energetic but do tend to sleep a lot. C A child with a hemoglobin level of 6.5 mg/dL has anemia. Gastrointestinal symptoms are not clinical manifestations associated with iron deficiency anemia. D A child with a hemoglobin level of 6.5 mg/dL has anemia. Although the infant with iron deficiency anemia may be tired and fussy, pallor, rather than a flushed appearance, is characteristic of a low hemoglobin level. PTS: 1 DIF: Cognitive Level: Application REF: p. 1242 OBJ: Nursing Process: Assessment MSC: Client Needs: Physiologic Integrity 3. What action is not appropriate for a 14-month-old child with iron deficiency anemia? a . Decreasing the infants daily milk intake to 24 oz or less b . Giving oral iron supplements between meals with orange juice c . Including apricots, dark-green leafy vegetables, and egg yolk in the infants diet d . Allowing the infant to drink the iron supplement from a small medicine cup ANS: D Feedback A A daily milk intake in toddlers of less than 24 oz will encourage the consumption of iron-rich solid foods. B Because food interferes with the absorption of iron, iron supplements are taken between meals. Administering this medication with foods rich in vitamin C facilitates absorption of iron. C Apricots, dark-green leafy vegetables, and egg yolks are rich sources of iron. Other iron-rich foods include liver, dried beans, Cream of Wheat, iron-fortified cereal, and prunes. D Iron supplements should be administered through a straw or by a medicine dropper placed at the back of the mouth because iron temporarily stains the teeth. PTS: 1 DIF: Cognitive Level: Application REF: p. 1243 OBJ: Nursing Process: Planning MSC: Client Needs: Physiologic Integrity 4. An accurate description of anemia is a . Increased blood viscosity b . Depressed hematopoietic system c . Presence of abnormal hemoglobin d . Decreased oxygen-carrying capacity of blood ANS: D Feedback A Increased blood viscosity is usually a function of too many cells or of dehydration, not of anemia. B A depressed hematopoietic system or abnormal hemoglobin can contribute to anemia, but the definition is dependent on the deceased oxygen-carrying capacity of the blood. C A depressed hematopoietic system or abnormal hemoglobin can contribute to anemia, but the definition is dependent on the decreased oxygen-carrying capacity of the blood. D Anemia is a condition in which the number of red blood cells or hemoglobin concentration is reduced below the normal values for age. This results in a decreased oxygen-carrying capacity of blood. PTS: 1 DIF: Cognitive Level: Comprehension REF: p. 1243 OBJ: Nursing Process: Assessment MSC: Client Needs: Physiologic Integrity 5. What is true about the genetic transmission of sickle cell disease? a Both parents must carry the sickle cell trait. . b Both parents must have sickle cell disease. . c One parent must have the sickle cell trait. . d Sickle cell disease has no known pattern of inheritance. . ANS: A Feedback A In this scenario, there is a 50% risk of having a child with sickle cell disease. B The sickle cell trait, not the disease itself, must be present in the parents for the child to have the disease. C An autosomal recessive pattern of inheritance means that both parents must be carriers of the sickle cell trait. D Sickle cell disease is known to have an autosomal recessive pattern of inheritance. PTS: 1 DIF: Cognitive Level: Application REF: p. 1246 OBJ: Nursing Process: Assessment MSC: Client Needs: Health Promotion and Maintenance 6. A condition in which the normal adult hemoglobin is partly or completely replaced by abnormal hemoglobin is known as a Aplastic anemia . b Sickle cell anemia . c Thalassemia major . d Iron-deficiency anemia . ANS: B Feedback A Aplastic anemia is a lack of cellular elements being produced. B Sickle cell anemia is one of a group of diseases collectively called hemoglobinopathies, in which normal adult hemoglobin is replaced by an abnormal hemoglobin. C Hemophilia refers to a group of bleeding disorders in which there is deficiency of one of the factors necessary for coagulation. D Iron-deficiency anemia affects size and depth of color and does not involve an abnormal hemoglobin. PTS: 1 DIF: Cognitive Level: Comprehension REF: p. 1245 OBJ: Nursing Process: Assessment MSC: Client Needs: Physiologic Integrity 7. What are the nursing priorities for a child with sickle cell disease in vaso-occlusive crisis? a Administration of antibiotics and nebulizer treatments . b Hydration and pain management . c Blood transfusions and an increased calorie diet . d School work and diversion . ANS: B Feedback A Antibiotics may be given prophylactically. Oxygen therapy rather than nebulizer treatments is used to prevent further sickling. B Hydration and pain management decrease the cells oxygen demands and prevent sickling. C Although blood transfusions and increased calories may be indicated, they are not primary considerations for vaso-occlusive crisis. D School work and diversion are not major considerations when the child is in a vaso-occlusive crisis PTS: 1 DIF: Cognitive Level: Application REF: p. 1248 OBJ: Nursing Process: Planning MSC: Client Needs: Physiologic Integrity 8. What describes the pathologic changes of sickle cell anemia? a . Sickle-shaped cells carry excess oxygen. b . Sickle-shaped cells decrease blood viscosity. c . Increased red blood cell destruction occurs. d . Decreased red blood cell destruction occurs. ANS: C Feedback A Sickled red cells have decreased oxygen-carrying capacity and transform into the sickle shape in conditions of low oxygen tension. B When the sickle cells change shape, they increase the viscosity in the area where they are involved in the microcirculation. C The clinical features of sickle cell anemia are primarily the result of increased red blood cell destruction and obstruction caused by the sickle-shaped red blood cells. D Increased red blood cell destruction occurs. PTS: 1 DIF: Cognitive Level: Application REF: p. 1246 OBJ: Nursing Process: Diagnosis MSC: Client Needs: Physiologic Integrity 9. Which clinical manifestation should the nurse expect when a child with sickle cell anemia experiences an acute vaso-occlusive crisis? a . Circulatory collapse b . Cardiomegaly, systolic murmurs c . Hepatomegaly, intrahepatic cholestasis d . Painful swelling of hands and feet; painful joints ANS: D Feedback A Circulatory collapse results from sequestration crises. B Cardiomegaly, systolic murmurs, hepatomegaly, and intrahepatic cholestasis result from chronic vaso-occlusive phenomena. C Cardiomegaly, systolic murmurs, hepatomegaly, and intrahepatic cholestasis result from chronic vaso-occlusive phenomena. D A vaso-occlusive crisis is characterized by severe pain in the area of involvement. If in the extremities, painful swelling of the hands and feet is seen; if in the abdomen, severe pain resembles that of acute surgical abdomen; and if in the head, stroke and visual disturbances occur. PTS: 1 DIF: Cognitive Level: Comprehension REF: p. 1246 OBJ: Nursing Process: Diagnosis MSC: Client Needs: Physiologic Integrity 10. What should the discharge plan for a school-age child with sickle cell disease include? a . Restricting the childs participation in outside activities b . Administering aspirin for pain or fever c . Limiting the childs interaction with peers d . Administering penicillin daily as ordered ANS: D Feedback A Sickle cell disease does not prohibit the child from outdoor play. Active and passive exercises help promote circulation. B Aspirin use should be avoided. Acetaminophen or ibuprofen should be administered for fever or pain. C The child needs to interact with peers to meet his developmental needs. D Children with sickle cell disease are at a high risk for pneumococcal infections and should receive long-term penicillin therapy and preventive immunizations. PTS: 1 DIF: Cognitive Level: Application REF: p. 1246 OBJ: Nursing Process: Planning MSC: Client Needs: Physiologic Integrity 11. How should the nurse respond when asked by the mother of a child with beta-thalassemia why the child is receiving deferoxamine? a To improve the anemia. . b . To decrease liver and spleen swelling. c . To eliminate excessive iron being stored in the organs. d . To prepare your child for a bone marrow transplant. ANS: C Feedback A Chronic transfusion therapy is the treatment for anemia. Deferoxamine is administered to prevent complications from repeated transfusions. B Deferoxamine is used to prevent organ damage, not as a treatment for existing conditions such as hepatosplenomegaly. C Multiple transfusions result in hemosiderosis. Deferoxamine is given to chelate iron and prevent organ damage. D Preparation for a bone marrow transplant does not include administration of deferoxamine. PTS: 1 DIF: Cognitive Level: Application REF: p. 1251 OBJ: Nursing Process: Implementation MSC: Client Needs: Physiologic Integrity 12. Which statement best describes beta-thalassemia major (Cooley anemia)? a All formed elements of the blood are depressed. . b Inadequate numbers of red blood cells are present. . c Increased incidence occurs in families of Mediterranean extraction. . d Increased incidence occurs in persons of West African descent. . ANS: C Feedback A An overproduction of red cells occurs. Although numerous, the red cells are relatively unstable. B An overproduction of red cells occurs. Although numerous, the red cells are relatively unstable. C Individuals who live near the Mediterranean Sea and their descendants have the highest incidence of thalassemia. D Sickle cell disease is common in blacks of West African descent. PTS: 1 DIF: Cognitive Level: Comprehension REF: p. 1250 OBJ: Nursing Process: Assessment MSC: Client Needs: Physiologic Integrity 13. What is the priority nursing intervention for a child hospitalized with hemarthrosis resulting from hemophilia? a Immobilization and elevation of the affected joint . b Administration of acetaminophen for pain relief . c Assessment of the childs response to hospitalization . d Assessment of the impact of hospitalization on the family system . ANS: A Feedback A Immobilization and elevation of the joint will prevent further injury until bleeding is resolved. B Although acetaminophen may help with pain associated with the treatment of hemarthrosis, it is not the priority nursing intervention. C Assessment of a childs response to hospitalization is relevant to all hospitalized children; however, in this situation, psychosocial concerns are secondary to physiologic concerns. A priority nursing concern for this child is the management of hemarthrosis. D Assessing the impact of hospitalization on the family system is relevant to all hospitalized children, but it is not the priority in this situation. PTS: 1 DIF: Cognitive Level: Application REF: p. 1253 OBJ: Nursing Process: Implementation MSC: Client Needs: Physiologic Integrity 14. What is descriptive of most cases of hemophilia? a . Autosomal dominant disorder causing deficiency is a factor involved in the blood-clotting reaction b . X-linked recessive inherited disorder causing deficiency of platelets and prolonged bleeding c . X-linked recessive inherited disorder in which a blood-clotting factor is deficient d . Y-linked recessive inherited disorder in which the red blood cells become moon shaped ANS: C Feedback A The inheritance pattern is X-linked recessive. B The disorder involves coagulation factors, not platelets. C The inheritance pattern in 80% of all of the cases of hemophilia is X-linked recessive. The two most common forms of the disorder are factor VIII deficiency, hemophilia A or classic hemophilia; and factor IX deficiency, hemophilia B or Christmas disease. D The disorder does not involve red cells or the Y chromosome. PTS: 1 DIF: Cognitive Level: Comprehension REF: p. 1252 OBJ: Nursing Process: Diagnosis MSC: Client Needs: Physiologic Integrity 15. The mother of a child with hemophilia asks the nurse how long her child will need to be treated for hemophilia. What is the best response to this question? a . Hemophilia is a lifelong blood disorder. b . There is a 25% chance that your child will have spontaneous remission and treatment will no longer be necessary. c . Treatment is indicated until after your child has progressed through the toddler years. d . It is unlikely that your child will need to be treated for his hemophilia because your first child does not have the disease. ANS: A Feedback A Hemophilia is a lifelong hereditary blood disorder with no cure. Prevention by avoiding activities that induce bleeding and by treatment is lifelong. The management of hemophilia is highly individual and depends on the severity of the illness. B This is an untrue statement. Hemophilia is a lifelong hereditary blood disorder with no cure. Treatment is lifelong. C This is an untrue statement. Hemophilia is a lifelong hereditary blood disorder with no cure. Treatment is lifelong. D Because hemophilia has an X chromosomelinked recessive inheritance, there is a risk with each pregnancy that a child will either have the disease or be a carrier. Hemophilia is a life-long hereditary blood disorder with no cure. Treatment is lifelong. PTS: 1 DIF: Cognitive Level: Application REF: p. 1252 OBJ: Nursing Process: Implementation MSC: Client Needs: Health Promotion and Maintenance 16. In teaching family members about their childs von Willebrand disease, what is the priority outcome for the child that the nurse should discuss? a . Prevention of injury b . Maintaining adequate hydration c . Compliance with chronic transfusion therapy d . Prevention of respiratory infections ANS: A Feedback A Hemorrhage as a result of injury is the childs greatest threat to life. B Fluid volume status becomes a concern when hemorrhage has occurred. C The treatment of von Willebrand disease is desmopressin acetate (DDAVP), which is administered intranasally or intravenously. D Respiratory infections do not constitute a major threat to the child with von Willebrand disease. PTS: 1 DIF: Cognitive Level: Application REF: p. 1256 OBJ: Nursing Process: Evaluation MSC: Client Needs: Health Promotion and Maintenance 17. A child who has been in good health has a platelet count of 45,000/mm3, petechiae, and excessive bruising that covers the body. The nurse is aware that these signs are clinical manifestations of a . Erythroblastopenia b . von Willebrand disease c . Hemophilia d . Immune thrombocytopenic purpura (ITP) ANS: D Feedback A The clinical manifestations of erythroblastopenia are pallor, lethargy, headache, fainting, and a history of upper respiratory infection. B The clinical manifestations of von Willebrand disease are bleeding from the gums or nose, prolonged bleeding from cuts, and excessive bleeding after surgery or trauma. C Bleeding is the clinical manifestation of hemophilia and results from a deficiency of normal factor activity necessary to produce blood clotting. D Excessive bruising and petechiae, especially involving the mucous membranes and gums in a child who is otherwise healthy, are the clinical manifestations of ITP, resulting from decreased platelets. The etiology of ITP is unknown, but it is considered to be an autoimmune process. PTS: 1 DIF: Cognitive Level: Application REF: p. 1256 OBJ: Nursing Process: Assessment MSC: Client Needs: Physiologic Integrity 18. What is the priority in the discharge plan for a child with immune thrombocytopenic purpura (ITP)? a . Teaching the parents to report excessive fatigue to the physician b . Monitoring the childs hemoglobin level every 2 weeks c . Providing a diet that contains iron-rich foods d . Establishing a safe, age-appropriate home environment ANS: D Feedback A Excessive fatigue is not a significant problem for the child with ITP. B ITP is associated with low platelet levels. C Increasing the childs intake of iron in the diet will not correct ITP. D Prevention of injury is a priority concern for a child with ITP. Chapter 47 Nursing Care of the Child With an Alteration in Immunity or Immunologic Disorder MULTIPLE CHOICE 1. A nurse in a well-child clinic is teaching parents about their childs immune system. Which statement by the nurse is correct? a The immune system distinguishes and actively protects the bodys own cells . from foreign substances. b The immune system is fully developed by 1 year of age. . c The immune system protects the child against communicable diseases in the . first 6 years of life. d The immune system responds to an offending agent by producing antigens. . ANS: A Feedback A The immune system responds to foreign substances, or antigens, by producing antibodies and storing information. Intact skin, mucous membranes, and processes such as coughing, sneezing, and tearing help maintain internal homeostasis. B Children up to age 6 or 7 years have limited antibodies against common bacteria. The immunoglobulins reach adult levels at different ages. C Immunization is the basis from which the immune system activates protection against some communicable diseases. D Antibodies are produced by the immune system against invading agents, or antigens. PTS: 1 DIF: Cognitive Level: Application REF: p. 1039 OBJ: Nursing Process: Implementation MSC: Client Needs: Health Promotion and Maintenance 2. A nurse is teaching parents about the importance of immunizations for infants because of immaturity of the immune system. The parents demonstrate that they understand the teaching if they make which statement? a . The spleen reaches full size by 1 year of age. b . IgM, IgE, and IgD levels are high at birth. c . IgG levels in the newborn infant are low at birth. d Absolute lymphocyte counts reach a peak during the first year. . ANS: D Feedback A The spleen reaches its full size during adulthood. B IgM, IgE, and IgD are normally in low concentration at birth. IgM, IgE, IgA, and IgD do not cross the placenta. C The term newborn infant receives an adult level of IgG as a result of transplacental transfer from the mother. D Absolute lymphocyte counts reach a peak during the first year. PTS: 1 DIF: Cognitive Level: Analysis REF: p. 1040 OBJ: Nursing Process: Evaluation MSC: Client Needs: Health Promotion and Maintenance 3. Which organs and tissues control the two types of specific immune functions? a . The spleen and mucous membranes b . Upper and lower intestinal lymphoid tissue c . The skin and lymph nodes d . The thymus and bone marrow ANS: D Feedback A Both the spleen and mucous membranes are secondary organs of the immune system that act as filters to remove debris and antigens and foster contact with T lymphocytes. B Gut-associated lymphoid tissue is a secondary organ of the immune system. This tissue filters antigens entering the gastrointestinal tract. C The skin and lymph nodes are secondary organs of the immune system. D The thymus controls cell-mediated immunity (cells that mature into T lymphocytes). The bone marrow controls humoral immunity (stem cells for B lymphocytes). PTS: 1 DIF: Cognitive Level: Comprehension REF: p. 1039 OBJ: Nursing Process: Assessment MSC: Client Needs: Physiologic Integrity 4. Which statement is true regarding how infants acquire immunity? a . The infant acquires humoral and cell-mediated immunity in response to infections and immunizations. b . The infant acquires maternal antibodies that ensure immunity up to 12 months age. c . Active immunity is acquired from the mother and lasts 6 to 7 months. d . Passive immunity develops in response to immunizations. ANS: A Feedback A Infants acquire long-term active immunity from exposure to antigens and vaccines. Immunity is acquired actively and passively. B The term infants passive immunity is acquired from the mother and begins to dissipate during the first 6 to 8 months of life. C Passive immunity is acquired from the mother. D Active immunity develops in response to immunizations. PTS: 1 DIF: Cognitive Level: Comprehension REF: p. 1042 OBJ: Nursing Process: Assessment MSC: Client Needs: Physiologic Integrity 5. What is the most common mode of transmission of human immunodeficiency virus (HIV) in the pediatric population? a . Perinatal transmission b . Sexual abuse c . Blood transfusions d . Poor handwashing ANS: A Feedback A Perinatal transmission accounts for the highest percentage (91%) of HIV infections in children. Infected women can transmit the virus to their infants across the placenta during pregnancy, at delivery, and through breastfeeding. B Cases of HIV infection from sexual abuse have been reported; however, perinatal transmission accounts for most pediatric HIV infections. C Although in the past some children became infected with HIV through blood transfusions, improved laboratory screening has significantly reduced the probability of contracting HIV from blood products. D Poor handwashing is not an etiology of HIV infection. PTS: 1 DIF: Cognitive Level: Application REF: p. 1045 OBJ: Nursing Process: Planning MSC: Client Needs: Physiologic Integrity 6. The Center for Disease Control (CDC, 2009) recommendation for immunizing infants who are HIV positive is a . Follow the routine immunization schedule. b . Routine immunizations are administered; assess CD4+ counts before administering the MMR and varicella vaccinations. c . Do not give immunizations because of the infants altered immune status. d . Eliminate the pertussis vaccination because of the risk of convulsions. ANS: B Feedback A Routine immunizations are appropriate; however, CD4+ cell counts should be assessed before administering the MMR and varicella vaccines to establish adequate immune system function. B Routine immunizations are appropriate. CD4+ cells are monitored when deciding whether to provide live virus vaccines. If the child is severely immunocompromised, the MMR vaccine is not given. The varicella vaccine can be considered on the basis of the childs CD4+ counts. Only inactivated polio virus (IPV) should be used for HIV-infected children. C Immunizations are given to infants who are HIV positive. D The pertussis vaccination is not eliminated for an infant who is HIV positive. PTS: 1 DIF: Cognitive Level: Comprehension REF: p. 1052 | Table 42-2 OBJ: Nursing Process: Implementation MSC: Client Needs: Health Promotion and Maintenance 7. Which suggestion is appropriate to teach a mother who has a preschool child who refuses to take the medications for HIV infection? a . Mix medications with chocolate syrup or follow with chocolate candy. b . Mix the medications with milk or an essential food. c . Skip the dose of medication if the child protests too much. d . Mix the medication in a syringe, hold the child down firmly, and administer the medication. ANS: A Feedback A Liquid forms of HIV medications may be foul tasting or have a gritty texture. Chocolate will help to make these foods more palatable and is liked by most children. B Medications should be mixed with nonessential foods. C Doses of medication should never be skipped. D Fighting with the child or using force should be avoided. A nonessential food that will make the taste of the medication more palatable for the child should be the correct action. The administration of medications for the child with HIV becomes part of the familys everyday routine for years. PTS: 1 DIF: Cognitive Level: Application REF: p. 1051 OBJ: Nursing Process: Implementation MSC: Client Needs: Physiologic Integrity 8. What is the primary nursing concern for a hospitalized child with HIV infection? a Maintaining growth and development . b . Eating foods that the family brings to the child c . Consideration of parental limitations and weaknesses d . Resting for 2 to 3 hours twice a day ANS: A Feedback A Maintaining growth and development is a major concern for the child with HIV infection. Frequent monitoring for failure to thrive, neurologic deterioration, or developmental delay is important for HIV-infected infants and children. B Nutrition, which contributes to a childs growth, is a nursing concern; however, it is not necessary for family members to bring food to the child. C Although an assessment of parental strengths and weaknesses is important, it will be imperative for health care providers to focus on the parental strengths, not weaknesses. This is not as important as the frequent assessment of the childs growth and development. D Rest is a nursing concern, but it is not as high a priority as maintaining growth and development. Rest periods twice a day for 2 to 3 hours may not be appropriate. PTS: 1 DIF: Cognitive Level: Application REF: p. 1047 OBJ: Nursing Process: Evaluation MSC: Client Needs: Physiologic Integrity 9. What should the nurse include in a teaching plan for the mother of a toddler who will be taking prednisone for several months? a The medication should be taken between meals. . b . The medication needs to be discontinued because of the risks associated with long-term usage. c . The medication should not be stopped abruptly. d . The medication may lower blood glucose, so the mother needs to observe for signs of hypoglycemia. ANS: C Feedback A Prednisone should be taken with food to minimize or prevent gastrointestinal bleeding. B Although there are adverse effects from long-term steroid use, the medication must not be discontinued without consulting a physician. Acute adrenal insufficiency can occur if the medication is withdrawn abruptly. The dosage needs to be tapered. C The dosage must be tapered before the drug is discontinued to allow the gradual return of function in the pituitary-adrenal axis. D The medication puts the child at risk for hyperglycemia. PTS: 1 DIF: Cognitive Level: Application REF: p. 1056 OBJ: Nursing Process: Planning MSC: Client Needs: Physiologic Integrity 10. Children receiving long-term systemic corticosteroid therapy are most at risk for a . Hypotension b . Dilation of blood vessels in the cheeks c . Growth delays d Decreased appetite and weight loss . ANS: C Feedback A Hypertension is a clinical manifestation of long-term systemic steroid administration. B Dilation of blood vessels in the cheeks is associated with an excess of topically administered steroids. C Growth delay is associated with long-term steroid use. D Increased appetite and weight gain are clinical manifestations of excess systemic corticosteroid therapy. PTS: 1 DIF: Cognitive Level: Comprehension REF: p. 1055 OBJ: Nursing Process: Assessment MSC: Client Needs: Physiologic Integrity 11. Which statement by a mother about antiretroviral agents for the management for her 5-yearold child with acquired immunodeficiency syndrome (AIDS) indicates that she has a good understanding? a . When my childs pain increases, I double the recommended dosage of antiretroviral medication. b . Addiction is a risk, so I only use the medication as ordered. c . Doses of the antiretroviral medication are selected on the basis of my childs age and growth. d . By the time my child is an adolescent she will not need her antiretroviral medications any longer. ANS: C Feedback A Antiretroviral medications are not administered for pain relief. Doubling the recommended dosage of any medication is not appropriate without an order from the physician. B Addiction is not a realistic concern with antiretroviral medications. C Doses of antiretroviral medication to treat HIV infection for infants and children are based on individualized age and growth considerations. D Antiretroviral medications are still needed during adolescence. Doses for adolescents are based on pubertal status by Tanner staging. PTS: 1 DIF: Cognitive Level: Analysis REF: p. 1048 OBJ: Nursing Process: Implementation MSC: Client Needs: Physiologic Integrity 12. Which intervention is appropriate for a child receiving high doses of steroids? a . Limit activity and receive home schooling. b . Decrease the amount of potassium in the diet. c . Substitute a killed virus vaccine for live virus vaccines. d . Monitor for seizure activity. ANS: C Feedback A Limiting activity and home schooling are not routine for a child receiving high doses of steroids. B The child receiving steroids is at risk for hypokalemia and needs potassium in the diet. C The child on high doses of steroids should not receive live virus vaccines because of immunosuppression. D Children on steroids are not typically at risk for seizures. PTS: 1 DIF: Cognitive Level: Application REF: p. 1055 OBJ: Nursing Process: Implementation MSC: Client Needs: Health Promotion and Maintenance 13. The nurse observes a red butterfly-shaped rash that spreads across the childs cheeks and nose. This assessment finding is characteristic of which condition? a . Systemic lupus erythematosus (SLE) b . Rheumatic fever c . Kawasaki disease d . Anaphylactic reaction ANS: A Feedback A A red, flat or raised malar butterfly rash over the cheeks and bridge of the nose is a clinical manifestation of SLE. B A major manifestation of rheumatic fever is erythema marginatum, which appears as red skin lesions spread peripherally over the trunk. C An erythematous rash, induration of the hands and feet, and erythema of the palms and soles are manifestations of Kawasaki disease. D Initial symptoms of anaphylaxis include severe itching and rapid development of erythema. PTS: 1 DIF: Cognitive Level: Application REF: p. 1057 OBJ: Nursing Process: Assessment MSC: Client Needs: Physiologic Integrity 14. What is the primary nursing concern for a child having an anaphylactic reaction? a . Identifying the offending allergen b . Ineffective breathing pattern c . Increased cardiac output d . Positioning to facilitate comfort ANS: B Feedback A Determining the cause of an anaphylactic reaction is important to implement the appropriate treatment, but the primary concern is the airway. B Laryngospasms resulting in ineffective breathing patterns is a life-threatening manifestation of anaphylaxis. The primary action is to assess airway patency, respiratory rate and effort, level of consciousness, oxygen saturation, and urine output. C During anaphylaxis, the cardiac output is decreased. D During the acute period of anaphylaxis, the nurses primary concern is the childs breathing. Positioning for comfort is not a primary concern during a crisis. PTS: 1 DIF: Cognitive Level: Application REF: p. 1060 OBJ: Nursing Process: Assessment MSC: Client Needs: Physiologic Integrity 15. What is the drug of choice the nurse should administer in the acute treatment of anaphylaxis? a . Diphenhydramine b . Histamine inhibitor (cimetidine) c . Epinephrine d . Albuterol ANS: C Feedback A Although diphenhydramine may be indicated, epinephrine is the first drug of choice in the immediate treatment of anaphylaxis. B Although a histamine inhibitor such as cimetidine may be indicated, epinephrine is the first drug of choice in immediate treatment of anaphylaxis. C Epinephrine is the first drug of choice in immediate treatment of anaphylaxis. Treatment must be initiated immediately because it may only be a matter of minutes before shock occurs. D Albuterol is not usually indicated for treatment of anaphylaxis. PTS: 1 DIF: Cognitive Level: Application REF: p. 1064 OBJ: Nursing Process: Implementation MSC: Client Needs: Physiologic Integrity 16. What is caused by a virus that primarily infects a specific subset of T lymphocytes, the CD4+ T cells? a . Wiskott-Aldrich syndrome b . Idiopathic thrombocytopenic purpura c . Acquired immunodeficiency syndrome (AIDS) d . Severe combined immunodeficiency disease ANS: C Feedback A Wiskott-Aldrich syndrome is not a viral illness. B Idiopathic thrombocytopenic purpura is not a viral illness. C Acquired immune deficiency is caused by the human immunodeficiency virus (HIV), which primarily attacks the CD4+ T cells. D Severe combined immunodeficiency disease is not a viral illness. Chapter 48 Nursing Care of the Child With an Alteration in Metabolism/Endocrine Disorder MULTIPLE CHOICE 1. New parents ask the nurse, Why is it necessary for our baby to have the newborn blood test? The nurse explains that the priority outcome of mandatory newborn screening for inborn errors of metabolism is a . Appropriate community referral for affected infants b . Parental education about raising a special needs child c . Early identification of serious genetically transmitted metabolic diseases d Early identification of electrolyte imbalances . ANS: C Feedback A Community referral is appropriate after a diagnosis is made. B With early identification and treatment, serious complications such as intellectual impairment are prevented. C Early identification of hypothyroidism is basic to the prevention of intellectual impairment in the child. D Although electrolyte imbalances could occur with some of the inborn errors of metabolism, this is not the priority outcome, nor would the newborn screen detect electrolyte imbalances. PTS: 1 DIF: Cognitive Level: Application REF: p. 1381 OBJ: Nursing Process: Planning MSC: Client Needs: Health Promotion and Maintenance 2. What is the priority nursing goal for a 14-year-old with Graves disease? a . Relieving constipation b . Allowing the adolescent to make decisions about whether or not to take medication c . Verbalizing the importance of adherence to the medication regimen d . Developing alternative educational goals ANS: C Feedback A The adolescent with Graves disease is not constipated. B Adherence to the medication schedule is important to ensure optimal health and wellness. Medications should not be skipped and dose regimens should not be tapered by the child without consultation with the childs medical provider. C To adhere to the medication schedule, children need to understand that the medication must be taken two or three times per day. D The management of Graves disease does not interfere with school attendance and does not require alternative educational plans. PTS: 1 DIF: Cognitive Level: Analysis REF: p. 1387 OBJ: Nursing Process: Planning MSC: Client Needs: Health Promotion and Maintenance 3. What information provided by the nurse would be helpful to a 15-year-old adolescent taking methimazole three times a day? a . Pill dispensers and alarms on her watch can remind her to take the medication as ordered. b . She can take the medication when she is nervous and feels she needs it. c . She can take two pills before school and one pill at dinner, which will be easier for her to remember. d . Her mother can be responsible for reminding her when it is time to take her medication. ANS: A Feedback A Methimazole is an antithyroid medication that should be taken three times a day. Reminders will facilitate taking medication as ordered. B This medication needs to be taken regularly, not on an as-needed basis. C The dosage cannot be combined to reduce the frequency of administration. D Because of the adolescents school schedule and activities, she, rather than her mother, needs to be responsible for her medication. PTS: 1 DIF: Cognitive Level: Comprehension REF: p. 1386 OBJ: Nursing Process: Evaluation MSC: Client Needs: Health Promotion and Maintenance 4. Diabetes insipidus is a disorder of the a . Anterior pituitary b . Posterior pituitary c . Adrenal cortex d . Adrenal medulla ANS: B Feedback A The anterior pituitary produces hormones such as growth hormone, thyroidstimulating hormone, adrenocorticotropic hormone, gonadotropin, prolactin, and melanocyte-stimulating hormone. B The principal disorder of posterior pituitary hypofunction is diabetes insipidus. C The adrenal cortex produces aldosterone, sex hormones, and glucocorticoids. D The adrenal medulla produces catecholamines. PTS: 1 DIF: Cognitive Level: Comprehension REF: p. 1389 OBJ: Nursing Process: Assessment MSC: Client Needs: Physiologic Integrity 5. Which sign, when exhibited by a hospitalized child, should the nurse recognize as a characteristic of diabetes insipidus? a . Weight gain b . Increased urine specific gravity c . Increased urination d . Serum sodium level of 130 mEq/L ANS: C Feedback A Weight gain results from retention of water when there is an excessive production of antidiuretic hormone; in diabetes insipidus there is a decreased production of antidiuretic hormone. B Concentrated urine is a sign of the syndrome of inappropriate antidiuretic hormone (SIADH), in which there is an excessive production of antidiuretic hormone. C The deficiency of antidiuretic hormone associated with diabetes insipidus causes the body to excrete large volumes of dilute urine. D A deficiency of antidiuretic hormone, as with diabetes insipidus, results in an increased serum sodium concentration (greater than 145 mEq/L). PTS: 1 DIF: Cognitive Level: Application REF: p. 1388 OBJ: Nursing Process: Assessment MSC: Client Needs: Physiologic Integrity 6. What should the nurse include in the teaching plan for parents of a child with diabetes insipidus who is receiving DDAVP? a . Increase the dosage of DDAVP as the urine specific gravity (SG) increases. b . Give DDAVP only if urine output decreases. c . The child should have free access to water and toilet facilities at school. d . Cleanse skin before administering the transdermal patch. ANS: C Feedback A DDAVP needs to be given as ordered by the physician. If the parents are monitoring urine SG at home, they would not increase the medication dose for increased SG; the physician may order an increased dosage for very dilute urine with decreased SG. B DDAVP needs to be given continuously as ordered by the physician. C The childs teachers should be aware of the diagnosis, and the child should have free access to water and toilet facilities at school. D DDAVP is typically given intranasally or by subcutaneous injection. For nocturnal enuresis, it may be given orally. PTS: 1 DIF: Cognitive Level: Comprehension REF: p. 1389 OBJ: Nursing Process: Planning MSC: Client Needs: Health Promotion and Maintenance 7. A child with GH deficiency is receiving GH therapy. What is the best time for the GH to be administered? a At bedtime . b . After meals c . Before meals d . On arising in the morning ANS: A Feedback A Injections are best given at bedtime to more closely approximate the physiologic release of GH. B This time does not mimic the physiologic release of the hormone. C This time does not mimic the physiologic release of the hormone. D This time does not mimic the physiologic release of the hormone. PTS: 1 DIF: Cognitive Level: Application REF: p. 1394 OBJ: Nursing Process: Implementation MSC: Client Needs: Physiologic Integrity 8. A nurse is explaining growth hormone deficiency to parents of a child admitted to rule out this problem. Which metabolic alteration that is related to growth hormone deficiency should the nurse explain to the parent? a Hypocalcemia . b Hypoglycemia . c Diabetes insipidus. . d Hyperglycemia . ANS: B Feedback A Symptoms of hypocalcemia are associated with hypoparathyroidism. B Growth hormone helps maintain blood sugar at normal levels. C Diabetes insipidus is a disorder of the posterior pituitary. Growth hormone is produced by the anterior pituitary. D Hyperglycemia results from an insufficiency of insulin, which is produced by the beta cells in the islets of Langerhans in the pancreas. PTS: 1 DIF: Cognitive Level: Application REF: p. 1393 OBJ: Nursing Process: Implementation MSC: Client Needs: Physiologic Integrity 9. At what age is sexual development in boys and girls considered to be precocious? a Boys, 11 years; girls, 9 years . b Boys, 12 years; girls, 10 years . c Boys, 9 years; girls, 8 years . d Boys, 10 years; girls, 9 1/2 years . ANS: C Feedback A These ages fall within the expected range of pubertal onset. B These ages fall within the expected range of pubertal onset. C Manifestations of sexual development before age 9 in boys and age 8 in girls is considered precocious and should be investigated. D These ages fall within the expected range of pubertal onset. PTS: 1 DIF: Cognitive Level: Comprehension REF: p. 1390 OBJ: Nursing Process: Assessment MSC: Client Needs: Physiologic Integrity 10. What is the most appropriate intervention for the parents of a 6-year-old child with precocious puberty? a . Advise the parents to consider birth control for their daughter. b . Explain the importance of having the child foster relationships with same-age peers. c . Assure the childs parents that there is no increased risk for sexual abuse because of her appearance. d . Counsel parents that there is no treatment currently available for this disorder. ANS: B Feedback A Advising the parents of a 6-year-old to put their daughter on birth control is not appropriate and will not reverse the effects of precocious puberty. B Despite the childs appearance, the child needs to be treated according to her chronologic age and to interact with children in the same age-group. An expected outcome is that the child will adjust socially by exhibiting ageappropriate behaviors and social interactions. C Parents need to be aware that there is an increased risk of sexual abuse for a child with precocious puberty. D Treatment for precocious puberty is the administration of gonadotropinreleasing hormone blocker, which slows or reverses the development of secondary sexual characteristics and slows rapid growth and bone aging. PTS: 1 DIF: Cognitive Level: Application REF: p. 1393 OBJ: Nursing Process: Implementation MSC: Client Needs: Health Promotion and Maintenance 11. A neonate is displaying mottled skin, has a large fontanel and tongue, is lethargic, and is having difficulty feeding. The nurse recognizes that this is most suggestive of a . Hypocalcemia b . Hypothyroidism c . Hypoglycemia d . Phenylketonuria (PKU) ANS: B Feedback A When hypocalcemia is present, neonates may display twitching, tremors, irritability, jitteriness, electrocardiographic changes, and, rarely, seizures. B An infant with hypothyroidism may exhibit skin mottling, a large fontanel, a large tongue, hypotonia, slow reflexes, a distended abdomen, prolonged jaundice, lethargy, constipation, feeding problems, and coldness to touch. C Hypoglycemia causes the neonate to exhibit jitteriness, poor feeding, lethargy, seizures, respiratory alterations including apnea, hypotonia, high-pitched cry, bradycardia, cyanosis, and temperature instability. D Infants with PKU may initially have digestive problems with vomiting, and they may have a musty or mousy odor to the urine, infantile eczema, hypertonia, and hyperactive behavior. PTS: 1 DIF: Cognitive Level: Application REF: p. 1383 OBJ: Nursing Process: Assessment MSC: Client Needs: Physiologic Integrity 12. A common clinical manifestation of juvenile hypothyroidism is a Insomnia . b Diarrhea . c Dry skin . d Accelerated growth . ANS: C Feedback A Children with hypothyroidism are usually sleepy. B Constipation is associated with hypothyroidism. C Thick, dry skin, mental decline, cold intolerance, and weight gain are associated with juvenile hypothyroidism. D Decelerated growth is common in juvenile hypothyroidism. PTS: 1 DIF: Cognitive Level: Comprehension REF: p. 1385 OBJ: Nursing Process: Assessment MSC: Client Needs: Physiologic Integrity 13. A goiter is an enlargement or hypertrophy of which gland? a . Thyroid b . Adrenal c . Anterior pituitary d . Posterior pituitary ANS: A Feedback A A goiter is an enlargement or hypertrophy of the thyroid gland. B Goiter is not associated with this secretory organ. C Goiter is not associated with this secretory organ. D Goiter is not associated with this secretory organ. PTS: 1 DIF: Cognitive Level: Comprehension REF: p. 1386 OBJ: Nursing Process: Diagnosis MSC: Client Needs: Physiologic Integrity 14. Exophthalmos (protruding eyeballs) may occur in children with which condition? a . Hypothyroidism b . Hyperthyroidism c . Hypoparathyroidism d Hyperparathyroidism . ANS: B Feedback A Hypothyroidism is not associated with exophthalmos. B Exophthalmos is a clinical manifestation of hyperthyroidism. C Hypoparathyroidism is not associated with exophthalmos. D Hyperparathyroidism is not associated with exophthalmos. PTS: 1 DIF: Cognitive Level: Comprehension REF: p. 1385 | Box 51-1 OBJ: Nursing Process: Assessment MSC: Client Needs: Physiologic Integrity 15. A neonate born with ambiguous genitalia is diagnosed with congenital adrenogenital hyperplasia. Therapeutic management includes administration of a . Vitamin D b . Cortisone c . Stool softeners d . Calcium carbonate ANS: B Feedback A Vitamin D has no role in the therapy of adrenogenital hyperplasia. B The most common biochemical defect with congenital adrenal hyperplasia is partial or complete 21-hydroxylase deficiency. With complete deficiency, insufficient amounts of aldosterone and cortisol are produced so that circulatory collapse occurs without immediate replacement. C Stool softeners have no role in the therapy of adrenogenital hyperplasia. D Calcium carbonate has no role in the therapy of adrenogenital hyperplasia. PTS: 1 DIF: Cognitive Level: Analysis REF: p. 1382 OBJ: Nursing Process: Implementation MSC: Client Needs: Physiologic Integrity 16. Type 1 diabetes mellitus is suspected in an adolescent. Which clinical manifestation may be present? a . Moist skin b . Weight gain c . Fluid overload d . Blurred vision ANS: D Feedback A Dry skin, weight loss, and dehydration are clinical manifestations of type 1 diabetes mellitus. B Dry skin, weight loss, and dehydration are clinical manifestations of type 1 diabetes mellitus. C Dry skin, weight loss, and dehydration are clinical manifestations of type 1 diabetes mellitus. D Fatigue and blurred vision are clinical manifestations of type 1 diabetes mellitus. PTS: 1 DIF: Cognitive Level: Comprehension REF: p. 1395 OBJ: Nursing Process: Diagnosis MSC: Client Needs: Physiologic Integrity 17. A parent asks the nurse why self-monitoring of blood glucose is being recommended for her child with diabetes. The nurse should base the explanation on the knowledge that a . It is a less expensive method of testing. b . It is not as accurate as laboratory testing. c . Children are better able to manage the diabetes. d . The parents are better able to manage the disease. ANS: C Feedback A Blood glucose monitoring is more expensive but provides improved management. B It is as accurate as equivalent testing done in laboratories. C Blood glucose self-management has improved diabetes management and can be used successfully by children from the time of diagnosis. Insulin dosages can be adjusted based on blood sugar results. D The ability to self-test allows the child to balance diet, exercise, and insulin. The parents are partners in the process, but the child should be taught how to manage the disease. PTS: 1 DIF: Cognitive Level: Analysis REF: p. 1400 OBJ: Nursing Process: Implementation MSC: Client Needs: Physiologic Integrity 18. What is the best time for the nurse to assess the peak effectiveness of subcutaneously administered Regular insulin? a . Two hours after administration b . Four hours after administration c . Immediately after administration d . Thirty minutes after administration ANS: A Feedback A The peak action for Regular (short-acting) insulin is 2 to 3 hours after subcutaneous administration. B The duration of Regular (short-acting) insulin is only 3 to 6 hours. Peak action occurs 2 to 3 hours after the insulin is administered. C Subcutaneously administered Regular (short-acting) insulin has an onset of action of 30 to 60 minutes after injection. The effectiveness of subcutaneously administered, short-acting insulin cannot be assessed immediately after administration. D Thirty minutes corresponds to the onset of action for Regular (short-acting) insulin. PTS: 1 DIF: Cognitive Level: Application REF: p. 1398 | Table 51-4 OBJ: Nursing Process: Evaluation MSC: Client Needs: Physiologic Integrity 19. What is the primary concern for a 7-year-old child with type 1 diabetes mellitus who asks his mother not to tell anyone at school that he has diabetes? a . The childs safety b . The privacy of the child c . Development of a sense of industry d . Peer group acceptance ANS: A Feedback A Safety is the primary issue. School personnel need to be aware of the signs and symptoms of hypoglycemia and hyperglycemia and the appropriate interventions. B Privacy is not a life-threatening concern. C The treatment of type 1 diabetes should not interfere with the school-age childs development of a sense of industry. D Peer group acceptance, along with body image, are issues for the early adolescent with type 1 diabetes. This is not of greater priority than the childs safety. PTS: 1 DIF: Cognitive Level: Analysis REF: p. 1400 OBJ: Nursing Process: Evaluation MSC: Client Needs: Health Promotion and Maintenance 20. What is the best nursing action when a child with type 1 diabetes mellitus is sweating, trembling, and pale? a . Offer the child a glass of water. b . Give the child 5 units of regular insulin subcutaneously. c . Give the child a glass of orange juice. d . Give the child glucagon subcutaneously. ANS: C Feedback A A glass of water is not indicated in this situation. An easily digested carbohydrate is indicated when a child exhibits symptoms of hypoglycemia. B Insulin would lower blood glucose and is contraindicated for a child with hypoglycemia. C Four ounces of orange juice is an appropriate treatment for the conscious child who is exhibiting signs of hypoglycemia. D Subcutaneous injection of glucagon is used to treat hypoglycemia when the child is unconscious. PTS: 1 DIF: Cognitive Level: Application REF: p. 1402 OBJ: Nursing Process: Implementation MSC: Client Needs: Physiologic Integrity 21. Which sign is the nurse most likely to assess in a child with hypoglycemia? a . Urine positive for ketones and serum glucose greater than 300 mg/dL b . Normal sensorium and serum glucose greater than 160 mg/dL c . Irritability and serum glucose less than 60 mg/dL d Increased urination and serum glucose less than 120 mg/dL . ANS: C Feedback A Serum glucose greater than 300 mg/dL and urine positive for ketones are indicative of diabetic ketoacidosis. B Normal sensorium and serum glucose greater than 160 mg/dL are associated with hyperglycemia. C Irritability and serum glucose less than 60 mg/dL are neuroglycopenic manifestations of hypoglycemia. D Increased urination is an indicator of hyperglycemia. A serum glucose level less than 120 mg/dL is within normal limits. PTS: 1 DIF: Cognitive Level: Application REF: p. 1396 | Table 51-3 OBJ: Nursing Process: Assessment MSC: Client Needs: Physiologic Integrity 22. When would a child diagnosed with type 1 diabetes mellitus most likely demonstrate a decreased need for insulin? a . During the honeymoon phase b . During adolescence c . During growth spurts d . During minor illnesses ANS: A Feedback A During the honeymoon phase, which may last from a few weeks to a year or longer, the child is likely to need less insulin. B During adolescence, physical growth and hormonal changes contribute to an increase in insulin requirements. C Insulin requirements are typically increased during growth spurts. D Stress either from illness or from events in the environment can cause hyperglycemia. Insulin requirements are increased during periods of minor illness. PTS: 1 DIF: Cognitive Level: Application REF: p. 1396 OBJ: Nursing Process: Evaluation MSC: Client Needs: Physiologic Integrity 23. What should a nurse advise the parents of a child with type 1 diabetes mellitus who is not eating as a result of a minor illness? a . Give the child half his regular morning dose of insulin. b . Substitute simple carbohydrates or calorie-containing liquids for solid foods. c . Give the child plenty of unsweetened, clear liquids to prevent dehydration. d . Take the child directly to the emergency department. ANS: B Feedback A The child should receive his regular dose of insulin even if he does not have an appetite. B A sick-day diet of simple carbohydrates or calorie-containing liquids will maintain normal serum glucose levels and decrease the risk of hypoglycemia. C If the child is not eating as usual, he needs calories to prevent hypoglycemia. D During periods of minor illness, the child with type 1 diabetes mellitus can be managed safely at home. PTS: 1 DIF: Cognitive Level: Analysis REF: p. 1405 | Box 51-4 OBJ: Nursing Process: Implementation MSC: Client Needs: Health Promotion and Maintenance 24. Which is the nurses best response to the parents of a 10-year-old child newly diagnosed with type 1 diabetes mellitus who are concerned about the childs continued participation in soccer? a . Consider the swim team as an alternative to soccer. b . Encourage intellectual activity rather than participation in sports. c . It is okay to play sports such as soccer unless the weather is too hot. d . Give the child an extra 15 to 30 g of carbohydrate snack before soccer practice. ANS: D Feedback A Soccer is an appropriate sport for a child with type 1 diabetes as long as the child prevents hypoglycemia by eating a snack. B Participation in sports is not contraindicated for a child with type 1 diabetes. C The child with type 1 diabetes may participate in sports activities regardless of climate. D Exercise lowers blood glucose levels. A snack with 15 to 30 g of carbohydrates before exercise will decrease the risk of hypoglycemia. PTS: 1 DIF: Cognitive Level: Application REF: p. 1400 OBJ: Nursing Process: Planning MSC: Client Needs: Health Promotion and Maintenance 25. Which comment by a 12-year-old child with type 1 diabetes indicates deficient knowledge? a . I rotate my insulin injection sites every time I give myself an injection. b . I keep records of my glucose levels and insulin sites and amounts. c . Ill be glad when I can take a pill for my diabetes like my uncle does. d . I keep Lifesavers in my school bag in case I have a low-sugar reaction. ANS: C Feedback A Rotating injection sites is appropriate because insulin absorption varies at different sites. B Keeping records of serum glucose and insulin sites and amounts is appropriate. C Children with type 1 diabetes will require life-long insulin therapy. D Prompt treatment of hypoglycemia reduces the possibility of a severe reaction. Keeping hard candy on hand is an appropriate action. Chapter 49 Nursing Care of the Child With an Alteration in Genetics MULTIPLE CHOICE 1. A parent whose child has been diagnosed with a cognitive deficit should be counseled that intellectual impairment a . Is usually due to a genetic defect b . May be caused by a variety of factors c . Is rarely due to first trimester events d . Is usually caused by parental intellectual impairment ANS: B Feedback A Only a small percentage of children with intellectual impairment are affected by a genetic defect. B There are a multitude of causes for intellectual impairment. In most cases, a specific cause has not been identified. C One third of children with intellectual impairment are affected by first trimester events. D Intellectual impairment can be transmitted to a child only if the parent has a genetic disorder. PTS: 1 DIF: Cognitive Level: Comprehension REF: p. 1480 OBJ: Nursing Process: Planning MSC: Client Needs: Health Promotion and Maintenance 2. A parent asks the nurse why a developmental assessment is being conducted for a child during a routine well-child visit. The nurse answers based on the knowledge that routine developmental assessments during well-child visits are a Not necessary unless the parents request them . b The best method for early detection of cognitive disorders . c Frightening to parents and children and should be avoided . d Valuable in measuring intelligence in children . ANS: B Feedback A Developmental assessment is a component of all well-child examinations. B Early detection of cognitive disorders can be facilitated through assessment of development at each well-child examination. C Developmental assessments are not frightening when the parent and child are educated about the purpose of the assessment. D Developmental assessments are not intended to measure intelligence. PTS: 1 DIF: Cognitive Level: Application REF: p. 1483 OBJ: Nursing Process: Implementation MSC: Client Needs: Health Promotion and Maintenance 3. The father of a child recently diagnosed with developmental delay is very rude and hostile toward the nurses. This father was cooperative during the childs evaluation a month ago. What is the best explanation for this change in parental behavior? a The father is exhibiting symptoms of a psychiatric illness. . b The father may be abusing the child. . c The father is resentful of the time he is missing from work for this appointment. . d The father is experiencing a symptom of grief. . ANS: D Feedback A One cannot determine that a parent is exhibiting symptoms of a psychiatric illness on the basis of a single situation. B The scenario does not give any information to suggest child abuse. C Although the father may have difficulty balancing his work schedule with medical appointments for his child, a more likely explanation for his behavior change is that he is grieving the loss of a normal child. D After a child is diagnosed with a developmental delay, families typically experience a cycle of grieving that is repeated when developmental milestones are not met. PTS: 1 DIF: Cognitive Level: Application REF: p. 1485 OBJ: Nursing Process: Evaluation MSC: Client Needs: Psychosocial Integrity 4. An appropriate nursing diagnosis for a child with a cognitive dysfunction who has a limited ability to anticipate danger is a Impaired social interaction . b Deficient knowledge . c Risk for injury . d Ineffective coping . ANS: C Feedback A Impaired social interaction is indeed a concern for the child with a cognitive disorder but does not address the limited ability to anticipate danger. B Because of the childs cognitive deficit, knowledge will not be retained and will not decrease the risk for injury. C The nurse needs to know that limited cognitive abilities to anticipate danger lead to risk for injury. D Ineffective individual coping does not address the limited ability to anticipate danger. PTS: 1 DIF: Cognitive Level: Application REF: p. 1485 OBJ: Nursing Process: Diagnosis MSC: Client Needs: Health Promotion and Maintenance 5. Anticipatory guidance for the family of a preadolescent with a cognitive dysfunction should include information about a . Institutional placement b . Sexual development c . Sterilization d . Clothing ANS: B Feedback A Preadolescence does not require the child to be institutionalized. B Preadolescents who have a cognitive dysfunction may have normal sexual development without the emotional and cognitive abilities to deal with it. It is important to assist the family and child through this developmental stage. C Sterilization is not an appropriate intervention when a child has a cognitive dysfunction. D By the time a child reaches preadolescence, the family should have received counseling on age-appropriate clothing. PTS: 1 DIF: Cognitive Level: Application REF: p. 1486 OBJ: Nursing Process: Planning MSC: Client Needs: Health Promotion and Maintenance 6. What should be the major consideration when selecting toys for a child with an intellectual or developmental disability? a . Safety b . Age appropriateness c . Ability to provide exercise d . Ability to teach useful skills ANS: A Feedback A Safety is the primary concern in selecting recreational and exercise activities for all children. This is especially true for children who are intellectually disabled. B Age appropriateness should be considered in the selection of toys, but safety is of paramount importance. C Ability to provide exercise should be considered in the selection of toys, but safety is of paramount importance. D Ability to teach useful skills should be considered in the selection of toys, but safety is of paramount importance. PTS: 1 DIF: Cognitive Level: Analysis REF: p. 1484 OBJ: Nursing Process: Implementation MSC: Client Needs: Safe and Effective Care Environment 7. Appropriate interventions to facilitate socialization of the cognitively impaired child include a . Providing age-appropriate toys and play activities b . Providing peer experiences, such as scouting, when older c . Avoiding exposure to strangers who may not understand cognitive development d . Emphasizing mastery of physical skills because they are delayed more often than verbal skills ANS: B Feedback A Providing age-appropriate toys and play activities is important. However, peer interactions will better facilitate social development. B The acquisition of social skills is a complex task. Children of all ages need peer relationships. Parents should enroll the child in preschool. When older, they should have peer experiences similar to other children such as group outings, Boy and Girl Scouts, and Special Olympics. C Parents should expose the child to strangers so that the child can practice social skills. D Verbal skills are delayed more than physical skills. PTS: 1 DIF: Cognitive Level: Comprehension REF: p. 1485 OBJ: Nursing Process: Implementation MSC: Client Needs: Psychosocial Integrity 8. A newborn assessment shows separated sagittal suture, oblique palpebral fissures, depressed nasal bridge, protruding tongue, and transverse palmar creases. These findings are most suggestive of a . Microcephaly b . Down syndrome c . Cerebral palsy d . Fragile X syndrome ANS: B Feedback A The infant with microcephaly has a small head. B These are characteristics associated with Down syndrome. C Cerebral palsy is a diagnosis not usually made at birth. No characteristic physical signs are present. D The infant with fragile X syndrome has increased head circumference; long, wide, and/or protruding ears; long, narrow face with prominent jaw; hypotonia; and high arched palate. PTS: 1 DIF: Cognitive Level: Comprehension REF: p. 1488 OBJ: Nursing Process: Assessment MSC: Client Needs: Physiologic Integrity 9. The infant with Down syndrome is closely monitored during the first year of life for what serious condition? a . Thyroid complications b . Orthopedic malformations c . Dental malformation d . Cardiac abnormalities ANS: D Feedback A Infants with Down syndrome are not known to have thyroid complications. B Orthopedic malformations may be present, but special attention is given to assessment for cardiac and gastrointestinal abnormalities. C Dental malformations are not a major concern compared with the lifethreatening complications of cardiac defects. D The high incidence of cardiac defects in children with Down syndrome makes assessment for signs and symptoms of these defects important during the first year. Clinicians recommend the child be monitored frequently throughout the first 12 months of life, including a full cardiac workup. PTS: 1 DIF: Cognitive Level: Comprehension REF: p. 1487 | Box 54-4 OBJ: Nursing Process: Assessment MSC: Client Needs: Health Promotion and Maintenance 10. Many of the physical characteristics of Down syndrome present feeding problems. Care of the infant should include a Delaying feeding solid foods until the tongue thrust has stopped . b Modifying diet as necessary to minimize the diarrhea that often occurs . c Providing calories appropriate to childs age . d Using special bottles that may assist the infant with feeding . ANS: D Feedback A The child has a protruding tongue, which makes feeding difficult. The parents must persist with feeding while the child continues the physiologic response of the tongue thrust. B The child is predisposed to constipation. C Calories should be appropriate to the childs weight and growth needs, not age. D Breastfeeding may not be possible if the infants muscle tone or sucking reflex is immature. Mothers should be encouraged to pump breast milk and use special bottles for assistance with feeding. Some children with Down syndrome can breastfeed adequately. PTS: 1 DIF: Cognitive Level: Analysis REF: p. 1489 OBJ: Nursing Process: Implementation MSC: Client Needs: Physiologic Integrity 11. What action is contraindicated when a child with Down syndrome is hospitalized? a Determine the childs vocabulary for specific body functions. . b Assess the childs hearing and visual capabilities. . c Encourage parents to leave the child alone for extended periods of time. . d Have meals served at the childs usual meal times. . ANS: C Feedback A To communicate effectively with the child, it is important to know the childs particular vocabulary for specific body functions. B Children with Down syndrome have a high incidence of hearing loss and vision problems and should have hearing and vision assessed whenever they are in a health care facility. C The child with Down syndrome needs routine schedules and consistency. Having familiar people present, especially parents, helps to decrease the childs anxiety. D Routine schedules and consistency are important to children. PTS: 1 DIF: Cognitive Level: Application REF: p. 1489 OBJ: Nursing Process: Planning MSC: Client Needs: Psychosocial Integrity 12. The child with Down syndrome should be evaluated for which condition before participating in some sports? a Hyperflexibility . b Cutis marmorata . c Atlantoaxial instability . d Speckling of iris (Brushfield spots) . ANS: C Feedback A Although hyperflexibility is characteristic of Down syndrome, it does not affect the childs ability to participate in sports. B Although cutis marmorata is characteristic of Down syndrome, it does not affect the childs ability to participate in sports. C Children with Down syndrome are at risk for atlantoaxial instability. Before participating in sports that put stress on the head and neck, a radiologic examination should be done. D Although Brushfield spots are characteristic of Down syndrome, they do not affect the childs ability to participate in sports. PTS: 1 DIF: Cognitive Level: Comprehension REF: p. 1488 OBJ: Nursing Process: Assessment MSC: Client Needs: Physiologic Integrity 13. A nurse is giving a parent information about autism. Which statement made by the parent indicates understanding of the teaching? a Autism is characterized by periods of remission and exacerbation. . b The onset of autism usually occurs before 3 years of age. . c Children with autism have imitation and gesturing skills. . d Autism can be treated effectively with medication. . ANS: B Feedback A Autism does not have periods of remission and exacerbation. B The onset of autism usually occurs before 3 years of age. C Autistic children lack imitative skills. D Medications are of limited use in children with autism. PTS: 1 DIF: Cognitive Level: Application REF: p. 1494 OBJ: Nursing Process: Evaluation MSC: Client Needs: Health Promotion and Maintenance 14. What should the nurse keep in mind when planning to communicate with a child who has autism? a . The child has normal verbal communication. b . Expect the child to use sign language. c . The child may exhibit monotone speech and echolalia. d . The child is not listening if she is not looking at the nurse. ANS: C Feedback A The child has impaired verbal communication and abnormalities in the production of speech. B Some autistic children may use sign language, but it is not assumed. C Children with autism have abnormalities in the production of speech such as a monotone voice or echolalia, or inappropriate volume, pitch, rate, rhythm, or intonation. D Children with autism often are reluctant to initiate direct eye contact. PTS: 1 DIF: Cognitive Level: Application REF: p. 1496 OBJ: Nursing Process: Planning MSC: Client Needs: Psychosocial Integrity 15. Developmental delays, self-injury, fecal smearing, and severe temper tantrums in a preschool child are symptoms of a . Down syndrome b . Intellectual disability c . Psychosocial deprivation d . Separation anxiety ANS: B Feedback A Down syndrome is often identified at birth by characteristic facial and head features, such as brachycephaly (disproportionate shortness of the head); flat profile; inner epicanthal folds; wide, flat nasal bridge; narrow, high-arched palate; protruding tongue; and small, short ears, which may be low set. Although intellectual impairment may be present, the symptoms listed are not the primary ones expected in the diagnosis of Down syndrome. B These are symptoms of intellectual disability. C Psychosocial deprivation may be a cause of mild intellectual disability. The symptoms listed are characteristic of severe intellectual disability. D Symptoms of separation anxiety include protest, despair, and detachment. PTS: 1 DIF: Cognitive Level: Comprehension REF: p. 1482 | Box 54-2 OBJ: Nursing Process: Assessment MSC: Client Needs: Physiologic Integrity 16. Throughout their life span, cognitively impaired children are less capable of managing environmental challenges and are at risk for a . Nutritional deficits b . Visual impairments c . Physical injuries d . Psychiatric problems ANS: C Feedback A Nutritional deficits are related more to dietary habits and the caregivers understanding of nutrition. B Visual impairments are unrelated to cognitive impairment. C Safety is a challenge for cognitively impaired children. Decreased capability to manage environmental challenges may lead to physical injuries. D Psychiatric problems may coexist with cognitive impairment; however, they are not environmental challenges. PTS: 1 DIF: Cognitive Level: Application REF: p. 1484 OBJ: Nursing Process: Assessment MSC: Client Needs: Health Promotion and Maintenance 17. Which statement best describes Fragile X syndrome? a . Chromosomal defect affecting only females b . Chromosomal defect that follows the pattern of X-linked recessive disorders c . Second most common genetic cause of cognitive impairment d Most common cause of noninherited cognitive impairment . ANS: C Feedback A Fragile X primarily affects males. B Fragile X follows the pattern of X-linked dominant with reduced manifestation of the syndrome in female and moderate to severe dysfunction in males. C Fragile X syndrome is the most common inherited cause of cognitive impairment and the second most common cause of cognitive impairment after Down syndrome. D Fragile X is inherited. PTS: 1 DIF: Cognitive Level: Comprehension REF: p. 1490 OBJ: Nursing Process: Assessment MSC: Client Needs: Physiologic Integrity 18. The nurse is providing counseling to the mother of a child diagnosed with fragile X syndrome. She explains to the mother that fragile X syndrome is a . Most commonly seen in girls b . Acquired after birth c . Usually transmitted by the male carrier d . Usually transmitted by the female carrier ANS: D Feedback A Fragile X syndrome is most common in males. B Fragile X syndrome is congenital. C Fragile X syndrome is not transmitted by a male carrier. D The gene causing fragile X syndrome is transmitted by the mother. PTS: 1 DIF: Cognitive Level: Application REF: p. 1490 OBJ: Nursing Process: Implementation MSC: Client Needs: Health Promotion and Maintenance 19. The best setting for daytime care for a 5-year-old autistic child whose mother works is a . Private day care b . Public school c . His own home with a sitter d . A specialized program that facilitates interaction by use of behavioral methods ANS: D Feedback A Daycare programs generally do not have resources to meet the needs of severely impaired children. B To best meet the needs of an autistic child, the public school may refer the child to a specialized program. C A sitter might not have the skills to interact with an autistic child. D Autistic children can benefit from specialized educational programs that address their special needs. PTS: 1 DIF: Cognitive Level: Application REF: pp. 1496-1497 OBJ: Nursing Process: Planning MSC: Client Needs: Health Promotion and Maintenance 20. Parents have learned that their 6-year-old child has autism. The nurse may help the parents to cope by explaining that the child may a . Have an extremely developed skill in a particular area b . Outgrow the condition by early adulthood c . Have average social skills d . Have age-appropriate language skills ANS: A Feedback A Some children with autism have an extremely developed skill in a particular area such as mathematics or music. B No evidence supports that autism is outgrown. C Autistic children have abnormal ways of relating to people (social skills). D Speech and language skills are usually delayed in autistic children. PTS: 1 DIF: Cognitive Level: Application REF: p. 1495 OBJ: Nursing Process: Implementation MSC: Client Needs: Psychosocial Integrity 21. A child with autism hospitalized with asthma. The nurse should plan care so that the a . Parents expectations are met. b . Childs routine habits and preferences are maintained. c . Child is supported through the autistic crisis. d . Parents need not be at the hospital. ANS: B Feedback A Focus of care is on the childs needs rather than on the parents desires. B Children with autism are often unable to tolerate even slight changes in routine. The childs routine habits and preferences are important to maintain. C Autism is a life-long condition. D The presence of the parents is almost always required when an autistic child is hospitalized. PTS: 1 DIF: Cognitive Level: Application REF: p. 1497 OBJ: Nursing Process: Planning MSC: Client Needs: Psychosocial Integrity MULTIPLE RESPONSE 1. You are the nurse assessing a 3-year-old child who has characteristics of autism. Which observed behaviors are associated with autism? Select all that apply. a The child flicks the light in the examination room on and off repetitiously. . b The child has a flat affect. . c . The child demonstrates imitation and gesturing skills. d . Mother reports the child has no interest in playing with other children. e . The child is able to make eye contact. ANS: A, B, D Feedback Corre Self-stimulation is common and generally involves repetition of a sensory ct stimulus. Autistic children generally show a fixed, unchanging response to a particular stimulus. Autistic children generally play alone or involve others only as mere objects. Incorr Autistic children lack imitative skills. These children lack social ability ect and make poor eye contact. PTS: 1 DIF: Cognitive Level: Analysis REF: pp. 1495-1496 OBJ: Nursing Process: Assessment MSC: Client Needs: Health Promotion and Maintenance 2. A nurse is assessing a newborn for facial feature characteristics associated with fetal alcohol syndrome: Which characteristics should the nurse expect to assess? Select all that apply. a Short palpebral fissures . b Smooth philtrum . c Low set ears . d Inner epicanthal folds . e Thin upper lip . ANS: A, B, E Feedback Corre Infants with fetal alcohol syndrome may have characteristic facial ct features, including short palpebral fissures, a smooth philtrum (the vertical groove in the median portion of the upper lip), and a thin upper lip. Incorr Low set ears and inner epicanthal folds are associated with Down ect syndrome. PTS: 1 DIF: Cognitive Level: Analysis REF: p. 1492 OBJ: Nursing Process: Assessment MSC: Client Needs: Physiologic Integrity 3. A nurse should plan to implement which interventions for a child admitted with inorganic failure to thrive? Select all that apply. a Observation of parent-child interactions . b Assignment of different nurses to care for the child from day to day . c Use of 28 calorie per ounce concentrated formulas . d Administration of daily multivitamin supplements . e Role modeling appropriate adult-child interactions . ANS: A, D, E Feedback Corre The nurse should plan to assess parent-child interactions when a child is ct admitted for nonorganic failure to thrive. The observations should include how the child is held and fed, how eye contact is initiated and maintained, and the facial expressions of both the child and the caregiver during interactions. Role modeling and teaching appropriate adult-child interactions (including holding, touching, and feeding the child) will facilitate appropriate parent-child relationships, enhance parents confidence in caring for their child, and facilitate expression by the parents of realistic expectations based on the childs developmental needs. Daily multivitamin supplements with minerals are often prescribed to ensure that specific nutritional deficiencies do not occur in the course of rapid growth. The nursing staff assigned to care for the child should be consistent. Incorr Providing a consistent caregiver from the nursing staff increases trust and ect provides the child with an adult who anticipates his or her needs and who is able to role model child care to the parent. Caloric enrichment of food is essential, and formula may be concentrated in titrated amounts up to 24 calories per ounce. Greater concentrations can lead to diarrhea and dehydration. PTS: 1 DIF: Cognitive Level: Analysis REF: p. 1493 OBJ: Nursing Process: Implementation MSC: Client Needs: Physiologic Integrity COMPLETION 1. Unlike fragile X syndrome, which affects primarily males, (RS) is almost exclusively linked to female gender. ANS: Rett syndrome An estimated 1:10,000 to 1:15,000 females are affected. RS is characterized by an initial period of normal development with symptoms emerging between the ages of 6 and 18 months. Social and intellectual development stops and seizures along with physical disabilities emerge. Chapter 50 Nursing Care of the Child With an Alteration in Behavior, Cognition, or Development MULTIPLE CHOICE 1. Which sign or symptom is likely to be manifested by an adolescent with a depressive disorder? a . Abuse of alcohol b . Impulsivity and distractibility c . Carelessness and inattention to details d . Refusal to leave the house ANS: A Feedback A Depression often manifests in conjunction with substance abuse, so children who abuse substances should be evaluated for depression as well. B Impulsivity and distractibility are manifestations of attention-deficit/ hyperactivity disorder (ADHD). C A diminished ability to think or concentrate, carelessness, and inattention to details is a clinical manifestation of ADHD. D A refusal to leave the house, even to play with friends, is characteristic of separation anxiety disorder. PTS: 1 DIF: Cognitive Level: Application REF: p. 1453 OBJ: Nursing Process: Assessment MSC: Client Needs: Psychosocial Integrity 2. Which statement about suicide is correct? a . Children younger than 10 years of age do not attempt suicide. b . Suicide risk decreases with age. c . Suicide is usually an isolated event in a school community. d . The prevalence of suicide attempts is higher among males. ANS: A Feedback A Suicide by children under the age of 10 is uncommon. B The risk of suicide increases with age. C It is common for suicide to occur in a cluster within a community (e.g., schools). D Males have a 4% rate of suicide attempts compared to 8% in females; however, males are more likely to die after a suicide attempt. PTS: 1 DIF: Cognitive Level: Comprehension REF: p. 1457 OBJ: Nursing Process: Assessment MSC: Client Needs: Psychosocial Integrity 3. The best response for the nurse to make to an adolescent who states, I am very sad. I wish I was not alive. is a . Everyone feels sad once in a while. b . You are just trying to escape your problems. c . Have you told your parents how you feel? d Have you thought about hurting yourself? . ANS: D Feedback A This is a judgmental response that ignores the adolescents obvious statement indicating a need for professional help. B This is a judgmental response that could increase the adolescents sense of isolation and rejection. C The parents should be made aware of an adolescents precarious mental state; however, this response does not address the adolescents statement. D This response acknowledges the adolescents suicide gesture and further assesses the adolescents condition. PTS: 1 DIF: Cognitive Level: Application REF: p. 1458 | Box 53-1 OBJ: Nursing Process: Implementation MSC: Client Needs: Psychosocial Integrity 4. The long-term treatment plan for an adolescent with an eating disorder focuses on a . Managing the effects of malnutrition b . Establishing sufficient caloric intake c . Improving family dynamics d . Restructuring perception of body image ANS: D Feedback A The treatment of eating disorders is initially focused on reestablishing physiologic homeostasis. B Once body systems are stabilized, the next goal of treatment for eating disorders is maintaining adequate caloric intake. C Although family therapy is indicated when dysfunctional family relationships exist, the primary focus of therapy for eating disorders is to help the adolescent cope with complex issues. D The focus of treatment in individual therapy for an eating disorder involves restructuring cognitive perceptions about the individuals body image. PTS: 1 DIF: Cognitive Level: Application REF: p. 1465 OBJ: Nursing Process: Implementation MSC: Client Needs: Psychosocial Integrity 5. A parent of a child with an anxiety disorder states, I dont know how my child developed this problem. On what information should the nurse base a response? a . Genetic factors, hormonal imbalances, and societal influences all contribute to the development of anxiety disorders in children. b . Like many conditions affecting children, the etiology of anxiety disorders is unknown. c . The majority of anxiety disorders have a clear pattern of genetic inheritance. d . Dysfunctional family patterns are usually identified as the cause of an anxiety disorder. ANS: A Feedback A Anxiety disorders are responses to stress and may be manifested as disturbances in feeling, body functions, behavior, or performance. Children with a history of verbal, physical, or sexual abuse; frequent separation from or loss of loved ones; drug use, incarceration, or lower socioeconomic status; homosexuality; chronic illness; behavioral disorders; and dysfunctional families are more likely than peers with healthy family patterns to have anxiety disorders. B The etiology of many anxiety disorders in children can be identified. C Some anxiety disorders are inheritable disorders. Others have been identified as having other origins. D Research consistently shows that psychosocial disorders are caused by a combination of predisposing or inherent factors and environmental or interactional factors. PTS: 1 DIF: Cognitive Level: Application REF: p. 1451 OBJ: Nursing Process: Implementation MSC: Client Needs: Psychosocial Integrity 6. In counseling an adolescent who is abusing alcohol, the nurse explains that alcohol abuse primarily affects which organ of the body? a Heart . b Liver . c Brain . d Lungs . ANS: C Feedback A Although an excessive amount of a chemical can cause cardiac abnormalities, the brain is the most commonly affected organ. B Long-term alcohol use is known to impair the liver; however, brain function is decreased by any amount of alcohol intake. C The primary effect of substance abuse is on the brain and residually on the rest of the body. Alcohol affects the entire brain by decreasing its responsiveness. D The pulmonary system is not the primary target; however, one commonly abused drug known to cause pulmonary problems is tobacco. PTS: 1 DIF: Cognitive Level: Application REF: p. 1466 OBJ: Nursing Process: Assessment MSC: Client Needs: Psychosocial Integrity 7. The outpatient nurse understands that the phase of substance abuse characterized by a 14-yearold child admitting to using marijuana every day with friends after attending school is a . Experimentation b . Early drug use c . True drug addiction d . Severe drug addiction ANS: C Feedback A With experimentation, the individual tries the drug to see what it is like or to satisfy peers. B Early drug use is identified as using drugs with some degree of regularity for their desirable effects. C True drug addiction is identified as regular use of drugs. Physical dependence may be present. Social functioning has a drug focus. D In severe drug addiction, the physical condition of the individual deteriorates and all activities are related to drug use. PTS: 1 DIF: Cognitive Level: Application REF: p. 1466 | Box 53-2 OBJ: Nursing Process: Assessment MSC: Client Needs: Psychosocial Integrity 8. The school nurse observes an unkempt child dressed in inappropriate clothing who repeatedly asks for food. About which problem is the nurse concerned? a . Physical abuse b . Physical neglect c . Emotional abuse d . Sexual abuse ANS: B Feedback A There are no physical indicators of actual abuse in this description. Behavioral indicators of physical abuse reflect an impaired relationship with parents and other adults. B These physical and behavioral indicators suggest that parental attention is not being given to the childs physical needs. The child is being neglected. C Emotional abuse is manifested by developmental problems or maladaptive behaviors. D Physical indicators of sexual abuse are focused on the genitourinary system. A variety of behavioral indicators range from bizarre sexual behavior to eating and sleeping disturbances. PTS: 1 DIF: Cognitive Level: Application REF: p. 1468 OBJ: Nursing Process: Assessment MSC: Client Needs: Psychosocial Integrity 9. A child who has symptoms of irritable mood, changes in sleep and appetite patterns, decreased self-esteem, and disengagement from family and friends lasting 3 weeks meets the criteria for which depressive disorder? a . Major depressive disorder b . Dysthymic disorder c . Cyclothymic disorder d . Panic disorder ANS: A Feedback A A 2-week (or longer) episode of depressed or irritable mood in addition to disturbances in appetite, sleep, energy, or self-esteem meets the criteria for a major depressive disorder. B A dysthymic disorder is associated with a depressed or irritable mood for at least a year. C A cyclothymic or bipolar mood disorder is characterized by chronic, fluctuating mood disturbances between depressive lows and highs for a year. D A panic disorder is a type of anxiety disorder. PTS: 1 DIF: Cognitive Level: Comprehension REF: p. 1453 OBJ: Nursing Process: Assessment MSC: Client Needs: Psychosocial Integrity 10. What is the goal of therapeutic management for a child diagnosed with ADHD? a . Administer stimulant medications. b . Assess the child for other psychosocial disorders. c . Correct nutritional imbalances. d . Reduce the frequency and intensity of unsocialized behaviors. ANS: D Feedback A Although medications are effective in managing behaviors associated with ADHD, all families do not choose to give their child medication. Administering medication is not the primary goal. B Children with ADHD may have other psychosocial or learning problems; however, diagnosing these is not the primary goal. C Interventions to correct nutritional imbalances are the primary focus of care for eating disorders. D The primary goal of therapeutic management for the child with ADHD is to reduce the intensity and frequency of unsocialized behaviors. PTS: 1 DIF: Cognitive Level: Application REF: p. 1460 OBJ: Nursing Process: Implementation MSC: Client Needs: Psychosocial Integrity 11. Which behavior demonstrated by an adolescent should alert the school nurse to a problem of substance abuse? a . States feelings of worthlessness b . Increased desire for social conformity c . Does not feel need for peer approval d . Deterioration of relationships with family members ANS: D Feedback A Feelings of worthlessness are suggestive of a depressive disorder. An adolescent with a substance abuse problem may be depressed, but this behavior is not a manifestation of substance abuse. B The clinical manifestations of substance abuse are marked by an increase in antisocial behavior as the desire for social conformity decreases and the need for the substance increases. C The adolescent with a substance abuse problem may demonstrate an excessive dependence on peer influence. D Deterioration of relationships with family members, irregular school attendance, low grades, rebellious or aggressive behavior, and excessive dependence on peer influence are behaviors that may indicate substance abuse. PTS: 1 DIF: Cognitive Level: Application REF: p. 1466 OBJ: Nursing Process: Assessment MSC: Client Needs: Psychosocial Integrity 12. Which behavior verbalized by a school-age child should alert the school nurse to a problem of possible obsessive-compulsive disorder (OCD)? a . States feelings of worthlessness and sadness everyday b . Feels need to ride a bike around the tree in front of the house seven times every day before entering the house c . Recurrent episodes of chest pain, heart palpations, and shortness of breath when entering the computer classroom d . Deterioration of relationships with family members ANS: B Feedback A Feelings of worthlessness and sadness are suggestive of a depressive disorder. B Obsessive-compulsive disorder (OCD) manifests repetitive unwanted thoughts (obsessions) or ritualistic actions (compulsions) or both. C Panic disorders often cause recurrent episodes of chest pain, heart palpations, and shortness of breath. These symptoms may be accompanied by a feeling of impending doom. D Deterioration of relationships with family members, irregular school attendance, low grades, rebellious or aggressive behavior, and excessive dependence on peer influence are behaviors that may indicate substance abuse. PTS: 1 DIF: Cognitive Level: Application REF: p. 1453 OBJ: Nursing Process: Assessment MSC: Client Needs: Psychosocial Integrity 13. Which finding noted by the nurse on a physical assessment is most suggestive that a child has been sexually abused? a . Swelling of the genitalia and pain on urination b . Smooth philtrum and thin upper lip c Speech and physical development delays . d History of constipation, drowsiness, and constricted pupils . ANS: A Feedback A Physical indicators of sexual abuse may include swelling or itching of the genitalia and pain on urination. Other indicators may include bruises, bleeding, or lacerations of the external genitalia, vagina, or anal area. B The infant with fetal alcohol syndrome may have microphthalmia or abnormally small eyes or short palpebral fissures, a thin upper lip, and a poorly developed philtrum. C Children who have been emotionally abused may exhibit speech disorders, lags in physical development, failure to thrive, or hyperactive and disruptive behaviors. Although there is a possibility for speech and developmental delays, these are not more suggestive of sexual abuse than swollen genitalia and pain on urination. D Opiates can cause detachment and apathy, drowsiness, constricted pupils, constipation, slurred speech, and impaired judgment. PTS: 1 DIF: Cognitive Level: Application REF: p. 1469 OBJ: Nursing Process: Assessment MSC: Client Needs: Psychosocial Integrity 14. Which manifestation is atypical of ADHD? a . Talking incessantly b . Blurting out the answers to questions before the questions have been completed c . Acting withdrawn in social situations d Fidgeting with hands or feet . ANS: C Feedback A Talking excessively is a characteristic of impulsivity/hyperactivity. B Blurting out the answers to questions before the questions have been completed is an indication of the impulse control that is often lacking in children with ADHD. C The child with ADHD tends to be talkative, often interrupting conversations, rather than withdrawn in social situations. D Fidgeting is typical of the overactivity that is associated with ADHD. PTS: 1 DIF: Cognitive Level: Application REF: p. 1459 OBJ: Nursing Process: Assessment MSC: Client Needs: Psychosocial Integrity MULTIPLE RESPONSE 1. The parents of a teen suspect their child is using amphetamines. Manifestations of amphetamine use include (select all that apply) a . Weight gain b . Excessive talking and activity c . Excessive sleeping d . Insomnia e . Agitation ANS: B, D, E Feedback Corre Euphoria, hyperactivity, agitation, irritability, insomnia, weight loss, ct tachycardia, and hypertension are expected behaviors and effects of amphetamine abuse. Incorr The adolescent using amphetamines is likely to have weight loss not ect weight gain. Excessive sleeping may be associated with alcohol abuse or abuse of barbiturates. PTS: 1 DIF: Cognitive Level: Analysis REF: p. 1465 | Table 53-1 OBJ: Nursing Process: Assessment MSC: Client Needs: Psychosocial Integrity 2. A nurse working on the pediatric unit should be aware that children admitted with which of the following assessment findings are suggestive of physical child abuse? Select all that apply. a . Bruises in various stages of healing b . Bruises over the shins or bony prominences c . Burns on the palms of the hands d . A fracture of the right wrist from a sports accident e . Rib fractures in an infant ANS: A, C, E Feedback Corre Bruises in various stages of healing and burns on the palms of the hand ct may be indicative of physical abuse. Rib fractures in an infant are another indicator of physical abuse. Incorr Bruises over the shins or bony prominences are seen in children ect beginning to walk. A fracture of the right wrist can occur as the child begins to participate in sports activities. PTS: 1 DIF: Cognitive Level: Analysis REF: p. 1468 OBJ: Nursing Process: Assessment MSC: Client Needs: Psychosocial Integrity 3. The nurse is aware that suicide risk increases if the child displays which characteristics? Select all that apply. a . Previous suicide attempt b . No previous exposure to violence in the home c . Recent loss d . Effective social network e . History of physical abuse ANS: A, C, E Feedback Corre The risk of suicide increases if the child has had a previous suicide ct attempt, a recent loss, or a history of physical abuse. Incorr No previous violence in the home or having an effective social network ect decreases the risk of suicide. PTS: 1 DIF: Cognitive Level: Analysis REF: p. 1457 OBJ: Nursing Process: Assessment MSC: Client Needs: Psychosocial Integrity COMPLETION 1. The rapid onset of physical, cognitive, and emotional symptoms that results in chest pain, shortness of breath, and the signs of impending doom is known as . ANS: panic disorder Chapter 51 Nursing Care During a Pediatric Emergency MULTIPLE CHOICE 1. Which nursing action facilitates care being provided to a child in an emergency situation? a . Encourage the family to remain in the waiting room. b . Include parents as partners in providing care for the child. c . Always reassure the child and family. d . Give explanations using professional terminology. ANS: B Feedback A Allowing the parents to remain with the child may help calm the child. B Include parents as partners in the childs treatments. Parents may need direct guidance in concrete terms to help distract the child. C Telling the truth is the most important thing. False reassurance does not facilitate a trusting relationship. D Professional terminology may not be understood. Speak to the child and family in language that they will understand. PTS: 1 DIF: Cognitive Level: Application REF: p. 842 OBJ: Nursing Process: Implementation MSC: Client Needs: Physiologic Integrity 2. The father of a child in the emergency department is yelling at the physician and nurses. Which action is contraindicated in this situation? a . Provide a nondefensive response. b . Encourage the father to talk about his feelings. c . Speak in simple, short sentences. d . Tell the father he must wait in the waiting room. ANS: D Feedback A When dealing with parents who are upset, it is important not to be defensive or attempt to justify anyones actions. B Encouraging the father to talk about his feelings may assist him to acknowledge his emotions and may defuse his angry reaction. C People who are upset need to be spoken to with simple words (no longer than five letters) and short sentences (no more than five words). D Because a parent who is upset may be aggravated by observers, he should be directed to a quiet area. PTS: 1 DIF: Cognitive Level: Application REF: p. 842 OBJ: Nursing Process: Implementation MSC: Client Needs: Psychosocial Integrity 3. What is an appropriate nursing intervention for a 6-month-old infant in the emergency department? a . Distract the infant with noise or bright lights. b . Avoid warming the infant. c . Remove any pacifiers from the baby. d . Encourage the parent to hold the infant. ANS: D Feedback A Distraction with noise or bright lights is most appropriate for a preschool-age child. B In an emergency health care facility, it is important to keep infants warm. C Infants use pacifiers to comfort themselves; therefore the pacifier should not be taken away. D Parents should be encouraged to hold the infant as much as possible while in the emergency department. Having the parent hold the infant may help to calm the child. PTS: 1 DIF: Cognitive Level: Application REF: p. 845 OBJ: Nursing Process: Implementation MSC: Client Needs: Psychosocial Integrity 4. Which action should the nurse working in the emergency department implement in order to decrease fear in a 2-year-old child? a . Keep the child physically restrained during nursing care. b . Allow the child to hold a favorite toy or blanket. c . Direct the parents to remain outside the treatment room. d . Let the child decide whether to sit up or lie down for procedures. ANS: B Feedback A It may be necessary to restrain the toddler for some nursing care or procedures. Because toddlers need autonomy and do not respond well to restrictions, the nurse should remove any restriction or restraint as soon as safety permits. B Allowing a child to hold a favorite toy or blanket is comforting. C Parents should remain with the child as much as possible to calm and reassure her. D The toddler should not be given the overwhelming choice of deciding which position she prefers. PTS: 1 DIF: Cognitive Level: Application REF: p. 844 OBJ: Nursing Process: Implementation MSC: Client Needs: Psychosocial Integrity 5. Which nursing action is most appropriate to assist a preschool-age child in coping with the emergency department experience? a . Explain procedures and give the child at least 1 hour to prepare. b . Remind the child that she is a big girl. c . Avoid the use of bandages. d . Use positive terms and avoid terms such as shot and cut. ANS: D Feedback A Preschool-age children should be told about procedures immediately before they are done. Allowing 1 hour of time to prepare only allows time for fantasies and increased anxiety. B Children should not be shamed into cooperation. C Bandages are important to preschool-age children. Children in this age-group believe that their insides can leak out and that bandages stop this from happening. D Using positive terms and avoiding words that have frightening connotations assist the child in coping. PTS: 1 DIF: Cognitive Level: Application REF: p. 845 OBJ: Nursing Process: Implementation MSC: Client Needs: Psychosocial Integrity 6. Which action should the nurse incorporate into a care plan for a 14-year-old child in the emergency department? a . Limit the number of choices to be made by the adolescent. b . Insist that parents remain with the adolescent. c Provide clear explanations and encourage questions. . d Give rewards for cooperation with procedures. . ANS: C Feedback A Because adolescents are capable of abstract thinking, they should be allowed to make decisions about their care. B Adolescents should have the choice of whether parents remain with them. They are very modest, and this modesty should be respected. C Adolescents are capable of abstract thinking and can understand explanations. They should be offered the opportunity to ask questions. D Giving rewards such as stickers for cooperation with treatments or procedures is more appropriate for the younger child. PTS: 1 DIF: Cognitive Level: Application REF: p. 844 | Box 34-1 OBJ: Nursing Process: Planning MSC: Client Needs: Psychosocial Integrity 7. The emergency department nurse notices that the mother of a young child is making a lot of phone calls and getting advice from her friends about what she should do. This behavior is an indication of a . Stress b . Healthy coping skills c . Attention-getting behaviors d . Low self-esteem ANS: A Feedback A Hyperactive behavior such as making a lot of phone calls and enlisting everyones opinions is a sign of stress. B The behavior described is not a healthy coping skill. C This may be an attention-getting behavior but is more likely an indicator of stress. D This mother may have low self-esteem, but the immediate provocation is stress. PTS: 1 DIF: Cognitive Level: Analysis REF: p. 846 OBJ: Nursing Process: Assessment MSC: Client Needs: Psychosocial Integrity 8. A preschool child in the emergency department has a respiratory rate of 10 breaths per minute. How should the nurse interpret this finding? a . The child is relaxed. b . Respiratory failure is likely. c . This child is in respiratory distress. d . The childs condition is improving. ANS: B Feedback A Although the respiratory rate slows when an individual is relaxed, a rate of 10 breaths per minute in an ill preschool child is not a normal finding and is cause for concern. B Very slow breathing in an ill child is an ominous sign, indicating respiratory failure. C A rapid respiratory rate indicates respiratory distress. Other signs of respiratory distress may include retractions, grunting, and nasal flaring. D A respiratory rate of 10 breaths per minute is not a normal finding for a preschool child. This conclusion is incorrect. PTS: 1 DIF: Cognitive Level: Analysis REF: p. 848 OBJ: Nursing Process: Assessment MSC: Client Needs: Physiologic Integrity 9. The nurse observes abdominal breathing in a 2-year-old child. What does this finding indicate? a . Imminent respiratory failure b . Hypoxia c . Normal respiration d . Airway obstruction ANS: C Feedback A A very slow respiration rate is an indicator of respiratory failure. B Nasal flaring with inspiration and grunting on expiration occurs when hypoxia is present. C Young children normally exhibit abdominal breathing. When measuring respiratory rate, the nurse should observe the rise and fall of the abdomen. D The child with an airway obstruction will use accessory muscles to breathe. PTS: 1 DIF: Cognitive Level: Analysis REF: p. 848 OBJ: Nursing Process: Assessment MSC: Client Needs: Physiologic Integrity 10. What should be the emergency department nurses next action when a 6-year-old child has a systolic blood pressure of 58 mm Hg? a . Alert the physician about the systolic blood pressure. b . Comfort the child and assess respiratory rate. c . Assess the childs responsiveness to the environment. d . Alert the physician that the child may need intravenous fluids. ANS: A Feedback A Hypotension is a late sign of shock in children. The lower limit for systolic blood pressure for a child more than 1 year old is 70 mm Hg plus two times the childs age in years. A systolic blood pressure of 58 mm Hg calls for immediate action. The nurse should be direct in relaying the childs condition to the physician. B This action does not address the problem of shock, which requires immediate intervention. C Assessing the childs responsiveness is included in a neurologic assessment. It does not address the systolic blood pressure of 58 mm Hg. D Although this child most likely requires intravenous fluids, the physician must be apprised of the systolic blood pressure so that appropriate intervention can be initiated. PTS: 1 DIF: Cognitive Level: Analysis REF: p. 854 OBJ: Nursing Process: Implementation MSC: Client Needs: Physiologic Integrity 11. You are the nurse caring for a child who is diagnosed with septic shock. He begins to develop an dysrhythmia and hemodynamic instability. Endotracheal intubation is necessary. The physician feels that cardiac arrest may soon develop. What drug do you anticipate the physician will order? a . Atropine sulfate b . Epinephrine c . Sodium bicarbonate d . Inotropic agents ANS: B Feedback A Atropine sulfate is used to treat symptomatic bradycardia. B Epinephrine is the drug of choice for the management of cardiac arrest, dysrhythmias, and hemodynamic instability. C Sodium bicarbonate is given to treat severe acidosis associated with cardiac arrest. D Inotropic agents are indicated for hypotension or poor peripheral circulation in a child. PTS: 1 DIF: Cognitive Level: Analysis REF: p. 843 OBJ: Nursing Process: Planning MSC: Client Needs: Physiologic Integrity 12. You are the nurse working triage in the emergency department. A school-age child is brought in for treatment, carried by her mother. What is part of a primary assessment that should be performed first on this child? a . Determine level of consciousness. b . Obtain a health history. c . Obtain a full set of vital signs. d . Evaluate for pain. ANS: A Feedback A A primary assessment consists of assessing the childs airway, breathing, circulation, level of consciousness, and exposure (ABCDEs). B Obtaining the childs health history is a component of a secondary assessment. C Vital signs are included in a secondary assessment, after the ABCDEs are assessed. D Assessing for pain is a component of a secondary assessment. PTS: 1 DIF: Cognitive Level: Application REF: p. 846 OBJ: Nursing Process: Assessment MSC: Client Needs: Physiologic Integrity 13. What is the goal of the initial intervention for a child in cardiopulmonary arrest? a . Establishing a patent airway b . Determining a pulse rate c . Removing clothing d . Reassuring the parents ANS: A Feedback A The first intervention for a child in cardiopulmonary arrest, as for an adult, is to establish a patent airway. B Assessment of pulse follows establishment of a patent airway. C Clothing may be removed from the upper body for chest compressions after a patent airway is established. D The first priority is to establish a patent airway. PTS: 1 DIF: Cognitive Level: Application REF: p. 847 | Table 34-1 OBJ: Nursing Process: Implementation MSC: Client Needs: Physiologic Integrity 14. What is the nurses immediate action when a child comes to the emergency department with sweating, chills, and fang bite marks on the thigh? a . Secure antivenin therapy. b . Apply a tourniquet to the leg. c . Ambulate the child. d Reassure the child and parent. . ANS: A Feedback A Antivenin therapy is essential to the childs survival because the child is showing signs of envenomation. B The use of a tourniquet is no longer recommended. C When a bite or envenomation is located on an extremity, the extremity should be immobilized. D Envenomation is a potentially life-threatening condition. False reassurance is not helpful for building a trusting relationship. PTS: 1 DIF: Cognitive Level: Application REF: p. 866 OBJ: Nursing Process: Implementation MSC: Client Needs: Physiologic Integrity 15. How should the nurse instruct the mother who calls the emergency department because her 9- year-old child has just fallen on his face and one of his front teeth fell out? a . Put the tooth back in the childs mouth and call the dentist right away. b . Place the tooth in milk or water and go directly to the emergency department. c . Gently place the tooth in a plastic zippered bag until she makes a dental appointment. d . Clean the tooth and call the dentist for an immediate appointment. ANS: B Feedback A The parent may replace the tooth incorrectly, so it is best not to advise the parent to do this. B The parent should be told to keep the tooth moist by placing it in a saline solution, water, milk, or a commercial tooth-preserving solution and get the child evaluated as soon as possible. C The tooth should be kept moist, not dry. The child should be evaluated as soon as possible. D Cleaning or scrubbing the tooth could damage it. It is essential for the child to have an immediate dental evaluation. PTS: 1 DIF: Cognitive Level: Application REF: p. 871 OBJ: Nursing Process: Implementation MSC: Client Needs: Physiologic Integrity 16. A 3-year-old is brought to the emergency department by ambulance after her body was found submerged in the family pool. The child has altered mental status and shallow respirations. She did not require resuscitative interventions. Which condition should the nurse monitor first in this child? a . Neurologic status b . Hypothermia c . Hypoglycemia d . Hypoxia ANS: D Feedback A Although a neurologic assessment will be required, it is not the area of primary assessment. The airway is always assessed first. B Hypothermia offers protection to the brain. It is a concern, but not the area of primary concern. C Although the child may have electrolyte imbalances, this is not the primary assessment area. D Hypoxia is responsible for the injury to organ systems during submersion injuries. Hypoxia can progress to cardiopulmonary arrest. Monitoring the airway is always the number one concern. PTS: 1 DIF: Cognitive Level: Analysis REF: pp. 867-868 OBJ: Nursing Process: Assessment MSC: Client Needs: Physiologic Integrity 17. Assessment of a child with a submersion injury focuses on which system? a . Cardiovascular b . Respiratory c . Neurologic d . Gastrointestinal ANS: B Feedback A Cardiovascular assessment is secondary to the airway and breathing. B Assessment of the child with a submersion injury focuses on the respiratory system. The airway and breathing are the priorities. C Preventing neurologic impairment is a goal of intervention. Because the primary problem in submersion injuries is hypoxia, the focus of assessment is the respiratory system. D Gastrointestinal assessment is less of a priority than assessment of other body systems. PTS: 1 DIF: Cognitive Level: Application REF: pp. 868-869 OBJ: Nursing Process: Assessment MSC: Client Needs: Physiologic Integrity 18. Which is the most critical element of pediatric emergency care? a . Airway management b . Prevention of neurologic impairment c . Maintaining adequate circulation d . Supporting the childs family ANS: A Feedback A Airway management is the most critical element in pediatric emergency care. B Prevention of neurologic impairment is certainly a concern in pediatric emergency care; however, it is not considered the most critical element. C Maintaining adequate circulation is accomplished after a patent airway is established. D The focus of emergency care is stabilizing the childs physiologic status. Supporting the family is important, but it is not considered to be the most critical element in pediatric emergency care. PTS: 1 DIF: Cognitive Level: Application REF: p. 846, 859 OBJ: Nursing Process: Implementation MSC: Client Needs: Physiologic Integrity 19. Which observations made by an emergency department nurse raises the suspicion that a 3- year-old child has been maltreated? a . The parents are extremely calm in the emergency department. b . The injury is unusual for a child of that age. c . The child does not remember how he got hurt. d . The child was doing something unsafe when the injury occurred. ANS: B Feedback A The nurse should observe the parents reaction to the child but must keep in mind that people behave very differently depending on culture, ethnicity, experience, and psychological makeup. B An injury that is rarely found in children or is inconsistent with the age and condition of the child should raise suspicion of child maltreatment. C The child may not remember what happened as a result of the injury itself, for example, sustaining a concussion. Also, a 3-year-old child may not be a reliable historian. D The fact that the child was not supervised might be an area for health teaching. The nurse needs to gather more information to determine whether the parents have been negligent in the care of their child. PTS: 1 DIF: Cognitive Level: Analysis REF: p. 860 OBJ: Nursing Process: Assessment MSC: Client Needs: Psychosocial Integrity 20. In which situation is the administration of milk or water indicated after ingestion? a . The child is suspected of ingesting lead paint chips. b . The child ingested approximately 15 tablets of baby aspirin. c . The child ingested an over-the-counter product containing acetaminophen. d . The child ingested an acid or alkali. ANS: D Feedback A Ingestion of leaded paint chips does not indicate treatment with administration of water or milk. B Ingestion of aspirin is not treated with administration of water or milk. The treatment may involve gastric lavage with activated charcoal, IV fluids with various additives to decrease absorption, treatment of electrolyte imbalances, and vitamin K for bleeding tendencies. C Ingestion of acetaminophen is not treated with administration of milk or water. Gastric lavage within 1 hour and administration of the antidote N-acetylcysteine (Mucomyst) is indicated. D Administering water or milk can dilute the toxic effects of acid or alkali ingestion. PTS: 1 DIF: Cognitive Level: Analysis REF: p. 864 OBJ: Nursing Process: Implementation MSC: Client Needs: Physiologic Integrity 21. Which initial assessment made by the triage nurse suggests that a child requires immediate intervention? a . The child has thick yellow rhinorrhea. b . The child has a frequent nonproductive cough. c . The childs oxygen saturation is 95% by pulse oximeter. d . The child is grunting. ANS: D Feedback A Nasal discharge indicates that the child has a respiratory condition but does not mean the child needs immediate attention. B A productive cough is not a finding that indicates that the child requires immediate attention. C An oxygen saturation of 95% is a normal finding. D One of the initial observations for triage is respiratory rate and effort. Grunting is a sign of hypoxemia and represents the bodys attempt to improve oxygenation by generating positive end-expiratory pressure. [Show More]

Last updated: 1 year ago

Preview 1 out of  pages

Reviews( 0 )

$15.00

Add to cart

Instant download

Can't find what you want? Try our AI powered Search

OR

GET ASSIGNMENT HELP
28
0

Document information


Connected school, study & course


About the document


Uploaded On

Jun 22, 2022

Number of pages

Written in

Seller


seller-icon
kevinmutula

Member since 2 years

0 Documents Sold


Additional information

This document has been written for:

Uploaded

Jun 22, 2022

Downloads

 0

Views

 28

Document Keyword Tags

Recommended For You


$15.00
What is Browsegrades

In Browsegrades, a student can earn by offering help to other student. Students can help other students with materials by upploading their notes and earn money.

We are here to help

We're available through e-mail, Twitter, Facebook, and live chat.
 FAQ
 Questions? Leave a message!

Follow us on
 Twitter

Copyright © Browsegrades · High quality services·